ΠΠ»Π΅ΠΊΡΡΠΎΠ΄Π²ΠΈΠΆΡΡΠ°Ρ ΡΠΈΠ»Π° ΠΈΠ½Π΄ΡΠΊΡΠΈΠΈ
ΠΠ»Π΅ΠΊΡΡΠΈΡΠ΅ΡΠΊΠΈΠ΅ ΡΠΎΠΊΠΈ ΠΏΠΎΡΠΎΠΆΠ΄Π°ΡΡ Π²ΠΎΠΊΡΡΠ³ ΡΠ΅Π±Ρ ΠΌΠ°Π³Π½ΠΈΡΠ½ΡΠ΅ ΠΏΠΎΠ»Ρ. ΠΠ°Π½Π½Π°Ρ ΡΠ²ΡΠ·Ρ Π΄Π°Π»Π° ΡΠΎΠ»ΡΠΎΠΊ ΠΊ ΠΌΠ½ΠΎΠ³ΠΎΡΠΈΡΠ»Π΅Π½Π½ΡΠΌ ΠΏΠΎΠΏΡΡΠΊΠ°ΠΌ ΡΠΎΠ·Π΄Π°ΡΡ ΡΠ»Π΅ΠΊΡΡΠΈΡΠ΅ΡΠΊΠΈΠΉ ΡΠΎΠΊ Π² ΠΊΠΎΠ½ΡΡΡΠ΅ ΠΏΡΠΈ ΠΏΠΎΠΌΠΎΡΠΈ ΠΌΠ°Π³Π½ΠΈΡΠ½ΠΎΠ³ΠΎ ΠΏΠΎΠ»Ρ.
ΠΠ°Π½Π½ΡΡ Π·Π°Π΄Π°ΡΡ ΡΠ΅ΡΠΈΠ» Π. Π€Π°ΡΠ°Π΄Π΅ΠΉ Π² 1831 Π³ΠΎΠ΄Ρ. Π£ΡΠ΅Π½ΡΠΉ ΠΎΡΠΊΡΡΠ» ΡΠ²Π»Π΅Π½ΠΈΠ΅ ΡΠ»Π΅ΠΊΡΡΠΎΠΌΠ°Π³Π½ΠΈΡΠ½ΠΎΠΉ ΠΈΠ½Π΄ΡΠΊΡΠΈΠΈ.
ΠΠ»Π΅ΠΊΡΡΠΎΠΌΠ°Π³Π½ΠΈΡΠ½Π°Ρ ΠΈΠ½Π΄ΡΠΊΡΠΈΡ
Π―Π²Π»Π΅Π½ΠΈΠ΅ ΡΠ»Π΅ΠΊΡΡΠΎΠΌΠ°Π³Π½ΠΈΡΠ½ΠΎΠΉ ΠΈΠ½Π΄ΡΠΊΡΠΈΠΈ Π·Π°ΠΊΠ»ΡΡΠ°Π΅ΡΡΡ Π² ΡΠΎΠΌ, ΡΡΠΎ Π² Π·Π°ΠΌΠΊΠ½ΡΡΠΎΠΌ ΠΏΡΠΎΠ²ΠΎΠ΄ΡΡΠ΅ΠΌ ΠΊΠΎΠ½ΡΡΡΠ΅, Π΅ΡΠ»ΠΈ ΠΈΠ·ΠΌΠ΅Π½ΡΠ΅ΡΡΡ ΠΏΠΎΡΠΎΠΊ ΠΌΠ°Π³Π½ΠΈΡΠ½ΠΎΠΉ ΠΈΠ½Π΄ΡΠΊΡΠΈΠΈ, ΠΊΠΎΡΠΎΡΡΠΉ ΡΠ°ΡΡΠΌΠ°ΡΡΠΈΠ²Π°Π΅ΠΌΡΠΉ ΠΊΠΎΠ½ΡΡΡ ΠΎΡ Π²Π°ΡΡΠ²Π°Π΅Ρ, ΠΏΠΎΡΠ²Π»ΡΠ΅ΡΡΡ ΡΠ»Π΅ΠΊΡΡΠΈΡΠ΅ΡΠΊΠΈΠΉ ΡΠΎΠΊ. ΠΠΎΠ·Π½ΠΈΠΊΠ°ΡΡΠΈΠΉ ΡΠ»Π΅ΠΊΡΡΠΈΡΠ΅ΡΠΊΠΈΠΉ ΡΠΎΠΊ Π½Π°Π·ΡΠ²Π°ΡΡ ΡΠΎΠΊΠΎΠΌ ΠΈΠ½Π΄ΡΠΊΡΠΈΠΈ.
ΠΠ½Π°Π»ΠΈΠ·ΠΈΡΡΡ ΡΠ²ΠΎΠΈ ΠΌΠ½ΠΎΠΆΠ΅ΡΡΠ²Π΅Π½Π½ΡΠ΅ ΡΠΊΡΠΏΠ΅ΡΠΈΠΌΠ΅Π½ΡΡ, Π. Π€Π°ΡΠ°Π΄Π΅ΠΉ ΠΏΡΠΈΡΠ΅Π» ΠΊ Π²ΡΠ²ΠΎΠ΄Ρ ΠΎ ΡΠΎΠΌ, ΡΡΠΎ:
- ΠΠ½Π΄ΡΠΊΡΠΈΠΎΠ½Π½ΡΠΉ ΡΠΎΠΊ ΠΏΠΎΡΠ²Π»ΡΠ΅ΡΡΡ Π²ΡΠ΅Π³Π΄Π° ΠΏΡΠΈ ΠΈΠ·ΠΌΠ΅Π½Π΅Π½ΠΈΠΈ ΠΌΠ°Π³Π½ΠΈΡΠ½ΠΎΠ³ΠΎ ΠΏΠΎΡΠΎΠΊΠ°, ΠΊΠΎΡΠΎΡΡΠΉ ΠΎΡ Π²Π°ΡΡΠ²Π°Π΅Ρ ΠΏΡΠΎΠ²ΠΎΠ΄ΡΡΠΈΠΉ ΠΊΠΎΠ½ΡΡΡ. Π’Π°ΠΊ, Π΅ΡΠ»ΠΈ Π² ΠΎΠ΄Π½ΠΎΡΠΎΠ΄Π½ΠΎΠΌ ΠΌΠ°Π³Π½ΠΈΡΠ½ΠΎΠΌ ΠΏΠΎΠ»Π΅ ΠΏΡΠΎΠ²ΠΎΠ΄ΡΡΠΈΠΉ Π·Π°ΠΌΠΊΠ½ΡΡΡΠΉ ΠΊΠΎΠ½ΡΡΡ ΠΏΠΎΠ²Π΅ΡΠ½ΡΡΡ, ΡΠΎ Π² ΠΌΠΎΠΌΠ΅Π½Ρ ΡΠ°Π·Π²ΠΎΡΠΎΡΠ° Π² Π½Π΅ΠΌ Π±ΡΠ΄Π΅Ρ ΡΠ΅ΡΡ ΡΠΎΠΊ ΠΈΠ½Π΄ΡΠΊΡΠΈΠΈ. Π ΡΡΠΎΠΌ ΡΠ»ΡΡΠ°Π΅ ΠΈΠ½Π΄ΡΠΊΡΠΈΡ ΠΌΠ°Π³Π½ΠΈΡΠ½ΠΎΠ³ΠΎ ΠΏΠΎΠ»Ρ ΠΏΠΎΡΡΠΎΡΠ½Π½Π° ΠΎΠΊΠΎΠ»ΠΎ ΠΏΡΠΎΠ²ΠΎΠ΄ΡΡΠ΅Π³ΠΎ ΠΊΠΎΠ½ΡΡΡΠ°, ΠΏΠ΅ΡΠ΅ΠΌΠ΅Π½Π½ΡΠΌ ΡΠ²Π»ΡΠ΅ΡΡΡ ΡΠΎΠ»ΡΠΊΠΎ ΠΏΠΎΡΠΎΠΊ ΠΌΠ°Π³Π½ΠΈΡΠ½ΠΎΠΉ ΠΈΠ½Π΄ΡΠΊΡΠΈΠΈ, ΠΊΠΎΡΠΎΡΡΠΉ ΠΈΠ·ΠΌΠ΅Π½ΡΠ΅ΡΡΡ Π·Π° ΡΡΠ΅Ρ ΠΈΠ·ΠΌΠ΅Π½Π΅Π½ΠΈΡ ΠΏΠ»ΠΎΡΠ°Π΄ΠΈ ΠΊΠΎΠ½ΡΡΡΠ°.
- ΠΠ΅Π»ΠΈΡΠΈΠ½Π° ΡΠΎΠΊΠ° ΠΈΠ½Π΄ΡΠΊΡΠΈΠΈ Π½Π΅ ΡΠ²ΡΠ·Π°Π½Π° ΡΠΎ ΡΠΏΠΎΡΠΎΠ±ΠΎΠΌ ΠΈΠ·ΠΌΠ΅Π½Π΅Π½ΠΈΡ ΠΌΠ°Π³Π½ΠΈΡΠ½ΠΎΠ³ΠΎ ΠΏΠΎΡΠΎΠΊΠ°. ΠΠ½Π° ΠΎΠΏΡΠ΅Π΄Π΅Π»Π΅Π½Π° ΡΠΎΠ»ΡΠΊΠΎ ΡΠΊΠΎΡΠΎΡΡΡΡ Π΅Π³ΠΎ ΠΈΠ·ΠΌΠ΅Π½Π΅Π½ΠΈΡ. Π‘ΠΈΠ»Π° ΡΠΎΠΊΠ° ΠΈΠ½Π΄ΡΠΊΡΠΈΠΈ ΡΠ΅ΠΌ Π±ΠΎΠ»ΡΡΠ΅, ΡΠ΅ΠΌ Π±ΠΎΠ»ΡΡΠ΅ ΡΠΊΠΎΡΠΎΡΡΡ ΠΏΠ΅ΡΠ΅ΠΌΠ΅ΡΠ΅Π½ΠΈΡ ΠΌΠ°Π³Π½ΠΈΡΠ°, ΠΈΠ»ΠΈ Π±ΡΡΡΡΠΎΡΠ° ΠΈΠ·ΠΌΠ΅Π½Π΅Π½ΠΈΡ ΡΠΈΠ»Ρ ΡΠΎΠΊΠ°, ΠΈΠ»ΠΈ ΡΠΊΠΎΡΠΎΡΡΡ ΠΏΠ΅ΡΠ΅ΠΌΠ΅ΡΠ΅Π½ΠΈΡ ΠΊΠ°ΡΡΡΠ΅ΠΊ.
ΠΠ»Π΅ΠΊΡΡΠΎΠΌΠ°Π³Π½ΠΈΡΠ½Π°Ρ ΠΈΠ½Π΄ΡΠΊΡΠΈΡ ΠΏΠΎΠ΄ΡΠ²Π΅ΡΠΆΠ΄Π°Π΅Ρ ΡΠ²ΡΠ·Ρ ΠΌΠ΅ΠΆΠ΄Ρ ΡΠ»Π΅ΠΊΡΡΠΈΡΠ΅ΡΠΊΠΈΠΌΠΈ ΠΈ ΠΌΠ°Π³Π½ΠΈΡΠ½ΡΠΌΠΈ ΡΠ²Π»Π΅Π½ΠΈΡΠΌΠΈ.
ΠΠ°ΠΊΠΎΠ½ Π€Π°ΡΠ°Π΄Π΅Ρ
ΠΠ½Π°Π»ΠΈΠ·ΠΈΡΡΡ Π΄Π°Π½Π½ΡΠ΅ ΡΠ²ΠΎΠΈΡ ΡΠΊΡΠΏΠ΅ΡΠΈΠΌΠ΅Π½ΡΠΎΠ², Π. Π€Π°ΡΠ°Π΄Π΅ΠΉ ΠΏΡΠ΅Π΄Π»ΠΎΠΆΠΈΠ» ΠΊΠΎΠ»ΠΈΡΠ΅ΡΡΠ²Π΅Π½Π½ΡΠΉ Π·Π°ΠΊΠΎΠ½, ΠΎΠΏΠΈΡΡΠ²Π°ΡΡΠΈΠΉ ΡΠ»Π΅ΠΊΡΡΠΎΠΌΠ°Π³Π½ΠΈΡΠ½ΡΡ ΠΈΠ½Π΄ΡΠΊΡΠΈΡ. Π£ΡΠ΅Π½ΡΠΉ Π΄ΠΎΠΊΠ°Π·Π°Π», ΡΡΠΎ ΠΊΠ°ΠΆΠ΄ΡΠΉ ΡΠ°Π· ΠΏΡΠΈ ΠΈΠ·ΠΌΠ΅Π½Π΅Π½ΠΈΠΈ ΠΏΠΎΡΠΎΠΊΠ° ΠΌΠ°Π³Π½ΠΈΡΠ½ΠΎΠΉ ΠΈΠ½Π΄ΡΠΊΡΠΈΠΈ, ΠΊΠΎΡΠΎΡΡΠΉ ΡΡΠ΅ΠΏΠ»Π΅Π½ Ρ ΠΏΡΠΎΠ²ΠΎΠ΄ΡΡΠΈΠΌ ΠΊΠΎΠ½ΡΡΡΠΎΠΌ, Π² ΠΏΡΠΎΠ²ΠΎΠ΄Π½ΠΈΠΊΠ΅ ΠΏΠΎΡΠ²Π»ΡΠ΅ΡΡΡ ΡΠΎΠΊ ΠΈΠ½Π΄ΡΠΊΡΠΈΠΈ. ΠΠ°Π»ΠΈΡΠΈΠ΅ ΠΈΠ½Π΄ΡΠΊΡΠΈΠΎΠ½Π½ΠΎΠ³ΠΎ ΡΠΎΠΊΠ° ΠΎΠ·Π½Π°ΡΠ°Π΅Ρ ΡΠΎ, ΡΡΠΎ Π² ΡΠ΅ΠΏΠΈ ΠΏΡΠΈΡΡΡΡΡΠ²ΡΠ΅Ρ ΡΠ»Π΅ΠΊΡΡΠΎΠ΄Π²ΠΈΠΆΡΡΠ°Ρ ΡΠΈΠ»Π° (ΠΠΠ‘), ΠΊΠΎΡΠΎΡΡΡ Π² Π΄Π°Π½Π½ΠΎΠΌ ΡΠ»ΡΡΠ°Π΅ Π½Π°Π·ΡΠ²Π°ΡΡ ΡΠ»Π΅ΠΊΡΡΠΎΠ΄Π²ΠΈΠΆΡΡΠ΅ΠΉ ΡΠΈΠ»ΠΎΠΉ ΡΠ»Π΅ΠΊΡΡΠΎΠΌΠ°Π³Π½ΠΈΡΠ½ΠΎΠΉ ΠΈΠ½Π΄ΡΠΊΡΠΈΠΈ ($Ζ_i$).
ΠΠ΅Π»ΠΈΡΠΈΠ½Π° ΡΠΎΠΊΠ° ΠΈΠ½Π΄ΡΠΊΡΠΈΠΈ, Π° Π·Π½Π°ΡΠΈΡ, ΠΈ Π²Π΅Π»ΠΈΡΠΈΠ½Π° $Ζ_i$ Π·Π°Π²ΠΈΡΠΈΡ ΠΎΡ ΡΠΊΠΎΡΠΎΡΡΠΈ ΠΈΠ·ΠΌΠ΅Π½Π΅Π½ΠΈΡ ΠΌΠ°Π³Π½ΠΈΡΠ½ΠΎΠ³ΠΎ ΠΏΠΎΡΠΎΠΊΠ°:
$\left|Ζ_{i} \right|=\frac{dΠ€}{dt}\left( 1 \right)$.
Π³Π΄Π΅ $Π€$ — ΠΏΠΎΡΠΎΠΊ ΠΌΠ°Π³Π½ΠΈΡΠ½ΠΎΠΉ ΠΈΠ½Π΄ΡΠΊΡΠΈΠΈ.
ΠΠΏΡΠ΅Π΄Π΅Π»ΠΈΠΌΡΡ ΡΠΎ Π·Π½Π°ΠΊΠΎΠΌ ΠΠΠ‘ ΠΈΠ½Π΄ΡΠΊΡΠΈΠΈ. ΠΠ½Π°ΠΊ ΠΏΠΎΡΠΎΠΊΠ° ΠΌΠ°Π³Π½ΠΈΡΠ½ΠΎΠΉ ΠΈΠ½Π΄ΡΠΊΡΠΈΠΈ ΡΠ²ΡΠ·Π°Π½ Ρ Π²ΡΠ±ΠΎΡΠΎΠΌ ΠΏΠΎΠ»ΠΎΠΆΠΈΡΠ΅Π»ΡΠ½ΠΎΠΉ Π½ΠΎΡΠΌΠ°Π»ΠΈ ΠΊ ΡΠ°ΡΡΠΌΠ°ΡΡΠΈΠ²Π°Π΅ΠΌΠΎΠΌΡ ΠΏΡΠΎΠ²ΠΎΠ΄ΡΡΠ΅ΠΌΡ ΠΊΠΎΠ½ΡΡΡΡ. Π Π½Π°ΠΏΡΠ°Π²Π»Π΅Π½ΠΈΠ΅ ΡΠΈΠ»Ρ ΡΠΎΠΊΠ° ΠΈ Π½Π°ΠΏΡΠ°Π²Π»Π΅Π½ΠΈΠ΅ Π½ΠΎΡΠΌΠ°Π»ΠΈ ΡΠ²ΡΠ·ΡΠ²Π°Π΅Ρ ΠΏΡΠ°Π²ΠΈΠ»ΠΎ ΠΏΡΠ°Π²ΠΎΠ³ΠΎ Π±ΡΡΠ°Π²ΡΠΈΠΊΠ° (Π²ΠΈΠ½ΡΠ°). ΠΠΎΠ»ΡΡΠ°Π΅ΡΡΡ, ΡΡΠΎ ΡΠΈΠΊΡΠΈΡΡΡ Π½Π°ΠΏΡΠ°Π²Π»Π΅Π½ΠΈΠ΅ Π½ΠΎΡΠΌΠ°Π»ΠΈ, ΠΌΡ ΡΡΡΠ°Π½Π°Π²Π»ΠΈΠ²Π°Π΅ΠΌ Π·Π½Π°ΠΊ ΠΌΠ°Π³Π½ΠΈΡΠ½ΠΎΠ³ΠΎ ΠΏΠΎΡΠΎΠΊΠ°, Π½Π°ΠΏΡΠ°Π²Π»Π΅Π½ΠΈΠ΅ ΡΠΎΠΊΠ° ΠΈ $Ζ_i$ Π² ΠΊΠΎΠ½ΡΡΡΠ΅.
Π‘ΡΠΎΡΠΌΡΠ»ΠΈΡΡΠ΅ΠΌ Π·Π°ΠΊΠΎΠ½ ΡΠ»Π΅ΠΊΡΡΠΎΠΌΠ°Π³Π½ΠΈΡΠ½ΠΎΠΉ ΠΈΠ½Π΄ΡΠΊΡΠΈΠΈ Π€Π°ΡΠ°Π΄Π΅Ρ Π² ΠΎΠΊΠΎΠ½ΡΠ°ΡΠ΅Π»ΡΠ½ΠΎΠΌ Π²ΠΈΠ΄Π΅:
ΠΠΏΡΠ΅Π΄Π΅Π»Π΅Π½ΠΈΠ΅ 1
ΠΠ΅ Π·Π°Π²ΠΈΡΠΈΠΌΠΎ ΠΎΡ ΠΏΡΠΈΡΠΈΠ½Ρ ΠΈΠ·ΠΌΠ΅Π½Π΅Π½ΠΈΡ ΠΌΠ°Π³Π½ΠΈΡΠ½ΠΎΠ³ΠΎ ΠΏΠΎΡΠΎΠΊΠ°, ΠΊΠΎΡΠΎΡΡΠΉ ΠΎΡ Π²Π°ΡΡΠ²Π°Π΅Ρ Π·Π°ΠΌΠΊΠ½ΡΡΡΠΉ ΠΏΡΠΎΠ²ΠΎΠ΄ΡΡΠΈΠΉ ΠΊΠΎΠ½ΡΡΡ, ΡΠ»Π΅ΠΊΡΡΠΎΠ΄Π²ΠΈΠΆΡΡΠ°Ρ ΡΠΈΠ»Π° ΠΈΠ½Π΄ΡΠΊΡΠΈΠΈ, ΠΏΠΎΡΠ²Π»ΡΡΡΠ°ΡΡΡ Π² ΡΡΠΎΠΌ ΠΊΠΎΠ½ΡΡΡΠ΅ ΡΠ°Π²Π½Π°:
$Ζ_{i}=-\frac{dΠ€}{dt}\left( 2 \right)$.
Π³Π΄Π΅ ΠΏΠΎΠ΄ $\frac{dΠ€}{dt}$ ΠΏΠΎΠ½ΠΈΠΌΠ°ΡΡ ΠΏΠΎΠ»Π½ΡΡ ΡΠΊΠΎΡΠΎΡΡΡ ΠΈΠ·ΠΌΠ΅Π½Π΅Π½ΠΈΡ ΠΏΠΎΡΠΎΠΊΠ° ΠΌΠ°Π³Π½ΠΈΡΠ½ΠΎΠΉ ΠΈΠ½Π΄ΡΠΊΡΠΈΠΈ, ΠΎΡ Π²Π°ΡΡΠ²Π°Π΅ΠΌΠΎΠ³ΠΎ ΠΏΡΠΎΠ²ΠΎΠ΄Π½ΠΈΠΊΠΎΠΌ.
ΠΠΈΠ½ΡΡ Π² ΡΠΎΡΠΌΡΠ»Π΅ (2) ΡΠΊΠ°Π·ΡΠ²Π°Π΅Ρ Π½Π° ΡΠΎ, ΡΡΠΎ:
- ΠΡΠΈ ΡΠΎΡΡΠ΅ ΠΏΠΎΡΠΎΠΊΠ° ΠΌΠ°Π³Π½ΠΈΡΠ½ΠΎΠΉ ΠΈΠ½Π΄ΡΠΊΡΠΈΠΈ (ΡΠΊΠΎΡΠΎΡΡΡ ΠΈΠ·ΠΌΠ΅Π½Π΅Π½ΠΈΡ ΠΌΠ°Π³Π½ΠΈΡΠ½ΠΎΠ³ΠΎ ΠΏΠΎΡΠΎΠΊΠ° Π±ΠΎΠ»ΡΡΠ΅ Π½ΡΠ»Ρ) ($\frac{dΠ€}{dt}>0)$, ΠΠΠ‘ ΠΈΠ½Π΄ΡΠΊΡΠΈΠΈ ΠΌΠ΅Π½ΡΡΠ΅ Π½ΡΠ»Ρ ($Ζ_i
- ΠΡΠΈ ΡΠΌΠ΅Π½ΡΡΠ΅Π½ΠΈΠΈ ΠΏΠΎΡΠΎΠΊΠ° ΠΌΠ°Π³Π½ΠΈΡΠ½ΠΎΠΉ ΠΈΠ½Π΄ΡΠΊΡΠΈΠΈ (ΡΠΊΠΎΡΠΎΡΡΡ ΠΈΠ·ΠΌΠ΅Π½Π΅Π½ΠΈΡ ΠΌΠ°Π³Π½ΠΈΡΠ½ΠΎΠ³ΠΎ ΠΏΠΎΡΠΎΠΊΠ° ΠΌΠ΅Π½ΡΡΠ΅ Π½ΡΠ»Ρ), ΠΠΠ‘ ΠΈΠ½Π΄ΡΠΊΡΠΈΠΈ Π±ΠΎΠ»ΡΡΠ΅ Π½ΡΠ»Ρ ($Ζ_i>0$).
ΠΠ°ΠΌΠ΅ΡΠ°Π½ΠΈΠ΅ 1
ΠΠ½Π°ΠΊ ΠΌΠΈΠ½ΡΡ Π² ΡΠΎΡΠΌΡΠ»Π΅ (2) β ΡΡΠΎ ΠΌΠ°ΡΠ΅ΠΌΠ°ΡΠΈΡΠ΅ΡΠΊΠΎΠ΅ ΠΎΡΠΎΠ±ΡΠ°ΠΆΠ΅Π½ΠΈΠ΅ ΠΏΡΠ°Π²ΠΈΠ»Π° ΠΠ΅Π½ΡΠ°, ΠΊΠΎΡΠΎΡΠΎΠ΅ ΠΈΡΠΏΠΎΠ»ΡΠ·ΡΡΡ Π΄Π»Ρ ΡΠΎΠ³ΠΎ, ΡΡΠΎΠ±Ρ Π½Π°ΠΉΡΠΈ Π½Π°ΠΏΡΠ°Π²Π»Π΅Π½ΠΈΠ΅ ΡΠΎΠΊΠ° ΠΈΠ½Π΄ΡΠΊΡΠΈΠΈ.
ΠΠ°ΠΊΠΎΠ½ Π€Π°ΡΠ°Π΄Π΅Ρ ΡΠΏΡΠ°Π²Π΅Π΄Π»ΠΈΠ² ΠΏΡΠΈ:
- ΠΏΡΠΎΠΈΠ·Π²ΠΎΠ»ΡΠ½ΡΡ ΠΏΠ΅ΡΠ΅ΠΌΠ΅ΡΠ΅Π½ΠΈΡΡ Π·Π°ΠΌΠΊΠ½ΡΡΠΎΠ³ΠΎ ΠΏΡΠΎΠ²ΠΎΠ΄ΡΡΠ΅Π³ΠΎ ΠΊΠΎΠ½ΡΡΡΠ°;
- ΠΏΡΠΈ Π»ΡΠ±ΡΡ Π΅Π³ΠΎ Π΄Π΅ΡΠΎΡΠΌΠ°ΡΠΈΡΡ ;
- ΠΈΠ·ΠΌΠ΅Π½Π΅Π½ΠΈΡΡ ΠΌΠ°Π³Π½ΠΈΡΠ½ΠΎΠ³ΠΎ ΠΏΠΎΠ»Ρ.
ΠΠΠ‘ ΠΈΠ½Π΄ΡΠΊΡΠΈΠΈ ΠΈΠ·ΠΌΠ΅ΡΡΠ΅ΡΡΡ Ρ ΠΠ΅ΠΆΠ΄ΡΠ½Π°ΡΠΎΠ΄Π½ΠΎΠΉ ΡΠΈΡΡΠ΅ΠΌΠ΅ Π΅Π΄ΠΈΠ½ΠΈΡ (Π‘Π) Π² Π²ΠΎΠ»ΡΡΠ°Ρ (Π).
$\left[ Ζ_{i} \right]=\left[ \frac{dΠ€}{dt} \right]=\frac{ΠΠ±}{Ρ}$=Π.
ΠΠ½Π°ΡΠ΅Π½ΠΈΠ΅ Π·Π°ΠΊΠΎΠ½Π° Π€Π°ΡΠ°Π΄Π΅Ρ
ΠΠ°ΠΊΠΎΠ½ Π€Π°ΡΠ°Π΄Π΅Ρ Π²ΡΡΠ°ΠΆΠ°Π΅Ρ Π½ΠΎΠ²ΠΎΠ΅ ΡΠΈΠ·ΠΈΡΠ΅ΡΠΊΠΎΠ΅ ΡΠ²Π»Π΅Π½ΠΈΠ΅, Π² ΠΊΠΎΡΠΎΡΠΎΠΌ ΠΏΠ΅ΡΠ΅ΠΌΠ΅Π½Π½ΠΎΠ΅ ΠΌΠ°Π³Π½ΠΈΡΠ½ΠΎΠ΅ ΠΏΠΎΠ»Π΅ ΠΏΠΎΡΠΎΠΆΠ΄Π°Π΅Ρ ΡΠ»Π΅ΠΊΡΡΠΈΡΠ΅ΡΠΊΠΎΠ΅ ΠΏΠΎΠ»Π΅. ΠΡΡΡΠ΄Π° Π΄Π΅Π»Π°Π΅ΡΡΡ Π²ΡΠ²ΠΎΠ΄ ΠΎ ΡΠΎΠΌ, ΡΡΠΎ ΡΠ»Π΅ΠΊΡΡΠΈΡΠ΅ΡΠΊΠΎΠ΅ ΠΏΠΎΠ»Π΅ ΠΌΠΎΠΆΠ΅Ρ ΠΏΠΎΡΠΎΠΆΠ΄Π°ΡΡΡΡ Π½Π΅ ΡΠΎΠ»ΡΠΊΠΎ ΡΠ»Π΅ΠΊΡΡΠΈΡΠ΅ΡΠΊΠΈΠΌΠΈ Π·Π°ΡΡΠ΄Π°ΠΌΠΈ, Π½ΠΎ ΠΈ ΠΈΠ·ΠΌΠ΅Π½ΡΡΡΠΈΠΌΡΡ ΠΌΠ°Π³Π½ΠΈΡΠ½ΡΠΌ ΠΏΠΎΠ»Π΅ΠΌ.
ΠΠ»Π΅ΠΊΡΡΠΎΠΌΠ°Π³Π½ΠΈΡΠ½Π°Ρ ΠΈΠ½Π΄ΡΠΊΡΠΈΡ β ΡΡΠΎ Π²ΡΠ΅ΠΎΠ±ΡΠΈΠΉ ΡΡΠ½Π΄Π°ΠΌΠ΅Π½ΡΠ°Π»ΡΠ½ΡΠΉ Π·Π°ΠΊΠΎΠ½ ΠΏΡΠΈΡΠΎΠ΄Ρ, ΡΠ΅Π°Π»ΠΈΠ·ΡΡΡΠΈΠΉ ΡΠ²ΡΠ·Ρ ΠΌΠ΅ΠΆΠ΄Ρ ΡΠ»Π΅ΠΊΡΡΠΈΡΠ΅ΡΠΊΠΈΠΌΠΈ ΠΈ ΠΌΠ°Π³Π½ΠΈΡΠ½ΡΠΌΠΈ ΠΏΠΎΠ»ΡΠΌΠΈ.
ΠΡΠΈΡΠΎΠ΄Π° ΡΠ»Π΅ΠΊΡΡΠΎΠ΄Π²ΠΈΠΆΡΡΠ΅ΠΉ ΡΠΈΠ»Ρ ΠΈΠ½Π΄ΡΠΊΡΠΈΠΈ
ΠΡΠ»ΠΈ ΠΏΡΠΎΠ²ΠΎΠ΄Π½ΠΈΠΊ ΠΏΠ΅ΡΠ΅ΠΌΠ΅ΡΠ°Π΅ΡΡΡ Π² ΠΌΠ°Π³Π½ΠΈΡΠ½ΠΎΠΌ ΠΏΠΎΠ»Π΅, ΡΠΎ Π½Π° ΡΠ²ΠΎΠ±ΠΎΠ΄Π½ΡΠ΅ ΡΠ»Π΅ΠΊΡΡΠΎΠ½Ρ Π΅Π³ΠΎ Π²Π΅ΡΠ΅ΡΡΠ²Π° Π΄Π΅ΠΉΡΡΠ²ΡΡΡ ΡΠΈΠ»Ρ ΠΠΎΡΠ΅Π½ΡΠ°. ΠΡΠΈ ΡΠ»Π΅ΠΊΡΡΠΎΠ½Ρ ΠΏΠΎΠ΄ Π²ΠΎΠ·Π΄Π΅ΠΉΡΡΠ²ΠΈΠ΅ΠΌ Π½Π°Π·Π²Π°Π½Π½ΠΎΠΉ ΡΠΈΠ»Ρ ΠΏΡΠΈΡ ΠΎΠ΄ΡΡ Π² Π΄Π²ΠΈΠΆΠ΅Π½ΠΈΠ΅ ΠΎΡΠ½ΠΎΡΠΈΡΠ΅Π»ΡΠ½ΠΎ ΠΏΡΠΎΠ²ΠΎΠ΄Π½ΠΈΠΊΠ°, ΡΡΠΎ ΠΎΠ·Π½Π°ΡΠ°Π΅Ρ: Π² ΠΏΡΠΎΠ²ΠΎΠ΄Π½ΠΈΠΊΠ΅ ΠΏΠΎΡΠ²Π»ΡΠ΅ΡΡΡ ΡΠΎΠΊ.
Π ΠΈΡΡΠ½ΠΎΠΊ 1. ΠΡΠΎΠ²ΠΎΠ΄Π½ΠΈΠΊΠΈ. ΠΠ²ΡΠΎΡ24 β ΠΈΠ½ΡΠ΅ΡΠ½Π΅Ρ-Π±ΠΈΡΠΆΠ° ΡΡΡΠ΄Π΅Π½ΡΠ΅ΡΠΊΠΈΡ ΡΠ°Π±ΠΎΡ
Π Π°ΡΡΠΌΠΎΡΡΠΈΠΌ ΠΏΡΡΠΌΠΎΠΉ ΡΡΠ°ΡΡΠΎΠΊ $DG$ ΠΏΡΠΎΠ²ΠΎΠ΄Π½ΠΈΠΊΠ° Π½Π° ΡΠΈΡ.1. ΠΡΠΎΡ ΡΡΠ°ΡΡΠΎΠΊ ΠΏΠ΅ΡΠ΅ΠΌΠ΅ΡΠ°Π΅ΡΡΡ ΡΠΎ ΡΠΊΠΎΡΠΎΡΡΡΡ $\vec v$ ΠΏΠΎ ΠΏΡΠΎΠ²ΠΎΠ΄Π½ΠΈΠΊΠ°ΠΌ $CK$ ΠΈ $AL$, ΠΊΠ°ΠΊ ΠΏΠΎ Π½Π°ΠΏΡΠ°Π²Π»ΡΡΡΠΈΠΌ. ΠΡΠΈ ΡΡΠΎΠΌ ΠΊΠΎΠ½ΡΡΡ $AGDCA$ ΠΏΠΎΡΡΠΎΡΠ½Π½ΠΎ Π·Π°ΠΌΠΊΠ½ΡΡ. ΠΠ΅ΠΊΡΠΎΡ ΠΈΠ½Π΄ΡΠΊΡΠΈΠΈ Π²Π½Π΅ΡΠ½Π΅Π³ΠΎ ΠΌΠ°Π³Π½ΠΈΡΠ½ΠΎΠ³ΠΎ ΠΏΠΎΠ»Ρ Π½ΠΎΡΠΌΠ°Π»Π΅Π½ ΠΏΠ»ΠΎΡΠΊΠΎΡΡΠΈ ΡΠ°ΡΡΠΌΠ°ΡΡΠΈΠ²Π°Π΅ΠΌΠΎΠ³ΠΎ ΠΊΠΎΠ½ΡΡΡΠ°. ΠΠ°Π³Π½ΠΈΡΠ½ΠΎΠ΅ ΠΏΠΎΠ»Π΅ Π±ΡΠ΄Π΅ΠΌ ΡΡΠΈΡΠ°ΡΡ ΠΎΠ΄Π½ΠΎΡΠΎΠ΄Π½ΡΠΌ. ΠΠ° Π·Π°ΡΡΠ΄Ρ, ΠΊΠΎΡΠΎΡΡΠ΅ ΠΏΠ΅ΡΠ΅ΠΌΠ΅ΡΠ°ΡΡΡΡ Π²ΠΌΠ΅ΡΡΠ΅ Ρ ΠΏΡΠΎΠ²ΠΎΠ΄Π½ΠΈΠΊΠΎΠΌ, Π΄Π΅ΠΉΡΡΠ²ΡΠ΅Ρ ΡΠΈΠ»Π° ΠΠΎΡΠ΅Π½ΡΠ°, ΡΠ°Π²Π½Π°Ρ:
$\vec{F}_{L}=q\left( \vec{v}\times \vec{B} \right)\left( 3 \right)$.
Π³Π΄Π΅$ \vec{B}$β ΠΈΠ½Π΄ΡΠΊΡΠΈΡ Π²Π½Π΅ΡΠ½Π΅Π³ΠΎ ΠΌΠ°Π³Π½ΠΈΡΠ½ΠΎΠ³ΠΎ ΠΏΠΎΠ»Ρ. ΠΠΎΠ΄ Π²ΠΎΠ·Π΄Π΅ΠΉΡΡΠ²ΠΈΠ΅ΠΌ ΡΠΈΠ»Ρ ΠΠΎΡΠ΅Π½ΡΠ° ΡΠ²ΠΎΠ±ΠΎΠ΄Π½ΡΠ΅ ΡΠ»Π΅ΠΊΡΡΠΎΠ½Ρ ΠΏΡΠΎΠ²ΠΎΠ΄Π½ΠΈΠΊΠ° ΠΏΡΠΈΡ ΠΎΠ΄ΡΡ Π² Π΄Π²ΠΈΠΆΠ΅Π½ΠΈΠ΅ ΠΈ ΠΎΠ±ΡΠ°Π·ΡΡΡ ΡΠ»Π΅ΠΊΡΡΠΈΡΠ΅ΡΠΊΠΈΠΉ ΡΠΎΠΊ. D {vBdl=vBl\, \left( 6 \right).} $
ΠΠ° Π½Π΅ Π΄Π²ΠΈΠΆΡΡΠΈΡ ΡΡ ΡΡΠ°ΡΡΠΊΠ°Ρ Π·Π°ΠΌΠΊΠ½ΡΡΠΎΠ³ΠΎ ΠΊΠΎΠ½ΡΡΡΠ°, ΠΊΠΎΡΠΎΡΡΠΉ ΠΌΡ ΡΠ°ΡΡΠΌΠ°ΡΡΠΈΠ²Π°Π΅ΠΌ, ΠΠΠ‘ Π½Π΅ Π²ΠΎΠ·Π½ΠΈΠΊΠ°Π΅Ρ. Π‘Π»Π΅Π΄ΠΎΠ²Π°ΡΠ΅Π»ΡΠ½ΠΎ, ΠΠΠ‘ ΠΊΠΎΠ½ΡΡΡΠ° ΡΠ°Π²Π½Π° ΠΠΠ‘ ΠΏΠΎΠ΄Π²ΠΈΠΆΠ½ΠΎΠ³ΠΎ ΠΏΡΠΎΠ²ΠΎΠ΄Π½ΠΈΠΊΠ° $DG$, ΠΏΠ΅ΡΠ΅ΠΌΠ΅ΡΠ°ΡΡΠ΅ΠΉΡΡ Π² ΠΌΠ°Π³Π½ΠΈΡΠ½ΠΎΠΌ ΠΏΠΎΠ»Π΅.
$Ζ_{i}=\int\limits_{AGDCA} {\vec{E}d\vec{l}=vBl\, \left( 7 \right).} $
Π‘ΠΊΠΎΡΠΎΡΡΡ ΠΏΠ΅ΡΠ΅ΠΌΠ΅ΡΠ΅Π½ΠΈΡ ΠΏΡΠΎΠ²ΠΎΠ΄Π½ΠΈΠΊΠ° Π²ΡΡΠ°Π·ΠΈΠΌ ΠΊΠ°ΠΊ:
$v=\frac{dx}{dt}\left( 8 \right)$.
Π³Π΄Π΅ $x$ — ΠΊΠΎΠΎΡΠ΄ΠΈΠ½Π°ΡΠ° ΠΊΠΎΠ½ΡΠ°ΠΊΡΠΎΠ² ΠΏΡΠΎΠ²ΠΎΠ΄Π½ΠΈΠΊΠ° Π² ΡΠΎΡΠΊΠ°Ρ $D$ ΠΈ $G$ Π½Π°ΠΏΡΠ°Π²Π»ΡΡΡΠΈΠΌΠΈ ΠΏΡΠΎΠ²ΠΎΠ΄Π½ΠΈΠΊΠ°ΠΌΠΈ:
$Ζ_{i}=Bl\frac{dx}{dt}\left( 9 \right)$.
Π£ΡΠΈΡΡΠ²Π°Ρ, ΡΡΠΎ:
$Π€=-xlB$ (10),
Π³Π΄Π΅ $Π€$ — ΠΏΠΎΡΠΎΠΊ ΠΌΠ°Π³Π½ΠΈΡΠ½ΠΎΠΉ ΠΈΠ½Π΄ΡΠΊΡΠΈΠΈ ΡΠ΅ΡΠ΅Π· ΠΏΠΎΠ²Π΅ΡΡ Π½ΠΎΡΡΡ, ΠΊΠΎΡΠΎΡΡΡ ΠΎΠ³ΡΠ°Π½ΠΈΡΠΈΠ²Π°Π΅Ρ ΠΊΠΎΠ½ΡΡΡ $AGDCA$. ΠΠ½Π°ΠΊ ΠΌΠΈΠ½ΡΡ ΡΠΊΠ°Π·ΡΠ²Π°Π΅Ρ Π½Π° ΡΠΎ, ΡΡΠΎ Π½Π°ΠΏΡΠ°Π²Π»Π΅Π½ΠΈΡ Π²Π΅ΠΊΡΠΎΡΠΎΠ² $\vec B$ ΠΈ $ d\vec S$ ΠΏΡΠΎΡΠΈΠ²ΠΎΠΏΠΎΠ»ΠΎΠΆΠ½Ρ,
ΠΎΠΊΠΎΠ½ΡΠ°ΡΠ΅Π»ΡΠ½ΠΎ ΠΈΠΌΠ΅Π΅ΠΌ:
$Ζ_{i}=-\frac{dΠ€}{dt}\left( 11 \right)$.
ΠΡΡΠ°ΠΆΠ΅Π½ΠΈΠ΅ (11) ΠΌΡ ΠΏΠΎΠ»ΡΡΠΈΠ»ΠΈ, ΡΠ°ΡΡΠΌΠ°ΡΡΠΈΠ²Π°Ρ Π΄Π²ΠΈΠΆΠ΅Π½ΠΈΠ΅ ΡΠ°ΡΡΠΈ ΠΏΡΠΎΠ²ΠΎΠ΄Π½ΠΈΠΊΠ°. ΠΡΠΈ ΠΏΠ΅ΡΠ΅ΠΌΠ΅ΡΠ΅Π½ΠΈΠΈ Π½Π΅ΡΠΊΠΎΠ»ΡΠΊΠΈΡ ΡΡΠ°ΡΡΠΊΠΎΠ² ΠΏΡΠΎΠ²ΠΎΠ΄Π½ΠΈΠΊΠ°, ΠΠΠ‘ ΠΈΠ½Π΄ΡΠΊΡΠΈΠΈ Π½Π°Ρ ΠΎΠ΄ΡΡ ΠΊΠ°ΠΊ Π°Π»Π³Π΅Π±ΡΠ°ΠΈΡΠ΅ΡΠΊΡΡ ΡΡΠΌΠΌΡ ΠΠΠ‘ ΠΈΠ½Π΄ΡΠΊΡΠΈΠΈ, ΠΏΠΎΡΠ²Π»ΡΡΡΠΈΡ ΡΡ Π½Π° ΠΊΠ°ΠΆΠ΄ΠΎΠΌ ΡΡΠ°ΡΡΠΊΠ΅.
ΡΠΎΡΠΌΡΠ»Π° ΡΠ΅ΡΠ΅Π· ΡΠΈΠ»Ρ ΡΠΎΠΊΠ°, ΠΈΠ½Π΄ΡΠΊΡΠΈΠ²Π½ΠΎΡΡΡ ΠΈΠ»ΠΈ ΠΏΠ»ΠΎΡΠ°Π΄Ρ, Π΅Π΄ΠΈΠ½ΠΈΡΠ° ΠΈΠ·ΠΌΠ΅ΡΠ΅Π½ΠΈΡ Π² ΡΠΈΠ·ΠΈΠΊΠ΅
- Π§ΡΠΎΒ ΡΠ°ΠΊΠΎΠ΅ ΠΠΠ‘Β ΠΈΠ½Π΄ΡΠΊΡΠΈΠΈ β ΠΊΠΎΠ³Π΄Π° Π²ΠΎΠ·Π½ΠΈΠΊΠ°Π΅Ρ, ΠΏΡΠΈΒ ΠΊΠ°ΠΊΠΈΡ ΡΡΠ»ΠΎΠ²ΠΈΡΡ
- ΠΠΠ‘Β Π²Β Π±ΡΡΡ, ΠΊΠ°ΠΊΒ ΠΎΠ±ΠΎΠ·Π½Π°ΡΠ°Π΅ΡΡΡ, Π΅Π΄ΠΈΠ½ΠΈΡΡ ΠΈΠ·ΠΌΠ΅ΡΠ΅Π½ΠΈΡ
- ΠΠ°ΠΊΠΎΠ½Ρ Π€Π°ΡΠ°Π΄Π΅Ρ ΠΈΒ ΠΠ΅Π½ΡΠ°
-
ΠΠ°ΠΊΒ ΡΠ°ΡΡΡΠΈΡΠ°ΡΡ ΡΠ»Π΅ΠΊΡΡΠΎΠ΄Π²ΠΈΠΆΡΡΡΡ ΡΠΈΠ»Ρ ΠΈΠ½Π΄ΡΠΊΡΠΈΠΈ, ΡΠΎΡΠΌΡΠ»Ρ
- Π§Π΅ΡΠ΅Π· ΠΌΠ°Π³Π½ΠΈΡΠ½ΡΠΉ ΠΏΠΎΡΠΎΠΊ
- Π§Π΅ΡΠ΅Π· ΡΠΈΠ»Ρ ΡΠΎΠΊΠ°
- Π§Π΅ΡΠ΅Π· ΡΠΎΠΏΡΠΎΡΠΈΠ²Π»Π΅Π½ΠΈΠ΅
- Π§Π΅ΡΠ΅Π· ΡΠ³Π»ΠΎΠ²ΡΡ ΡΠΊΠΎΡΠΎΡΡΡ
- Π§Π΅ΡΠ΅Π· ΠΏΠ»ΠΎΡΠ°Π΄Ρ
Π‘ΠΎΠ΄Π΅ΡΠΆΠ°Π½ΠΈΠ΅
- Π§ΡΠΎΒ ΡΠ°ΠΊΠΎΠ΅ ΠΠΠ‘Β ΠΈΠ½Π΄ΡΠΊΡΠΈΠΈ β ΠΊΠΎΠ³Π΄Π° Π²ΠΎΠ·Π½ΠΈΠΊΠ°Π΅Ρ, ΠΏΡΠΈΒ ΠΊΠ°ΠΊΠΈΡ ΡΡΠ»ΠΎΠ²ΠΈΡΡ
- ΠΠΠ‘Β Π²Β Π±ΡΡΡ, ΠΊΠ°ΠΊΒ ΠΎΠ±ΠΎΠ·Π½Π°ΡΠ°Π΅ΡΡΡ, Π΅Π΄ΠΈΠ½ΠΈΡΡ ΠΈΠ·ΠΌΠ΅ΡΠ΅Π½ΠΈΡ
- ΠΠ°ΠΊΠΎΠ½Ρ Π€Π°ΡΠ°Π΄Π΅Ρ ΠΈΒ ΠΠ΅Π½ΡΠ°
-
ΠΠ°ΠΊΒ ΡΠ°ΡΡΡΠΈΡΠ°ΡΡ ΡΠ»Π΅ΠΊΡΡΠΎΠ΄Π²ΠΈΠΆΡΡΡΡ ΡΠΈΠ»Ρ ΠΈΠ½Π΄ΡΠΊΡΠΈΠΈ, ΡΠΎΡΠΌΡΠ»Ρ
- Π§Π΅ΡΠ΅Π· ΠΌΠ°Π³Π½ΠΈΡΠ½ΡΠΉ ΠΏΠΎΡΠΎΠΊ
- Π§Π΅ΡΠ΅Π· ΡΠΈΠ»Ρ ΡΠΎΠΊΠ°
- Π§Π΅ΡΠ΅Π· ΡΠΎΠΏΡΠΎΡΠΈΠ²Π»Π΅Π½ΠΈΠ΅
- Π§Π΅ΡΠ΅Π· ΡΠ³Π»ΠΎΠ²ΡΡ ΡΠΊΠΎΡΠΎΡΡΡ
- Π§Π΅ΡΠ΅Π· ΠΏΠ»ΠΎΡΠ°Π΄Ρ
Π§ΡΠΎΒ ΡΠ°ΠΊΠΎΠ΅ ΠΠΠ‘Β ΠΈΠ½Π΄ΡΠΊΡΠΈΠΈ β ΠΊΠΎΠ³Π΄Π° Π²ΠΎΠ·Π½ΠΈΠΊΠ°Π΅Ρ, ΠΏΡΠΈΒ ΠΊΠ°ΠΊΠΈΡ ΡΡΠ»ΠΎΠ²ΠΈΡΡ
ΠΠΏΡΠ΅Π΄Π΅Π»Π΅Π½ΠΈΠ΅
ΠΠ»Π΅ΠΊΡΡΠΎΠ΄Π²ΠΈΠΆΡΡΠ°Ρ ΡΠΈΠ»Π°, ΠΠΠ‘Β β ΡΠΈΠ·ΠΈΡΠ΅ΡΠΊΠ°Ρ Π²Π΅Π»ΠΈΡΠΈΠ½Π°, ΠΎΠΏΠΈΡΡΠ²Π°ΡΡΠ°Ρ ΡΠ°Π±ΠΎΡΡ Π»ΡΠ±ΡΡ ΡΠΈΠ», ΠΊΠΎΡΠΎΡΡΠ΅ Π΄Π΅ΠΉΡΡΠ²ΡΡΡ Π²Β ΠΊΠ²Π°Π·ΠΈΡΡΠ°ΡΠΈΠΎΠ½Π°ΡΠ½ΡΡ ΡΠ΅ΠΏΡΡ ΠΏΠΎΡΡΠΎΡΠ½Π½ΠΎΠ³ΠΎ ΠΈΠ»ΠΈΒ ΠΏΠ΅ΡΠ΅ΠΌΠ΅Π½Π½ΠΎΠ³ΠΎ ΡΠΎΠΊΠ°, Π·Π°Β ΠΈΡΠΊΠ»ΡΡΠ΅Π½ΠΈΠ΅ΠΌ Π΄ΠΈΡΡΠΈΠΏΠ°ΡΠΈΠ²Π½ΡΡ ΠΈΒ ΡΠ»Π΅ΠΊΡΡΠΎΡΡΠ°ΡΠΈΡΠ΅ΡΠΊΠΈΡ ΡΠΈΠ».
ΠΡΠΈΒ Π·Π°ΠΌΠΊΠ½ΡΡΠΎΠΉ ΡΠ΅ΠΏΠΈ ΠΌΠΎΠΆΠ½ΠΎ Π½Π°ΠΉΡΠΈ ΠΠΠ‘, Π²ΠΎΡΠΏΠΎΠ»ΡΠ·ΠΎΠ²Π°Π²ΡΠΈΡΡ Π·Π°ΠΊΠΎΠ½ΠΎΠΌ ΠΠΌΠ°:
\(\varepsilon\;=\;I\;\times\;(R\;+\;r).\)
R Π·Π΄Π΅ΡΡ β ΡΠΎΠΏΡΠΎΡΠΈΠ²Π»Π΅Π½ΠΈΠ΅ ΡΠ΅ΠΏΠΈ, r β Π²Π½ΡΡΡΠ΅Π½Π½Π΅Π΅ ΡΠΎΠΏΡΠΎΡΠΈΠ²Π»Π΅Π½ΠΈΠ΅ ΠΈΡΡΠΎΡΠ½ΠΈΠΊΠ°.
Π‘ΠΎΠ·Π΄Π°Π½ΠΈΠ΅ ΠΠ»Π΅ΡΡΠ°Π½Π΄ΡΠΎ ΠΠΎΠ»ΡΡΠΎΠΉ Π½Π°Π΄Π΅ΠΆΠ½ΠΎΠ³ΠΎ ΠΈΡΡΠΎΡΠ½ΠΈΠΊΠ° ΡΠ»Π΅ΠΊΡΡΠΈΡΠ΅ΡΡΠ²Π°, Π³Π°Π»ΡΠ²Π°Π½ΠΈΡΠ΅ΡΠΊΠΎΠ³ΠΎ ΡΠ»Π΅ΠΌΠ΅Π½ΡΠ°, ΠΈΒ ΠΎΡΠΊΡΡΡΠΈΠ΅ Π₯Π°Π½ΡΠΎΠΌ ΠΡΠΈΡΡΠΈΠ°Π½ΠΎΠΌ ΠΡΡΡΠ΅Π΄ΠΎΠΌ ΠΌΠ°Π³Π½ΠΈΡΠ½ΠΎΠ³ΠΎ Π΄Π΅ΠΉΡΡΠ²ΠΈΡ ΡΠ»Π΅ΠΊΡΡΠΈΡΠ΅ΡΠΊΠΎΠ³ΠΎ ΡΠΎΠΊΠ° ΠΏΠΎΡΠ»ΡΠΆΠΈΠ»ΠΈ ΡΠΎΠ»ΡΠΊΠΎΠΌ ΠΊΒ ΠΈΠ½ΡΠ΅Π½ΡΠΈΠ²Π½ΠΎΠΌΡ ΡΠ°Π·Π²ΠΈΡΠΈΡ ΡΠ΅Ρ
Π½ΠΈΠΊΠΈ ΡΠ»Π΅ΠΊΡΡΠΈΡΠ΅ΡΠΊΠΈΡ
ΠΈΠ·ΠΌΠ΅ΡΠ΅Π½ΠΈΠΉ Π² XIXΒ Π²Π΅ΠΊΠ΅.
ΠΡΡΠΎΡΠΎΠΆΠ½ΠΎ! ΠΡΠ»ΠΈ ΠΏΡΠ΅ΠΏΠΎΠ΄Π°Π²Π°ΡΠ΅Π»Ρ ΠΎΠ±Π½Π°ΡΡΠΆΠΈΡ ΠΏΠ»Π°Π³ΠΈΠ°Ρ Π² ΡΠ°Π±ΠΎΡΠ΅, Π½Π΅ ΠΈΠ·Π±Π΅ΠΆΠ°ΡΡ ΠΊΡΡΠΏΠ½ΡΡ ΠΏΡΠΎΠ±Π»Π΅ΠΌ (Π²ΠΏΠ»ΠΎΡΡ Π΄ΠΎ ΠΎΡΡΠΈΡΠ»Π΅Π½ΠΈΡ). ΠΡΠ»ΠΈ Π½Π΅Ρ Π²ΠΎΠ·ΠΌΠΎΠΆΠ½ΠΎΡΡΠΈ Π½Π°ΠΏΠΈΡΠ°ΡΡ ΡΠ°ΠΌΠΎΠΌΡ, Π·Π°ΠΊΠ°ΠΆΠΈΡΠ΅ ΡΡΡ.
ΠΡΠ΄Π°ΡΡΠ°ΡΡΡ ΡΠΎΠ»Ρ Π·Π΄Π΅ΡΡ ΠΏΡΠΈΠ½Π°Π΄Π»Π΅ΠΆΠΈΡ Π½Π΅ΠΌΠ΅ΡΠΊΠΎΠΌΡ ΡΠΈΠ·ΠΈΠΊΡ ΠΠ΅ΠΎΡΠ³Ρ Π‘ΠΈΠΌΠΎΠ½Ρ ΠΠΌΡ. ΠΠ»ΡΒ ΠΎΠΏΡΠ΅Π΄Π΅Π»Π΅Π½ΠΈΡ ΡΠΈΠ»Ρ ΡΠΎΠΊΠ° ΠΎΠ½Β ΠΈΡΠΏΠΎΠ»ΡΠ·ΠΎΠ²Π°Π» ΠΏΡΠΈΠ½ΡΠΈΠΏ ΠΊΡΡΡΠΈΠ»ΡΠ½ΡΡ Π²Π΅ΡΠΎΠ² ΠΡΠ»ΠΎΠ½Π°. ΠΠ°Β Π΄Π»ΠΈΠ½Π½ΠΎΠΉ ΡΠΎΠ½ΠΊΠΎΠΉ Π½ΠΈΡΠΈ ΠΏΠΎΠ΄Π²Π΅ΡΠ΅Π½ΠΎ Π³ΠΎΡΠΈΠ·ΠΎΠ½ΡΠ°Π»ΡΠ½ΠΎΠ΅ ΠΊΠΎΡΠΎΠΌΡΡΠ»ΠΎ ΡΒ Π·Π°ΡΡΠΆΠ΅Π½Π½ΡΠΌ ΡΠ°ΡΠΈΠΊΠΎΠΌ Π½Π°Β ΠΊΠΎΠ½ΡΠ΅. ΠΡΠΎΡΠΎΠΉ Π·Π°ΡΡΠ΄ Π·Π°ΠΊΡΠ΅ΠΏΠ»Π΅Π½ Π½Π° ΡΠΏΠΈΡΠ΅, ΠΏΡΠΎΠΏΡΡΠ΅Π½Π½ΠΎΠΉ ΡΠΊΠ²ΠΎΠ·Ρ ΠΊΡΡΡΠΊΡ Π²Π΅ΡΠΎΠ².
ΠΡΠΈΒ ΠΈΡ Β Π²Π·Π°ΠΈΠΌΠΎΠ΄Π΅ΠΉΡΡΠ²ΠΈΠΈ ΠΊΠΎΡΠΎΠΌΡΡΠ»ΠΎ ΠΏΠΎΠ²ΠΎΡΠ°ΡΠΈΠ²Π°Π΅ΡΡΡ. ΠΡΠ°ΡΠ΅Π½ΠΈΠ΅ Π³ΠΎΠ»ΠΎΠ²ΠΊΠΈ Π²Β Π²Π΅ΡΡ Π½Π΅ΠΉ ΡΠ°ΡΡΠΈ Π²Π΅ΡΠΎΠ² Π·Π°ΠΊΡΡΡΠΈΠ²Π°Π»ΠΎ Π½ΠΈΡΡ, Π²ΠΎΠ·Π²ΡΠ°ΡΠ°Ρ ΠΊΠΎΡΠΎΠΌΡΡΠ»ΠΎ Π²Β ΠΈΡΡ ΠΎΠ΄Π½ΠΎΠ΅ ΡΠΎΡΡΠΎΡΠ½ΠΈΠ΅. ΠΠΎΒ ΡΠ³Π»Ρ Π·Π°ΠΊΡΡΡΠΈΠ²Π°Π½ΠΈΡ ΠΌΠΎΠΆΠ½ΠΎ ΡΠ°ΡΡΡΠΈΡΠ°ΡΡ ΡΠΈΠ»Ρ Π²Π·Π°ΠΈΠΌΠΎΠ΄Π΅ΠΉΡΡΠ²ΠΈΡ Π·Π°ΡΡΠ΄ΠΎΠ² Π²Β Π·Π°Π²ΠΈΡΠΈΠΌΠΎΡΡΠΈ ΠΎΡΒ ΡΠ°ΡΡΡΠΎΡΠ½ΠΈΡ ΠΌΠ΅ΠΆΠ΄Ρ Π½ΠΈΠΌΠΈ.
ΠΠΌΒ ΠΏΠΎΒ Π²Π΅Π»ΠΈΡΠΈΠ½Π΅ ΡΠ³Π»Π° Π·Π°ΠΊΡΡΡΠΊΠΈ ΡΡΠ΄ΠΈΠ» ΠΎΒ ΡΠΈΠ»Π΅ ΡΠΎΠΊΠ° I Π²Β ΠΏΡΠΎΠ²ΠΎΠ΄Π½ΠΈΠΊΠ΅, Ρ. Π΅. ΠΊΠΎΠ»ΠΈΡΠ΅ΡΡΠ²Π΅ ΡΠ»Π΅ΠΊΡΡΠΈΡΠ΅ΡΡΠ²Π°, ΠΏΠ΅ΡΠ΅Π½Π΅ΡΠ΅Π½Π½ΠΎΠΌ ΡΠ΅ΡΠ΅Π· ΠΏΠΎΠΏΠ΅ΡΠ΅ΡΠ½ΠΎΠ΅ ΡΠ΅ΡΠ΅Π½ΠΈΠ΅ ΠΏΡΠΎΠ²ΠΎΠ΄Π½ΠΈΠΊΠ° Π·Π°Β Π΅Π΄ΠΈΠ½ΠΈΡΡ Π²ΡΠ΅ΠΌΠ΅Π½ΠΈ.
ΠΒ ΠΊΠ°ΡΠ΅ΡΡΠ²Π΅ ΠΎΡΠ½ΠΎΠ²Π½ΠΎΠΉ Ρ Π°ΡΠ°ΠΊΡΠ΅ΡΠΈΡΡΠΈΠΊΠΈ ΠΈΡΡΠΎΡΠ½ΠΈΠΊΠ° ΡΠΎΠΊΠ° ΠΠΌΒ Π±ΡΠ°Π» Π²Π΅Π»ΠΈΡΠΈΠ½Ρ Π½Π°ΠΏΡΡΠΆΠ΅Π½ΠΈΡ \varepsilon Π½Π°Β ΡΠ»Π΅ΠΊΡΡΠΎΠ΄Π°Ρ Π³Π°Π»ΡΠ²Π°Π½ΠΈΡΠ΅ΡΠΊΠΎΠ³ΠΎ ΡΠ»Π΅ΠΌΠ΅Π½ΡΠ° ΠΏΡΠΈΒ ΡΠ°Π·ΠΎΠΌΠΊΠ½ΡΡΠΎΠΉ ΡΠ΅ΠΏΠΈ. ΠΡΡΒ Π²Π΅Π»ΠΈΡΠΈΠ½Ρ \varepsilon ΠΎΠ½Β Π½Π°Π·Π²Π°Π» ΡΠ»Π΅ΠΊΡΡΠΎΠ΄Π²ΠΈΠΆΡΡΠ΅ΠΉ ΡΠΈΠ»ΠΎΠΉ, ΡΠΎΠΊΡΠ°ΡΠ΅Π½Π½ΠΎ ΠΠΠ‘.
ΠΠ²ΠΈΠΆΡΡΠΈΠ΅ΡΡ Π·Π°ΡΡΠ΄Ρ ΡΠΎΠ·Π΄Π°ΡΡ Π²ΠΎΠΊΡΡΠ³ ΡΠ΅Π±Ρ ΠΌΠ°Π³Π½ΠΈΡΠ½ΠΎΠ΅ ΠΏΠΎΠ»Π΅. ΠΠ΄Π½Π°ΠΊΠΎ Π΄Π΅ΠΉΡΡΠ²ΡΡΡΠ°Ρ Π²Β Π½Π΅ΠΌΒ Π½Π°Β ΠΌΠ°Π³Π½ΠΈΡ ΠΈΠ»ΠΈΒ Π΄ΡΡΠ³ΠΎΠΉ ΡΠΎΠΊΒ ΡΠΈΠ»Π° ΠΎΡΠ»ΠΈΡΠ°Π΅ΡΡΡ ΠΎΡΒ ΡΠ»Π΅ΠΊΡΡΠΈΡΠ΅ΡΠΊΠΎΠΉ ΡΠ²ΠΎΠΈΠΌ Π½Π°ΠΏΡΠ°Π²Π»Π΅Π½ΠΈΠ΅ΠΌ β ΠΌΠ°Π³Π½ΠΈΡΠ½Π°Ρ ΡΡΡΠ΅Π»ΠΊΠ° ΡΡΠ°ΡΠ°Π΅ΡΡΡ ΡΠ°Π·Π²Π΅ΡΠ½ΡΡΡΡΡ ΠΏΠ΅ΡΠΏΠ΅Π½Π΄ΠΈΠΊΡΠ»ΡΡΠ½ΠΎ ΠΏΡΠΎΠ²ΠΎΠ΄Ρ.
ΠΠ·ΡΡΠ΅Π½ΠΈΠ΅ Π΄Π΅ΠΉΡΡΠ²ΡΡΡΠ΅ΠΉ Π½Π°Β Π΄ΡΡΠ³ΠΎΠΉ ΡΠΎΠΊΒ ΡΠΈΠ»Ρ ΠΏΠ΅ΡΠ΅ΡΠΎΡΠ»ΠΎ Π²Β ΠΎΡΠ΄Π΅Π»ΡΠ½ΠΎΠ΅ ΠΈΡΡΠ»Π΅Π΄ΠΎΠ²Π°Π½ΠΈΠ΅ ΡΒ Π½Π΅ΠΎΠΆΠΈΠ΄Π°Π½Π½ΡΠΌ ΡΠ΅Π·ΡΠ»ΡΡΠ°ΡΠΎΠΌ: ΡΠΈΠ»Π° ΠΎΠΊΠ°Π·Π°Π»Π°ΡΡ Π½Π°ΠΏΡΠ°Π²Π»Π΅Π½Π½ΠΎΠΉ Π²ΡΠ΅Π³Π΄Π° ΠΏΠ΅ΡΠΏΠ΅Π½Π΄ΠΈΠΊΡΠ»ΡΡΠ½ΠΎ Π²Π½Π΅ΡΠ΅Π½Π½ΠΎΠΌΡ Π²Β ΠΌΠ°Π³Π½ΠΈΡΠ½ΠΎΠ΅ ΠΏΠΎΠ»Π΅ ΠΏΡΠΎΠ²ΠΎΠ΄Π½ΠΈΠΊΡ, ΠΊΠΎΡΠΎΡΡΠΉ Π΄Π»ΡΒ ΠΏΡΠΎΡΡΠΎΡΡ ΠΈΡΡΠ»Π΅Π΄ΠΎΠ²Π°Π½ΠΈΡ Π±ΡΠ»Β ΠΏΡΡΠΌΠΎΠ»ΠΈΠ½Π΅ΠΉΠ½ΡΠΌ.
ΠΠ°ΡΠ΅ΠΌΠ°ΡΠΈΡΠ΅ΡΠΊΠΎΠ΅ Π²ΡΡΠ°ΠΆΠ΅Π½ΠΈΠ΅ Π΄Π»ΡΒ ΡΡΠΎΠΉ ΡΠΈΠ»Ρ, Π½Π°Π·Π²Π°Π½Π½ΠΎΠΉ ΡΠΈΠ»ΠΎΠΉ ΠΠΌΠΏΠ΅ΡΠ°, ΠΏΡΠΎΡΠ΅ Π²ΡΠ΅Π³ΠΎ Π·Π°ΠΏΠΈΡΠ°ΡΡ Π²Β Π²ΠΈΠ΄Π΅ Π²Π΅ΠΊΡΠΎΡΠ½ΠΎΠ³ΠΎ ΠΏΡΠΎΠΈΠ·Π²Π΅Π΄Π΅Π½ΠΈΡ:
\(d\overrightarrow F\;=\;Id\overrightarrow l\;\times\;\overrightarrow B\).
I Π·Π΄Π΅ΡΡ β ΡΠΈΠ»Π° ΡΠΎΠΊΠ°, ΠΏΡΠΎΡΠ΅ΠΊΠ°ΡΡΠ΅Π³ΠΎ ΡΠ΅ΡΠ΅Π· ΠΏΡΠΎΠ²ΠΎΠ΄Π½ΠΈΠΊ; l β Π²Π΅ΠΊΡΠΎΡ Π΄Π»ΠΈΠ½Ρ ΠΏΡΠΎΠ²ΠΎΠ΄Π½ΠΈΠΊΠ°, Π½Π°ΠΏΡΠ°Π²Π»Π΅Π½Π½ΡΠΉ Π²Β ΡΡΒ ΠΆΠ΅Β ΡΡΠΎΡΠΎΠ½Ρ, ΠΊΡΠ΄Π° ΡΠ΅ΡΠ΅Ρ ΡΠΎΠΊ; ΠΒ β Ρ Π°ΡΠ°ΠΊΡΠ΅ΡΠΈΡΡΠΈΠΊΠ° ΠΏΠΎΠ»Ρ. ΠΠ΅Π»ΠΈΡΠΈΠ½Π° ΠΒ Π½Π°Π·ΡΠ²Π°Π΅ΡΡΡ ΠΌΠ°Π³Π½ΠΈΡΠ½ΠΎΠΉ ΠΈΠ½Π΄ΡΠΊΡΠΈΠ΅ΠΉ ΠΈΒ ΡΠ²Π»ΡΠ΅ΡΡΡ Π°Π½Π°Π»ΠΎΠ³ΠΎΠΌ ΡΠ»Π΅ΠΊΡΡΠΈΡΠ΅ΡΠΊΠΎΠΉ Π½Π°ΠΏΡΡΠΆΠ΅Π½Π½ΠΎΡΡΠΈ.
ΠΠ°ΠΊΡΠ²Π΅Π»Π» ΠΏΠΎΡΡΠ°Π²ΠΈΠ» ΡΠ΅Π»ΡΡ ΡΠΎΠ·Π΄Π°ΡΡ ΡΠ΅ΠΎΡΠΈΡ ΡΡΠΈΡΠ°, ΡΠ²ΡΠ·Π°Π² Π΅Π³ΠΎΒ ΠΌΠ΅Ρ Π°Π½ΠΈΡΠ΅ΡΠΊΠΈΠ΅ Ρ Π°ΡΠ°ΠΊΡΠ΅ΡΠΈΡΡΠΈΠΊΠΈ ΡΒ ΡΠ»Π΅ΠΊΡΡΠΈΡΠ΅ΡΠΊΠΈΠΌΠΈ ΠΈΒ ΠΌΠ°Π³Π½ΠΈΡΠ½ΡΠΌΠΈ ΡΠΈΠ»Π°ΠΌΠΈ. Π’ΡΠ°ΡΠ΅Π»ΡΠ½ΠΎ ΠΈΠ·ΡΡΠΈΠ² ΡΡΡΠ΄Ρ Π€Π°ΡΠ°Π΄Π΅Ρ, ΠΎΠ½Β ΠΏΡΠΈΡΠ΅Π» ΠΊΒ Π²ΡΠ²ΠΎΠ΄Ρ, ΡΡΠΎΒ Π½Π°ΠΏΡΡΠΆΠ΅Π½Π½ΠΎΡΡΡ \(\overrightarrow Π\)Β ΡΠ»Π΅ΠΊΡΡΠΈΡΠ΅ΡΠΊΠΎΠ³ΠΎ ΠΏΠΎΠ»Ρ ΠΎΠ±ΡΡΡΠ½ΡΠ΅ΡΡΡ ΡΠΏΡΡΠ³ΠΈΠΌΠΈ Π½Π°ΠΏΡΡΠΆΠ΅Π½ΠΈΡΠΌΠΈ Π²Β ΡΡΠΈΡΠ΅, Π°Β ΠΌΠ°Π³Π½ΠΈΡΠ½Π°Ρ ΠΈΠ½Π΄ΡΠΊΡΠΈΡΒ \(\overrightarrow B\) β Π΅Π³ΠΎΒ Π²ΠΈΡ ΡΠ΅Π²ΡΠΌΠΈ Π΄Π²ΠΈΠΆΠ΅Π½ΠΈΡΠΌΠΈ.
Π Π°ΡΡΠΌΠ°ΡΡΠΈΠ²Π°Ρ Π·Π°ΠΌΠΊΠ½ΡΡΡΠΉ ΠΏΡΠΎΠ²ΠΎΠ΄ΡΡΠΈΠΉ ΠΊΠΎΠ½ΡΡΡ Π‘, Π³Π΄Π΅Β Π΄Π΅ΠΉΡΡΠ²ΡΠ΅Ρ ΠΠΠ‘Β ΠΈΠ½Π΄ΡΠΊΡΠΈΠΈ \(\varepsilon_i\), ΠΠ°ΠΊΡΠ²Π΅Π»Π» Π΄Π»ΡΒ ΠΏΠΎΠ»ΡΡΠ΅Π½ΠΈΡ ΡΠΈΡΠ»Π° ΡΠΈΠ»ΠΎΠ²ΡΡ Π»ΠΈΠ½ΠΈΠΉ ΠΌΠ°Π³Π½ΠΈΡΠ½ΠΎΠ³ΠΎ ΠΏΠΎΡΠΎΠΊΠ° \(\triangle Π€\), ΠΏΠ΅ΡΠ΅ΡΠ΅ΠΊΠ°Π΅ΠΌΡΡ ΠΊΠΎΠ½ΡΡΡΠΎΠΌ Π·Π°Β Π²ΡΠ΅ΠΌΡ \triangle t, Β«Π½Π°ΡΡΠ³ΠΈΠ²Π°Π»Β» Π½Π°Β Π½Π΅Π³ΠΎ Π½Π΅ΠΊΡΡ ΠΏΠΎΠ²Π΅ΡΡ Π½ΠΎΡΡΡ S, ΡΠ°Π·Π±ΠΈΡΡΡ Π½Π°Β ΡΠ»Π΅ΠΌΠ΅Π½ΡΠ°ΡΠ½ΡΠ΅ ΠΏΠ»ΠΎΡΠ°Π΄ΠΊΠΈ \(\triangle S\), ΠΈΒ ΠΎΡΠΎΠΆΠ΄Π΅ΡΡΠ²Π»ΡΠ» Π€Β ΡΒ ΠΌΠ°Π³Π½ΠΈΡΠ½ΡΠΌ ΠΏΠΎΡΠΎΠΊΠΎΠΌ ΡΠΊΠ²ΠΎΠ·Ρ Π²ΡΡΒ ΠΏΠΎΠ²Π΅ΡΡ Π½ΠΎΡΡΡ. ΠΠ°ΡΠ΅ΠΌΠ°ΡΠΈΡΠ΅ΡΠΊΠΈ ΡΡΠΎΒ ΠΌΠΎΠΆΠ½ΠΎ Π²ΡΡΠ°Π·ΠΈΡΡ ΡΠ°ΠΊ:
\(Π€\;=\;\sum_{\triangle S}\;\;B\triangle S. \)
ΠΠ±ΡΠ΅Π΄ΠΈΠ½ΠΈΠ² ΡΡΠΎΒ ΡΠΎΠΎΡΠ½ΠΎΡΠ΅Π½ΠΈΠ΅ ΡΒ ΠΈΠ΄Π΅Π΅ΠΉ Π€Π°ΡΠ°Π΄Π΅Ρ, ΠΠ°ΠΊΡΠ²Π΅Π»Π» ΠΏΡΠΈΡΠ΅Π» ΠΊΒ ΡΠΎΠ±ΡΡΠ²Π΅Π½Π½ΠΎΠΉ ΡΠΎΡΠΌΡΠ»Π΅:
\(\varepsilon_i\;=\;-\;\frac1Ρ\;\times\;\frac{dΠ€}{dt}.\)
ΠΡΠ±ΠΎΡ ΠΊΠΎΡΡΡΠΈΡΠΈΠ΅Π½ΡΠ° ΠΏΡΠΎΠΏΠΎΡΡΠΈΠΎΠ½Π°Π»ΡΠ½ΠΎΡΡΠΈΒ \(\alpha\) Π·Π΄Π΅ΡΡ ΠΎΠ±ΡΡΠ»ΠΎΠ²Π»Π΅Π½ Π½Π΅ΠΎΠ±Ρ ΠΎΠ΄ΠΈΠΌΠΎΡΡΡΡ ΡΠΎΠ³Π»Π°ΡΠΎΠ²Π°Π½ΠΈΡ ΡΠΎΡΠΌΡΠ»Ρ ΡΒ Π·Π°ΠΊΠΎΠ½ΠΎΠΌ ΠΠΈΠΎΒ β Π‘Π°Π²Π°ΡΠ° β ΠΠ°ΠΏΠ»Π°ΡΠ°, Π²Β ΠΊΠΎΡΠΎΡΠΎΠΌ ΠΏΠΎΡΠ²Π»ΡΠ΅ΡΡΡ ΡΠ°Β ΠΆΠ΅Β ΡΠ»Π΅ΠΊΡΡΠΎΠ΄ΠΈΠ½Π°ΠΌΠΈΡΠ΅ΡΠΊΠ°Ρ ΠΏΠΎΡΡΠΎΡΠ½Π½Π°Ρ Ρ.
ΠΠΏΡΠ΅Π΄Π΅Π»Π΅Π½ΠΈΠ΅
ΠΠ»Π΅ΠΊΡΡΠΎΠ΄ΠΈΠ½Π°ΠΌΠΈΡΠ΅ΡΠΊΠ°Ρ ΠΏΠΎΡΡΠΎΡΠ½Π½Π°Ρ ΡΒ β ΡΠ½ΠΈΠ²Π΅ΡΡΠ°Π»ΡΠ½Π°Ρ ΠΏΠΎΡΡΠΎΡΠ½Π½Π°Ρ, ΡΠ°Π²Π½Π°Ρ ΡΠΊΠΎΡΠΎΡΡΠΈ ΡΠ°ΡΠΏΡΠΎΡΡΡΠ°Π½Π΅Π½ΠΈΡ ΡΠ»Π΅ΠΊΡΡΠΎΠΌΠ°Π³Π½ΠΈΡΠ½ΡΡ Π²ΠΎΠ»Π½ Π²Β Π²Π°ΠΊΡΡΠΌΠ΅.
ΠΠΎΒ Π²Β ΠΎΠΏΡΡΠ°Ρ Π€Π°ΡΠ°Π΄Π΅Ρ ΠΠΠ‘Β ΠΈΠ½Π΄ΡΠΊΡΠΈΠΈ ΡΠ΅Π³ΠΈΡΡΡΠΈΡΠΎΠ²Π°Π»Π°ΡΡ ΠΊΠ°ΠΊ Π² Π΄Π²ΠΈΠΆΡΡΠ΅ΠΌΡΡ, ΡΠ°ΠΊΒ ΠΈΒ Π²Β ΠΏΠΎΠΊΠΎΡΡΠ΅ΠΌΡΡ ΠΏΡΠΎΠ²ΠΎΠ΄ΡΡΠ΅ΠΌ ΠΊΠΎΠ½ΡΡΡΠ΅ Π‘, Π΅ΡΠ»ΠΈ ΠΏΠΎΡΠ»Π΅Π΄Π½ΠΈΠΉ Π½Π°Ρ ΠΎΠ΄ΠΈΠ»ΡΡ Π²Β ΠΏΠ΅ΡΠ΅ΠΌΠ΅Π½Π½ΠΎΠΌ ΠΌΠ°Π³Π½ΠΈΡΠ½ΠΎΠΌ ΠΏΠΎΠ»Π΅. ΠΒ Π·Π΄Π΅ΡΡ Π²ΡΡΠ°Π» Π²ΠΎΠΏΡΠΎΡ, ΡΡΠΎ ΠΊΠΎΠ½ΠΊΡΠ΅ΡΠ½ΠΎ ΠΏΠ΅ΡΠ΅ΠΌΠ΅ΡΠ°Π΅Ρ Π·Π°ΡΡΠ΄Ρ Π²Β Π½Π΅ΠΏΠΎΠ΄Π²ΠΈΠΆΠ½ΠΎΠΌ ΠΏΡΠΎΠ²ΠΎΠ΄Π½ΠΈΠΊΠ΅.
Π‘Π°ΠΌΠΎ ΠΏΠΎ ΡΠ΅Π±Π΅ ΠΌΠ°Π³Π½ΠΈΡΠ½ΠΎΠ΅ ΠΏΠΎΠ»Π΅ Π½Π΅ Π²ΠΎΠ·Π΄Π΅ΠΉΡΡΠ²ΡΠ΅Ρ Π½Π° Π·Π°ΡΡΠ΄Ρ, Π½Π°Ρ ΠΎΠ΄ΡΡΠΈΠ΅ΡΡ Π² ΠΏΠΎΠΊΠΎΠ΅, ΠΈΠ· ΡΠ΅Π³ΠΎ ΡΠ»Π΅Π΄ΡΠ΅Ρ: ΡΡΠ»ΠΎΠ²ΠΈΠ΅ Π²ΠΎΠ·Π½ΠΈΠΊΠ½ΠΎΠ²Π΅Π½ΠΈΡ ΠΈΠ½Π΄ΡΠΊΡΠΈΠΎΠ½Π½ΠΎΠ³ΠΎ ΡΠΎΠΊΠ° β Π²ΠΎΠ·Π½ΠΈΠΊΠ°ΡΡΠ΅Π΅ Π²Β ΠΊΠΎΠ½ΡΡΡΠ΅ ΡΠ»Π΅ΠΊΡΡΠΈΡΠ΅ΡΠΊΠΎΠ΅ ΠΏΠΎΠ»Π΅ \overrightarrow Π. Β Π’Π°ΠΊ ΠΊΠ°ΠΊ ΡΠ»Π΅ΠΊΡΡΠΎΡΡΠ°ΡΠΈΡΠ΅ΡΠΊΠΎΠ΅ ΠΏΠΎΠ»Π΅ Π² Π·Π°ΠΌΠΊΠ½ΡΡΠΎΠΌ ΠΊΠΎΠ½ΡΡΡΠ΅ Π½Π΅ ΡΠΎΠ²Π΅ΡΡΠ°Π΅Ρ ΡΠ°Π±ΠΎΡΡ, Π·Π½Π°ΡΠΈΡ, ΠΏΡΠΎΠΈΡΡ ΠΎΠ΄ΠΈΡ ΡΠ°Π±ΠΎΡΠ° Π²ΠΈΡ ΡΠ΅Π²ΠΎΠ³ΠΎ ΠΏΠΎΠ»Ρ, ΠΈΒ ΠΎΠ½Π° ΡΠ°Π²Π½Π° ΠΠΠ‘Β ΠΈΠ½Π΄ΡΠΊΡΠΈΠΈ:
\(\varepsilon_i\;=\;\underset Π‘{\oint\;}\;(\overrightarrow{Π\;}\times\;d\overrightarrow l)\)
ΠΠΏΡΠ΅Π΄Π΅Π»Π΅Π½ΠΈΠ΅
Π‘Π°ΠΌΠΎΠΈΠ½Π΄ΡΠΊΡΠΈΡ β ΡΠ°ΡΡΠ½ΡΠΉ ΡΠ»ΡΡΠ°ΠΉ ΠΌΠ°Π³Π½ΠΈΡΠ½ΠΎΠΉ ΠΈΠ½Π΄ΡΠΊΡΠΈΠΈ, Π²ΠΎΠ·Π½ΠΈΠΊΠ½ΠΎΠ²Π΅Π½ΠΈΠ΅ ΠΠΠ‘Β ΠΈΠ½Π΄ΡΠΊΡΠΈΠΈ Π²Β ΠΏΡΠΎΠ²ΠΎΠ΄ΡΡΠ΅ΠΌ ΠΊΠΎΠ½ΡΡΡΠ΅, ΠΊΠΎΠ³Π΄Π° Π²Β Π½Π΅ΠΌΒ ΠΌΠ΅Π½ΡΠ΅ΡΡΡ ΡΠΎΠΊ.
ΠΡΡΠΎΡΠ½ΠΈΠΊΠΎΠΌ ΡΠ½Π΅ΡΠ³ΠΈΠΈ, Π²ΠΎΠ·Π½ΠΈΠΊΠ°ΡΡΠ΅ΠΉ Π²Β ΡΠ΅ΠΏΠΈ, ΡΠ²Π»ΡΠ΅ΡΡΡ Π²Β ΡΡΠΎΠΌ ΡΠ»ΡΡΠ°Π΅ Π·Π°ΠΏΠ°Ρ ΡΠ½Π΅ΡΠ³ΠΈΠΈ ΠΌΠ°Π³Π½ΠΈΡΠ½ΠΎΠ³ΠΎ ΠΏΠΎΠ»Ρ. ΠΠΎΠ»Π½ΠΎΠ΅ ΠΊΠΎΠ»ΠΈΡΠ΅ΡΡΠ²ΠΎ Π²ΡΠ΄Π΅Π»ΠΈΠ²ΡΠ΅ΠΉΡΡ Π΄ΠΆΠΎΡΠ»Π΅Π²ΠΎΠΉ ΡΠ΅ΠΏΠ»ΠΎΡΡ ΠΌΠΎΠΆΠ½ΠΎ Π²ΡΡΠΈΡΠ»ΠΈΡΡ, ΠΈΠ·ΠΎΠ±ΡΠ°Π·ΠΈΠ² Π½Π°Β Π³ΡΠ°ΡΠΈΠΊΠ΅ Π·Π°Π²ΠΈΡΠΈΠΌΠΎΡΡΡ ΠΌΠ°Π³Π½ΠΈΡΠ½ΠΎΠ³ΠΎ ΠΏΠΎΡΠΎΠΊΠ° Π€(I) ΠΎΡΒ ΡΠΈΠ»Ρ ΡΠΎΠΊΠ° I:
ΠΡΡΠΎΡΠ½ΠΈΠΊ: physics.ruΠΠΠ‘Β Π²Β Π±ΡΡΡ, ΠΊΠ°ΠΊΒ ΠΎΠ±ΠΎΠ·Π½Π°ΡΠ°Π΅ΡΡΡ, Π΅Π΄ΠΈΠ½ΠΈΡΡ ΠΈΠ·ΠΌΠ΅ΡΠ΅Π½ΠΈΡ
ΠΒ Π±ΡΡΡ ΡΠ²Π»Π΅Π½ΠΈΠ΅ ΡΠ»Π΅ΠΊΡΡΠΎΠΌΠ°Π³Π½ΠΈΡΠ½ΠΎΠΉ ΠΈΠ½Π΄ΡΠΊΡΠΈΠΈ ΠΈΡΠΏΠΎΠ»ΡΠ·ΡΡΡ Π΄Π»ΡΒ ΠΈΠ·ΠΌΠ΅Π½Π΅Π½ΠΈΡ Π²Π΅Π»ΠΈΡΠΈΠ½Ρ Π½Π°ΠΏΡΡΠΆΠ΅Π½ΠΈΡ ΡΠΎΠΊΠ° Π²Β ΡΡΠ°Π½ΡΡΠΎΡΠΌΠ°ΡΠΎΡΠ°Ρ ΠΈΒ Π΄ΡΠΎΡΡΠ΅Π»ΡΡ . ΠΠ°Β ΠΏΡΠΈΠ½ΡΠΈΠΏΠ΅ ΠΌΠ°Π³Π½ΠΈΡΠ½ΠΎΠΉ ΠΈΠ½Π΄ΡΠΊΡΠΈΠΈ ΡΠ°Π±ΠΎΡΠ°ΡΡ ΡΠ»Π΅ΠΊΡΡΠΈΡΠ΅ΡΠΊΠΈΠ΅ ΡΡΠ΅ΡΡΠΈΠΊΠΈ, ΡΠ΅Π»Π΅ ΠΌΠΎΡΠ½ΠΎΡΡΠΈ, ΡΡΠΏΠΎΠΊΠΎΠΈΡΠ΅Π»ΡΠ½ΡΠ΅ ΡΠΈΡΡΠ΅ΠΌΡ ΡΡΡΠ΅Π»ΠΎΡΠ½ΡΡ ΠΈΠ·ΠΌΠ΅ΡΠΈΡΠ΅Π»ΡΠ½ΡΡ ΠΏΡΠΈΠ±ΠΎΡΠΎΠ².
Π‘ΡΡΠ΅ΡΡΠ²ΡΡΡ ΡΠ°ΠΊΠΆΠ΅ ΠΌΠ°Π³Π½ΠΈΡΠ½ΡΠ΅ Π³Π°Π·ΠΎΠ²ΡΠ΅ Π³Π΅Π½Π΅ΡΠ°ΡΠΎΡΡ, Π² ΠΊΠΎΡΠΎΡΡΡ Π±Π»Π°Π³ΠΎΠ΄Π°ΡΡ ΠΌΠ°Π³Π½ΠΈΡΠ½ΠΎΠΌΡ ΠΏΠΎΠ»Ρ Π²ΠΎΠ·Π½ΠΈΠΊΠ°Π΅Ρ ΡΠ»Π΅ΠΊΡΡΠΎΠ΄Π²ΠΈΠΆΡΡΠ°Ρ ΡΠΈΠ»Π°, ΡΠΎΠ·Π΄Π°ΡΡΠ°Ρ ΡΠΎΠΊ.
ΠΠ»Π΅ΠΊΡΡΠΎΠ΄Π²ΠΈΠΆΡΡΠ°Ρ ΡΠΈΠ»Π° ΠΈΠ½Π΄ΡΠΊΡΠΈΠΈ Π²Β ΡΠΈΡΡΠ΅ΠΌΠ΅ Π‘ΠΒ ΠΈΠ·ΠΌΠ΅ΡΡΠ΅ΡΡΡ Π²Β Π²ΠΎΠ»ΡΡΠ°Ρ . ΠΡΠΎΡΡΠΎ ΡΠ»Π΅ΠΊΡΡΠΎΠ΄Π²ΠΈΠΆΡΡΠ°Ρ ΡΠΈΠ»Π° ΠΎΠ±ΠΎΠ·Π½Π°ΡΠ°Π΅ΡΡΡ Π³ΡΠ΅ΡΠ΅ΡΠΊΠΎΠΉ Π±ΡΠΊΠ²ΠΎΠΉ \(\varepsilon \), ΡΠ»Π΅ΠΊΡΡΠΎΠ΄Π²ΠΈΠΆΡΡΠ°Ρ ΡΠΈΠ»Π° ΠΈΠ½Π΄ΡΠΊΡΠΈΠΈ β\( \varepsilon_i.\)
ΠΠ°ΠΊΠΎΠ½Ρ Π€Π°ΡΠ°Π΄Π΅Ρ ΠΈΒ ΠΠ΅Π½ΡΠ°
Π€Π°ΡΠ°Π΄Π΅ΠΉ ΠΎΠΏΡΡΠ½ΡΠΌ ΠΏΡΡΠ΅ΠΌ Π²ΡΡΡΠ½ΠΈΠ», ΡΡΠΎΒ ΠΏΡΠΈ ΠΏΠ΅ΡΠ΅ΡΠ΅ΡΠ΅Π½ΠΈΠΈ ΠΏΡΠΎΠ²ΠΎΠ΄Π½ΠΈΠΊΠΎΠΌ ΠΌΠ°Π³Π½ΠΈΡΠ½ΡΡ ΡΠΈΠ»ΠΎΠ²ΡΡ Π»ΠΈΠ½ΠΈΠΉ ΠΏΠΎΒ Π½Π΅ΠΌΡ ΠΏΡΠΎΡ ΠΎΠ΄ΠΈΡ Π·Π°ΡΡΠ΄ \(\triangle Q\).Β ΠΠ½Β ΡΠ²ΡΠ·Π°Π½ Ρ ΡΠΈΡΠ»ΠΎΠΌ ΠΏΠ΅ΡΠ΅ΡΠ΅ΡΠ΅Π½Π½ΡΡ ΡΠΈΠ»ΠΎΠ²ΡΡ Π»ΠΈΠ½ΠΈΠΉΒ \( \triangle Π€\) ΠΈ ΡΠ»Π΅ΠΊΡΡΠΈΡΠ΅ΡΠΊΠΈΠΌ ΡΠΎΠΏΡΠΎΡΠΈΠ²Π»Π΅Π½ΠΈΠ΅ΠΌ ΠΊΠΎΠ½ΡΡΡΠ° R, ΡΡΠΎ Π²ΡΡΠ°ΠΆΠ°Π΅ΡΡΡ Π·Π°ΠΊΠΎΠ½ΠΎΠΌ Π€Π°ΡΠ°Π΄Π΅Ρ:
\(\triangle Q\;=\;\alpha\frac{\triangle Π€}R. \)
Π‘ΠΎΠΏΡΠΈΠΊΠΎΡΠ½ΠΎΠ²Π΅Π½ΠΈΠ΅ ΠΏΠΎΠ»Ρ ΠΈ ΠΏΡΠΎΠ²ΠΎΠ΄Π½ΠΈΠΊΠ° Π²ΡΠ·Π²Π°Π½ΠΎ Π»ΠΈΠ±ΠΎ Π΄Π²ΠΈΠΆΠ΅Π½ΠΈΠ΅ΠΌ ΠΏΡΠΎΠ²ΠΎΠ΄Π½ΠΈΠΊΠ°, Π»ΠΈΠ±ΠΎ ΠΈΠ·ΠΌΠ΅Π½Π΅Π½ΠΈΡΠΌΠΈ ΡΠ°ΠΌΠΎΠ³ΠΎ ΠΌΠ°Π³Π½ΠΈΡΠ½ΠΎΠ³ΠΎ ΠΏΠΎΠ»Ρ.Β
Π‘Π°ΠΌΡ ΡΠ»Π΅ΠΊΡΡΠΎΠ΄Π²ΠΈΠΆΡΡΡΡ ΡΠΈΠ»Ρ ΠΈΠ½Π΄ΡΠΊΡΠΈΠΈ, ΡΠ²ΡΠ·Π°Π½Π½ΡΡ ΡΒ ΡΠΎΠΏΡΠΎΡΠΈΠ²Π»Π΅Π½ΠΈΠ΅ΠΌ ΠΊΠΎΠ½ΡΡΡΠ° ΠΈΒ ΡΠΈΠ»ΠΎΠΉ ΡΠΎΠΊΠ° ΡΠΎΠ³Π»Π°ΡΠ½ΠΎ Π·Π°ΠΊΠΎΠ½Ρ ΠΠΌΠ°, ΠΌΠΎΠΆΠ½ΠΎ Π½Π°ΠΉΡΠΈ ΠΏΠΎΒ ΡΠΎΡΠΌΡΠ»Π΅
\(\varepsilon_i\;=\;\alpha\frac{\triangle Π€}{\triangle t}. \)
\(\triangle t\)Β Π·Π΄Π΅ΡΡ β Π²ΡΠ΅ΠΌΡ, Π·Π°Β ΠΊΠΎΡΠΎΡΠΎΠ΅ ΠΏΡΠΎΡ
ΠΎΠ΄ΠΈΡ ΡΠ΅ΡΠ΅Π· ΠΏΠΎΠΏΠ΅ΡΠ΅ΡΠ½ΠΎΠ΅ ΡΠ΅ΡΠ΅Π½ΠΈΠ΅ ΠΏΡΠΎΠ²ΠΎΠ΄Π½ΠΈΠΊΠ° ΠΊΠΎΠ»ΠΈΡΠ΅ΡΡΠ²ΠΎ ΡΠ»Π΅ΠΊΡΡΠΈΡΠ΅ΡΡΠ²Π°Β \(\triangle Q.\)
ΠΠ΅Π½Ρ Π΄ΠΎΠΊΠ°Π·Π°Π», ΡΡΠΎ ΠΈΠ½Π΄ΡΠΊΡΠΈΠΎΠ½Π½ΡΠΉ ΡΠΎΠΊΒ Π²ΡΠ΅Π³Π΄Π° Π½Π°ΠΏΡΠ°Π²Π»Π΅Π½ ΡΠ°ΠΊ, ΡΡΠΎΠ±Ρ ΠΏΡΠΎΡΠΈΠ²ΠΎΠ΄Π΅ΠΉΡΡΠ²ΠΎΠ²Π°ΡΡ Π²ΡΠ·Π²Π°Π²ΡΠ΅ΠΉ Π΅Π³ΠΎΒ ΠΏΡΠΈΡΠΈΠ½Π΅. Π‘ΠΎΠ³Π»Π°ΡΠ½ΠΎ ΠΏΡΠ°Π²ΠΈΠ»Ρ ΠΠ΅Π½ΡΠ°, Π²Β Π²ΡΡΠ΅ΠΏΡΠΈΠ²Π΅Π΄Π΅Π½Π½ΠΎΠΌ ΡΠΎΠΎΡΠ½ΠΎΡΠ΅Π½ΠΈΠΈ ΡΠ»Π΅Π΄ΡΠ΅Ρ Π²ΡΠ±ΡΠ°ΡΡ ΠΎΡΡΠΈΡΠ°ΡΠ΅Π»ΡΠ½ΡΠΉ Π·Π½Π°ΠΊ, ΡΡΠΈΡΠ°Ρ ΠΊΠΎΡΡΡΠΈΡΠΈΠ΅Π½Ρ\( \alpha \) ΠΏΠΎΠ»ΠΎΠΆΠΈΡΠ΅Π»ΡΠ½ΡΠΌ:Β
\(\varepsilon_i\;=\;-\;\alpha\frac{\triangle Π€}{\triangle t}.\)
ΠΠ°ΠΊΒ ΡΠ°ΡΡΡΠΈΡΠ°ΡΡ ΡΠ»Π΅ΠΊΡΡΠΎΠ΄Π²ΠΈΠΆΡΡΡΡ ΡΠΈΠ»Ρ ΠΈΠ½Π΄ΡΠΊΡΠΈΠΈ, ΡΠΎΡΠΌΡΠ»Ρ
Π§Π΅ΡΠ΅Π· ΠΌΠ°Π³Π½ΠΈΡΠ½ΡΠΉ ΠΏΠΎΡΠΎΠΊ
\(\varepsilon_i\;=\;-\;\alpha\frac{\triangle Π€}{\triangle t}. \)
Π§Π΅ΡΠ΅Π· ΡΠΈΠ»Ρ ΡΠΎΠΊΠ°
ΠΠΠ‘Β ΡΠ°ΠΌΠΎΠΈΠ½Π΄ΡΠΊΡΠΈΠΈ Π·Π°Π²ΠΈΡΠΈΡ ΠΎΡΒ ΠΈΠ·ΠΌΠ΅Π½Π΅Π½ΠΈΡ ΡΠΈΠ»Ρ ΡΠΎΠΊΠ°, ΠΏΡΠΈΒ ΡΡΠΎΠΌ ΠΌΠ°Π³Π½ΠΈΡΠ½ΡΠΉ ΠΏΠΎΡΠΎΠΊ ΡΠΎΠ±ΡΡΠ²Π΅Π½Π½ΠΎΠ³ΠΎ ΠΏΠΎΠ»Ρ ΡΠ΅ΡΠ΅Π· ΡΠ΅ΠΏΡ ΠΏΡΠΎΠΏΠΎΡΡΠΈΠΎΠ½Π°Π»Π΅Π½ ΡΠΎΠΊΡ Π²Β Π½Π΅ΠΉ:
\(\varepsilon_{is\;}\;=\;-\;L\frac{\triangle I}{\triangle t}. \)
L Π·Π΄Π΅ΡΡ β ΠΈΠ½Π΄ΡΠΊΡΠΈΠ²Π½ΠΎΡΡΡ ΠΏΡΠΎΠ²ΠΎΠ΄Π½ΠΈΠΊΠ°.
Π§Π΅ΡΠ΅Π· ΡΠΎΠΏΡΠΎΡΠΈΠ²Π»Π΅Π½ΠΈΠ΅
ΠΠ»Ρ ΠΠΠ‘ ΠΈΠ½Π΄ΡΠΊΡΠΈΠΈ ΡΡΠ°Π²Π½Π΅Π½ΠΈΠ΅ Π·Π°ΠΊΠΎΠ½Π° ΠΠΌΠ°Β ΠΌΠΎΠΆΠ½ΠΎ ΠΏΠ΅ΡΠ΅ΠΏΠΈΡΠ°ΡΡ Π²Β Π²ΠΈΠ΄Π΅:
\(\varepsilon_{i\;}\;=\;IR\;-\;\varepsilon.\)
Π§Π΅ΡΠ΅Π· ΡΠ³Π»ΠΎΠ²ΡΡ ΡΠΊΠΎΡΠΎΡΡΡ
\(\varepsilon_i\;=\;Π\omega SN\sin\left(\alpha\right). \)
BΒ Π·Π΄Π΅ΡΡ β ΠΈΠ½Π΄ΡΠΊΡΠΈΡ ΠΌΠ°Π³Π½ΠΈΡΠ½ΠΎΠ³ΠΎ ΠΏΠΎΠ»Ρ,Β \(\omega\) β ΡΠ³Π»ΠΎΠ²Π°Ρ ΡΠΊΠΎΡΠΎΡΡΡ Π²ΡΠ°ΡΠ΅Π½ΠΈΡ ΡΠ°ΠΌΠΊΠΈ, S β ΠΏΠ»ΠΎΡΠ°Π΄Ρ ΡΠ°ΠΌΠΊΠΈ, N β ΡΠΈΡΠ»ΠΎ Π²ΠΈΡΠΊΠΎΠ²,Β \(\alpha\) β ΡΠ³ΠΎΠ» ΠΌΠ΅ΠΆΠ΄Ρ Π²Π΅ΠΊΡΠΎΡΠ°ΠΌΠΈ ΠΈΠ½Π΄ΡΠΊΡΠΈΠΈ ΠΌΠ°Π³Π½ΠΈΡΠ½ΠΎΠ³ΠΎ ΠΏΠΎΠ»Ρ ΠΈΒ ΡΠΊΠΎΡΠΎΡΡΠΈ Π΄Π²ΠΈΠΆΠ΅Π½ΠΈΡ ΠΏΡΠΎΠ²ΠΎΠ΄Π½ΠΈΠΊΠ°.
Π§Π΅ΡΠ΅Π· ΠΏΠ»ΠΎΡΠ°Π΄Ρ
ΠΡΠ»ΠΈ ΠΌΠ°Π³Π½ΠΈΡΠ½ΡΠΉ ΠΏΠΎΡΠΎΠΊ ΠΈΠ·ΠΌΠ΅Π½ΡΠ΅ΡΡΡ Π±Π΅Π·Β Π΄Π΅ΡΠΎΡΠΌΠ°ΡΠΈΠΈ Π²ΠΈΡΠΊΠΎΠ², Ρ. Π΅. ΠΈΡ
Β ΠΊΠΎΠ»ΠΈΡΠ΅ΡΡΠ²ΠΎ ΠΈΒ ΠΏΠ»ΠΎΡΠ°Π΄Ρ Π½Π΅Β ΠΌΠ΅Π½ΡΡΡΡΡ, ΡΠΎΒ ΠΌΠΎΠΆΠ½ΠΎ Π½Π°ΠΉΡΠΈ ΡΠ»Π΅ΠΊΡΡΠΎΠ΄Π²ΠΈΠΆΡΡΡΡ ΡΠΈΠ»Ρ ΠΈΠ½Π΄ΡΠΊΡΠΈΠΈ ΡΠ΅ΡΠ΅Π· ΠΏΠ»ΠΎΡΠ°Π΄Ρ.
Π£Π³ΠΎΠ» \alpha ΠΌΠ΅ΠΆΠ΄Ρ Π²Π΅ΠΊΡΠΎΡΠΎΠΌ ΠΌΠ°Π³Π½ΠΈΡΠ½ΠΎΠ³ΠΎ ΠΏΠΎΠ»Ρ ΠΈΒ Π½ΠΎΡΠΌΠ°Π»ΡΡ ΠΊΒ ΠΏΠ»ΠΎΡΠΊΠΎΡΡΠΈ Π²ΠΈΡΠΊΠΎΠ² Π±ΡΠ΄Π΅Ρ ΡΠ°Π²Π΅Π½:
\(2\mathrm\pi\;\times\;\mathrm v\;\times\;\mathrm t. \)ΠΠΎΠ»Π½ΡΠΉ ΠΌΠ°Π³Π½ΠΈΡΠ½ΡΠΉ ΠΏΠΎΡΠΎΠΊ Π²Β ΠΌΠΎΠΌΠ΅Π½Ρ Π²ΡΠ΅ΠΌΠ΅Π½ΠΈ t Π±ΡΠ΄Π΅Ρ ΡΠ°Π²Π΅Π½:
\(\psi_B\;=\;N\;\times\;B\;\times\;S\;\times\;\cos\left(\alpha\right)=\;N\;\times\;B\;\times\;S\;\times\;\cos\left(2\mathrm\pi\;\times\;\mathrm v\;\times\;\mathrm t\right). \)
Π’ΠΎΠ³Π΄Π°Β \(\varepsilon_i\;=\;-\;\frac{d\psi_B}{dt}=\;2\mathrm{pivNBSsin}\left(2\mathrm{pivt}\right).\)
ΠΠ°ΡΠΊΠΎΠ»ΡΠΊΠΎ ΠΏΠΎΠ»Π΅Π·Π½ΠΎΠΉ Π±ΡΠ»Π° Π΄Π»Ρ Π²Π°Ρ ΡΡΠ°ΡΡΡ?
Π Π΅ΠΉΡΠΈΠ½Π³: 5.00 (ΠΠΎΠ»ΠΎΡΠΎΠ²: 1)
ΠΡΠ΄Π΅Π»ΠΈΡΠ΅ ΡΠ΅ΠΊΡΡ ΠΈ Π½Π°ΠΆΠΌΠΈΡΠ΅ ΠΎΠ΄Π½ΠΎΠ²ΡΠ΅ΠΌΠ΅Π½Π½ΠΎ ΠΊΠ»Π°Π²ΠΈΡΠΈ Β«CtrlΒ» ΠΈ Β«EnterΒ»
ΠΠΎΠΈΡΠΊ ΠΏΠΎ ΡΠΎΠ΄Π΅ΡΠΆΠΈΠΌΠΎΠΌΡ
Π§ΡΠΎ ΡΠ°ΠΊΠΎΠ΅ ΡΠ»Π΅ΠΊΡΡΠΎΠΌΠ°Π³Π½ΠΈΡΠ½Π°Ρ ΠΈΠ½Π΄ΡΠΊΡΠΈΡ? | HomeElectronics
ΠΡΠ΅ΠΌ Π΄ΠΎΠ±ΡΠΎΠ³ΠΎ Π²ΡΠ΅ΠΌΠ΅Π½ΠΈ ΡΡΡΠΎΠΊ. Π ΠΏΡΠΎΡΠ»ΡΡ ΡΡΠ°ΡΡΡΡ Ρ ΡΠ°ΡΡΠΊΠ°Π·Π°Π» ΠΎ ΠΌΠ°Π³Π½ΠΈΡΠ½ΠΎΠΌ ΠΏΠΎΠ»Π΅ Π² Π²Π΅ΡΠ΅ΡΡΠ²Π΅, Π° ΡΠ°ΠΊ ΠΆΠ΅ ΠΌΠ°Π³Π½ΠΈΡΠ½ΡΡ ΡΠ΅ΠΏΡΡ ΠΈ ΠΌΠ΅ΡΠΎΠ΄Π°Ρ ΠΈΡ ΡΠ°ΡΡΡΡΠ°. ΠΠ°Π½Π½Π°Ρ ΡΡΠ°ΡΡΡ ΠΏΠΎΡΠ²ΡΡΠ΅Π½Π° ΡΠ°ΠΊΠΎΠΌΡ ΡΠ²Π»Π΅Π½ΠΈΡ, ΠΊΠ°ΠΊ ΠΠΠ‘ ΠΈΠ½Π΄ΡΠΊΡΠΈΠΈ, Π² ΠΊΠ°ΠΊΠΈΡ ΡΠ»ΡΡΠ°ΡΡ ΠΎΠ½Π° Π²ΠΎΠ·Π½ΠΈΠΊΠ°Π΅Ρ, Π° ΡΠ°ΠΊ ΠΆΠ΅ Π·Π°ΡΡΠΎΠ½Ρ ΠΏΠΎΠ½ΡΡΠΈΠ΅ ΠΈΠ½Π΄ΡΠΊΡΠΈΠ²Π½ΠΎΡΡΠΈ, ΠΊΠ°ΠΊ ΠΎΡΠ½ΠΎΠ²Π½ΠΎΠ³ΠΎ ΠΏΠ°ΡΠ°ΠΌΠ΅ΡΡΠ° Ρ Π°ΡΠ°ΠΊΡΠ΅ΡΠΈΠ·ΡΡΡΠ΅Π³ΠΎ Π²ΠΎΠ·Π½ΠΈΠΊΠ½ΠΎΠ²Π΅Π½ΠΈΠ΅ ΠΌΠ°Π³Π½ΠΈΡΠ½ΠΎΠ³ΠΎ ΠΏΠΎΡΠΎΠΊΠ° ΠΏΡΠΈ Π²ΠΎΠ·Π½ΠΈΠΊΠ½ΠΎΠ²Π΅Π½ΠΈΠΈ ΡΠ»Π΅ΠΊΡΡΠΈΡΠ΅ΡΠΊΠΎΠ³ΠΎ ΠΏΠΎΠ»Ρ Π² ΠΏΡΠΎΠ²ΠΎΠ΄Π½ΠΈΠΊΠ΅.
ΠΠ»Ρ ΡΠ±ΠΎΡΠΊΠΈ ΡΠ°Π΄ΠΈΠΎΡΠ»Π΅ΠΊΡΡΠΎΠ½Π½ΠΎΠ³ΠΎ ΡΡΡΡΠΎΠΉΡΡΠ²Π° ΠΌΠΎΠΆΠ½ΠΎ ΠΏΡΠ΅ΠΎΠ±ΡΠ΅ΡΠΊΠΈ DIY KIT Π½Π°Π±ΠΎΡ ΠΏΠΎ ΡΡΡΠ»ΠΊΠ΅.
ΠΠ°ΠΊ Π²ΠΎΠ·Π½ΠΈΠΊΠ°Π΅Ρ ΠΠΠ‘ ΠΈΠ½Π΄ΡΠΊΡΠΈΠΈ ΠΈ ΠΈΠ½Π΄ΡΠΊΡΠΈΠΎΠ½Π½ΡΠΉ ΡΠΎΠΊ?
ΠΠ°ΠΊ Ρ Π³ΠΎΠ²ΠΎΡΠΈΠ» Π² ΠΏΡΠ΅Π΄ΡΠ΄ΡΡΠΈΡ ΡΡΠ°ΡΡΡΡ Π²ΠΎΠΊΡΡΠ³ ΠΏΡΠΎΠ²ΠΎΠ΄Π½ΠΈΠΊΠ°, ΠΏΠΎ ΠΊΠΎΡΠΎΡΠΎΠΌΡ ΠΏΡΠΎΡΠ΅ΠΊΠ°Π΅Ρ ΡΠ»Π΅ΠΊΡΡΠΈΡΠ΅ΡΠΊΠΈΠΉ ΡΠΎΠΊ, Π²ΠΎΠ·Π½ΠΈΠΊΠ°Π΅Ρ ΡΠ»Π΅ΠΊΡΡΠΎΠΌΠ°Π³Π½ΠΈΡΠ½ΠΎΠ΅ ΠΏΠΎΠ»Π΅. ΠΠ°Π½Π½ΠΎΠ΅ ΠΌΠ°Π³Π½ΠΈΡΠ½ΠΎΠ΅ ΠΏΠΎΠ»Π΅ Ρ ΡΠ°ΡΡΠΌΠΎΡΡΠ΅Π» Π·Π΄Π΅ΡΡ ΠΈ Π·Π΄Π΅ΡΡ. ΠΠ΄Π½Π°ΠΊΠΎ ΡΡΡΠ΅ΡΡΠ²ΡΠ΅Ρ ΠΈ ΠΎΠ±ΡΠ°ΡΠ½ΠΎΠ΅ ΡΠ²Π»Π΅Π½ΠΈΠ΅, ΠΊΠΎΡΠΎΡΠΎΠ΅ Π½Π°Π·ΡΠ²Π°Π΅ΡΡΡ ΡΠ»Π΅ΠΊΡΡΠΎΠΌΠ°Π³Π½ΠΈΡΠ½Π°Ρ ΠΈΠ½Π΄ΡΠΊΡΠΈΡ. ΠΠ°Π½Π½ΠΎΠ΅ ΡΠ²Π»Π΅Π½ΠΈΠ΅ ΠΎΡΠΊΡΡΠ» Π°Π½Π³Π»ΠΈΠΉΡΠΊΠΈΠΉ ΡΠΈΠ·ΠΈΠΊ Π. Π€Π°ΡΠ°Π΄Π΅ΠΉ.
ΠΠ»Ρ ΡΠ°ΡΡΠΌΠΎΡΡΠ΅Π½ΠΈΡ Π΄Π°Π½Π½ΠΎΠ³ΠΎ ΡΠ²Π»Π΅Π½ΠΈΡ ΡΠ°ΡΡΠΌΠΎΡΡΠΈΠΌ ΡΠ»Π΅Π΄ΡΡΡΠΈΠΉ ΡΠΈΡΡΠ½ΠΎΠΊ
Π ΠΈΡΡΠ½ΠΎΠΊ, ΠΈΠ»Π»ΡΡΡΡΠΈΡΡΡΡΠΈΠΉ ΡΠ»Π΅ΠΊΡΡΠΎΠΌΠ°Π³Π½ΠΈΡΠ½ΡΡ ΠΈΠ½Π΄ΡΠΊΡΠΈΡ.
ΠΠ° Π΄Π°Π½Π½ΠΎΠΌ ΡΠΈΡΡΠ½ΠΊΠ΅ ΠΏΠΎΠΊΠ°Π·Π°Π½Π° ΡΠ°ΠΌΠΊΠ° ΠΈΠ· ΠΏΡΠΎΠ²ΠΎΠ΄Π½ΠΈΠΊΠ°, ΠΏΠΎΠΌΠ΅ΡΡΠ½Π½Π°Ρ Π² ΡΠ»Π΅ΠΊΡΡΠΈΡΠ΅ΡΠΊΠΎΠ΅ ΠΏΠΎΠ»Π΅ Ρ ΠΈΠ½Π΄ΡΠΊΡΠΈΠ΅ΠΉ Π. ΠΡΠ»ΠΈ Π΄Π°Π½Π½ΡΡ ΡΠ°ΠΌΠΊΡ Π΄Π²ΠΈΠ³Π°ΡΡ Π²Π²Π΅ΡΡ -Π²Π½ΠΈΠ· ΠΏΠΎ Π½Π°ΠΏΡΠ°Π²Π»Π΅Π½ΠΈΡ ΠΌΠ°Π³Π½ΠΈΡΠ½ΡΡ ΡΠΈΠ»ΠΎΠ²ΡΡ Π»ΠΈΠ½ΠΈΠΉ ΠΈΠ»ΠΈ Π²Π»Π΅Π²ΠΎ β Π²ΠΏΡΠ°Π²ΠΎ ΠΏΠ΅ΡΠΏΠ΅Π½Π΄ΠΈΠΊΡΠ»ΡΡΠ½ΠΎ ΡΠΈΠ»ΠΎΠ²ΡΠΌ Π»ΠΈΠ½ΠΈΡΠΌ, ΡΠΎ ΠΌΠ°Π³Π½ΠΈΡΠ½ΡΠΉ ΠΏΠΎΡΠΎΠΊ Ξ¦ ΠΏΡΠΎΠ½ΠΈΠ·ΡΠ²Π°ΡΡΠΈΠΉ ΡΠ°ΠΌΠΊΡ Π±ΡΠ΄Π΅Π½ ΠΏΡΠ°ΠΊΡΠΈΡΠ΅ΡΠΊΠΈ ΠΏΠΎΡΡΠΎΡΠ½Π½ΡΠΌ. ΠΡΠ»ΠΈ ΠΆΠ΅ Π²ΡΠ°ΡΠ°ΡΡ ΡΠ°ΠΌΠΊΡ Π²ΠΎΠΊΡΡΠ³ ΠΎΡΠΈ Π, ΡΠΎ Π·Π° Π½Π΅ΠΊΠΎΡΠΎΡΡΠΉ ΠΏΡΠΎΠΌΠ΅ΠΆΡΡΠΎΠΊ Π²ΡΠ΅ΠΌΠ΅Π½ΠΈ βt Β ΠΌΠ°Π³Π½ΠΈΡΠ½ΡΠΉ ΠΏΠΎΡΠΎΠΊ ΠΈΠ·ΠΌΠ΅Π½ΠΈΡΡΡ Π½Π° Π½Π΅ΠΊΠΎΡΠΎΡΡΡ Π²Π΅Π»ΠΈΡΠΈΠ½Ρ βΞ¦ ΠΈ Π² ΡΠ΅Π·ΡΠ»ΡΡΠ°ΡΠ΅ Π² ΡΠ°ΠΌΠΊΠ΅ ΠΏΠΎΡΠ²ΠΈΡΡΡ ΠΠΠ‘ ΠΈΠ½Π΄ΡΠΊΡΠΈΠΈ Πi ΠΈ ΠΏΠΎΡΠ΅ΡΡΡ ΡΠΎΠΊ I, Π½Π°Π·ΡΠ²Π°Π΅ΠΌΡΠΌ ΠΈΠ½Π΄ΡΠΊΡΠΈΠΎΠ½Π½ΡΠΌ ΡΠΎΠΊΠΎΠΌ.
Π§Π΅ΠΌΡ ΡΠ°Π²Π½ΠΎ ΠΠΠ‘ ΠΈΠ½Π΄ΡΠΊΡΠΈΠΈ?
ΠΠ»Ρ ΠΎΠΏΡΠ΅Π΄Π΅Π»Π΅Π½ΠΈΡ Π²Π΅Π»ΠΈΡΠΈΠ½Ρ Π²ΠΎΠ·Π½ΠΈΠΊΠ°ΡΡΠ΅ΠΉ ΠΠΠ‘ ΡΠ°ΡΡΠΌΠΎΡΡΠΈΠΌ ΠΊΠΎΠ½ΡΡΡ ΠΏΠΎΠΌΠ΅ΡΠ΅Π½Π½ΡΠΉ Π² ΠΎΠ΄Π½ΠΎΡΠΎΠ΄Π½ΠΎΠ΅ ΠΌΠ°Π³Π½ΠΈΡΠ½ΠΎΠ΅ ΠΏΠΎΠ»Π΅ Ρ ΠΈΠ½Π΄ΡΠΊΡΠΈΠ΅ΠΉ Π, ΠΏΠΎ Π΄Π°Π½Π½ΠΎΠΌΡ ΠΊΠΎΠ½ΡΡΡΡ ΡΠ²ΠΎΠ±ΠΎΠ΄Π½ΠΎ ΠΌΠΎΠΆΠ΅Ρ ΠΏΠ΅ΡΠ΅ΠΌΠ΅ΡΠ°ΡΡΡΡ ΠΏΡΠΎΠ²ΠΎΠ΄Π½ΠΈΠΊ Π΄Π»ΠΈΠ½ΠΎΠΉ l.
Β
ΠΠΎΠ·Π½ΠΈΠΊΠ½ΠΎΠ²Π΅Π½ΠΈΠ΅ ΠΠΠ‘ ΠΈΠ½Π΄ΡΠΊΡΠΈΠΈ Π² ΠΏΡΡΠΌΠΎΠ»ΠΈΠ½Π΅ΠΉΠ½ΠΎΠΌ ΠΏΡΠΎΠ²ΠΎΠ΄Π½ΠΈΠΊΠ΅.
ΠΠΎΠ΄ Π΄Π΅ΠΉΡΡΠ²ΠΈΠ΅ΠΌ ΡΠΈΠ»Ρ F ΠΏΡΠΎΠ²ΠΎΠ΄Π½ΠΈΠΊ Π½Π°ΡΠΈΠ½Π°Π΅Ρ Π΄Π²ΠΈΠ³Π°ΡΡΡΡ ΡΠΎ ΡΠΊΠΎΡΠΎΡΡΡΡ v. ΠΠ° Π½Π΅ΠΊΠΎΡΠΎΡΠΎΠ΅ Π²ΡΠ΅ΠΌΡ βt ΠΏΡΠΎΠ²ΠΎΠ΄Π½ΠΈΠΊ ΠΏΡΠΎΠΉΠ΄ΡΡ ΠΏΡΡΡ db. Π’Π°ΠΊΠΈΠΌ ΠΎΠ±ΡΠ°Π·ΠΎΠΌ, Π·Π°ΡΡΠ°ΡΠΈΠ²Π°Π΅ΠΌΠ°Ρ ΡΠ°Π±ΠΎΡΠ° Π½Π° ΠΏΠ΅ΡΠ΅ΠΌΠ΅ΡΠ΅Π½ΠΈΠ΅ ΠΏΡΠΎΠ²ΠΎΠ΄Π½ΠΈΠΊΠ° ΡΠΎΡΡΠ°Π²ΠΈΡ
Π’Π°ΠΊ ΠΊΠ°ΠΊ ΠΏΡΠΎΠ²ΠΎΠ΄Π½ΠΈΠΊ ΡΠΎΡΡΠΎΠΈΡ ΠΈΠ· Π·Π°ΡΡΠΆΠ΅Π½Π½ΡΡ ΡΠ°ΡΡΠΈΡ β ΡΠ»Π΅ΠΊΡΡΠΎΠ½ΠΎΠ² ΠΈ ΠΏΡΠΎΡΠΎΠ½ΠΎΠ², ΡΠΎ ΠΎΠ½ΠΈ ΡΠ°ΠΊΠΆΠ΅ Π΄Π²ΠΈΠΆΡΡΡΡ Π²ΠΌΠ΅ΡΡΠ΅ Ρ ΠΏΡΠΎΠ²ΠΎΠ΄Π½ΠΈΠΊΠΎΠΌ. ΠΠ°ΠΊ ΠΈΠ·Π²Π΅ΡΡΠ½ΠΎ Π½Π° Π΄Π²ΠΈΠΆΡΡΡΡΡΡ Π·Π°ΡΡΠΆΠ΅Π½Π½ΡΡ ΡΠ°ΡΡΠΈΡΡ Π΄Π΅ΠΉΡΡΠ²ΡΠ΅Ρ ΡΠΈΠ»Π° ΠΠΎΡΠ΅Π½ΡΠ°, ΠΊΠΎΡΠΎΡΠ°Ρ ΠΏΠ΅ΡΠΏΠ΅Π½Π΄ΠΈΠΊΡΠ»ΡΡΠ½Π° ΠΊ Π½Π°ΠΏΡΠ°Π²Π»Π΅Π½ΠΈΡ Π΄Π²ΠΈΠΆΠ΅Π½ΠΈΡ ΡΠ°ΡΡΠΈΡΡ ΠΈ ΠΊ Π²Π΅ΠΊΡΠΎΡΡ ΠΌΠ°Π³Π½ΠΈΡΠ½ΠΎΠΉ ΠΈΠ½Π΄ΡΠΊΡΠΈΠΈ Π, ΡΠΎ Π΅ΡΡΡ ΡΠ»Π΅ΠΊΡΡΠΎΠ½Ρ Π½Π°ΡΠΈΠ½Π°ΡΡ Π΄Π²ΠΈΠ³Π°ΡΡΡΡ Π²Π΄ΠΎΠ»Ρ ΠΏΡΠΎΠ²ΠΎΠ΄Π½ΠΈΠΊΠ° ΠΏΡΠΈΠ²ΠΎΠ΄ΡΒ ΠΊ Π²ΠΎΠ·Π½ΠΈΠΊΠ½ΠΎΠ²Π΅Π½ΠΈΡ ΡΠ»Π΅ΠΊΡΡΠΈΡΠ΅ΡΠΊΠΎΠ³ΠΎ ΡΠΎΠΊΠ° Π² Π½ΡΠΌ.
ΠΠ΄Π½Π°ΠΊΠΎ Π½Π° ΠΏΡΠΎΠ²ΠΎΠ΄Π½ΠΈΠΊ Ρ ΡΠΎΠΊΠΎΠΌ Π² ΠΌΠ°Π³Π½ΠΈΡΠ½ΠΎΠΌ ΠΏΠΎΠ»Π΅ Π΄Π΅ΠΉΡΡΠ²ΡΠ΅Ρ Π½Π΅ΠΊΠΎΡΠΎΡΠ°Ρ ΡΠΈΠ»Π° FΡ, ΠΊΠΎΡΠΎΡΠ°Ρ Π² ΡΠΎΠΎΡΠ²Π΅ΡΡΡΠ²ΠΈΠΈ Ρ ΠΏΡΠ°Π²ΠΈΠ»ΠΎΠΌ Π»Π΅Π²ΠΎΠΉ ΡΡΠΊΠΈ Π±ΡΠ΄Π΅Ρ ΠΏΡΠΎΡΠΈΠ²ΠΎΠΏΠΎΠ»ΠΎΠΆΠ½Π° Π΄Π΅ΠΉΡΡΠ²ΠΈΡ ΡΠΈΠ»Ρ F, Π·Π° ΡΡΡΡ ΠΊΠΎΡΠΎΡΠΎΠΉ ΠΏΡΠΎΠ²ΠΎΠ΄Π½ΠΈΠΊ Π΄Π²ΠΈΠΆΠ΅ΡΡΡ. Π’Π°ΠΊ ΠΊΠ°ΠΊ ΠΏΡΠΎΠ²ΠΎΠ΄Π½ΠΈΠΊ Π΄Π²ΠΈΠΆΠ΅ΡΡΡ ΡΠ°Π²Π½ΠΎΠΌΠ΅ΡΠ½ΠΎ, ΡΠΎ Π΅ΡΡΡ Ρ ΠΏΠΎΡΡΠΎΡΠ½Π½ΠΎΠΉ ΡΠΊΠΎΡΠΎΡΡΡΡ, ΡΠΎ ΡΠΈΠ»ΡΒ FΡ ΠΈ F ΡΠ°Π²Π½Ρ ΠΏΠΎ Π°Π±ΡΠΎΠ»ΡΡΠ½ΠΎΠΌΡ Π·Π½Π°ΡΠ΅Π½ΠΈΡ
Π³Π΄Π΅ Π β ΠΈΠ½Π΄ΡΠΊΡΠΈΡ ΠΌΠ°Π³Π½ΠΈΡΠ½ΠΎΠ³ΠΎ ΠΏΠΎΠ»Ρ,
I β ΡΠΈΠ»Π° ΡΠΎΠΊΠ° Π² ΠΏΡΠΎΠ²ΠΎΠ΄Π½ΠΈΠΊΠ°, Π²ΠΎΠ·Π½ΠΈΠΊΠ°ΡΡΠ°Ρ ΠΏΠΎ Π΄Π΅ΠΉΡΡΠ²ΠΈΠ΅ΠΌ ΠΠΠ‘ ΠΈΠ½Π΄ΡΠΊΡΠΈΠΈ,
l β Π΄Π»ΠΈΠ½Π° ΠΏΡΠΎΠ²ΠΎΠ΄Π½ΠΈΠΊΠ°.
Π’Π°ΠΊ ΠΊΠ°ΠΊ ΠΏΡΡΡ db ΠΏΡΠΎΠΉΠ΄Π΅Π½Π½ΡΠΉ ΠΏΡΠΎΠ²ΠΎΠ΄Π½ΠΈΠΊΠΎΠΌ Π·Π°Π²ΠΈΡΠΈΡ ΠΎΡ ΡΠΊΠΎΡΠΎΡΡΠΈ v ΠΈ Π²ΡΠ΅ΠΌΠ΅Π½ΠΈ t, ΡΠΎ ΡΠ°Π±ΠΎΡΠ°, Π·Π°ΡΡΠ°ΡΠΈΠ²Π°Π΅ΠΌΠ°Ρ Π½Π° ΠΏΠ΅ΡΠ΅ΠΌΠ΅ΡΠ΅Π½ΠΈΡ ΠΏΡΠΎΠ²ΠΎΠ΄Π½ΠΈΠΊΠ°, Π² ΠΌΠ°Π³Π½ΠΈΡΠ½ΠΎΠΌ ΠΏΠΎΠ»Π΅ ΡΠΎΡΡΠ°Π²ΠΈΡ
ΠΡΠΈ ΠΏΠ΅ΡΠ΅ΠΌΠ΅ΡΠ΅Π½ΠΈΠΈ ΠΏΡΠΎΠ²ΠΎΠ΄Π½ΠΈΠΊΠ° Π² ΠΌΠ°Π³Π½ΠΈΡΠ½ΠΎΠΌ ΠΏΠΎΠ»Π΅ ΠΏΡΠ°ΠΊΡΠΈΡΠ΅ΡΠΊΠΈ Π²ΡΡ Π·Π°ΡΡΠ°ΡΠΈΠ²Π°Π΅ΠΌΠ°Ρ Π½Π° ΡΡΡ ΡΠ°Π±ΠΎΡΡ ΠΌΠ΅Ρ Π°Π½ΠΈΡΠ΅ΡΠΊΠ°Ρ ΡΠ½Π΅ΡΠ³ΠΈΡ ΠΏΠ΅ΡΠ΅Ρ ΠΎΠ΄ΠΈΡ Π² ΡΠ»Π΅ΠΊΡΡΠΈΡΠ΅ΡΠΊΡΡ ΡΠ½Π΅ΡΠ³ΠΈΡ, ΡΠΎ Π΅ΡΡΡ
Π’Π°ΠΊΠΈΠΌ ΠΎΠ±ΡΠ°Π·ΠΎΠΌ, ΠΏΡΠ΅ΠΎΠ±ΡΠ°Π·ΠΎΠ²Π°Π² ΠΏΠΎΡΠ»Π΅Π΄Π½Π΅Π΅ Π²ΡΡΠ°ΠΆΠ΅Π½ΠΈΠ΅, ΠΏΠΎΠ»ΡΡΠΈΠΌ Π·Π½Π°ΡΠ΅Π½ΠΈΠ΅ ΠΠΠ‘ ΠΈΠ½Π΄ΡΠΊΡΠΈΠΈ ΠΏΡΠΈ Π΄Π²ΠΈΠΆΠ΅Π½ΠΈΠΈ ΠΏΡΡΠΌΠΎΠ»ΠΈΠ½Π΅ΠΉΠ½ΠΎΠ³ΠΎ ΠΏΡΠΎΠ²ΠΎΠ΄Π½ΠΈΠΊΠ° Π² ΠΌΠ°Π³Π½ΠΈΡΠ½ΠΎΠΌ ΠΏΠΎΠ»Π΅
Π³Π΄Π΅ Π β ΠΈΠ½Π΄ΡΠΊΡΠΈΡ ΠΌΠ°Π³Π½ΠΈΡΠ½ΠΎΠ³ΠΎ ΠΏΠΎΠ»Ρ,
lΒ β Π΄Π»ΠΈΠ½Π° ΠΏΡΠΎΠ²ΠΎΠ΄Π½ΠΈΠΊΠ°,
vΒ β ΡΠΊΠΎΡΠΎΡΡΡ ΠΏΠ΅ΡΠ΅ΠΌΠ΅ΡΠ΅Π½ΠΈΡ ΠΏΡΠΎΠ²ΠΎΠ΄Π½ΠΈΠΊΠ°.
ΠΠ°Π½Π½ΠΎΠ΅ Π²ΡΡΠ°ΠΆΠ΅Π½ΠΈΠ΅ ΡΠΎΠΎΡΠ²Π΅ΡΡΡΠ²ΡΠ΅Ρ Π΄Π²ΠΈΠΆΠ΅Π½ΠΈΡ ΠΏΡΠΎΠ²ΠΎΠ΄Π½ΠΈΠΊΠ° ΠΏΠ΅ΡΠΏΠ΅Π½Π΄ΠΈΠΊΡΠ»ΡΡΠ½ΠΎ Π»ΠΈΠ½ΠΈΡΠΌ ΠΌΠ°Π³Π½ΠΈΡΠ½ΠΎΠΉ ΠΈΠ½Π΄ΡΠΊΡΠΈΠΈ. ΠΡΠ»ΠΈ ΠΏΡΠΎΠΈΡΡ ΠΎΠ΄ΠΈΡ Π΄Π²ΠΈΠΆΠ΅Π½ΠΈΠ΅ ΠΏΠΎΠ΄ Π½Π΅ΠΊΠΎΡΠΎΡΡΠΌ ΡΠ³Π»ΠΎΠΌ ΠΊ Π»ΠΈΠ½ΠΈΡΠΌ ΠΌΠ°Π³Π½ΠΈΡΠ½ΠΎΠΉ ΠΈΠ½Π΄ΡΠΊΡΠΈΠΈ, ΡΠΎ Π²ΡΡΠ°ΠΆΠ΅Π½ΠΈΠ΅ ΠΏΡΠΈΠΎΠ±ΡΠ΅ΡΠ°Π΅Ρ Π²ΠΈΠ΄
ΠΠ° ΠΏΡΠ°ΠΊΡΠΈΠΊΠ΅ Π΄ΠΎΡΡΠ°ΡΠΎΡΠ½ΠΎ ΡΡΡΠ΄Π½ΠΎ ΠΏΠΎΡΡΠΈΡΠ°ΡΡ ΡΠΊΠΎΡΠΎΡΡΡ ΠΏΠ΅ΡΠ΅ΠΌΠ΅ΡΠ΅Π½ΠΈΡ ΠΏΡΠΎΠ²ΠΎΠ΄Π½ΠΈΠΊΠ°, ΠΏΠΎΡΡΠΎΠΌΡ ΠΏΡΠ΅ΠΎΠ±ΡΠ°Π·ΡΠ΅ΠΌ Π²ΡΡΠ°ΠΆΠ΅Π½ΠΈΠ΅ ΠΊ ΡΠ»Π΅Π΄ΡΡΡΠ΅ΠΌΡ Π²ΠΈΠ΄Ρ
Π³Π΄Π΅ dS β ΠΏΠ»ΠΎΡΠ°Π΄ΠΊΠ°, ΠΊΠΎΡΠΎΡΡΡ ΠΏΠ΅ΡΠ΅ΡΠ΅ΠΊΠ°Π΅Ρ ΠΏΡΠΎΠ²ΠΎΠ΄Π½ΠΈΠΊ ΠΏΡΠΈ ΡΠ²ΠΎΡΠΌ Π΄Π²ΠΈΠΆΠ΅Π½ΠΈΠΈ,
dΞ¦ β ΠΌΠ°Π³Π½ΠΈΡΠ½ΡΠΉ ΠΏΠΎΡΠΎΠΊ ΠΏΡΠΎΠ½ΠΈΠ·ΡΠ²Π°ΡΡΠΈΠΉ ΠΏΠ»ΠΎΡΠ°Π΄ΠΊΡ dS.
Π’Π°ΠΊΠΈΠΌ ΠΎΠ±ΡΠ°Π·ΠΎΠΌ, ΠΠΠ‘ ΠΈΠ½Π΄ΡΠΊΡΠΈΠΈ ΡΠ°Π²Π½Π° ΡΠΊΠΎΡΠΎΡΡΠΈ ΠΈΠ·ΠΌΠ΅Π½Π΅Π½ΠΈΡ ΠΌΠ°Π³Π½ΠΈΡΠ½ΠΎΠ³ΠΎ ΠΏΠΎΡΠΎΠΊΠ°, ΠΊΠΎΡΠΎΡΡΠΉ ΠΏΡΠΎΠ½ΠΈΠ·ΡΠ²Π°Π΅Ρ ΠΊΠΎΠ½ΡΡΡ.
ΠΠ»Ρ ΠΎΠ±ΠΎΠ·Π½Π°ΡΠ΅Π½ΠΈΡ Π½Π°ΠΏΡΠ°Π²Π»Π΅Π½ΠΈΡ Π΄Π²ΠΈΠΆΠ΅Π½ΠΈΡ ΡΠΎΠΊΠ° Π² ΠΊΠΎΠ½ΡΡΡΠ΅ Π²Π²ΠΎΠ΄ΡΡ Π·Π½Π°ΠΊ Β«βΒ», ΠΊΠΎΡΠΎΡΡΠΉ ΡΠΊΠ°Π·ΡΠ²Π°Π΅Ρ, ΡΡΠΎ ΡΠΎΠΊ Π² ΠΊΠΎΠ½ΡΡΡΠ΅ Π½Π°ΠΏΡΠ°Π²Π»Π΅Π½ ΠΏΡΠΎΡΠΈΠ² ΠΏΠΎΠ»ΠΎΠΆΠΈΡΠ΅Π»ΡΠ½ΠΎΠ³ΠΎ ΠΎΠ±Ρ ΠΎΠ΄Π° ΠΊΠΎΠ½ΡΡΡΠ°. Π’Π°ΠΊΠΈΠΌ ΠΎΠ±ΡΠ°Π·ΠΎΠΌ
ΠΠ°ΡΠ°ΡΡΡΡ Π² ΠΌΠ°Π³Π½ΠΈΡΠ½ΠΎΠΌ ΠΏΠΎΠ»Π΅ Π΄Π²ΠΈΠΆΠ΅ΡΡΡ ΠΊΠΎΠ½ΡΡΡ, ΡΠΎΡΡΠΎΡΡΠΈΠΉ ΠΈΠ· ΠΌΠ½ΠΎΠΆΠ΅ΡΡΠ²Π° Π²ΠΈΡΠΊΠΎΠ² ΠΏΡΠΎΠ²ΠΎΠ΄Π°, ΠΏΠΎΡΡΠΎΠΌΡ ΠΠΠ‘ ΠΈΠ½Π΄ΡΠΊΡΠΈΠΈ Π±ΡΠ΄Π΅Ρ ΠΈΠΌΠ΅ΡΡ Π²ΠΈΠ΄
Π³Π΄Π΅ w β ΠΊΠΎΠ»ΠΈΡΠ΅ΡΡΠ²ΠΎ Π²ΠΈΡΠΊΠΎΠ² Π² ΠΊΠΎΠ½ΡΡΡΠ΅,
dΞ¨ = wdΞ¦ β ΡΠ»Π΅ΠΌΠ΅Π½ΡΠ°ΡΠ½ΠΎΠ΅ ΠΏΠΎΡΠΎΠΊΠΎΡΡΠ΅ΠΏΠ»Π΅Π½ΠΈΠ΅.
ΠΠ΅ΡΠ΅ΡΡΠ°Π·ΠΈΡΡΡ ΠΏΡΠ΅Π΄ΡΠ΄ΡΡΠ΅Π΅ ΠΎΠΏΡΠ΅Π΄Π΅Π»Π΅Π½ΠΈΠ΅, ΠΠΠ‘ ΠΈΠ½Π΄ΡΠΊΡΠΈΠΈ Π² ΠΊΠΎΠ½ΡΡΡΠ΅ ΡΠ°Π²Π½Π° ΡΠΊΠΎΡΠΎΡΡΠΈ ΠΈΠ·ΠΌΠ΅Π½Π΅Π½ΠΈΡ ΠΏΠΎΡΠΎΠΊΠΎΡΡΠ΅ΠΏΠ»Π΅Π½ΠΈΡ ΡΡΠΎΠ³ΠΎ ΠΊΠΎΠ½ΡΡΡΠ°.
Π§ΡΠΎ ΡΠ°ΠΊΠΎΠ΅ ΠΠΠ‘ ΡΠ°ΠΌΠΎΠΈΠ΄ΡΠΊΡΠΈΠΈ? ΠΠ½Π΄ΡΠΊΡΠΈΠ²Π½ΠΎΡΡΡ
ΒΠΠ°ΠΊ ΠΈΠ·Π²Π΅ΡΡΠ½ΠΎ Π²ΠΎΠΊΡΡΠ³ ΠΏΡΠΎΠ²ΠΎΠ΄Π½ΠΈΠΊΠ° Ρ ΡΠΎΠΊΠΎΠΌ ΡΡΡΠ΅ΡΡΠ²ΡΠ΅Ρ ΠΌΠ°Π³Π½ΠΈΡΠ½ΠΎΠ΅ ΠΏΠΎΠ»Π΅. Π’Π°ΠΊ ΠΊΠ°ΠΊ ΠΈΠ½Π΄ΡΠΊΡΠΈΡ ΠΌΠ°Π³Π½ΠΈΡΠ½ΠΎΠ³ΠΎ ΠΏΠΎΠ»Ρ ΠΏΡΠΎΠΏΠΎΡΡΠΈΠΎΠ½Π°Π»ΡΠ½Π° ΡΠΈΠ»Π΅ ΡΠΎΠΊΠ° ΠΏΡΠΎΡΠ΅ΠΊΠ°ΡΡΠ΅Π³ΠΎ ΡΠ΅ΡΠ΅Π· ΠΏΡΠΎΠ²ΠΎΠ΄Π½ΠΈΠΊ, Π° ΠΌΠ°Π³Π½ΠΈΡΠ½ΡΠΉ ΠΏΠΎΡΠΎΠΊ ΠΏΡΠΎΠΏΠΎΡΡΠΈΠΎΠ½Π°Π»Π΅Π½ ΠΌΠ°Π³Π½ΠΈΡΠ½ΠΎΠΉ ΠΈΠ½Π΄ΡΠΊΡΠΈΠΈ, ΡΠ»Π΅Π΄ΠΎΠ²Π°ΡΠ΅Π»ΡΠ½ΠΎ, ΠΌΠ°Π³Π½ΠΈΡΠ½ΡΠΉ ΠΏΠΎΡΠΎΠΊ ΠΏΡΠΎΠΏΠΎΡΡΠΈΠΎΠ½Π°Π»Π΅Π½ ΡΠΈΠ»Π΅ ΡΠΎΠΊΠ°, ΠΏΡΠΎΡΠ΅ΠΊΠ°ΡΡΠ΅ΠΉ ΡΠ΅ΡΠ΅Π· ΠΏΡΠΎΠ²ΠΎΠ΄Π½ΠΈΠΊ.
Π’Π°ΠΊΠΈΠΌ ΠΎΠ±ΡΠ°Π·ΠΎΠΌ, ΠΏΡΠΈ ΠΈΠ·ΠΌΠ΅Π½Π΅Π½ΠΈΠΈ ΡΠΈΠ»Ρ ΡΠΎΠΊΠ° ΠΏΡΠΎΠΈΡΡ ΠΎΠ΄ΠΈΡ ΠΈΠ·ΠΌΠ΅Π½Π΅Π½ΠΈΠ΅ ΠΌΠ°Π³Π½ΠΈΡΠ½ΠΎΠ³ΠΎ ΠΏΠΎΡΠΎΠΊΠ° (ΠΈΠ»ΠΈ ΠΏΠΎΡΠΎΠΊΠΎΡΡΠ΅ΠΏΠ»Π΅Π½ΠΈΡ). ΠΠ΄Π½Π°ΠΊΠΎ Π² ΡΠΎΠΎΡΠ²Π΅ΡΡΡΠ²ΠΈΠ΅ Ρ Π·Π°ΠΊΠΎΠ½ΠΎΠΌ ΡΠ»Π΅ΠΊΡΡΠΎΠΌΠ°Π³Π½ΠΈΡΠ½ΠΎΠΉ ΠΈΠ½Π΄ΡΠΊΡΠΈΠΈ, ΠΈΠ·ΠΌΠ΅Π½Π΅Π½ΠΈΠ΅ ΠΏΠΎΡΠΎΠΊΠΎΡΡΠ΅ΠΏΠ»Π΅Π½ΠΈΡ ΠΏΡΠΈΠ²ΠΎΠ΄ΠΈΡ ΠΊ Π²ΠΎΠ·Π½ΠΈΠΊΠ½ΠΎΠ²Π΅Π½ΠΈΡ Π² ΠΏΡΠΎΠ²ΠΎΠ΄Π½ΠΈΠΊΠ΅ ΠΠΠ‘ ΠΈΠ½Π΄ΡΠΊΡΠΈΠΈ.
ΠΠ°Π½Π½ΠΎΠ΅ ΡΠ²Π»Π΅Π½ΠΈΠ΅ (Π²ΠΎΠ·Π½ΠΈΠΊΠ½ΠΎΠ²Π΅Π½ΠΈΠ΅ ΠΠΠ‘) Π² ΠΏΡΠΎΠ²ΠΎΠ΄Π½ΠΈΠΊΠ΅ ΠΏΡΠΈ ΠΈΠ·ΠΌΠ΅Π½Π΅Π½ΠΈΠΈ ΠΏΡΠΎΡ ΠΎΠ΄ΡΡΠ΅Π³ΠΎ ΠΏΠΎ Π½Π΅ΠΌΡ ΡΠΎΠΊΠ° Π½Π°Π·ΡΠ²Π°Π΅ΡΡΡ ΡΠ°ΠΌΠΎΠΈΠ½Π΄ΡΠΊΡΠΈΠ΅ΠΉ. ΠΠΎΠ·Π½ΠΈΠΊΠ°ΡΡΠ°Ρ Π²ΡΠ»Π΅Π΄ΡΡΠ²ΠΈΠ΅ ΡΠ°ΠΌΠΎΠΈΠ½Π΄ΡΠΊΡΠΈΠΈ ΠΠΠ‘ Π½Π°Π·ΡΠ²Π°Π΅ΡΡΡ ΠΠΠ‘ ΡΠ°ΠΌΠΎΠΈΠ½Π΄ΡΠΊΡΠΈΠΈ ΠL, ΠΊΠΎΡΠΎΡΠ°Ρ ΡΠ°Π²Π½Π°
Π³Π΄Π΅ dΞ¨L β ΠΈΠ·ΠΌΠ΅Π½Π΅Π½ΠΈΠ΅ ΠΏΠΎΡΠΎΠΊΠΎΡΡΠ΅ΠΏΠ»Π΅Π½ΠΈΡ.
Π‘Π»Π΅Π΄ΠΎΠ²Π°ΡΠ΅Π»ΡΠ½ΠΎ ΠΌΠ΅ΠΆΠ΄Ρ ΡΠ»Π΅ΠΊΡΡΠΈΡΠ΅ΡΠΊΠΈΠΌ ΡΠΎΠΊΠΎΠΌ Π² ΠΏΡΠΎΠ²ΠΎΠ΄Π½ΠΈΠΊΠ΅ ΠΈ ΠΏΠΎΡΠΎΠΊΠΎΡΡΠ΅ΠΏΠ»Π΅Π½ΠΈΠ΅ΠΌ, Π²ΠΎΠ·Π½ΠΈΠΊΠ°ΡΡΠ΅Π³ΠΎ Π²ΠΎΠΊΡΡΠ³ ΠΏΡΠΎΠ²ΠΎΠ΄Π½ΠΈΠΊΠ° ΠΌΠ°Π³Π½ΠΈΡΠ½ΠΎΠ³ΠΎ ΠΏΠΎΠ»Ρ ΡΡΡΠ΅ΡΡΠ²ΡΠ΅Ρ Π½Π΅ΠΊΠΎΡΠΎΡΡΠΉ ΠΊΠΎΡΡΡΠΈΡΠΈΠ΅Π½Ρ ΠΏΡΠΎΠΏΠΎΡΡΠΈΠΎΠ½Π°Π»ΡΠ½ΠΎΡΡΠΈ ΡΠ²ΡΠ·ΡΠ²Π°ΡΡΠΈΠΉ ΠΈΡ . Π’Π°ΠΊΠΈΠΌ ΠΊΠΎΡΡΡΠΈΡΠΈΠ΅Π½ΡΠΎΠΌ ΡΠ²Π»ΡΠ΅ΡΡΡ ΠΈΠ½Π΄ΡΠΊΡΠΈΠ²Π½ΠΎΡΡΡ β ΠΎΠ±ΠΎΠ·Π½Π°ΡΠ°Π΅ΡΡΡ L (ΠΈΠΌΠ΅Π΅Ρ ΡΡΠ°ΡΠΎΠ΅ Π½Π°Π·Π²Π°Π½ΠΈΠ΅ ΠΊΠΎΡΡΡΠΈΡΠΈΠ΅Π½Ρ ΡΠ°ΠΌΠΎΠΈΠ½Π΄ΡΠΊΡΠΈΠΈ)
ΠΠ΅Π»ΠΈΡΠΈΠ½Π° ΠΈΠ½Π΄ΡΠΊΡΠΈΠ²Π½ΠΎΡΡΠΈ Ρ Π°ΡΠ°ΠΊΡΠ΅ΡΠΈΠ·ΡΠ΅Ρ ΡΠΏΠΎΡΠΎΠ±Π½ΠΎΡΡΡ ΡΠ»Π΅ΠΊΡΡΠΈΡΠ΅ΡΠΊΠΎΠΉ ΡΠ΅ΠΏΠΈ ΡΠΎΠ·Π΄Π°Π²Π°ΡΡ ΠΏΠΎΡΠΎΠΊΠΎΡΡΠ΅ΠΏΠ»Π΅Π½ΠΈΠ΅ (ΠΌΠ°Π³Π½ΠΈΡΠ½ΡΠΉ ΠΏΠΎΡΠΎΠΊ) ΠΏΡΠΈ ΠΏΡΠΎΡΠ΅ΠΊΠ°Π½ΠΈΠΈ ΠΏΠΎ Π½Π΅ΠΉ ΡΠ»Π΅ΠΊΡΡΠΈΡΠ΅ΡΠΊΠΎΠ³ΠΎ ΡΠΎΠΊΠ°. ΠΠ΄ΠΈΠ½ΠΈΡΠ΅ΠΉ ΠΈΠ½Π΄ΡΠΊΡΠΈΠ²Π½ΠΎΡΡΠΈ ΡΠ²Π»ΡΠ΅ΡΡΡ ΠΠ΅Π½ΡΠΈ (ΠΎΠ±ΠΎΠ·Π½Π°ΡΠ°Π΅ΡΡΡ ΠΠ½)
Π’Π°ΠΊΠΈΠΌ ΠΎΠ±ΡΠ°Π·ΠΎΠΌ, ΠΈΠ½Π΄ΡΠΊΡΠΈΠ²Π½ΠΎΡΡΡ Π·Π°Π²ΠΈΡΠΈΡ ΠΎΡ Π³Π΅ΠΎΠΌΠ΅ΡΡΠΈΡΠ΅ΡΠΊΠΈΡ ΡΠ°Π·ΠΌΠ΅ΡΠΎΠ² ΠΏΡΠΎΠ²ΠΎΠ΄Π½ΠΈΠΊΠ° Ρ ΡΠΎΠΊΠΎΠΌ ΠΈ ΠΎΡ ΠΌΠ°Π³Π½ΠΈΡΠ½ΡΡ ΡΠ²ΠΎΠΉΡΡΠ² ΠΌΠ°Π³Π½ΠΈΡΠ½ΠΎΠΉ ΡΠ΅ΠΏΠΈ, ΡΠ΅ΡΠ΅Π· ΠΊΠΎΡΠΎΡΡΡ Π·Π°ΠΌΡΠΊΠ°Π΅ΡΡΡ ΠΌΠ°Π³Π½ΠΈΡΠ½ΡΠΉ ΠΏΠΎΡΠΎΠΊ, ΡΠΎΠ·Π΄Π°Π²Π°Π΅ΠΌΡΠΉ ΠΏΡΠΎΠ²ΠΎΠ΄Π½ΠΈΠΊΠΎΠΌ Ρ ΡΠΎΠΊΠΎΠΌ.
Π ΡΠ»Π΅Π΄ΡΡΡΠ΅ΠΉ ΡΡΠ°ΡΡΠ΅ Ρ ΡΠ°ΡΡΠΊΠ°ΠΆΡ, ΠΊΠ°ΠΊ ΡΠ°ΡΡΡΠΈΡΠ°ΡΡ ΠΈΠ½Π΄ΡΠΊΡΠΈΠ²Π½ΠΎΡΡΡ ΡΠ°Π·Π»ΠΈΡΠ½ΡΡ ΠΏΠΎ ΡΠΎΡΠΌΠ΅ ΠΏΡΠΎΠ²ΠΎΠ΄Π½ΠΈΠΊΠΎΠ² Ρ ΡΠΎΠΊΠΎΠΌ.
Β
Π§ΡΠΎ ΡΠ°ΠΊΠΎΠ΅ Π²Π·Π°ΠΈΠΌΠ½Π°Ρ ΠΈΠ½Π΄ΡΠΊΡΠΈΡ? ΠΠ·Π°ΠΈΠΌΠ½Π°Ρ ΠΈΠ½Π΄ΡΠΊΡΠΈΠ²Π½ΠΎΡΡΡ
Β
ΠΠ»Ρ ΡΠ°Π·ΡΡΡΠ½Π΅Π½ΠΈΡ ΠΏΠΎΠ½ΡΡΠΈΡ Π²Π·Π°ΠΈΠΌΠ½ΠΎΠΉ ΠΈΠ½Π΄ΡΠΊΡΠΈΠΈ ΡΠ°ΡΡΠΌΠΎΡΡΠΈΠΌ Π΄Π²Π΅ ΠΊΠ°ΡΡΡΠΊΠΈ Π1 ΠΈ Π2 ΡΠ°ΡΠΏΠΎΠ»ΠΎΠΆΠ΅Π½Π½ΡΠ΅ Π±Π»ΠΈΠ·ΠΊΠΎ Π΄ΡΡΠ³ ΠΎΡ Π΄ΡΡΠ³Π°
ΠΠ·Π°ΠΈΠΌΠ½Π°Ρ ΠΈΠ½Π΄ΡΠΊΡΠΈΡ Π΄Π²ΡΡ
ΠΊΠ°ΡΡΡΠ΅ΠΊ ΡΠ°ΡΠΏΠΎΠ»ΠΎΠΆΠ΅Π½Π½ΡΡ
ΡΡΠ΄ΠΎΠΌ.
ΠΡΠ»ΠΈ ΠΏΠΎ ΠΎΠ΄Π½ΠΎΠΉ ΠΈΠ· ΠΊΠ°ΡΡΡΠ΅ΠΊ ΠΏΡΠΎΠΏΡΡΠΊΠ°ΡΡ ΡΠ»Π΅ΠΊΡΡΠΈΡΠ΅ΡΠΊΠΈΠΉ ΡΠΎΠΊ i1, ΡΠΎ Π²ΠΎΠΊΡΡΠ³ Π΄Π°Π½Π½ΠΎΠΉ ΠΊΠ°ΡΡΡΠΊΠΈ Π²ΠΎΠ·Π½ΠΈΠΊΠ½Π΅Ρ ΠΌΠ°Π³Π½ΠΈΡΠ½ΠΎΠ΅ ΠΏΠΎΠ»Π΅ Ρ ΠΏΠΎΡΠΎΠΊΠΎΠΌ Ξ¦1, ΡΠ°ΡΡΡ ΠΌΠ°Π³Π½ΠΈΡΠ½ΡΡ ΡΠΈΠ»ΠΎΠ²ΡΡ Π»ΠΈΠ½ΠΈΠΉ ΠΊΠΎΡΠΎΡΠΎΠ³ΠΎ Π±ΡΠ΄Π΅Ρ ΠΏΠ΅ΡΠ΅ΡΠ΅ΠΊΠ°ΡΡ ΠΈ Π²ΡΠΎΡΡΡ ΠΊΠ°ΡΡΡΠΊΡ, Π²ΠΎΠΊΡΡΠ³ ΠΊΠΎΡΠΎΡΠΎΠΉ ΠΎΠ±ΡΠ°Π·ΡΠ΅ΡΡΡ ΠΌΠ°Π³Π½ΠΈΡΠ½ΡΠΉ ΠΏΠΎΡΠΎΠΊ Ξ¦12. Π’Π°ΠΊΠΈΠΌ ΠΎΠ±ΡΠ°Π·ΠΎΠΌ, ΠΏΡΠΈ ΠΈΠ·ΠΌΠ΅Π½Π΅Π½ΠΈΠΈ ΡΠΎΠΊΠ° i1Π² ΠΏΠ΅ΡΠ²ΠΎΠΉ ΠΊΠ°ΡΡΡΠΊΠ΅ Π±ΡΠ΄Π΅Ρ ΠΈΠ·ΠΌΠ΅Π½ΡΡΡΡΡ ΠΌΠ°Π³Π½ΠΈΡΠ½ΡΠΉ ΠΏΠΎΡΠΎΠΊ Ξ¦1, Π°, ΡΠ»Π΅Π΄ΠΎΠ²Π°ΡΠ΅Π»ΡΠ½ΠΎ, ΠΈ ΠΌΠ°Π³Π½ΠΈΡΠ½ΡΠΉ ΠΏΠΎΡΠΎΠΊ Ξ¦12, ΠΏΠ΅ΡΠ΅ΡΠ΅ΠΊΠ°ΡΡΠΈΠΉ Π²ΡΠΎΡΡΡ ΠΊΠ°ΡΡΡΠΊΡ, ΡΡΠΎ Π½Π΅ΠΏΡΠ΅ΠΌΠ΅Π½Π½ΠΎ ΠΏΡΠΈΠ²Π΅Π΄ΡΡ ΠΊ ΠΈΠ·ΠΌΠ΅Π½Π΅Π½ΠΈΡ ΡΠ»Π΅ΠΊΡΡΠΈΡΠ΅ΡΠΊΠΎΠ³ΠΎ ΡΠΎΠΊΠ° Π²ΠΎ Π²ΡΠΎΡΠΎΠΉ ΠΊΠ°ΡΡΡΠΊΠ΅ ΠΈ ΡΠΎΠΎΡΠ²Π΅ΡΡΡΠ²Π΅Π½Π½ΠΎ Π²ΠΎΠ·Π½ΠΈΠΊΠ½ΠΎΠ²Π΅Π½ΠΈΡ ΠΠΠ‘.
Π’Π°ΠΊΠΈΠΌ ΠΎΠ±ΡΠ°Π·ΠΎΠΌ, Π²ΠΎΠ·Π½ΠΈΠΊΠ½ΠΎΠ²Π΅Π½ΠΈΠ΅ ΠΠΠ‘ Π² ΠΊΠΎΠ½ΡΡΡΠ΅ ΠΏΠΎΠ΄ Π΄Π΅ΠΉΡΡΠ²ΠΈΠ΅ΠΌ ΠΈΠ·ΠΌΠ΅Π½ΡΡΡΠ΅Π³ΠΎΡΡ ΡΠΎΠΊΠ° Π² Π±Π»ΠΈΠ·ΠΊΠΎΡΠ°ΡΠΏΠΎΠ»ΠΎΠΆΠ΅Π½Π½ΠΎΠΌ ΡΠΎΡΠ΅Π΄Π½Π΅ΠΉ ΠΊΠ°ΡΡΡΠΊΠ΅, ΠΈΠΌΠ΅Π΅Ρ Π½Π°Π·Π²Π°Π½ΠΈΠ΅ Π²Π·Π°ΠΈΠΌΠ½ΠΎΠΉ ΠΈΠ½Π΄ΡΠΊΡΠΈΠΈ.
ΠΠ°ΠΊ Π±ΡΠ»ΠΎ ΡΠΊΠ°Π·Π°Π½ΠΎ Π²ΡΡΠ΅, ΡΠ²Π»Π΅Π½ΠΈΠ΅ ΡΠ°ΠΌΠΎΠΈΠ½Π΄ΡΠΊΡΠΈΠΈ Π² ΠΊΠΎΠ»ΠΈΡΠ΅ΡΡΠ²Π΅Π½Π½ΠΎΠΉ ΡΠΎΡΠΌΠ΅ Π²ΡΡΠ°ΠΆΠ°Π΅ΡΡΡ ΠΈΠ½Π΄ΡΠΊΡΠΈΠ²Π½ΠΎΡΡΡΡ L, Π°Π½Π°Π»ΠΎΠ³ΠΈΡΠ½ΠΎ ΠΈ Π²Π·Π°ΠΈΠΌΠ½Π°Ρ ΠΈΠ½Π΄ΡΠΊΡΠΈΡ ΠΎΠΏΡΠ΅Π΄Π΅Π»ΡΠ΅ΡΡΡ ΡΠΈΠ·ΠΈΡΠ΅ΡΠΊΠΎΠΉ Π²Π΅Π»ΠΈΡΠΈΠ½ΠΎΠΉ Π½Π°Π·ΡΠ²Π°Π΅ΠΌΠΎΠΉ Π²Π·Π°ΠΈΠΌΠ½ΠΎΠΉ ΠΈΠ½Π΄ΡΠΊΡΠΈΠ²Π½ΠΎΡΡΡΡ Π (ΠΈΠΌΠ΅Π΅Ρ ΡΠ°Π·ΠΌΠ΅ΡΠ½ΠΎΡΡΡ ΠΠ΅Π½ΡΠΈ β Β«ΠΠ½Β»). ΠΠ°Π½Π½Π°Ρ Π²Π΅Π»ΠΈΡΠΈΠ½Π° ΠΎΠΏΡΠ΅Π΄Π΅Π»ΡΠ΅ΡΡΡ ΠΎΡΠ½ΠΎΡΠ΅Π½ΠΈΠ΅ΠΌ ΠΏΠΎΡΠΎΠΊΠΎΡΡΠ΅ΠΏΠ»Π΅Π½ΠΈΡ Π²ΠΎ Π²ΡΠΎΡΠΈΡΠ½ΠΎΠΉ ΠΊΠ°ΡΡΡΠΊΠ΅ Ξ¨12Β ΠΊ ΡΠΎΠΊΡ Π² ΠΏΠ΅ΡΠ²ΠΈΡΠ½ΠΎΠΉ ΠΊΠ°ΡΡΡΠΊΠ΅ i1
ΠΠ΄Π½Π°ΠΊΠΎ, ΠΎΠΏΡΠ΅Π΄Π΅Π»ΠΈΡΡ Π²Π·Π°ΠΈΠΌΠ½ΡΡ ΠΈΠ½Π΄ΡΠΊΡΠΈΡ ΠΌΠΎΠΆΠ½ΠΎ ΠΈ ΠΎΠ±ΡΠ°ΡΠ½ΡΠΌ ΡΠΏΠΎΡΠΎΠ±ΠΎΠΌ, ΡΠΎ Π΅ΡΡΡ ΠΏΡΠΎΠΏΡΡΠΊΠ°Ρ ΡΠΎΠΊ i2Β ΡΠ΅ΡΠ΅Π· Π²ΡΠΎΡΠΈΡΠ½ΡΡ ΠΊΠ°ΡΡΡΠΊΡ. Π ΡΡΠΎΠΌ ΡΠ»ΡΡΠ°Π΅ Π±ΡΠ΄Π΅Ρ ΡΠΎΠ·Π΄Π°Π²Π°ΡΡΡΡ ΠΌΠ°Π³Π½ΠΈΡΠ½ΡΠΉ ΠΏΠΎΡΠΎΠΊ Ξ¦2, ΡΠ°ΡΡΡ ΠΊΠΎΡΠΎΡΠΎΠ³ΠΎ Ξ¦21 Π±ΡΠ΄Π΅Ρ ΠΏΡΠΎΠ½ΠΈΠ·ΡΠ²Π°ΡΡ ΠΏΠ΅ΡΠ²ΠΈΡΠ½ΡΡ ΠΊΠ°ΡΡΡΠΊΡ, ΡΠΎΠ³Π΄Π° Π²Π·Π°ΠΈΠΌΠ½Π°Ρ ΠΈΠ½Π΄ΡΠΊΡΠΈΡ Π±ΡΠ΄Π΅Ρ ΠΎΠΏΡΠ΅Π΄Π΅Π»ΡΡΡΡΡ ΡΠ»Π΅Π΄ΡΡΡΠΈΠΌ Π²ΡΡΠ°ΠΆΠ΅Π½ΠΈΠ΅ΠΌ
Π’Π°ΠΊ ΠΆΠ΅ ΠΊΠ°ΠΊ ΠΈ Π² ΡΠ»ΡΡΠ°Π΅ Ρ ΡΠ°ΠΌΠΎΠΈΠ½Π΄ΡΠΊΡΠΈΠ΅ΠΉ, ΠΠΠ‘ Π²Π·Π°ΠΈΠΌΠ½ΠΎΠΉ ΠΈΠ½Π΄ΡΠΊΡΠΈΠΈ Π²ΠΎ Π²ΡΠΎΡΠΈΡΠ½ΠΎΠΉ ΠΊΠ°ΡΡΡΠΊΠ΅ Π±ΡΠ΄Π΅Ρ Π·Π°Π²ΠΈΡΠ΅ΡΡ ΠΎΡ ΡΠΊΠΎΡΠΎΡΡΠΈ ΠΈΠ·ΠΌΠ΅Π½Π΅Π½ΠΈΡ ΠΌΠ°Π³Π½ΠΈΡΠ½ΠΎΠ³ΠΎ ΠΏΠΎΡΠΎΠΊΠ° ΠΈΠ»ΠΈ ΠΏΠΎΡΠΎΠΊΠΎΡΡΠ΅ΠΏΠ»Π΅Π½ΠΈΡ
ΠΠ·Π°ΠΈΠΌΠ½Π°Ρ ΠΈΠ½Π΄ΡΠΊΡΠΈΠ²Π½ΠΎΡΡΡ Π ΠΈΠΌΠ΅Π΅Ρ Π·Π°Π²ΠΈΡΠΈΠΌΠΎΡΡΡ ΠΎΡ ΠΈΠ½Π΄ΡΠΊΡΠΈΠ²Π½ΠΎΡΡΠΈ Π΄Π²ΡΡ ΠΊΠ°ΡΡΡΠ΅ΠΊ ΠΈ ΠΎΠΏΡΠ΅Π΄Π΅Π»ΡΠ΅ΡΡΡ ΡΠΎΠ³Π»Π°ΡΠ½ΠΎ ΡΠ»Π΅Π΄ΡΡΡΠ΅ΠΌΡ Π²ΡΡΠ°ΠΆΠ΅Π½ΠΈΡ
Π³Π΄Π΅ kΒ β ΠΊΠΎΡΡΡΠΈΡΠΈΠ΅Π½Ρ ΡΠ²ΡΠ·ΠΈ, Π·Π°Π²ΠΈΡΡΡΠΈΠΉ ΠΎΡ ΡΡΠ΅ΠΏΠ΅Π½ΠΈ ΠΈΠ½Π΄ΡΠΊΡΠΈΠ²Π½ΠΎΠΉ ΡΠ²ΡΠ·ΠΈ ΠΌΠ΅ΠΆΠ΄Ρ ΠΊΠ°ΡΡΡΠΊΠ°ΠΌΠΈ;
L1Β β ΠΈΠ½Π΄ΡΠΊΡΠΈΠ²Π½ΠΎΡΡΡ ΠΏΠ΅ΡΠ²ΠΎΠΉ ΠΊΠ°ΡΡΡΠΊΠΈ;
L2Β β ΠΈΠ½Π΄ΡΠΊΡΠΈΠ²Π½ΠΎΡΡΡ Π²ΡΠΎΡΠΎΠΉ ΠΊΠ°ΡΡΡΠΊΠΈ.
ΠΠΎΡΡΡΠΈΡΠΈΠ΅Π½Ρ ΠΈΠ½Π΄ΡΠΊΡΠΈΠ²Π½ΠΎΠΉ ΡΠ²ΡΠ·ΠΈ kΒ ΠΎΠΏΡΠ΅Π΄Π΅Π»ΡΠ΅ΡΡΡ ΡΠ»Π΅Π΄ΡΡΡΠΈΠΌ Π²ΡΡΠ°ΠΆΠ΅Π½ΠΈΠ΅ΠΌ
ΠΠ· Π΄Π°Π½Π½ΠΎΠ³ΠΎ Π²ΡΡΠ°ΠΆΠ΅Π½ΠΈΡ Π²ΠΈΠ΄Π½ΠΎ, ΡΡΠΎ ΠΊΠΎΡΡΡΠΈΡΠΈΠ΅Π½Ρ ΡΠ²ΡΠ·ΠΈ Π²ΡΠ΅Π³Π΄Π° Π±ΡΠ΄Π΅Ρ ΠΌΠ΅Π½ΡΡΠ΅ Π΅Π΄ΠΈΠ½ΠΈΡΡ, ΡΠ°ΠΊ ΠΊΠ°ΠΊ Ξ¦12< Ξ¦1 ΠΈ Ξ¦21< Ξ¦2.
Π’Π΅ΠΎΡΠΈΡ ΡΡΠΎ Ρ ΠΎΡΠΎΡΠΎ, Π½ΠΎ Π½Π΅ΠΎΠ±Ρ ΠΎΠ΄ΠΈΠΌΠΎ ΠΎΡΡΠ°Π±Π°ΡΡΠ²Π°ΡΡ ΡΡΠΎ Π²ΡΡ ΠΏΡΠ°ΠΊΡΠΈΡΠ΅ΡΠΊΠΈ ΠΠΠΠ ΠΠΠ«ΠΠΠ’Π¬ ΠΠΠΠΠ ΠΠΠΠ‘Π¬
ΠΠ΄Ρ ΠΈΠ½Π΄ΡΠΊΡΠΈΠΈ Π² Π·Π°ΠΌΠΊΠ½ΡΡΠΎΠΌ ΠΊΠΎΠ½ΡΡΡΠ΅ ΡΡΠΎ
Π‘ΠΎΠ΄Π΅ΡΠΆΠ°Π½ΠΈΠ΅
- Π―Π²Π»Π΅Π½ΠΈΠ΅ ΡΠ»Π΅ΠΊΡΡΠΎΠΌΠ°Π³Π½ΠΈΡΠ½ΠΎΠΉ ΠΈΠ½Π΄ΡΠΊΡΠΈΠΈ
- ΠΠ°Π³Π½ΠΈΡΠ½ΡΠΉ ΠΏΠΎΡΠΎΠΊ
- ΠΠ°ΠΊΠΎΠ½ ΡΠ»Π΅ΠΊΡΡΠΎΠΌΠ°Π³Π½ΠΈΡΠ½ΠΎΠΉ ΠΈΠ½Π΄ΡΠΊΡΠΈΠΈ Π€Π°ΡΠ°Π΄Π΅Ρ
- ΠΡΠ°Π²ΠΈΠ»ΠΎ ΠΠ΅Π½ΡΠ°
- Π‘Π°ΠΌΠΎΠΈΠ½Π΄ΡΠΊΡΠΈΡ
- ΠΠ½Π΄ΡΠΊΡΠΈΠ²Π½ΠΎΡΡΡ
- ΠΠ½Π΅ΡΠ³ΠΈΡ ΠΌΠ°Π³Π½ΠΈΡΠ½ΠΎΠ³ΠΎ ΠΏΠΎΠ»Ρ
- ΠΡΠ½ΠΎΠ²Π½ΡΠ΅ ΡΠΎΡΠΌΡΠ»Ρ ΡΠ°Π·Π΄Π΅Π»Π° Β«ΠΠ»Π΅ΠΊΡΡΠΎΠΌΠ°Π³Π½ΠΈΡΠ½Π°Ρ ΠΈΠ½Π΄ΡΠΊΡΠΈΡΒ»
- ΠΠ±ΠΎΠ·Π½Π°ΡΠ΅Π½ΠΈΠ΅ ΠΈ Π΅Π΄ΠΈΠ½ΠΈΡΡ ΠΈΠ·ΠΌΠ΅ΡΠ΅Π½ΠΈΡ
ΠΠ»Π΅ΠΊΡΡΠΎΠΌΠ°Π³Π½ΠΈΡΠ½Π°Ρ ΠΈΠ½Π΄ΡΠΊΡΠΈΡ (ΠΈΠ½Π΄ΡΠΊΡΠΈΡ Π·Π½Π°ΡΠΈΡ Π½Π°Π²Π΅Π΄Π΅Π½ΠΈΠ΅) ΡΡΠΎ ΡΠ²Π»Π΅Π½ΠΈΠ΅, ΠΏΡΠΈ ΠΊΠΎΡΠΎΡΠΎΠΌ Π² Π·Π°ΠΌΠΊΠ½ΡΡΠΎΠΌ ΠΊΠΎΠ½ΡΡΡΠ΅ Π²ΠΎΠ·Π½ΠΈΠΊΠ°Π΅Ρ ΡΠ»Π΅ΠΊΡΡΠΈΡΠ΅ΡΠΊΠΈΠΉ ΡΠΎΠΊ ΠΏΡΠΈ ΠΈΠ·ΠΌΠ΅Π½Π΅Π½ΠΈΠΈ ΠΌΠ°Π³Π½ΠΈΡΠ½ΠΎΠ³ΠΎ ΠΏΠΎΡΠΎΠΊΠ°, ΠΏΡΠΎΠ½ΠΈΠ·ΡΠ²Π°ΡΡΠ΅Π³ΠΎ Π΅Π³ΠΎ.
Π―Π²Π»Π΅Π½ΠΈΠ΅ ΡΠ»Π΅ΠΊΡΡΠΎΠΌΠ°Π³Π½ΠΈΡΠ½ΠΎΠΉ ΠΈΠ½Π΄ΡΠΊΡΠΈΠΈ Π±ΡΠ»ΠΎ ΠΎΠ±Π½Π°ΡΡΠΆΠ΅Π½ΠΎ Π² 1831 Π³.
Π. Π€Π°ΡΠ°Π΄Π΅Π΅ΠΌ. Π’ΠΎΠΊ, Π²ΠΎΠ·Π½ΠΈΠΊΠ°ΡΡΠΈΠΉ ΠΏΡΠΈ ΡΠ»Π΅ΠΊΡΡΠΎΠΌΠ°Π³Π½ΠΈΡΠ½ΠΎΠΉ ΠΈΠ½Π΄ΡΠΊΡΠΈΠΈ Π½Π°Π·ΡΠ²Π°ΡΡ ΠΈΠ½Π΄ΡΠΊΡΠΈΠΎΠ½Π½ΡΠΌ.
ΠΠ°ΠΊΠΎΠ½ ΡΠ»Π΅ΠΊΡΡΠΎΠΌΠ°Π³Π½ΠΈΡΠ½ΠΎΠΉ ΠΈΠ½Π΄ΡΠΊΡΠΈΠΈΠΠΠ‘ ΠΈΠ½Π΄ΡΠΊΡΠΈΠΈ Π² ΠΊΠΎΠ½ΡΡΡΠ΅ ΡΠ°Π²Π½Π° ΡΠΊΠΎΡΠΎΡΡΠΈ ΠΈΠ·ΠΌΠ΅Π½Π΅Π½ΠΈΡ ΠΌΠ°Π³Π½ΠΈΡΠ½ΠΎΠ³ΠΎ ΠΏΠΎΠ»Ρ ΡΠΊΠ²ΠΎΠ·Ρ ΠΏΠΎΠ²Π΅ΡΡ Π½ΠΎΡΡΡ, ΠΎΠ³ΡΠ°Π½ΠΈΡΠ΅Π½Π½ΡΡ ΠΊΠΎΠ½ΡΡΡΠΎΠΌ.
ΠΠ»Π΅ΠΊΡΡΠΎΠΌΠ°Π³Π½ΠΈΡΠ½Π°Ρ ΠΈΠ½Π΄ΡΠΊΡΠΈΡ | |
1831 Π³. β Π. Π€Π°ΡΠ°Π΄Π΅ΠΉ ΠΎΠ±Π½Π°ΡΡΠΆΠΈΠ», ΡΡΠΎ Π² Π·Π°ΠΌΠΊΠ½ΡΡΠΎΠΌ ΠΏΡΠΎΠ²ΠΎΠ΄ΡΡΠ΅ΠΌ ΠΊΠΎΠ½ΡΡΡΠ΅ ΠΏΡΠΈ ΠΈΠ·ΠΌΠ΅Π½Π΅Π½ΠΈΠΈ ΠΌΠ°Π³Π½ΠΈΡΠ½ΠΎΠ³ΠΎ ΠΏΠΎΠ»Ρ Π²ΠΎΠ·Π½ΠΈΠΊΠ°Π΅Ρ ΡΠ°ΠΊ Π½Π°Π·ΡΠ²Π°Π΅ΠΌΡΠΉ ΠΈΠ½Π΄ΡΠΊΡΠΈΠΎΠ½Π½ΡΠΉ ΡΠΎΠΊ. (ΠΠ½Π΄ΡΠΊΡΠΈΡ, Π² Π΄Π°Π½Π½ΠΎΠΌ ΡΠ»ΡΡΠ°Π΅, β ΠΏΠΎΡΠ²Π»Π΅Π½ΠΈΠ΅, Π²ΠΎΠ·Π½ΠΈΠΊΠ½ΠΎΠ²Π΅Π½ΠΈΠ΅). | |
ΠΠ½Π΄ΡΠΊΡΠΈΠΎΠ½Π½ΡΠΉ ΡΠΎΠΊ Π² ΠΊΠ°ΡΡΡΠΊΠ΅ Π²ΠΎΠ·Π½ΠΈΠΊΠ°Π΅Ρ ΠΏΡΠΈ ΠΏΠ΅ΡΠ΅ΠΌΠ΅ΡΠ΅Π½ΠΈΠΈ ΠΏΠΎΡΡΠΎΡΠ½Π½ΠΎΠ³ΠΎ ΠΌΠ°Π³Π½ΠΈΡΠ° ΠΎΡΠ½ΠΎΡΠΈΡΠ΅Π»ΡΠ½ΠΎ ΠΊΠ°ΡΡΡΠΊΠΈ; ΠΏΡΠΈ ΠΏΠ΅ΡΠ΅ΠΌΠ΅ΡΠ΅Π½ΠΈΠΈ ΡΠ»Π΅ΠΊΡΡΠΎΠΌΠ°Π³Π½ΠΈΡΠ° ΠΎΡΠ½ΠΎΡΠΈΡΠ΅Π»ΡΠ½ΠΎ ΠΊΠ°ΡΡΡΠΊΠΈ; ΠΏΡΠΈ ΠΏΠ΅ΡΠ΅ΠΌΠ΅ΡΠ΅Π½ΠΈΠΈ ΡΠ΅ΡΠ΄Π΅ΡΠ½ΠΈΠΊΠ° ΠΎΡΠ½ΠΎΡΠΈΡΠ΅Π»ΡΠ½ΠΎ ΡΠ»Π΅ΠΊΡΡΠΎΠΌΠ°Π³Π½ΠΈΡΠ°, Π²ΡΡΠ°Π²Π»Π΅Π½Π½ΠΎΠ³ΠΎ Π² ΠΊΠ°ΡΡΡΠΊΡ; ΠΏΡΠΈ ΡΠ΅Π³ΡΠ»ΠΈΡΠΎΠ²Π°Π½ΠΈΠΈ ΡΠΎΠΊΠ° Π² ΡΠ΅ΠΏΠΈ ΡΠ»Π΅ΠΊΡΡΠΎΠΌΠ°Π³Π½ΠΈΡΠ°; ΠΏΡΠΈ Π·Π°ΠΌΡΠΊΠ°Π½ΠΈΠΈ ΠΈ ΡΠ°Π·ΠΌΡΠΊΠ°Π½ΠΈΠΈ ΡΠ΅ΠΏΠΈ | |
ΠΠΎΡΠ²Π»Π΅Π½ΠΈΠ΅ ΡΠΎΠΊΠ° Π² Π·Π°ΠΌΠΊΠ½ΡΡΠΎΠΌ ΠΊΠΎΠ½ΡΡΡΠ΅ ΠΏΡΠΈ ΠΈΠ·ΠΌΠ΅Π½Π΅Π½ΠΈΠΈ ΠΌΠ°Π³Π½ΠΈΡΒΠ½ΠΎΠ³ΠΎ ΠΏΠΎΠ»Ρ, ΠΏΡΠΎΠ½ΠΈΠ·ΡΠ²Π°ΡΡΠ΅Π³ΠΎ ΠΊΠΎΠ½ΡΡΡ, ΡΠ²ΠΈΠ΄Π΅ΡΠ΅Π»ΡΡΡΠ²ΡΠ΅Ρ ΠΎ Π΄Π΅ΠΉΡΡΠ²ΠΈΠΈ Π² ΠΊΠΎΠ½ΡΡΡΠ΅ ΡΡΠΎΡΠΎΠ½Π½ΠΈΡ ΡΠΈΠ» (ΠΈΠ»ΠΈ ΠΎ Π²ΠΎΠ·Π½ΠΈΠΊΠ½ΠΎΒΠ²Π΅Π½ΠΈΠΈ ΠΠΠ‘ ΠΈΠ½Π΄ΡΠΊΡΠΈΠΈ). Π―Π²Π»Π΅Π½ΠΈΠ΅ Π²ΠΎΠ·Π½ΠΈΠΊΠ½ΠΎΠ²Π΅Π½ΠΈΡ ΠΠΠ‘ Π² Π·Π°ΠΌΠΊΠ½ΡΡΠΎΠΌ ΠΏΡΠΎΠ²ΠΎΠ΄ΡΡΠ΅ΠΌ ΠΊΠΎΠ½ΡΡΡΠ΅ ΠΏΡΠΈ ΠΈΠ·ΠΌΠ΅Π½Π΅Π½ΠΈΠΈ ΠΌΠ°Π³Π½ΠΈΡΠ½ΠΎΠ³ΠΎ ΠΏΠΎΠ»Ρ (ΠΏΠΎΡΠΎΠΊΠ°), ΠΏΡΠΎΠ½ΠΈΠ·ΡΠ²Π°ΡΡΠ΅Π³ΠΎ ΠΊΠΎΠ½ΡΡΡ, Π½Π°Π·ΡΒΠ²Π°Π΅ΡΡΡ ΡΠ»Π΅ΠΊΡΡΠΎΠΌΠ°Π³Π½ΠΈΡΠ½ΠΎΠΉ ΠΈΠ½Π΄ΡΠΊΡΠΈΠ΅ΠΉ. ΠΠ»ΠΈ:ΡΠ²Π»Π΅Π½ΠΈΠ΅ Π²ΠΎΠ·Π½ΠΈΠΊΠ½ΠΎΠ²Π΅Π½ΠΈΡ ΡΠ»Π΅ΠΊΡΡΠΈΡΠ΅ΡΠΊΠΎΠ³ΠΎ ΠΏΠΎΠ»Ρ ΠΏΡΠΈ ΠΈΠ·ΠΌΠ΅Π½Π΅Π½ΠΈΠΈ ΠΌΠ°Π³Π½ΠΈΡΠ½ΠΎΠ³ΠΎ ΠΏΠΎΠ»Ρ (ΠΏΠΎΡΠΎΠΊΠ°), Π½Π°Π·ΡΠ²Π°Π΅ΡΡΡ ΡΠ»Π΅ΠΊΡΡΠΎΠΌΠ°Π³Π½ΠΈΡΠ½ΠΎΠΉ ΠΈΠ½Π΄ΡΠΊΡΠΈΠ΅ΠΉ. | |
ΠΠ°ΠΊΠΎΠ½ ΡΠ»Π΅ΠΊΡΡΠΎΠΌΠ°Π³Π½ΠΈΡΠ½ΠΎΠΉ ΠΈΠ½Π΄ΡΠΊΡΠΈΠΈ ΠΡΠΈ Π²ΡΡΠΊΠΎΠΌ ΠΈΠ·ΠΌΠ΅Π½Π΅Π½ΠΈΠΈ ΠΌΠ°Π³Π½ΠΈΡΠ½ΠΎΠ³ΠΎ ΠΏΠΎΡΠΎΠΊΠ° ΡΠ΅ΡΠ΅Π· ΠΏΡΠΎΠ²ΠΎΠ΄ΡΡΠΈΠΉ Π·Π°ΠΌΠΊΠ½ΡΡΡΠΉ ΠΊΠΎΠ½ΡΡΡ Π² ΡΡΠΎΠΌ ΠΊΠΎΠ½ΡΡΡΠ΅ Π²ΠΎΠ·Π½ΠΈΠΊΠ°Π΅Ρ ΡΠ»Π΅ΠΊΡΡΠΈΡΠ΅ΡΠΊΠΈΠΉ ΡΠΎΠΊ. I Π·Π°Π²ΠΈΡΠΈΡ ΠΎΡ ΡΠ²ΠΎΠΉΡΡΠ² ΠΊΠΎΠ½ΡΡΡΠ° (ΡΠΎΠΏΡΠΎΡΠΈΠ²Π»Π΅Π½ΠΈΠ΅): . e Π½Π΅ Π·Π°Π²ΠΈΡΠΈΡ ΠΎΡ ΡΠ²ΠΎΠΉΡΡΠ² ΠΊΠΎΠ½ΡΡΡΠ°: . ΠΠΠ‘ ΠΈΠ½Π΄ΡΠΊΡΠΈΠΈ Π² Π·Π°ΠΌΠΊΠ½ΡΡΠΎΠΌ ΠΊΠΎΠ½ΡΡΡΠ΅ ΠΏΡΡΠΌΠΎ ΠΏΡΠΎΠΏΠΎΡΡΠΈΠΎΠ½Π°Π»ΡΠ½Π° ΡΠΊΠΎΡΠΎΡΡΠΈ ΠΈΠ·ΠΌΠ΅Π½Π΅Π½ΠΈΡ ΠΌΠ°Π³Π½ΠΈΡΠ½ΠΎΠ³ΠΎ ΠΏΠΎΡΠΎΠΊΠ° ΡΠ΅ΡΠ΅Π· ΠΏΠ»ΠΎΡΠ°Π΄Ρ, ΠΎΠ³ΡΠ°Π½ΠΈΡΠ΅Π½Π½ΡΡ ΡΡΠΈΠΌ ΠΊΠΎΠ½ΡΡΡΠΎΠΌ. | |
ΠΡΠ½ΠΎΠ²Π½ΡΠ΅ ΠΏΡΠΈΠΌΠ΅Π½Π΅Π½ΠΈΡ ΡΠ»Π΅ΠΊΡΡΠΎΠΌΠ°Π³Π½ΠΈΡΠ½ΠΎΠΉ ΠΈΠ½Π΄ΡΠΊΡΠΈΠΈ: Π³Π΅Π½Π΅ΡΠΈΡΠΎΠ²Π°Π½ΠΈΠ΅ ΡΠΎΠΊΠ° (ΠΈΠ½Π΄ΡΠΊΡΠΈΠΎΠ½Π½ΡΠ΅ Π³Π΅Π½Π΅ΡΠ°ΡΠΎΡΡ Π½Π° Π²ΡΠ΅Ρ ΡΠ»Π΅ΠΊΡΡΠΎΡΡΠ°Π½ΡΠΈΡΡ , Π΄ΠΈΠ½Π°ΠΌΠΎΠΌΠ°ΡΠΈΠ½Ρ), ΡΡΠ°Π½ΡΡΠΎΡΠΌΠ°ΡΠΎΡΡ. |
ΠΡΠ°Π²ΠΈΠ»ΠΎ ΠΠ΅Π½ΡΠ° ΠΠΎΠ·Π½ΠΈΠΊΠ½ΠΎΠ²Π΅Π½ΠΈΠ΅ ΠΈΠ½Π΄ΡΠΊΡΠΈΠΎΠ½Π½ΠΎΠ³ΠΎ ΡΠΎΠΊΠ° β ΡΠ»Π΅Π΄ΡΡΠ²ΠΈΠ΅ Π·Π°ΠΊΠΎΠ½Π° ΡΠΎΡ ΡΠ°Π½Π΅Π½ΠΈΡ ΡΠ½Π΅ΡΠ³ΠΈΠΈ! Π ΡΠ»ΡΡΠ°Π΅ 1: ΠΡΠΈ ΠΏΡΠΈΠ±Π»ΠΈΠΆΠ΅Π½ΠΈΠΈ ΠΌΠ°Π³Π½ΠΈΡΠ°, ΡΠ²Π΅Π»ΠΈΡΠ΅Π½ΠΈΠΈ ΡΠΎΠΊΠ°, Π·Π°ΠΌΡΠΊΠ°Π½ΠΈΠΈ ΡΠ΅ΠΏΠΈ: ; ΠΠ°Π³Π½ΠΈΡΠ½ΡΠΉ ΠΏΠΎΡΠΎΠΊ Π€Β β ΞΠ€>0. Π§ΡΠΎΠ±Ρ ΠΊΠΎΠΌΠΏΠ΅Π½ΡΠΈΡΠΎΠ²Π°ΡΡ ΡΡΠΎ ΠΈΠ·ΠΌΠ΅Π½Π΅Π½ΠΈΠ΅ (ΡΠ²Π΅Π»ΠΈΡΠ΅Π½ΠΈΠ΅) Π²Π½Π΅ΡΠ½Π΅Π³ΠΎ ΠΏΠΎΠ»Ρ, Π½Π΅ΠΎΠ±Ρ ΠΎΠ΄ΠΈΠΌΠΎ ΠΌΠ°Π³Π½ΠΈΡΠ½ΠΎΠ΅ ΠΏΠΎΠ»Π΅, Π½Π°ΠΏΡΠ°Π²Π»Π΅Π½Π½ΠΎΠ΅ Π² ΡΡΠΎΡΠΎΠ½Ρ, ΠΏΡΠΎΡΠΈΠ²ΠΎΠΏΠΎΠ»ΠΎΠΆΠ½ΡΡ Π²Π½Π΅ΡΠ½Π΅ΠΌΡ ΠΏΠΎΠ»Ρ: , Π³Π΄Π΅ β Ρ.Π½. ΠΈΠ½Π΄ΡΠΊΡΠΈΠΎΠ½Π½ΠΎΠ΅ ΠΌΠ°Π³Π½ΠΈΡΠ½ΠΎΠ΅ ΠΏΠΎΠ»Π΅. Π ΡΠ»ΡΡΠ°Π΅ 2: ΠΏΡΠΈ ΡΠ΄Π°Π»Π΅Π½ΠΈΠΈ ΠΌΠ°Π³Π½ΠΈΡΠ°, ΡΠΌΠ΅Π½ΡΡΠ΅Π½ΠΈΠΈ ΡΠΎΠΊΠ°, ΡΠ°Π·ΠΌΡΠΊΠ°Π½ΠΈΠΈ ΡΠ΅ΠΏΠΈ: . ΠΠ°Π³Π½ΠΈΡΠ½ΡΠΉ ΠΏΠΎΡΠΎΠΊ Π€ β ΞΠ€ 0). Π’ΠΎΠΊ Π² ΠΊΠΎΠ½ΡΡΡΠ΅ ΠΈΠΌΠ΅Π΅Ρ ΠΏΠΎΠ»ΠΎΠΆΠΈΡΠ΅Π»ΡΠ½ΠΎΠ΅ Π½Π°ΠΏΡΠ°Π²Π»Π΅Π½ΠΈΠ΅ ( ), Π΅ΡΠ»ΠΈ ΡΠΎΠ²ΠΏΠ°ΒΠ΄Π°Π΅Ρ Ρ , (Ρ.Π΅. ΞΞ¦ |
ΠΠ΅ Π½Π°ΡΠ»ΠΈ ΡΠΎ, ΡΡΠΎ ΠΈΡΠΊΠ°Π»ΠΈ? ΠΠΎΡΠΏΠΎΠ»ΡΠ·ΡΠΉΡΠ΅ΡΡ ΠΏΠΎΠΈΡΠΊΠΎΠΌ:
Π―Π²Π»Π΅Π½ΠΈΠ΅ ΡΠ»Π΅ΠΊΡΡΠΎΠΌΠ°Π³Π½ΠΈΡΠ½ΠΎΠΉ ΠΈΠ½Π΄ΡΠΊΡΠΈΠΈ
ΠΠ»Π΅ΠΊΡΡΠΎΠΌΠ°Π³Π½ΠΈΡΠ½Π°Ρ ΠΈΠ½Π΄ΡΠΊΡΠΈΡ β ΡΠ²Π»Π΅Π½ΠΈΠ΅ Π²ΠΎΠ·Π½ΠΈΠΊΠ½ΠΎΠ²Π΅Π½ΠΈΡ ΡΠΎΠΊΠ° Π² Π·Π°ΠΌΠΊΠ½ΡΡΠΎΠΌ ΠΏΡΠΎΠ²ΠΎΠ΄ΡΡΠ΅ΠΌ ΠΊΠΎΠ½ΡΡΡΠ΅ ΠΏΡΠΈ ΠΈΠ·ΠΌΠ΅Π½Π΅Π½ΠΈΠΈ ΠΌΠ°Π³Π½ΠΈΡΠ½ΠΎΠ³ΠΎ ΠΏΠΎΡΠΎΠΊΠ°, ΠΏΡΠΎΠ½ΠΈΠ·ΡΠ²Π°ΡΡΠ΅Π³ΠΎ Π΅Π³ΠΎ.
Π―Π²Π»Π΅Π½ΠΈΠ΅ ΡΠ»Π΅ΠΊΡΡΠΎΠΌΠ°Π³Π½ΠΈΡΠ½ΠΎΠΉ ΠΈΠ½Π΄ΡΠΊΡΠΈΠΈ Π±ΡΠ»ΠΎ ΠΎΡΠΊΡΡΡΠΎ Π. Π€Π°ΡΠ°Π΄Π΅Π΅ΠΌ.
- ΠΠ° ΠΎΠ΄Π½Ρ Π½Π΅ΠΏΡΠΎΠ²ΠΎΠ΄ΡΡΡΡ ΠΎΡΠ½ΠΎΠ²Ρ Π±ΡΠ»ΠΈ Π½Π°ΠΌΠΎΡΠ°Π½Ρ Π΄Π²Π΅ ΠΊΠ°ΡΡΡΠΊΠΈ: Π²ΠΈΡΠΊΠΈ ΠΏΠ΅ΡΠ²ΠΎΠΉ ΠΊΠ°ΡΡΡΠΊΠΈ Π±ΡΠ»ΠΈ ΡΠ°ΡΠΏΠΎΠ»ΠΎΠΆΠ΅Π½Ρ ΠΌΠ΅ΠΆΠ΄Ρ Π²ΠΈΡΠΊΠ°ΠΌΠΈ Π²ΡΠΎΡΠΎΠΉ. ΠΠΈΡΠΊΠΈ ΠΎΠ΄Π½ΠΎΠΉ ΠΊΠ°ΡΡΡΠΊΠΈ Π±ΡΠ»ΠΈ Π·Π°ΠΌΠΊΠ½ΡΡΡ Π½Π° Π³Π°Π»ΡΠ²Π°Π½ΠΎΠΌΠ΅ΡΡ, Π° Π²ΡΠΎΡΠΎΠΉ β ΠΏΠΎΠ΄ΠΊΠ»ΡΡΠ΅Π½Ρ ΠΊ ΠΈΡΡΠΎΡΠ½ΠΈΠΊΡ ΡΠΎΠΊΠ°. ΠΡΠΈ Π·Π°ΠΌΡΠΊΠ°Π½ΠΈΠΈ ΠΊΠ»ΡΡΠ° ΠΈ ΠΏΡΠΎΡΠ΅ΠΊΠ°Π½ΠΈΠΈ ΡΠΎΠΊΠ° ΠΏΠΎ Π²ΡΠΎΡΠΎΠΉ ΠΊΠ°ΡΡΡΠΊΠ΅ Π² ΠΏΠ΅ΡΠ²ΠΎΠΉ Π²ΠΎΠ·Π½ΠΈΠΊΠ°Π» ΠΈΠΌΠΏΡΠ»ΡΡ ΡΠΎΠΊΠ°. ΠΡΠΈ ΡΠ°Π·ΠΌΡΠΊΠ°Π½ΠΈΠΈ ΠΊΠ»ΡΡΠ° ΡΠ°ΠΊΠΆΠ΅ Π½Π°Π±Π»ΡΠ΄Π°Π»ΡΡ ΠΈΠΌΠΏΡΠ»ΡΡ ΡΠΎΠΊΠ°, Π½ΠΎ ΡΠΎΠΊ ΡΠ΅ΡΠ΅Π· Π³Π°Π»ΡΠ²Π°Π½ΠΎΠΌΠ΅ΡΡ ΡΠ΅ΠΊ Π² ΠΏΡΠΎΡΠΈΠ²ΠΎΠΏΠΎΠ»ΠΎΠΆΠ½ΠΎΠΌ Π½Π°ΠΏΡΠ°Π²Π»Π΅Π½ΠΈΠΈ.
- ΠΠ΅ΡΠ²Π°Ρ ΠΊΠ°ΡΡΡΠΊΠ° Π±ΡΠ»Π° ΠΏΠΎΠ΄ΠΊΠ»ΡΡΠ΅Π½Π° ΠΊ ΠΈΡΡΠΎΡΠ½ΠΈΠΊΡ ΡΠΎΠΊΠ°, Π²ΡΠΎΡΠ°Ρ, ΠΏΠΎΠ΄ΠΊΠ»ΡΡΠ΅Π½Π½Π°Ρ ΠΊ Π³Π°Π»ΡΠ²Π°Π½ΠΎΠΌΠ΅ΡΡΡ, ΠΏΠ΅ΡΠ΅ΠΌΠ΅ΡΠ°Π»Π°ΡΡ ΠΎΡΠ½ΠΎΡΠΈΡΠ΅Π»ΡΠ½ΠΎ Π½Π΅Π΅. ΠΡΠΈ ΠΏΡΠΈΠ±Π»ΠΈΠΆΠ΅Π½ΠΈΠΈ ΠΈΠ»ΠΈ ΡΠ΄Π°Π»Π΅Π½ΠΈΠΈ ΠΊΠ°ΡΡΡΠΊΠΈ ΡΠΈΠΊΡΠΈΡΠΎΠ²Π°Π»ΡΡ ΡΠΎΠΊ.
- ΠΠ°ΡΡΡΠΊΠ° Π·Π°ΠΌΠΊΠ½ΡΡΠ° Π½Π° Π³Π°Π»ΡΠ²Π°Π½ΠΎΠΌΠ΅ΡΡ, Π° ΠΌΠ°Π³Π½ΠΈΡ Π΄Π²ΠΈΠΆΠ΅ΡΡΡ β Π²Π΄Π²ΠΈΠ³Π°Π΅ΡΡΡ (Π²ΡΠ΄Π²ΠΈΠ³Π°Π΅ΡΡΡ) β ΠΎΡΠ½ΠΎΡΠΈΡΠ΅Π»ΡΠ½ΠΎ ΠΊΠ°ΡΡΡΠΊΠΈ.
ΠΠΏΡΡΡ ΠΏΠΎΠΊΠ°Π·Π°Π»ΠΈ, ΡΡΠΎ ΠΈΠ½Π΄ΡΠΊΡΠΈΠΎΠ½Π½ΡΠΉ ΡΠΎΠΊ Π²ΠΎΠ·Π½ΠΈΠΊΠ°Π΅Ρ ΡΠΎΠ»ΡΠΊΠΎ ΠΏΡΠΈ ΠΈΠ·ΠΌΠ΅Π½Π΅Π½ΠΈΠΈ Π»ΠΈΠ½ΠΈΠΉ ΠΌΠ°Π³Π½ΠΈΡΠ½ΠΎΠΉ ΠΈΠ½Π΄ΡΠΊΡΠΈΠΈ. ΠΠ°ΠΏΡΠ°Π²Π»Π΅Π½ΠΈΠ΅ ΡΠΎΠΊΠ° Π±ΡΠ΄Π΅Ρ ΡΠ°Π·Π»ΠΈΡΠ½ΠΎ ΠΏΡΠΈ ΡΠ²Π΅Π»ΠΈΡΠ΅Π½ΠΈΠΈ ΡΠΈΡΠ»Π° Π»ΠΈΠ½ΠΈΠΉ ΠΈ ΠΏΡΠΈ ΠΈΡ ΡΠΌΠ΅Π½ΡΡΠ΅Π½ΠΈΠΈ.
Π‘ΠΈΠ»Π° ΠΈΠ½Π΄ΡΠΊΡΠΈΠΎΠ½Π½ΠΎΠ³ΠΎ ΡΠΎΠΊΠ° Π·Π°Π²ΠΈΡΠΈΡ ΠΎΡ ΡΠΊΠΎΡΠΎΡΡΠΈ ΠΈΠ·ΠΌΠ΅Π½Π΅Π½ΠΈΡ ΠΌΠ°Π³Π½ΠΈΡΠ½ΠΎΠ³ΠΎ ΠΏΠΎΡΠΎΠΊΠ°. ΠΠΎΠΆΠ΅Ρ ΠΈΠ·ΠΌΠ΅Π½ΡΡΡΡΡ ΡΠ°ΠΌΠΎ ΠΏΠΎΠ»Π΅, ΠΈΠ»ΠΈ ΠΊΠΎΠ½ΡΡΡ ΠΌΠΎΠΆΠ΅Ρ ΠΏΠ΅ΡΠ΅ΠΌΠ΅ΡΠ°ΡΡΡΡ Π² Π½Π΅ΠΎΠ΄Π½ΠΎΡΠΎΠ΄Π½ΠΎΠΌ ΠΌΠ°Π³Π½ΠΈΡΠ½ΠΎΠΌ ΠΏΠΎΠ»Π΅.
ΠΠ±ΡΡΡΠ½Π΅Π½ΠΈΡ Π²ΠΎΠ·Π½ΠΈΠΊΠ½ΠΎΠ²Π΅Π½ΠΈΡ ΠΈΠ½Π΄ΡΠΊΡΠΈΠΎΠ½Π½ΠΎΠ³ΠΎ ΡΠΎΠΊΠ°
Π’ΠΎΠΊ Π² ΡΠ΅ΠΏΠΈ ΠΌΠΎΠΆΠ΅Ρ ΡΡΡΠ΅ΡΡΠ²ΠΎΠ²Π°ΡΡ, ΠΊΠΎΠ³Π΄Π° Π½Π° ΡΠ²ΠΎΠ±ΠΎΠ΄Π½ΡΠ΅ Π·Π°ΡΡΠ΄Ρ Π΄Π΅ΠΉΡΡΠ²ΡΡΡ ΡΡΠΎΡΠΎΠ½Π½ΠΈΠ΅ ΡΠΈΠ»Ρ. Π Π°Π±ΠΎΡΠ° ΡΡΠΈΡ ΡΠΈΠ» ΠΏΠΎ ΠΏΠ΅ΡΠ΅ΠΌΠ΅ΡΠ΅Π½ΠΈΡ Π΅Π΄ΠΈΠ½ΠΈΡΠ½ΠΎΠ³ΠΎ ΠΏΠΎΠ»ΠΎΠΆΠΈΡΠ΅Π»ΡΠ½ΠΎΠ³ΠΎ Π·Π°ΡΡΠ΄Π° Π²Π΄ΠΎΠ»Ρ Π·Π°ΠΌΠΊΠ½ΡΡΠΎΠ³ΠΎ ΠΊΠΎΠ½ΡΡΡΠ° ΡΠ°Π²Π½Π° ΠΠΠ‘. ΠΠ½Π°ΡΠΈΡ, ΠΏΡΠΈ ΠΈΠ·ΠΌΠ΅Π½Π΅Π½ΠΈΠΈ ΡΠΈΡΠ»Π° ΠΌΠ°Π³Π½ΠΈΡΠ½ΡΡ Π»ΠΈΠ½ΠΈΠΉ ΡΠ΅ΡΠ΅Π· ΠΏΠΎΠ²Π΅ΡΡ Π½ΠΎΡΡΡ, ΠΎΠ³ΡΠ°Π½ΠΈΡΠ΅Π½Π½ΡΡ ΠΊΠΎΠ½ΡΡΡΠΎΠΌ, Π² Π½Π΅ΠΌ ΠΏΠΎΡΠ²Π»ΡΠ΅ΡΡΡ ΠΠΠ‘, ΠΊΠΎΡΠΎΡΡΡ Π½Π°Π·ΡΠ²Π°ΡΡ ΠΠΠ‘ ΠΈΠ½Π΄ΡΠΊΡΠΈΠΈ.
ΠΠ»Π΅ΠΊΡΡΠΎΠ½Ρ Π² Π½Π΅ΠΏΠΎΠ΄Π²ΠΈΠΆΠ½ΠΎΠΌ ΠΏΡΠΎΠ²ΠΎΠ΄Π½ΠΈΠΊΠ΅ ΠΌΠΎΠ³ΡΡ ΠΏΡΠΈΠ²ΠΎΠ΄ΠΈΡΡΡΡ Π² Π΄Π²ΠΈΠΆΠ΅Π½ΠΈΠ΅ ΡΠΎΠ»ΡΠΊΠΎ ΡΠ»Π΅ΠΊΡΡΠΈΡΠ΅ΡΠΊΠΈΠΌ ΠΏΠΎΠ»Π΅ΠΌ. ΠΡΠΎ ΡΠ»Π΅ΠΊΡΡΠΈΡΠ΅ΡΠΊΠΎΠ΅ ΠΏΠΎΠ»Π΅ ΠΏΠΎΡΠΎΠΆΠ΄Π°Π΅ΡΡΡ ΠΈΠ·ΠΌΠ΅Π½ΡΡΡΠΈΠΌΡΡ Π²ΠΎ Π²ΡΠ΅ΠΌΠ΅Π½ΠΈ ΠΌΠ°Π³Π½ΠΈΡΠ½ΡΠΌ ΠΏΠΎΠ»Π΅ΠΌ. ΠΠ³ΠΎ Π½Π°Π·ΡΠ²Π°ΡΡ Π²ΠΈΡ ΡΠ΅Π²ΡΠΌ ΡΠ»Π΅ΠΊΡΡΠΈΡΠ΅ΡΠΊΠΈΠΌ ΠΏΠΎΠ»Π΅ΠΌ. ΠΡΠ΅Π΄ΡΡΠ°Π²Π»Π΅Π½ΠΈΠ΅ ΠΎ Π²ΠΈΡ ΡΠ΅Π²ΠΎΠΌ ΡΠ»Π΅ΠΊΡΡΠΈΡΠ΅ΡΠΊΠΎΠΌ ΠΏΠΎΠ»Π΅ Π±ΡΠ»ΠΎ Π²Π²Π΅Π΄Π΅Π½ΠΎ Π² ΡΠΈΠ·ΠΈΠΊΡ Π²Π΅Π»ΠΈΠΊΠΈΠΌ Π°Π½Π³Π»ΠΈΠΉΡΠΊΠΈΠΌ ΡΠΈΠ·ΠΈΠΊΠΎΠΌ ΠΠΆ. ΠΠ°ΠΊΡΠ²Π΅Π»Π»ΠΎΠΌ Π² 1861 Π³ΠΎΠ΄Ρ.
Π‘Π²ΠΎΠΉΡΡΠ²Π° Π²ΠΈΡ ΡΠ΅Π²ΠΎΠ³ΠΎ ΡΠ»Π΅ΠΊΡΡΠΈΡΠ΅ΡΠΊΠΎΠ³ΠΎ ΠΏΠΎΠ»Ρ:
- ΠΈΡΡΠΎΡΠ½ΠΈΠΊ β ΠΏΠ΅ΡΠ΅ΠΌΠ΅Π½Π½ΠΎΠ΅ ΠΌΠ°Π³Π½ΠΈΡΠ½ΠΎΠ΅ ΠΏΠΎΠ»Π΅;
- ΠΎΠ±Π½Π°ΡΡΠΆΠΈΠ²Π°Π΅ΡΡΡ ΠΏΠΎ Π΄Π΅ΠΉΡΡΠ²ΠΈΡ Π½Π° Π·Π°ΡΡΠ΄;
- Π½Π΅ ΡΠ²Π»ΡΠ΅ΡΡΡ ΠΏΠΎΡΠ΅Π½ΡΠΈΠ°Π»ΡΠ½ΡΠΌ;
- Π»ΠΈΠ½ΠΈΠΈ ΠΏΠΎΠ»Ρ Π·Π°ΠΌΠΊΠ½ΡΡΡΠ΅.
Π Π°Π±ΠΎΡΠ° ΡΡΠΎΠ³ΠΎ ΠΏΠΎΠ»Ρ ΠΏΡΠΈ ΠΏΠ΅ΡΠ΅ΠΌΠ΅ΡΠ΅Π½ΠΈΠΈ Π΅Π΄ΠΈΠ½ΠΈΡΠ½ΠΎΠ³ΠΎ ΠΏΠΎΠ»ΠΎΠΆΠΈΡΠ΅Π»ΡΠ½ΠΎΠ³ΠΎ Π·Π°ΡΡΠ΄Π° ΠΏΠΎ Π·Π°ΠΌΠΊΠ½ΡΡΠΎΠΌΡ ΠΊΠΎΠ½ΡΡΡΡ ΡΠ°Π²Π½Π° ΠΠΠ‘ ΠΈΠ½Π΄ΡΠΊΡΠΈΠΈ Π² Π½Π΅ΠΏΠΎΠ΄Π²ΠΈΠΆΠ½ΠΎΠΌ ΠΏΡΠΎΠ²ΠΎΠ΄Π½ΠΈΠΊΠ΅.
ΠΠ°Π³Π½ΠΈΡΠ½ΡΠΉ ΠΏΠΎΡΠΎΠΊ
ΠΠ°Π³Π½ΠΈΡΠ½ΡΠΌ ΠΏΠΎΡΠΎΠΊΠΎΠΌ ΡΠ΅ΡΠ΅Π· ΠΏΠ»ΠΎΡΠ°Π΄Ρ β ( S ) β ΠΊΠΎΠ½ΡΡΡΠ° Π½Π°Π·ΡΠ²Π°ΡΡ ΡΠΊΠ°Π»ΡΡΠ½ΡΡ ΡΠΈΠ·ΠΈΡΠ΅ΡΠΊΡΡ Π²Π΅Π»ΠΈΡΠΈΠ½Ρ, ΡΠ°Π²Π½ΡΡ ΠΏΡΠΎΠΈΠ·Π²Π΅Π΄Π΅Π½ΠΈΡ ΠΌΠΎΠ΄ΡΠ»Ρ Π²Π΅ΠΊΡΠΎΡΠ° ΠΌΠ°Π³Π½ΠΈΡΠ½ΠΎΠΉ ΠΈΠ½Π΄ΡΠΊΡΠΈΠΈ β ( B ) β, ΠΏΠ»ΠΎΡΠ°Π΄ΠΈ ΠΏΠΎΠ²Π΅ΡΡ Π½ΠΎΡΡΠΈ β ( S ) β, ΠΏΡΠΎΠ½ΠΈΠ·ΡΠ²Π°Π΅ΠΌΠΎΠΉ Π΄Π°Π½Π½ΡΠΌ ΠΏΠΎΡΠΎΠΊΠΎΠΌ, ΠΈ ΠΊΠΎΡΠΈΠ½ΡΡΠ° ΡΠ³Π»Π° β ( alpha ) β ΠΌΠ΅ΠΆΠ΄Ρ Π½Π°ΠΏΡΠ°Π²Π»Π΅Π½ΠΈΠ΅ΠΌ Π²Π΅ΠΊΡΠΎΡΠ° ΠΌΠ°Π³Π½ΠΈΡΠ½ΠΎΠΉ ΠΈΠ½Π΄ΡΠΊΡΠΈΠΈ ΠΈ Π²Π΅ΠΊΡΠΎΡΠ° Π½ΠΎΡΠΌΠ°Π»ΠΈ (ΠΏΠ΅ΡΠΏΠ΅Π½Π΄ΠΈΠΊΡΠ»ΡΡΠ° ΠΊ ΠΏΠ»ΠΎΡΠΊΠΎΡΡΠΈ Π΄Π°Π½Π½ΠΎΠΉ ΠΏΠΎΠ²Π΅ΡΡ Π½ΠΎΡΡΠΈ):
ΠΠ±ΠΎΠ·Π½Π°ΡΠ΅Π½ΠΈΠ΅ β β ( Phi ) β, Π΅Π΄ΠΈΠ½ΠΈΡΠ° ΠΈΠ·ΠΌΠ΅ΡΠ΅Π½ΠΈΡ Π² Π‘Π β Π²Π΅Π±Π΅Ρ (ΠΠ±).
ΠΠ°Π³Π½ΠΈΡΠ½ΡΠΉ ΠΏΠΎΡΠΎΠΊ Π² 1 Π²Π΅Π±Π΅Ρ ΡΠΎΠ·Π΄Π°Π΅ΡΡΡ ΠΎΠ΄Π½ΠΎΡΠΎΠ΄Π½ΡΠΌ ΠΌΠ°Π³Π½ΠΈΡΠ½ΡΠΌ ΠΏΠΎΠ»Π΅ΠΌ Ρ ΠΈΠ½Π΄ΡΠΊΡΠΈΠ΅ΠΉ 1 Π’Π» ΡΠ΅ΡΠ΅Π· ΠΏΠΎΠ²Π΅ΡΡ Π½ΠΎΡΡΡ ΠΏΠ»ΠΎΡΠ°Π΄ΡΡ 1 ΠΌ 2 , ΡΠ°ΡΠΏΠΎΠ»ΠΎΠΆΠ΅Π½Π½ΡΡ ΠΏΠ΅ΡΠΏΠ΅Π½Π΄ΠΈΠΊΡΠ»ΡΡΠ½ΠΎ Π²Π΅ΠΊΡΠΎΡΡ ΠΌΠ°Π³Π½ΠΈΡΠ½ΠΎΠΉ ΠΈΠ½Π΄ΡΠΊΡΠΈΠΈ:
ΠΠ°Π³Π½ΠΈΡΠ½ΡΠΉ ΠΏΠΎΡΠΎΠΊ ΠΌΠΎΠΆΠ½ΠΎ Π½Π°Π³Π»ΡΠ΄Π½ΠΎ ΠΏΡΠ΅Π΄ΡΡΠ°Π²ΠΈΡΡ ΠΊΠ°ΠΊ Π²Π΅Π»ΠΈΡΠΈΠ½Ρ, ΠΏΡΠΎΠΏΠΎΡΡΠΈΠΎΠ½Π°Π»ΡΠ½ΡΡ ΡΠΈΡΠ»Ρ ΠΌΠ°Π³Π½ΠΈΡΠ½ΡΡ Π»ΠΈΠ½ΠΈΠΉ, ΠΏΡΠΎΡ ΠΎΠ΄ΡΡΠΈΡ ΡΠ΅ΡΠ΅Π· Π΄Π°Π½Π½ΡΡ ΠΏΠ»ΠΎΡΠ°Π΄Ρ.
Π Π·Π°Π²ΠΈΡΠΈΠΌΠΎΡΡΠΈ ΠΎΡ ΡΠ³Π»Π° β ( alpha ) β ΠΌΠ°Π³Π½ΠΈΡΠ½ΡΠΉ ΠΏΠΎΡΠΎΠΊ ΠΌΠΎΠΆΠ΅Ρ Π±ΡΡΡ ΠΏΠΎΠ»ΠΎΠΆΠΈΡΠ΅Π»ΡΠ½ΡΠΌ ( ( alpha ) ( alpha ) > 90Β°). ΠΡΠ»ΠΈ ( alpha ) = 90Β°, ΡΠΎ ΠΌΠ°Π³Π½ΠΈΡΠ½ΡΠΉ ΠΏΠΎΡΠΎΠΊ ΡΠ°Π²Π΅Π½ 0.
ΠΠ·ΠΌΠ΅Π½ΠΈΡΡ ΠΌΠ°Π³Π½ΠΈΡΠ½ΡΠΉ ΠΏΠΎΡΠΎΠΊ ΠΌΠΎΠΆΠ½ΠΎ ΠΌΠ΅Π½ΡΡ ΠΏΠ»ΠΎΡΠ°Π΄Ρ ΠΊΠΎΠ½ΡΡΡΠ°, ΠΌΠΎΠ΄ΡΠ»Ρ ΠΈΠ½Π΄ΡΠΊΡΠΈΠΈ ΠΏΠΎΠ»Ρ ΠΈΠ»ΠΈ ΡΠ°ΡΠΏΠΎΠ»ΠΎΠΆΠ΅Π½ΠΈΠ΅ ΠΊΠΎΠ½ΡΡΡΠ° Π² ΠΌΠ°Π³Π½ΠΈΡΠ½ΠΎΠΌ ΠΏΠΎΠ»Π΅ (ΠΏΠΎΠ²ΠΎΡΠ°ΡΠΈΠ²Π°Ρ Π΅Π³ΠΎ).
Π ΡΠ»ΡΡΠ°Π΅ Π½Π΅ΠΎΠ΄Π½ΠΎΡΠΎΠ΄Π½ΠΎΠ³ΠΎ ΠΌΠ°Π³Π½ΠΈΡΠ½ΠΎΠ³ΠΎ ΠΏΠΎΠ»Ρ ΠΈ Π½Π΅ΠΏΠ»ΠΎΡΠΊΠΎΠ³ΠΎ ΠΊΠΎΠ½ΡΡΡΠ° ΠΌΠ°Π³Π½ΠΈΡΠ½ΡΠΉ ΠΏΠΎΡΠΎΠΊ Π½Π°Ρ ΠΎΠ΄ΡΡ ΠΊΠ°ΠΊ ΡΡΠΌΠΌΡ ΠΌΠ°Π³Π½ΠΈΡΠ½ΡΡ ΠΏΠΎΡΠΎΠΊΠΎΠ², ΠΏΡΠΎΠ½ΠΈΠ·ΡΠ²Π°ΡΡΠΈΡ ΠΏΠ»ΠΎΡΠ°Π΄Ρ ΠΊΠ°ΠΆΠ΄ΠΎΠ³ΠΎ ΠΈΠ· ΡΡΠ°ΡΡΠΊΠΎΠ², Π½Π° ΠΊΠΎΡΠΎΡΡΠ΅ ΠΌΠΎΠΆΠ½ΠΎ ΡΠ°Π·Π±ΠΈΡΡ Π΄Π°Π½Π½ΡΡ ΠΏΠΎΠ²Π΅ΡΡ Π½ΠΎΡΡΡ.
ΠΠ°ΠΊΠΎΠ½ ΡΠ»Π΅ΠΊΡΡΠΎΠΌΠ°Π³Π½ΠΈΡΠ½ΠΎΠΉ ΠΈΠ½Π΄ΡΠΊΡΠΈΠΈ Π€Π°ΡΠ°Π΄Π΅Ρ
ΠΠ°ΠΊΠΎΠ½ ΡΠ»Π΅ΠΊΡΡΠΎΠΌΠ°Π³Π½ΠΈΡΠ½ΠΎΠΉ ΠΈΠ½Π΄ΡΠΊΡΠΈΠΈ (Π·Π°ΠΊΠΎΠ½ Π€Π°ΡΠ°Π΄Π΅Ρ):
ΠΠΠ‘ ΠΈΠ½Π΄ΡΠΊΡΠΈΠΈ Π² Π·Π°ΠΌΠΊΠ½ΡΡΠΎΠΌ ΠΊΠΎΠ½ΡΡΡΠ΅ ΡΠ°Π²Π½Π° ΠΈ ΠΏΡΠΎΡΠΈΠ²ΠΎΠΏΠΎΠ»ΠΎΠΆΠ½Π° ΠΏΠΎ Π·Π½Π°ΠΊΡ ΡΠΊΠΎΡΠΎΡΡΠΈ ΠΈΠ·ΠΌΠ΅Π½Π΅Π½ΠΈΡ ΠΌΠ°Π³Π½ΠΈΡΠ½ΠΎΠ³ΠΎ ΠΏΠΎΡΠΎΠΊΠ° ΡΠ΅ΡΠ΅Π· ΠΏΠΎΠ²Π΅ΡΡ Π½ΠΎΡΡΡ, ΠΎΠ³ΡΠ°Π½ΠΈΡΠ΅Π½Π½ΡΡ ΠΊΠΎΠ½ΡΡΡΠΎΠΌ:
ΠΠ½Π°ΠΊ Β«βΒ» Π² ΡΠΎΡΠΌΡΠ»Π΅ ΠΏΠΎΠ·Π²ΠΎΠ»ΡΠ΅Ρ ΡΡΠ΅ΡΡΡ Π½Π°ΠΏΡΠ°Π²Π»Π΅Π½ΠΈΠ΅ ΠΈΠ½Π΄ΡΠΊΡΠΈΠΎΠ½Π½ΠΎΠ³ΠΎ ΡΠΎΠΊΠ°. ΠΠ½Π΄ΡΠΊΡΠΈΠΎΠ½Π½ΡΠΉ ΡΠΎΠΊ Π² Π·Π°ΠΌΠΊΠ½ΡΡΠΎΠΌ ΠΊΠΎΠ½ΡΡΡΠ΅ ΠΈΠΌΠ΅Π΅Ρ Π²ΡΠ΅Π³Π΄Π° ΡΠ°ΠΊΠΎΠ΅ Π½Π°ΠΏΡΠ°Π²Π»Π΅Π½ΠΈΠ΅, ΡΡΠΎΠ±Ρ ΠΌΠ°Π³Π½ΠΈΡΠ½ΡΠΉ ΠΏΠΎΡΠΎΠΊ ΠΏΠΎΠ»Ρ, ΡΠΎΠ·Π΄Π°Π½Π½ΠΎΠ³ΠΎ ΡΡΠΈΠΌ ΡΠΎΠΊΠΎΠΌ ΡΠΊΠ²ΠΎΠ·Ρ ΠΏΠΎΠ²Π΅ΡΡ Π½ΠΎΡΡΡ, ΠΎΠ³ΡΠ°Π½ΠΈΡΠ΅Π½Π½ΡΡ ΠΊΠΎΠ½ΡΡΡΠΎΠΌ, ΡΠΌΠ΅Π½ΡΡΠ°Π» Π±Ρ ΡΠ΅ ΠΈΠ·ΠΌΠ΅Π½Π΅Π½ΠΈΡ ΠΏΠΎΠ»Ρ, ΠΊΠΎΡΠΎΡΡΠ΅ Π²ΡΠ·Π²Π°Π»ΠΈ ΠΏΠΎΡΠ²Π»Π΅Π½ΠΈΠ΅ ΠΈΠ½Π΄ΡΠΊΡΠΈΠΎΠ½Π½ΠΎΠ³ΠΎ ΡΠΎΠΊΠ°.
ΠΡΠ»ΠΈ ΠΊΠΎΠ½ΡΡΡ ΡΠΎΡΡΠΎΠΈΡ ΠΈΠ· β ( N ) β Π²ΠΈΡΠΊΠΎΠ², ΡΠΎ ΠΠΠ‘ ΠΈΠ½Π΄ΡΠΊΡΠΈΠΈ:
Π‘ΠΈΠ»Π° ΠΈΠ½Π΄ΡΠΊΡΠΈΠΎΠ½Π½ΠΎΠ³ΠΎ ΡΠΎΠΊΠ° Π² Π·Π°ΠΌΠΊΠ½ΡΡΠΎΠΌ ΠΏΡΠΎΠ²ΠΎΠ΄ΡΡΠ΅ΠΌ ΠΊΠΎΠ½ΡΡΡΠ΅ Ρ ΡΠΎΠΏΡΠΎΡΠΈΠ²Π»Π΅Π½ΠΈΠ΅ΠΌ β ( R ) β:
ΠΡΠΈ Π΄Π²ΠΈΠΆΠ΅Π½ΠΈΠΈ ΠΏΡΠΎΠ²ΠΎΠ΄Π½ΠΈΠΊΠ° Π΄Π»ΠΈΠ½ΠΎΠΉ β ( l ) β ΡΠΎ ΡΠΊΠΎΡΠΎΡΡΡΡ β ( v ) β Π² ΠΏΠΎΡΡΠΎΡΠ½Π½ΠΎΠΌ ΠΎΠ΄Π½ΠΎΡΠΎΠ΄Π½ΠΎΠΌ ΠΌΠ°Π³Π½ΠΈΡΠ½ΠΎΠΌ ΠΏΠΎΠ»Π΅ Ρ ΠΈΠ½Π΄ΡΠΊΡΠΈΠ΅ΠΉ β ( vec ) β ΠΠΠ‘ ΡΠ»Π΅ΠΊΡΡΠΎΠΌΠ°Π³Π½ΠΈΡΠ½ΠΎΠΉ ΠΈΠ½Π΄ΡΠΊΡΠΈΠΈ ΡΠ°Π²Π½Π°:
Π³Π΄Π΅ β ( alpha ) β β ΡΠ³ΠΎΠ» ΠΌΠ΅ΠΆΠ΄Ρ Π²Π΅ΠΊΡΠΎΡΠ°ΠΌΠΈ β ( vec ) β ΠΈ ( vec ) .
ΠΠΎΠ·Π½ΠΈΠΊΠ½ΠΎΠ²Π΅Π½ΠΈΠ΅ ΠΠΠ‘ ΠΈΠ½Π΄ΡΠΊΡΠΈΠΈ Π² Π΄Π²ΠΈΠΆΡΡΠ΅ΠΌΡΡ Π² ΠΌΠ°Π³Π½ΠΈΡΠ½ΠΎΠΌ ΠΏΠΎΠ»Π΅ ΠΏΡΠΎΠ²ΠΎΠ΄Π½ΠΈΠΊΠ΅ ΠΎΠ±ΡΡΡΠ½ΡΠ΅ΡΡΡ Π΄Π΅ΠΉΡΡΠ²ΠΈΠ΅ΠΌ ΡΠΈΠ»Ρ ΠΠΎΡΠ΅Π½ΡΠ° Π½Π° ΡΠ²ΠΎΠ±ΠΎΠ΄Π½ΡΠ΅ Π·Π°ΡΡΠ΄Ρ Π² Π΄Π²ΠΈΠΆΡΡΠΈΡ ΡΡ ΠΏΡΠΎΠ²ΠΎΠ΄Π½ΠΈΠΊΠ°Ρ . Π‘ΠΈΠ»Π° ΠΠΎΡΠ΅Π½ΡΠ° ΠΈΠ³ΡΠ°Π΅Ρ Π² ΡΡΠΎΠΌ ΡΠ»ΡΡΠ°Π΅ ΡΠΎΠ»Ρ ΡΡΠΎΡΠΎΠ½Π½Π΅ΠΉ ΡΠΈΠ»Ρ.
ΠΠ²ΠΈΠΆΡΡΠΈΠΉΡΡ Π² ΠΌΠ°Π³Π½ΠΈΡΠ½ΠΎΠΌ ΠΏΠΎΠ»Π΅ ΠΏΡΠΎΠ²ΠΎΠ΄Π½ΠΈΠΊ, ΠΏΠΎ ΠΊΠΎΡΠΎΡΠΎΠΌΡ ΠΏΡΠΎΡΠ΅ΠΊΠ°Π΅Ρ ΠΈΠ½Π΄ΡΠΊΡΠΈΠΎΠ½Π½ΡΠΉ ΡΠΎΠΊ, ΠΈΡΠΏΡΡΡΠ²Π°Π΅Ρ ΠΌΠ°Π³Π½ΠΈΡΠ½ΠΎΠ΅ ΡΠΎΡΠΌΠΎΠΆΠ΅Π½ΠΈΠ΅. ΠΠΎΠ»Π½Π°Ρ ΡΠ°Π±ΠΎΡΠ° ΡΠΈΠ»Ρ ΠΠΎΡΠ΅Π½ΡΠ° ΡΠ°Π²Π½Π° Π½ΡΠ»Ρ.
ΠΠΎΠ»ΠΈΡΠ΅ΡΡΠ²ΠΎ ΡΠ΅ΠΏΠ»ΠΎΡΡ Π² ΠΊΠΎΠ½ΡΡΡΠ΅ Π²ΡΠ΄Π΅Π»ΡΠ΅ΡΡΡ Π»ΠΈΠ±ΠΎ Π·Π° ΡΡΠ΅Ρ ΡΠ°Π±ΠΎΡΡ Π²Π½Π΅ΡΠ½Π΅ΠΉ ΡΠΈΠ»Ρ, ΠΊΠΎΡΠΎΡΠ°Ρ ΠΏΠΎΠ΄Π΄Π΅ΡΠΆΠΈΠ²Π°Π΅Ρ ΡΠΊΠΎΡΠΎΡΡΡ ΠΏΡΠΎΠ²ΠΎΠ΄Π½ΠΈΠΊΠ° Π½Π΅ΠΈΠ·ΠΌΠ΅Π½Π½ΠΎΠΉ, Π»ΠΈΠ±ΠΎ Π·Π° ΡΡΠ΅Ρ ΡΠΌΠ΅Π½ΡΡΠ΅Π½ΠΈΡ ΠΊΠΈΠ½Π΅ΡΠΈΡΠ΅ΡΠΊΠΎΠΉ ΡΠ½Π΅ΡΠ³ΠΈΠΈ ΠΏΡΠΎΠ²ΠΎΠ΄Π½ΠΈΠΊΠ°.
ΠΠ°ΠΆΠ½ΠΎ!
ΠΠ·ΠΌΠ΅Π½Π΅Π½ΠΈΠ΅ ΠΌΠ°Π³Π½ΠΈΡΠ½ΠΎΠ³ΠΎ ΠΏΠΎΡΠΎΠΊΠ°, ΠΏΡΠΎΠ½ΠΈΠ·ΡΠ²Π°ΡΡΠ΅Π³ΠΎ Π·Π°ΠΌΠΊΠ½ΡΡΡΠΉ ΠΊΠΎΠ½ΡΡΡ, ΠΌΠΎΠΆΠ΅Ρ ΠΏΡΠΎΠΈΡΡ
ΠΎΠ΄ΠΈΡΡ ΠΏΠΎ Π΄Π²ΡΠΌ ΠΏΡΠΈΡΠΈΠ½Π°ΠΌ:
- ΠΌΠ°Π³Π½ΠΈΡΠ½ΡΠΉ ΠΏΠΎΡΠΎΠΊ ΠΈΠ·ΠΌΠ΅Π½ΡΠ΅ΡΡΡ Π²ΡΠ»Π΅Π΄ΡΡΠ²ΠΈΠ΅ ΠΏΠ΅ΡΠ΅ΠΌΠ΅ΡΠ΅Π½ΠΈΡ ΠΊΠΎΠ½ΡΡΡΠ° ΠΈΠ»ΠΈ Π΅Π³ΠΎ ΡΠ°ΡΡΠ΅ΠΉ Π² ΠΏΠΎΡΡΠΎΡΠ½Π½ΠΎΠΌ Π²ΠΎ Π²ΡΠ΅ΠΌΠ΅Π½ΠΈ ΠΌΠ°Π³Π½ΠΈΡΠ½ΠΎΠΌ ΠΏΠΎΠ»Π΅. ΠΡΠΎ ΡΠ»ΡΡΠ°ΠΉ, ΠΊΠΎΠ³Π΄Π° ΠΏΡΠΎΠ²ΠΎΠ΄Π½ΠΈΠΊΠΈ, Π° Π²ΠΌΠ΅ΡΡΠ΅ Ρ Π½ΠΈΠΌΠΈ ΠΈ ΡΠ²ΠΎΠ±ΠΎΠ΄Π½ΡΠ΅ Π½ΠΎΡΠΈΡΠ΅Π»ΠΈ Π·Π°ΡΡΠ΄Π°, Π΄Π²ΠΈΠΆΡΡΡΡ Π² ΠΌΠ°Π³Π½ΠΈΡΠ½ΠΎΠΌ ΠΏΠΎΠ»Π΅;
- Π²ΡΠΎΡΠ°Ρ ΠΏΡΠΈΡΠΈΠ½Π° ΠΈΠ·ΠΌΠ΅Π½Π΅Π½ΠΈΡ ΠΌΠ°Π³Π½ΠΈΡΠ½ΠΎΠ³ΠΎ ΠΏΠΎΡΠΎΠΊΠ°, ΠΏΡΠΎΠ½ΠΈΠ·ΡΠ²Π°ΡΡΠ΅Π³ΠΎ ΠΊΠΎΠ½ΡΡΡ, β ΠΈΠ·ΠΌΠ΅Π½Π΅Π½ΠΈΠ΅ Π²ΠΎ Π²ΡΠ΅ΠΌΠ΅Π½ΠΈ ΠΌΠ°Π³Π½ΠΈΡΠ½ΠΎΠ³ΠΎ ΠΏΠΎΠ»Ρ ΠΏΡΠΈ Π½Π΅ΠΏΠΎΠ΄Π²ΠΈΠΆΠ½ΠΎΠΌ ΠΊΠΎΠ½ΡΡΡΠ΅. Π ΡΡΠΎΠΌ ΡΠ»ΡΡΠ°Π΅ Π²ΠΎΠ·Π½ΠΈΠΊΠ½ΠΎΠ²Π΅Π½ΠΈΠ΅ ΠΠΠ‘ ΠΈΠ½Π΄ΡΠΊΡΠΈΠΈ ΡΠΆΠ΅ Π½Π΅Π»ΡΠ·Ρ ΠΎΠ±ΡΡΡΠ½ΠΈΡΡ Π΄Π΅ΠΉΡΡΠ²ΠΈΠ΅ΠΌ ΡΠΈΠ»Ρ ΠΠΎΡΠ΅Π½ΡΠ°. Π―Π²Π»Π΅Π½ΠΈΠ΅ ΡΠ»Π΅ΠΊΡΡΠΎΠΌΠ°Π³Π½ΠΈΡΠ½ΠΎΠΉ ΠΈΠ½Π΄ΡΠΊΡΠΈΠΈ Π² Π½Π΅ΠΏΠΎΠ΄Π²ΠΈΠΆΠ½ΡΡ ΠΏΡΠΎΠ²ΠΎΠ΄Π½ΠΈΠΊΠ°Ρ , Π²ΠΎΠ·Π½ΠΈΠΊΠ°ΡΡΠ΅Π΅ ΠΏΡΠΈ ΠΈΠ·ΠΌΠ΅Π½Π΅Π½ΠΈΠΈ ΠΎΠΊΡΡΠΆΠ°ΡΡΠ΅Π³ΠΎ ΠΌΠ°Π³Π½ΠΈΡΠ½ΠΎΠ³ΠΎ ΠΏΠΎΠ»Ρ, ΡΠ°ΠΊΠΆΠ΅ ΠΎΠΏΠΈΡΡΠ²Π°Π΅ΡΡΡ ΡΠΎΡΠΌΡΠ»ΠΎΠΉ Π€Π°ΡΠ°Π΄Π΅Ρ.
Π’Π°ΠΊΠΈΠΌ ΠΎΠ±ΡΠ°Π·ΠΎΠΌ, ΡΠ²Π»Π΅Π½ΠΈΡ ΠΈΠ½Π΄ΡΠΊΡΠΈΠΈ Π² Π΄Π²ΠΈΠΆΡΡΠΈΡ ΡΡ ΠΈ Π½Π΅ΠΏΠΎΠ΄Π²ΠΈΠΆΠ½ΡΡ ΠΏΡΠΎΠ²ΠΎΠ΄Π½ΠΈΠΊΠ°Ρ ΠΏΡΠΎΡΠ΅ΠΊΠ°ΡΡ ΠΎΠ΄ΠΈΠ½Π°ΠΊΠΎΠ²ΠΎ, Π½ΠΎ ΡΠΈΠ·ΠΈΡΠ΅ΡΠΊΠ°Ρ ΠΏΡΠΈΡΠΈΠ½Π° Π²ΠΎΠ·Π½ΠΈΠΊΠ½ΠΎΠ²Π΅Π½ΠΈΡ ΠΈΠ½Π΄ΡΠΊΡΠΈΠΎΠ½Π½ΠΎΠ³ΠΎ ΡΠΎΠΊΠ° ΠΎΠΊΠ°Π·ΡΠ²Π°Π΅ΡΡΡ Π² ΡΡΠΈΡ Π΄Π²ΡΡ ΡΠ»ΡΡΠ°ΡΡ ΡΠ°Π·Π»ΠΈΡΠ½ΠΎΠΉ:
- Π² ΡΠ»ΡΡΠ°Π΅ Π΄Π²ΠΈΠΆΡΡΠΈΡ ΡΡ ΠΏΡΠΎΠ²ΠΎΠ΄Π½ΠΈΠΊΠΎΠ² ΠΠΠ‘ ΠΈΠ½Π΄ΡΠΊΡΠΈΠΈ ΠΎΠ±ΡΡΠ»ΠΎΠ²Π»Π΅Π½Π° ΡΠΈΠ»ΠΎΠΉ ΠΠΎΡΠ΅Π½ΡΠ°;
- Π² ΡΠ»ΡΡΠ°Π΅ Π½Π΅ΠΏΠΎΠ΄Π²ΠΈΠΆΠ½ΡΡ ΠΏΡΠΎΠ²ΠΎΠ΄Π½ΠΈΠΊΠΎΠ² ΠΠΠ‘ ΠΈΠ½Π΄ΡΠΊΡΠΈΠΈ ΡΠ²Π»ΡΠ΅ΡΡΡ ΡΠ»Π΅Π΄ΡΡΠ²ΠΈΠ΅ΠΌ Π΄Π΅ΠΉΡΡΠ²ΠΈΡ Π½Π° ΡΠ²ΠΎΠ±ΠΎΠ΄Π½ΡΠ΅ Π·Π°ΡΡΠ΄Ρ Π²ΠΈΡ ΡΠ΅Π²ΠΎΠ³ΠΎ ΡΠ»Π΅ΠΊΡΡΠΈΡΠ΅ΡΠΊΠΎΠ³ΠΎ ΠΏΠΎΠ»Ρ, Π²ΠΎΠ·Π½ΠΈΠΊΠ°ΡΡΠ΅Π³ΠΎ ΠΏΡΠΈ ΠΈΠ·ΠΌΠ΅Π½Π΅Π½ΠΈΠΈ ΠΌΠ°Π³Π½ΠΈΡΠ½ΠΎΠ³ΠΎ ΠΏΠΎΠ»Ρ.
ΠΡΠ°Π²ΠΈΠ»ΠΎ ΠΠ΅Π½ΡΠ°
ΠΠ°ΠΏΡΠ°Π²Π»Π΅Π½ΠΈΠ΅ ΠΈΠ½Π΄ΡΠΊΡΠΈΠΎΠ½Π½ΠΎΠ³ΠΎ ΡΠΎΠΊΠ° ΠΎΠΏΡΠ΅Π΄Π΅Π»ΡΠ΅ΡΡΡ ΠΏΠΎ ΠΏΡΠ°Π²ΠΈΠ»Ρ ΠΠ΅Π½ΡΠ°: ΠΈΠ½Π΄ΡΠΊΡΠΈΠΎΠ½Π½ΡΠΉ ΡΠΎΠΊ, Π²ΠΎΠ·Π±ΡΠΆΠ΄Π°Π΅ΠΌΡΠΉ Π² Π·Π°ΠΌΠΊΠ½ΡΡΠΎΠΌ ΠΊΠΎΠ½ΡΡΡΠ΅ ΠΏΡΠΈ ΠΈΠ·ΠΌΠ΅Π½Π΅Π½ΠΈΠΈ ΠΌΠ°Π³Π½ΠΈΡΠ½ΠΎΠ³ΠΎ ΠΏΠΎΡΠΎΠΊΠ°, Π²ΡΠ΅Π³Π΄Π° Π½Π°ΠΏΡΠ°Π²Π»Π΅Π½ ΡΠ°ΠΊ, ΡΡΠΎ ΡΠΎΠ·Π΄Π°Π²Π°Π΅ΠΌΠΎΠ΅ ΠΈΠΌ ΠΌΠ°Π³Π½ΠΈΡΠ½ΠΎΠ΅ ΠΏΠΎΠ»Π΅ ΠΏΡΠ΅ΠΏΡΡΡΡΠ²ΡΠ΅Ρ ΠΈΠ·ΠΌΠ΅Π½Π΅Π½ΠΈΡ ΠΌΠ°Π³Π½ΠΈΡΠ½ΠΎΠ³ΠΎ ΠΏΠΎΡΠΎΠΊΠ°, Π²ΡΠ·ΡΠ²Π°ΡΡΠ΅Π³ΠΎ ΠΈΠ½Π΄ΡΠΊΡΠΈΠΎΠ½Π½ΡΠΉ ΡΠΎΠΊ.
ΠΠ»Π³ΠΎΡΠΈΡΠΌ ΡΠ΅ΡΠ΅Π½ΠΈΡ Π·Π°Π΄Π°Ρ Ρ ΠΈΡΠΏΠΎΠ»ΡΠ·ΠΎΠ²Π°Π½ΠΈΠ΅ΠΌ ΠΏΡΠ°Π²ΠΈΠ»Π° ΠΠ΅Π½ΡΠ°:
- ΠΎΠΏΡΠ΅Π΄Π΅Π»ΠΈΡΡ Π½Π°ΠΏΡΠ°Π²Π»Π΅Π½ΠΈΠ΅ Π»ΠΈΠ½ΠΈΠΉ ΠΌΠ°Π³Π½ΠΈΡΠ½ΠΎΠΉ ΠΈΠ½Π΄ΡΠΊΡΠΈΠΈ Π²Π½Π΅ΡΠ½Π΅Π³ΠΎ ΠΌΠ°Π³Π½ΠΈΡΠ½ΠΎΠ³ΠΎ ΠΏΠΎΠ»Ρ;
- Π²ΡΡΡΠ½ΠΈΡΡ, ΠΊΠ°ΠΊ ΠΈΠ·ΠΌΠ΅Π½ΡΠ΅ΡΡΡ ΠΌΠ°Π³Π½ΠΈΡΠ½ΡΠΉ ΠΏΠΎΡΠΎΠΊ;
- ΠΎΠΏΡΠ΅Π΄Π΅Π»ΠΈΡΡ Π½Π°ΠΏΡΠ°Π²Π»Π΅Π½ΠΈΠ΅ Π»ΠΈΠ½ΠΈΠΉ ΠΌΠ°Π³Π½ΠΈΡΠ½ΠΎΠΉ ΠΈΠ½Π΄ΡΠΊΡΠΈΠΈ ΠΌΠ°Π³Π½ΠΈΡΠ½ΠΎΠ³ΠΎ ΠΏΠΎΠ»Ρ ΠΈΠ½Π΄ΡΠΊΡΠΈΠΎΠ½Π½ΠΎΠ³ΠΎ ΡΠΎΠΊΠ°: Π΅ΡΠ»ΠΈ ΠΌΠ°Π³Π½ΠΈΡΠ½ΡΠΉ ΠΏΠΎΡΠΎΠΊ ΡΠΌΠ΅Π½ΡΡΠ°Π΅ΡΡΡ, ΡΠΎ ΠΎΠ½ΠΈ ΡΠΎΠ½Π°ΠΏΡΠ°Π²Π»Π΅Π½Ρ Ρ Π»ΠΈΠ½ΠΈΡΠΌΠΈ Π²Π½Π΅ΡΠ½Π΅Π³ΠΎ ΠΌΠ°Π³Π½ΠΈΡΠ½ΠΎΠ³ΠΎ ΠΏΠΎΠ»Ρ; Π΅ΡΠ»ΠΈ ΠΌΠ°Π³Π½ΠΈΡΠ½ΡΠΉ ΠΏΠΎΡΠΎΠΊ ΡΠ²Π΅Π»ΠΈΡΠΈΠ²Π°Π΅ΡΡΡ, β ΠΏΡΠΎΡΠΈΠ²ΠΎΠΏΠΎΠ»ΠΎΠΆΠ½ΠΎ Π½Π°ΠΏΡΠ°Π²Π»Π΅Π½ΠΈΡ Π»ΠΈΠ½ΠΈΠΉ ΠΌΠ°Π³Π½ΠΈΡΠ½ΠΎΠΉ ΠΈΠ½Π΄ΡΠΊΡΠΈΠΈ Π²Π½Π΅ΡΠ½Π΅Π³ΠΎ ΠΏΠΎΠ»Ρ;
- ΠΏΠΎ ΠΏΡΠ°Π²ΠΈΠ»Ρ Π±ΡΡΠ°Π²ΡΠΈΠΊΠ°, Π·Π½Π°Ρ Π½Π°ΠΏΡΠ°Π²Π»Π΅Π½ΠΈΠ΅ Π»ΠΈΠ½ΠΈΠΉ ΠΈΠ½Π΄ΡΠΊΡΠΈΠΈ ΠΌΠ°Π³Π½ΠΈΡΠ½ΠΎΠ³ΠΎ ΠΏΠΎΠ»Ρ ΠΈΠ½Π΄ΡΠΊΡΠΈΠΎΠ½Π½ΠΎΠ³ΠΎ ΡΠΎΠΊΠ°, ΠΎΠΏΡΠ΅Π΄Π΅Π»ΠΈΡΡ Π½Π°ΠΏΡΠ°Π²Π»Π΅Π½ΠΈΠ΅ ΠΈΠ½Π΄ΡΠΊΡΠΈΠΎΠ½Π½ΠΎΠ³ΠΎ ΡΠΎΠΊΠ°.
ΠΡΠ°Π²ΠΈΠ»ΠΎ ΠΠ΅Π½ΡΠ° ΠΈΠΌΠ΅Π΅Ρ Π³Π»ΡΠ±ΠΎΠΊΠΈΠΉ ΡΠΈΠ·ΠΈΡΠ΅ΡΠΊΠΈΠΉ ΡΠΌΡΡΠ» β ΠΎΠ½ΠΎ Π²ΡΡΠ°ΠΆΠ°Π΅Ρ Π·Π°ΠΊΠΎΠ½ ΡΠΎΡ ΡΠ°Π½Π΅Π½ΠΈΡ ΡΠ½Π΅ΡΠ³ΠΈΠΈ.
Π‘Π°ΠΌΠΎΠΈΠ½Π΄ΡΠΊΡΠΈΡ
Π‘Π°ΠΌΠΎΠΈΠ½Π΄ΡΠΊΡΠΈΡ β ΡΡΠΎ ΡΠ²Π»Π΅Π½ΠΈΠ΅ Π²ΠΎΠ·Π½ΠΈΠΊΠ½ΠΎΠ²Π΅Π½ΠΈΡ ΠΠΠ‘ ΠΈΠ½Π΄ΡΠΊΡΠΈΠΈ Π² ΠΏΡΠΎΠ²ΠΎΠ΄Π½ΠΈΠΊΠ΅ Π² ΡΠ΅Π·ΡΠ»ΡΡΠ°ΡΠ΅ ΠΈΠ·ΠΌΠ΅Π½Π΅Π½ΠΈΡ ΡΠΎΠΊΠ° Π² Π½Π΅ΠΌ.
ΠΡΠΈ ΠΈΠ·ΠΌΠ΅Π½Π΅Π½ΠΈΠΈ ΡΠΈΠ»Ρ ΡΠΎΠΊΠ° Π² ΠΊΠ°ΡΡΡΠΊΠ΅ ΠΏΡΠΎΠΈΡΡ ΠΎΠ΄ΠΈΡ ΠΈΠ·ΠΌΠ΅Π½Π΅Π½ΠΈΠ΅ ΠΌΠ°Π³Π½ΠΈΡΠ½ΠΎΠ³ΠΎ ΠΏΠΎΡΠΎΠΊΠ°, ΡΠΎΠ·Π΄Π°Π²Π°Π΅ΠΌΠΎΠ³ΠΎ ΡΡΠΈΠΌ ΡΠΎΠΊΠΎΠΌ. ΠΠ·ΠΌΠ΅Π½Π΅Π½ΠΈΠ΅ ΠΌΠ°Π³Π½ΠΈΡΠ½ΠΎΠ³ΠΎ ΠΏΠΎΡΠΎΠΊΠ°, ΠΏΡΠΎΠ½ΠΈΠ·ΡΠ²Π°ΡΡΠ΅Π³ΠΎ ΠΊΠ°ΡΡΡΠΊΡ, Π΄ΠΎΠ»ΠΆΠ½ΠΎ Π²ΡΠ·ΡΠ²Π°ΡΡ ΠΏΠΎΡΠ²Π»Π΅Π½ΠΈΠ΅ ΠΠΠ‘ ΠΈΠ½Π΄ΡΠΊΡΠΈΠΈ Π² ΠΊΠ°ΡΡΡΠΊΠ΅.
Π ΡΠΎΠΎΡΠ²Π΅ΡΡΡΠ²ΠΈΠΈ Ρ ΠΏΡΠ°Π²ΠΈΠ»ΠΎΠΌ ΠΠ΅Π½ΡΠ° ΠΠΠ‘ ΡΠ°ΠΌΠΎΠΈΠ½Π΄ΡΠΊΡΠΈΠΈ ΠΏΡΠ΅ΠΏΡΡΡΡΠ²ΡΠ΅Ρ Π½Π°ΡΠ°ΡΡΠ°Π½ΠΈΡ ΡΠΈΠ»Ρ ΡΠΎΠΊΠ° ΠΏΡΠΈ Π²ΠΊΠ»ΡΡΠ΅Π½ΠΈΠΈ ΠΈ ΡΠ±ΡΠ²Π°Π½ΠΈΡ ΡΠΈΠ»Ρ ΡΠΎΠΊΠ° ΠΏΡΠΈ Π²ΡΠΊΠ»ΡΡΠ΅Π½ΠΈΠΈ ΡΠ΅ΠΏΠΈ.
ΠΡΠΎ ΠΏΡΠΈΠ²ΠΎΠ΄ΠΈΡ ΠΊ ΡΠΎΠΌΡ, ΡΡΠΎ ΠΏΡΠΈ Π·Π°ΠΌΡΠΊΠ°Π½ΠΈΠΈ ΡΠ΅ΠΏΠΈ, Π² ΠΊΠΎΡΠΎΡΠΎΠΉ Π΅ΡΡΡ ΠΈΡΡΠΎΡΠ½ΠΈΠΊ ΡΠΎΠΊΠ° Ρ ΠΏΠΎΡΡΠΎΡΠ½Π½ΠΎΠΉ ΠΠΠ‘, ΡΠΈΠ»Π° ΡΠΎΠΊΠ° ΡΡΡΠ°Π½Π°Π²Π»ΠΈΠ²Π°Π΅ΡΡΡ ΡΠ΅ΡΠ΅Π· Π½Π΅ΠΊΠΎΡΠΎΡΠΎΠ΅ Π²ΡΠ΅ΠΌΡ.
ΠΡΠΈ ΠΎΡΠΊΠ»ΡΡΠ΅Π½ΠΈΠΈ ΠΈΡΡΠΎΡΠ½ΠΈΠΊΠ° ΡΠΎΠΊ ΡΠ°ΠΊΠΆΠ΅ Π½Π΅ ΠΏΡΠ΅ΠΊΡΠ°ΡΠ°Π΅ΡΡΡ ΠΌΠ³Π½ΠΎΠ²Π΅Π½Π½ΠΎ. ΠΠΎΠ·Π½ΠΈΠΊΠ°ΡΡΠ°Ρ ΠΏΡΠΈ ΡΡΠΎΠΌ ΠΠΠ‘ ΡΠ°ΠΌΠΎΠΈΠ½Π΄ΡΠΊΡΠΈΠΈ ΠΌΠΎΠΆΠ΅Ρ ΠΏΡΠ΅Π²ΡΡΠ°ΡΡ ΠΠΠ‘ ΠΈΡΡΠΎΡΠ½ΠΈΠΊΠ°.
Π―Π²Π»Π΅Π½ΠΈΠ΅ ΡΠ°ΠΌΠΎΠΈΠ½Π΄ΡΠΊΡΠΈΠΈ ΠΌΠΎΠΆΠ½ΠΎ Π½Π°Π±Π»ΡΠ΄Π°ΡΡ, ΡΠΎΠ±ΡΠ°Π² ΡΠ»Π΅ΠΊΡΡΠΈΡΠ΅ΡΠΊΡΡ ΡΠ΅ΠΏΡ ΠΈΠ· ΠΊΠ°ΡΡΡΠΊΠΈ Ρ Π±ΠΎΠ»ΡΡΠΎΠΉ ΠΈΠ½Π΄ΡΠΊΡΠΈΠ²Π½ΠΎΡΡΡΡ, ΡΠ΅Π·ΠΈΡΡΠΎΡΠ°, Π΄Π²ΡΡ ΠΎΠ΄ΠΈΠ½Π°ΠΊΠΎΠ²ΡΡ Π»Π°ΠΌΠΏ Π½Π°ΠΊΠ°Π»ΠΈΠ²Π°Π½ΠΈΡ ΠΈ ΠΈΡΡΠΎΡΠ½ΠΈΠΊΠ° ΡΠΎΠΊΠ°. Π Π΅Π·ΠΈΡΡΠΎΡ Π΄ΠΎΠ»ΠΆΠ΅Π½ ΠΈΠΌΠ΅ΡΡ ΡΠ°ΠΊΠΎΠ΅ ΠΆΠ΅ ΡΠ»Π΅ΠΊΡΡΠΈΡΠ΅ΡΠΊΠΎΠ΅ ΡΠΎΠΏΡΠΎΡΠΈΠ²Π»Π΅Π½ΠΈΠ΅, ΠΊΠ°ΠΊ ΠΈ ΠΏΡΠΎΠ²ΠΎΠ΄ ΠΊΠ°ΡΡΡΠΊΠΈ.
ΠΠΏΡΡ ΠΏΠΎΠΊΠ°Π·ΡΠ²Π°Π΅Ρ, ΡΡΠΎ ΠΏΡΠΈ Π·Π°ΠΌΡΠΊΠ°Π½ΠΈΠΈ ΡΠ΅ΠΏΠΈ ΡΠ»Π΅ΠΊΡΡΠΈΡΠ΅ΡΠΊΠ°Ρ Π»Π°ΠΌΠΏΠ°, Π²ΠΊΠ»ΡΡΠ΅Π½Π½Π°Ρ ΠΏΠΎΡΠ»Π΅Π΄ΠΎΠ²Π°ΡΠ΅Π»ΡΠ½ΠΎ Ρ ΠΊΠ°ΡΡΡΠΊΠΎΠΉ, Π·Π°Π³ΠΎΡΠ°Π΅ΡΡΡ Π½Π΅ΡΠΊΠΎΠ»ΡΠΊΠΎ ΠΏΠΎΠ·ΠΆΠ΅, ΡΠ΅ΠΌ Π»Π°ΠΌΠΏΠ°, Π²ΠΊΠ»ΡΡΠ΅Π½Π½Π°Ρ ΠΏΠΎΡΠ»Π΅Π΄ΠΎΠ²Π°ΡΠ΅Π»ΡΠ½ΠΎ Ρ ΡΠ΅Π·ΠΈΡΡΠΎΡΠΎΠΌ. ΠΠ°ΡΠ°ΡΡΠ°Π½ΠΈΡ ΡΠΎΠΊΠ° Π² ΡΠ΅ΠΏΠΈ ΠΊΠ°ΡΡΡΠΊΠΈ ΠΏΡΠΈ Π·Π°ΠΌΡΠΊΠ°Π½ΠΈΠΈ ΠΏΡΠ΅ΠΏΡΡΡΡΠ²ΡΠ΅Ρ ΠΠΠ‘ ΡΠ°ΠΌΠΎΠΈΠ½Π΄ΡΠΊΡΠΈΠΈ, Π²ΠΎΠ·Π½ΠΈΠΊΠ°ΡΡΠ°Ρ ΠΏΡΠΈ Π²ΠΎΠ·ΡΠ°ΡΡΠ°Π½ΠΈΠΈ ΠΌΠ°Π³Π½ΠΈΡΠ½ΠΎΠ³ΠΎ ΠΏΠΎΡΠΎΠΊΠ° Π² ΠΊΠ°ΡΡΡΠΊΠ΅.
ΠΡΠΈ ΠΎΡΠΊΠ»ΡΡΠ΅Π½ΠΈΠΈ ΠΈΡΡΠΎΡΠ½ΠΈΠΊΠ° ΡΠΎΠΊΠ° Π²ΡΠΏΡΡ ΠΈΠ²Π°ΡΡ ΠΎΠ±Π΅ Π»Π°ΠΌΠΏΡ. Π ΡΡΠΎΠΌ ΡΠ»ΡΡΠ°Π΅ ΡΠΎΠΊ Π² ΡΠ΅ΠΏΠΈ ΠΏΠΎΠ΄Π΄Π΅ΡΠΆΠΈΠ²Π°Π΅ΡΡΡ ΠΠΠ‘ ΡΠ°ΠΌΠΎΠΈΠ½Π΄ΡΠΊΡΠΈΠΈ, Π²ΠΎΠ·Π½ΠΈΠΊΠ°ΡΡΠ΅ΠΉ ΠΏΡΠΈ ΡΠ±ΡΠ²Π°Π½ΠΈΠΈ ΠΌΠ°Π³Π½ΠΈΡΠ½ΠΎΠ³ΠΎ ΠΏΠΎΡΠΎΠΊΠ° Π² ΠΊΠ°ΡΡΡΠΊΠ΅.
ΠΠΠ‘ ΡΠ°ΠΌΠΎΠΈΠ½Π΄ΡΠΊΡΠΈΠΈ β ( varepsilon_ ) β, Π²ΠΎΠ·Π½ΠΈΠΊΠ°ΡΡΠ°Ρ Π² ΠΊΠ°ΡΡΡΠΊΠ΅ Ρ ΠΈΠ½Π΄ΡΠΊΡΠΈΠ²Π½ΠΎΡΡΡΡ β ( L ) β, ΠΏΠΎ Π·Π°ΠΊΠΎΠ½Ρ ΡΠ»Π΅ΠΊΡΡΠΎΠΌΠ°Π³Π½ΠΈΡΠ½ΠΎΠΉ ΠΈΠ½Π΄ΡΠΊΡΠΈΠΈ ΡΠ°Π²Π½Π°:
ΠΠΠ‘ ΡΠ°ΠΌΠΎΠΈΠ½Π΄ΡΠΊΡΠΈΠΈ ΠΏΡΡΠΌΠΎ ΠΏΡΠΎΠΏΠΎΡΡΠΈΠΎΠ½Π°Π»ΡΠ½Π° ΠΈΠ½Π΄ΡΠΊΡΠΈΠ²Π½ΠΎΡΡΠΈ ΠΊΠ°ΡΡΡΠΊΠΈ ΠΈ ΡΠΊΠΎΡΠΎΡΡΠΈ ΠΈΠ·ΠΌΠ΅Π½Π΅Π½ΠΈΡ ΡΠΈΠ»Ρ ΡΠΎΠΊΠ° Π² ΠΊΠ°ΡΡΡΠΊΠ΅.
ΠΠ½Π΄ΡΠΊΡΠΈΠ²Π½ΠΎΡΡΡ
ΠΠ»Π΅ΠΊΡΡΠΈΡΠ΅ΡΠΊΠΈΠΉ ΡΠΎΠΊ, ΠΏΡΠΎΡ ΠΎΠ΄ΡΡΠΈΠΉ ΠΏΠΎ ΠΏΡΠΎΠ²ΠΎΠ΄Π½ΠΈΠΊΡ, ΡΠΎΠ·Π΄Π°Π΅Ρ Π²ΠΎΠΊΡΡΠ³ Π½Π΅Π³ΠΎ ΠΌΠ°Π³Π½ΠΈΡΠ½ΠΎΠ΅ ΠΏΠΎΠ»Π΅. ΠΠ°Π³Π½ΠΈΡΠ½ΡΠΉ ΠΏΠΎΡΠΎΠΊ β ( Phi ) β ΡΠ΅ΡΠ΅Π· ΠΊΠΎΠ½ΡΡΡ ΠΈΠ· ΡΡΠΎΠ³ΠΎ ΠΏΡΠΎΠ²ΠΎΠ΄Π½ΠΈΠΊΠ° ΠΏΡΠΎΠΏΠΎΡΡΠΈΠΎΠ½Π°Π»Π΅Π½ ΠΌΠΎΠ΄ΡΠ»Ρ ΠΈΠ½Π΄ΡΠΊΡΠΈΠΈ β ( vec ) β ΠΌΠ°Π³Π½ΠΈΡΠ½ΠΎΠ³ΠΎ ΠΏΠΎΠ»Ρ Π²Π½ΡΡΡΠΈ ΠΊΠΎΠ½ΡΡΡΠ°, Π° ΠΈΠ½Π΄ΡΠΊΡΠΈΡ ΠΌΠ°Π³Π½ΠΈΡΠ½ΠΎΠ³ΠΎ ΠΏΠΎΠ»Ρ, Π² ΡΠ²ΠΎΡ ΠΎΡΠ΅ΡΠ΅Π΄Ρ, ΠΏΡΠΎΠΏΠΎΡΡΠΈΠΎΠ½Π°Π»ΡΠ½Π° ΡΠΈΠ»Π΅ ΡΠΎΠΊΠ° Π² ΠΏΡΠΎΠ²ΠΎΠ΄Π½ΠΈΠΊΠ΅.
Π‘Π»Π΅Π΄ΠΎΠ²Π°ΡΠ΅Π»ΡΠ½ΠΎ, ΠΌΠ°Π³Π½ΠΈΡΠ½ΡΠΉ ΠΏΠΎΡΠΎΠΊ ΡΠ΅ΡΠ΅Π· ΠΊΠΎΠ½ΡΡΡ ΠΏΡΡΠΌΠΎ ΠΏΡΠΎΠΏΠΎΡΡΠΈΠΎΠ½Π°Π»Π΅Π½ ΡΠΈΠ»Π΅ ΡΠΎΠΊΠ° Π² ΠΊΠΎΠ½ΡΡΡΠ΅:
ΠΠ½Π΄ΡΠΊΡΠΈΠ²Π½ΠΎΡΡΡ β ΠΊΠΎΡΡΡΠΈΡΠΈΠ΅Π½Ρ ΠΏΡΠΎΠΏΠΎΡΡΠΈΠΎΠ½Π°Π»ΡΠ½ΠΎΡΡΠΈ β ( L ) β ΠΌΠ΅ΠΆΠ΄Ρ ΡΠΈΠ»ΠΎΠΉ ΡΠΎΠΊΠ° β ( I ) β Π² ΠΊΠΎΠ½ΡΡΡΠ΅ ΠΈ ΠΌΠ°Π³Π½ΠΈΡΠ½ΡΠΌ ΠΏΠΎΡΠΎΠΊΠΎΠΌ β ( Phi ) β, ΡΠΎΠ·Π΄Π°Π²Π°Π΅ΠΌΡΠΌ ΡΡΠΈΠΌ ΡΠΎΠΊΠΎΠΌ:
ΠΠ½Π΄ΡΠΊΡΠΈΠ²Π½ΠΎΡΡΡ Π·Π°Π²ΠΈΡΠΈΡ ΠΎΡ ΡΠ°Π·ΠΌΠ΅ΡΠΎΠ² ΠΈ ΡΠΎΡΠΌΡ ΠΏΡΠΎΠ²ΠΎΠ΄Π½ΠΈΠΊΠ°, ΠΎΡ ΠΌΠ°Π³Π½ΠΈΡΠ½ΡΡ ΡΠ²ΠΎΠΉΡΡΠ² ΡΡΠ΅Π΄Ρ, Π² ΠΊΠΎΡΠΎΡΠΎΠΉ Π½Π°Ρ ΠΎΠ΄ΠΈΡΡΡ ΠΏΡΠΎΠ²ΠΎΠ΄Π½ΠΈΠΊ.
ΠΠ΄ΠΈΠ½ΠΈΡΠ° ΠΈΠ½Π΄ΡΠΊΡΠΈΠ²Π½ΠΎΡΡΠΈ Π² Π‘Π β Π³Π΅Π½ΡΠΈ (ΠΠ½). ΠΠ½Π΄ΡΠΊΡΠΈΠ²Π½ΠΎΡΡΡ ΠΊΠΎΠ½ΡΡΡΠ° ΡΠ°Π²Π½Π° 1 Π³Π΅Π½ΡΠΈ, Π΅ΡΠ»ΠΈ ΠΏΡΠΈ ΡΠΈΠ»Π΅ ΠΏΠΎΡΡΠΎΡΠ½Π½ΠΎΠ³ΠΎ ΡΠΎΠΊΠ° 1 Π°ΠΌΠΏΠ΅Ρ ΠΌΠ°Π³Π½ΠΈΡΠ½ΡΠΉ ΠΏΠΎΡΠΎΠΊ ΡΠ΅ΡΠ΅Π· ΠΊΠΎΠ½ΡΡΡ ΡΠ°Π²Π΅Π½ 1 Π²Π΅Π±Π΅Ρ:
ΠΠΎΠΆΠ½ΠΎ Π΄Π°ΡΡ Π²ΡΠΎΡΠΎΠ΅ ΠΎΠΏΡΠ΅Π΄Π΅Π»Π΅Π½ΠΈΠ΅ Π΅Π΄ΠΈΠ½ΠΈΡΡ ΠΈΠ½Π΄ΡΠΊΡΠΈΠ²Π½ΠΎΡΡΠΈ: ΡΠ»Π΅ΠΌΠ΅Π½Ρ ΡΠ»Π΅ΠΊΡΡΠΈΡΠ΅ΡΠΊΠΎΠΉ ΡΠ΅ΠΏΠΈ ΠΎΠ±Π»Π°Π΄Π°Π΅Ρ ΠΈΠ½Π΄ΡΠΊΡΠΈΠ²Π½ΠΎΡΡΡΡ Π² 1 ΠΠ½, Π΅ΡΠ»ΠΈ ΠΏΡΠΈ ΡΠ°Π²Π½ΠΎΠΌΠ΅ΡΠ½ΠΎΠΌ ΠΈΠ·ΠΌΠ΅Π½Π΅Π½ΠΈΠΈ ΡΠΈΠ»Ρ ΡΠΎΠΊΠ° Π² ΡΠ΅ΠΏΠΈ Π½Π° 1 Π°ΠΌΠΏΠ΅Ρ Π·Π° 1 Ρ Π² Π½Π΅ΠΌ Π²ΠΎΠ·Π½ΠΈΠΊΠ°Π΅Ρ ΠΠΠ‘ ΡΠ°ΠΌΠΎΠΈΠ½Π΄ΡΠΊΡΠΈΠΈ 1 Π²ΠΎΠ»ΡΡ.
ΠΠ½Π΅ΡΠ³ΠΈΡ ΠΌΠ°Π³Π½ΠΈΡΠ½ΠΎΠ³ΠΎ ΠΏΠΎΠ»Ρ
ΠΡΠΈ ΠΎΡΠΊΠ»ΡΡΠ΅Π½ΠΈΠΈ ΠΊΠ°ΡΡΡΠΊΠΈ ΠΈΠ½Π΄ΡΠΊΡΠΈΠ²Π½ΠΎΡΡΠΈ ΠΎΡ ΠΈΡΡΠΎΡΠ½ΠΈΠΊΠ° ΡΠΎΠΊΠ° Π»Π°ΠΌΠΏΠ° Π½Π°ΠΊΠ°Π»ΠΈΠ²Π°Π½ΠΈΡ, Π²ΠΊΠ»ΡΡΠ΅Π½Π½Π°Ρ ΠΏΠ°ΡΠ°Π»Π»Π΅Π»ΡΠ½ΠΎ ΠΊΠ°ΡΡΡΠΊΠ΅, Π΄Π°Π΅Ρ ΠΊΡΠ°ΡΠΊΠΎΠ²ΡΠ΅ΠΌΠ΅Π½Π½ΡΡ Π²ΡΠΏΡΡΠΊΡ. Π’ΠΎΠΊ Π² ΡΠ΅ΠΏΠΈ Π²ΠΎΠ·Π½ΠΈΠΊΠ°Π΅Ρ ΠΏΠΎΠ΄ Π΄Π΅ΠΉΡΡΠ²ΠΈΠ΅ΠΌ ΠΠΠ‘ ΡΠ°ΠΌΠΎΠΈΠ½Π΄ΡΠΊΡΠΈΠΈ.
ΠΡΡΠΎΡΠ½ΠΈΠΊΠΎΠΌ ΡΠ½Π΅ΡΠ³ΠΈΠΈ, Π²ΡΠ΄Π΅Π»ΡΡΡΠ΅ΠΉΡΡ ΠΏΡΠΈ ΡΡΠΎΠΌ Π² ΡΠ»Π΅ΠΊΡΡΠΈΡΠ΅ΡΠΊΠΎΠΉ ΡΠ΅ΠΏΠΈ, ΡΠ²Π»ΡΠ΅ΡΡΡ ΠΌΠ°Π³Π½ΠΈΡΠ½ΠΎΠ΅ ΠΏΠΎΠ»Π΅ ΠΊΠ°ΡΡΡΠΊΠΈ.
ΠΠ»Ρ ΡΠΎΠ·Π΄Π°Π½ΠΈΡ ΡΠΎΠΊΠ° Π² ΠΊΠΎΠ½ΡΡΡΠ΅ Ρ ΠΈΠ½Π΄ΡΠΊΡΠΈΠ²Π½ΠΎΡΡΡΡ Π½Π΅ΠΎΠ±Ρ ΠΎΠ΄ΠΈΠΌΠΎ ΡΠΎΠ²Π΅ΡΡΠΈΡΡ ΡΠ°Π±ΠΎΡΡ Π½Π° ΠΏΡΠ΅ΠΎΠ΄ΠΎΠ»Π΅Π½ΠΈΠ΅ ΠΠΠ‘ ΡΠ°ΠΌΠΎΠΈΠ½Π΄ΡΠΊΡΠΈΠΈ. ΠΠ½Π΅ΡΠ³ΠΈΡ ΠΌΠ°Π³Π½ΠΈΡΠ½ΠΎΠ³ΠΎ ΠΏΠΎΠ»Ρ ΡΠΎΠΊΠ° Π²ΡΡΠΈΡΠ»ΡΠ΅ΡΡΡ ΠΏΠΎ ΡΠΎΡΠΌΡΠ»Π΅:
ΠΡΠ½ΠΎΠ²Π½ΡΠ΅ ΡΠΎΡΠΌΡΠ»Ρ ΡΠ°Π·Π΄Π΅Π»Π° Β«ΠΠ»Π΅ΠΊΡΡΠΎΠΌΠ°Π³Π½ΠΈΡΠ½Π°Ρ ΠΈΠ½Π΄ΡΠΊΡΠΈΡΒ»
ΠΠ»Π³ΠΎΡΠΈΡΠΌ ΡΠ΅ΡΠ΅Π½ΠΈΡ Π·Π°Π΄Π°Ρ ΠΏΠΎ ΡΠ΅ΠΌΠ΅ Β«ΠΠ»Π΅ΠΊΡΡΠΎΠΌΠ°Π³Π½ΠΈΡΠ½Π°Ρ ΠΈΠ½Π΄ΡΠΊΡΠΈΡΒ»:
1. ΠΠ½ΠΈΠΌΠ°ΡΠ΅Π»ΡΠ½ΠΎ ΠΏΡΠΎΡΠΈΡΠ°ΡΡ ΡΡΠ»ΠΎΠ²ΠΈΠ΅ Π·Π°Π΄Π°ΡΠΈ. Π£ΡΡΠ°Π½ΠΎΠ²ΠΈΡΡ ΠΏΡΠΈΡΠΈΠ½Ρ ΠΈΠ·ΠΌΠ΅Π½Π΅Π½ΠΈΡ ΠΌΠ°Π³Π½ΠΈΡΠ½ΠΎΠ³ΠΎ ΠΏΠΎΡΠΎΠΊΠ°, ΠΏΡΠΎΠ½ΠΈΠ·ΡΠ²Π°ΡΡΠ΅Π³ΠΎ ΠΊΠΎΠ½ΡΡΡ.
2. ΠΠ°ΠΏΠΈΡΠ°ΡΡ ΡΠΎΡΠΌΡΠ»Ρ:
- Π·Π°ΠΊΠΎΠ½Π° ΡΠ»Π΅ΠΊΡΡΠΎΠΌΠ°Π³Π½ΠΈΡΠ½ΠΎΠΉ ΠΈΠ½Π΄ΡΠΊΡΠΈΠΈ;
- ΠΠΠ‘ ΠΈΠ½Π΄ΡΠΊΡΠΈΠΈ Π² Π΄Π²ΠΈΠΆΡΡΠ΅ΠΌΡΡ ΠΏΡΠΎΠ²ΠΎΠ΄Π½ΠΈΠΊΠ΅, Π΅ΡΠ»ΠΈ Π² Π·Π°Π΄Π°ΡΠ΅ ΡΠ°ΡΡΠΌΠ°ΡΡΠΈΠ²Π°Π΅ΡΡΡ ΠΏΠΎΡΡΡΠΏΠ°ΡΠ΅Π»ΡΠ½ΠΎ Π΄Π²ΠΈΠΆΡΡΠΈΠΉΡΡ ΠΏΡΠΎΠ²ΠΎΠ΄Π½ΠΈΠΊ; Π΅ΡΠ»ΠΈ Π² Π·Π°Π΄Π°ΡΠ΅ ΡΠ°ΡΡΠΌΠ°ΡΡΠΈΠ²Π°Π΅ΡΡΡ ΡΠ»Π΅ΠΊΡΡΠΈΡΠ΅ΡΠΊΠ°Ρ ΡΠ΅ΠΏΡ, ΡΠΎΠ΄Π΅ΡΠΆΠ°ΡΠ°Ρ ΠΈΡΡΠΎΡΠ½ΠΈΠΊ ΡΠΎΠΊΠ°, ΠΈ Π²ΠΎΠ·Π½ΠΈΠΊΠ°ΡΡΠ°Ρ Π½Π° ΠΎΠ΄Π½ΠΎΠΌ ΠΈΠ· ΡΡΠ°ΡΡΠΊΠΎΠ² ΠΠΠ‘ ΠΈΠ½Π΄ΡΠΊΡΠΈΠΈ, Π²ΡΠ·Π²Π°Π½Π½Π°Ρ Π΄Π²ΠΈΠΆΠ΅Π½ΠΈΠ΅ΠΌ ΠΏΡΠΎΠ²ΠΎΠ΄Π½ΠΈΠΊΠ° Π² ΠΌΠ°Π³Π½ΠΈΡΠ½ΠΎΠΌ ΠΏΠΎΠ»Π΅, ΡΠΎ ΡΠ½Π°ΡΠ°Π»Π° Π½ΡΠΆΠ½ΠΎ ΠΎΠΏΡΠ΅Π΄Π΅Π»ΠΈΡΡ Π²Π΅Π»ΠΈΡΠΈΠ½Ρ ΠΈ Π½Π°ΠΏΡΠ°Π²Π»Π΅Π½ΠΈΠ΅ ΠΠΠ‘ ΠΈΠ½Π΄ΡΠΊΡΠΈΠΈ. ΠΠΎΡΠ»Π΅ ΡΡΠΎΠ³ΠΎ Π·Π°Π΄Π°ΡΠ° ΡΠ΅ΡΠ°Π΅ΡΡΡ ΠΏΠΎ Π°Π½Π°Π»ΠΎΠ³ΠΈΠΈ Ρ Π·Π°Π΄Π°ΡΠ°ΠΌΠΈ Π½Π° ΡΠ°ΡΡΠ΅Ρ ΡΠ΅ΠΏΠΈ ΠΏΠΎΡΡΠΎΡΠ½Π½ΠΎΠ³ΠΎ ΡΠΎΠΊΠ° Ρ Π½Π΅ΡΠΊΠΎΠ»ΡΠΊΠΈΠΌΠΈ ΠΈΡΡΠΎΡΠ½ΠΈΠΊΠ°ΠΌΠΈ.
3. ΠΠ°ΠΏΠΈΡΠ°ΡΡ Π²ΡΡΠ°ΠΆΠ΅Π½ΠΈΠ΅ Π΄Π»Ρ ΠΈΠ·ΠΌΠ΅Π½Π΅Π½ΠΈΡ ΠΌΠ°Π³Π½ΠΈΡΠ½ΠΎΠ³ΠΎ ΠΏΠΎΡΠΎΠΊΠ° ΠΈ ΠΏΠΎΠ΄ΡΡΠ°Π²ΠΈΡΡ Π² ΡΠΎΡΠΌΡΠ»Ρ Π·Π°ΠΊΠΎΠ½Π° ΡΠ»Π΅ΠΊΡΡΠΎΠΌΠ°Π³Π½ΠΈΡΠ½ΠΎΠΉ ΠΈΠ½Π΄ΡΠΊΡΠΈΠΈ.
4. ΠΠ°ΠΏΠΈΡΠ°ΡΡ ΠΌΠ°ΡΠ΅ΠΌΠ°ΡΠΈΡΠ΅ΡΠΊΠΈ Π²ΡΠ΅ Π΄ΠΎΠΏΠΎΠ»Π½ΠΈΡΠ΅Π»ΡΠ½ΡΠ΅ ΡΡΠ»ΠΎΠ²ΠΈΡ (ΡΠ°ΡΠ΅ Π²ΡΠ΅Π³ΠΎ ΡΡΠΎ ΡΠΎΡΠΌΡΠ»Ρ Π·Π°ΠΊΠΎΠ½Π° ΠΠΌΠ° Π΄Π»Ρ ΠΏΠΎΠ»Π½ΠΎΠΉ ΡΠ΅ΠΏΠΈ, ΡΠΈΠ»Ρ ΠΠΌΠΏΠ΅ΡΠ° ΠΈΠ»ΠΈ ΡΠΈΠ»Ρ ΠΠΎΡΠ΅Π½ΡΠ°, ΡΠΎΡΠΌΡΠ»Ρ ΠΊΠΈΠ½Π΅ΠΌΠ°ΡΠΈΠΊΠΈ ΠΈ Π΄ΠΈΠ½Π°ΠΌΠΈΠΊΠΈ).
5. Π Π΅ΡΠΈΡΡ ΠΏΠΎΠ»ΡΡΠ΅Π½Π½ΡΡ ΡΠΈΡΡΠ΅ΠΌΡ ΡΡΠ°Π²Π½Π΅Π½ΠΈΠΉ ΠΎΡΠ½ΠΎΡΠΈΡΠ΅Π»ΡΠ½ΠΎ ΠΈΡΠΊΠΎΠΌΠΎΠΉ Π²Π΅Π»ΠΈΡΠΈΠ½Ρ.
ΠΡΠΈΡΠΈΠ½ΠΎΠΉ ΡΠ»Π΅ΠΊΡΡΠΎΠ΄Π²ΠΈΠΆΡΡΠ΅ΠΉ ΡΠΈΠ»Ρ ΠΌΠΎΠΆΠ΅Ρ ΡΡΠ°ΡΡ ΠΈΠ·ΠΌΠ΅Π½Π΅Π½ΠΈΠ΅ ΠΌΠ°Π³Π½ΠΈΡΠ½ΠΎΠ³ΠΎ ΠΏΠΎΠ»Ρ Π² ΠΎΠΊΡΡΠΆΠ°ΡΡΠ΅ΠΌ ΠΏΡΠΎΡΡΡΠ°Π½ΡΡΠ²Π΅. ΠΡΠΎ ΡΠ²Π»Π΅Π½ΠΈΠ΅ Π½Π°Π·ΡΠ²Π°Π΅ΡΡΡΡΠ»Π΅ΠΊΡΡΠΎΠΌΠ°Π³Π½ΠΈΡΠ½ΠΎΠΉ ΠΈΠ½Π΄ΡΠΊΡΠΈΠ΅ΠΉ. ΠΠ΅Π»ΠΈΡΠΈΠ½Π° ΠΠΠ‘ ΠΈΠ½Π΄ΡΠΊΡΠΈΠΈ Π² ΠΊΠΎΠ½ΡΡΡΠ΅ ΠΎΠΏΡΠ΅Π΄Π΅Π»ΡΠ΅ΡΡΡ Π²ΡΡΠ°ΠΆΠ΅Π½ΠΈΠ΅ΠΌ
Π³Π΄Π΅ β ΠΏΠΎΡΠΎΠΊ ΠΌΠ°Π³Π½ΠΈΡΠ½ΠΎΠ³ΠΎ ΠΏΠΎΠ»Ρ ΡΠ΅ΡΠ΅Π· Π·Π°ΠΌΠΊΠ½ΡΡΡΡ ΠΏΠΎΠ²Π΅ΡΡ Π½ΠΎΡΡΡ , ΠΎΠ³ΡΠ°Π½ΠΈΡΠ΅Π½Π½ΡΡ ΠΊΠΎΠ½ΡΡΡΠΎΠΌ. ΠΠ½Π°ΠΊ Β«βΒ» ΠΏΠ΅ΡΠ΅Π΄ Π²ΡΡΠ°ΠΆΠ΅Π½ΠΈΠ΅ΠΌ ΠΏΠΎΠΊΠ°Π·ΡΠ²Π°Π΅Ρ, ΡΡΠΎ ΠΈΠ½Π΄ΡΠΊΡΠΈΠΎΠ½Π½ΡΠΉ ΡΠΎΠΊ, ΡΠΎΠ·Π΄Π°Π½Π½ΡΠΉ ΠΠΠ‘ ΠΈΠ½Π΄ΡΠΊΡΠΈΠΈ, ΠΏΡΠ΅ΠΏΡΡΡΡΠ²ΡΠ΅Ρ ΠΈΠ·ΠΌΠ΅Π½Π΅Π½ΠΈΡ ΠΌΠ°Π³Π½ΠΈΡΠ½ΠΎΠ³ΠΎ ΠΏΠΎΡΠΎΠΊΠ° Π² ΠΊΠΎΠ½ΡΡΡΠ΅ (ΡΠΌ. ΠΏΡΠ°Π²ΠΈΠ»ΠΎ ΠΠ΅Π½ΡΠ°).
41. ΠΠ½Π΄ΡΠΊΡΠΈΠ²Π½ΠΎΡΡΡ, Π΅Π΅ Π΅Π΄ΠΈΠ½ΠΈΡΠ° Π‘Π. ΠΠ½Π΄ΡΠΊΡΠΈΠ²Π½ΠΎΡΡΡ Π΄Π»ΠΈΠ½Π½ΠΎΠ³ΠΎ ΡΠΎΠ»Π΅Π½ΠΎΠΈΠ΄Π°.
ΠΠ½Π΄ΡΠΊΡΠΈΜΠ²Π½ΠΎΡΡΡ (ΠΈΠ»ΠΈ ΠΊΠΎΡΡΡΠΈΡΠΈΠ΅Π½Ρ ΡΠ°ΠΌΠΎΠΈΠ½Π΄ΡΠΊΡΠΈΠΈ) β ΠΊΠΎΡΡΡΠΈΡΠΈΠ΅Π½Ρ ΠΏΡΠΎΠΏΠΎΡΡΠΈΠΎΠ½Π°Π»ΡΠ½ΠΎΡΡΠΈ ΠΌΠ΅ΠΆΠ΄Ρ ΡΠ»Π΅ΠΊΡΡΠΈΡΠ΅ΡΠΊΠΈΠΌ ΡΠΎΠΊΠΎΠΌ, ΡΠ΅ΠΊΡΡΠΈΠΌ Π² ΠΊΠ°ΠΊΠΎΠΌ-Π»ΠΈΠ±ΠΎ Π·Π°ΠΌΠΊΠ½ΡΡΠΎΠΌ ΠΊΠΎΠ½ΡΡΡΠ΅, ΠΈ ΠΌΠ°Π³Π½ΠΈΡΠ½ΡΠΌ ΠΏΠΎΡΠΎΠΊΠΎΠΌ, ΡΠΎΠ·Π΄Π°Π²Π°Π΅ΠΌΡΠΌ ΡΡΠΈΠΌ ΡΠΎΠΊΠΎΠΌ ΡΠ΅ΡΠ΅Π· ΠΏΠΎΠ²Π΅ΡΡ Π½ΠΎΡΡΡ [1] , ΠΊΡΠ°Π΅ΠΌ ΠΊΠΎΡΠΎΡΠΎΠΉ ΡΠ²Π»ΡΠ΅ΡΡΡ ΡΡΠΎΡ ΠΊΠΎΠ½ΡΡΡ. [2][3][4] .
β ΠΌΠ°Π³Π½ΠΈΡΠ½ΡΠΉ ΠΏΠΎΡΠΎΠΊ, β ΡΠΎΠΊ Π² ΠΊΠΎΠ½ΡΡΡΠ΅, β ΠΈΠ½Π΄ΡΠΊΡΠΈΠ²Π½ΠΎΡΡΡ.
ΠΠ΅ΡΠ΅Π΄ΠΊΠΎ Π³ΠΎΠ²ΠΎΡΡΡ ΠΎΠ± ΠΈΠ½Π΄ΡΠΊΡΠΈΠ²Π½ΠΎΡΡΠΈ ΠΏΡΡΠΌΠΎΠ³ΠΎ Π΄Π»ΠΈΠ½Π½ΠΎΠ³ΠΎ ΠΏΡΠΎΠ²ΠΎΠ΄Π°(ΡΠΌ.). Π ΡΡΠΎΠΌ ΡΠ»ΡΡΠ°Π΅ ΠΈ Π΄ΡΡΠ³ΠΈΡ (ΠΎΡΠΎΠ±Π΅Π½Π½ΠΎ β Π² Π½Π΅ ΠΎΡΠ²Π΅ΡΠ°ΡΡΠΈΡ ΠΊΠ²Π°Π·ΠΈΡΡΠ°ΡΠΈΠΎΠ½Π°ΡΠ½ΠΎΠΌΡ ΠΏΡΠΈΠ±Π»ΠΈΠΆΠ΅Π½ΠΈΡ) ΡΠ»ΡΡΠ°ΡΡ , ΠΊΠΎΠ³Π΄Π° Π·Π°ΠΌΠΊΠ½ΡΡΡΠΉ ΠΊΠΎΠ½ΡΡΡ Π½Π΅ΠΏΡΠΎΡΡΠΎ Π°Π΄Π΅ΠΊΠ²Π°ΡΠ½ΠΎ ΠΈ ΠΎΠ΄Π½ΠΎΠ·Π½Π°ΡΠ½ΠΎ ΡΠΊΠ°Π·Π°ΡΡ, ΠΏΡΠΈΠ²Π΅Π΄Π΅Π½Π½ΠΎΠ΅ Π²ΡΡΠ΅ ΠΎΠΏΡΠ΅Π΄Π΅Π»Π΅Π½ΠΈΠ΅ ΡΡΠ΅Π±ΡΠ΅Ρ ΠΎΡΠΎΠ±ΡΡ ΡΡΠΎΡΠ½Π΅Π½ΠΈΠΉ; ΠΎΡΡΠ°ΡΡΠΈ ΠΏΠΎΠ»Π΅Π·Π½ΡΠΌ Π΄Π»Ρ ΡΡΠΎΠ³ΠΎ ΠΎΠΊΠ°Π·ΡΠ²Π°Π΅ΡΡΡ ΠΏΠΎΠ΄Ρ ΠΎΠ΄ (ΡΠΏΠΎΠΌΠΈΠ½Π°Π΅ΠΌΡΠΉ Π½ΠΈΠΆΠ΅), ΡΠ²ΡΠ·ΡΠ²Π°ΡΡΠΈΠΉ ΠΈΠ½Π΄ΡΠΊΡΠΈΠ²Π½ΠΎΡΡΡ Ρ ΡΠ½Π΅ΡΠ³ΠΈΠ΅ΠΉ ΠΌΠ°Π³Π½ΠΈΡΠ½ΠΎΠ³ΠΎ ΠΏΠΎΠ»Ρ.
Π§Π΅ΡΠ΅Π· ΠΈΠ½Π΄ΡΠΊΡΠΈΠ²Π½ΠΎΡΡΡ Π²ΡΡΠ°ΠΆΠ°Π΅ΡΡΡ ΠΠΠ‘ ΡΠ°ΠΌΠΎΠΈΠ½Π΄ΡΠΊΡΠΈΠΈ Π² ΠΊΠΎΠ½ΡΡΡΠ΅, Π²ΠΎΠ·Π½ΠΈΠΊΠ°ΡΡΠ°Ρ ΠΏΡΠΈ ΠΈΠ·ΠΌΠ΅Π½Π΅Π½ΠΈΠΈ Π² Π½ΡΠΌ ΡΠΎΠΊΠ° [4] :
.
ΠΠ· ΡΡΠΎΠΉ ΡΠΎΡΠΌΡΠ»Ρ ΡΠ»Π΅Π΄ΡΠ΅Ρ, ΡΡΠΎ ΠΈΠ½Π΄ΡΠΊΡΠΈΠ²Π½ΠΎΡΡΡ ΡΠΈΡΠ»Π΅Π½Π½ΠΎ ΡΠ°Π²Π½Π° ΠΠΠ‘ ΡΠ°ΠΌΠΎΠΈΠ½Π΄ΡΠΊΡΠΈΠΈ, Π²ΠΎΠ·Π½ΠΈΠΊΠ°ΡΡΠ΅ΠΉ Π² ΠΊΠΎΠ½ΡΡΡΠ΅ ΠΏΡΠΈ ΠΈΠ·ΠΌΠ΅Π½Π΅Π½ΠΈΠΈ ΡΠΈΠ»Ρ ΡΠΎΠΊΠ° Π½Π° 1 Π Π·Π° 1 Ρ.
ΠΡΠΈ Π·Π°Π΄Π°Π½Π½ΠΎΠΉ ΡΠΈΠ»Π΅ ΡΠΎΠΊΠ° ΠΈΠ½Π΄ΡΠΊΡΠΈΠ²Π½ΠΎΡΡΡ ΠΎΠΏΡΠ΅Π΄Π΅Π»ΡΠ΅Ρ ΡΠ½Π΅ΡΠ³ΠΈΡ ΠΌΠ°Π³Π½ΠΈΡΠ½ΠΎΠ³ΠΎ ΠΏΠΎΠ»Ρ, ΡΠΎΠ·Π΄Π°Π²Π°Π΅ΠΌΠΎΠ³ΠΎ ΡΡΠΈΠΌ ΡΠΎΠΊΠΎΠΌ [4] :
.
ΠΠ±ΠΎΠ·Π½Π°ΡΠ΅Π½ΠΈΠ΅ ΠΈ Π΅Π΄ΠΈΠ½ΠΈΡΡ ΠΈΠ·ΠΌΠ΅ΡΠ΅Π½ΠΈΡ
Π ΡΠΈΡΡΠ΅ΠΌΠ΅ Π΅Π΄ΠΈΠ½ΠΈΡ Π‘Π ΠΈΠ½Π΄ΡΠΊΡΠΈΠ²Π½ΠΎΡΡΡ ΠΈΠ·ΠΌΠ΅ΡΡΠ΅ΡΡΡ Π² Π³Π΅Π½ΡΠΈ [7] , ΡΠΎΠΊΡΠ°ΡΠ΅Π½Π½ΠΎ ΠΠ½, Π² ΡΠΈΡΡΠ΅ΠΌΠ΅ Π‘ΠΠ‘ β Π² ΡΠ°Π½ΡΠΈΠΌΠ΅ΡΡΠ°Ρ (1 ΠΠ½ = 10 9 ΡΠΌ) [4] . ΠΠΎΠ½ΡΡΡ ΠΎΠ±Π»Π°Π΄Π°Π΅Ρ ΠΈΠ½Π΄ΡΠΊΡΠΈΠ²Π½ΠΎΡΡΡΡ Π² ΠΎΠ΄ΠΈΠ½ Π³Π΅Π½ΡΠΈ, Π΅ΡΠ»ΠΈ ΠΏΡΠΈ ΠΈΠ·ΠΌΠ΅Π½Π΅Π½ΠΈΠΈ ΡΠΎΠΊΠ° Π½Π° ΠΎΠ΄ΠΈΠ½ Π°ΠΌΠΏΠ΅Ρ Π² ΡΠ΅ΠΊΡΠ½Π΄Ρ Π½Π° Π²ΡΠ²ΠΎΠ΄Π°Ρ ΠΊΠΎΠ½ΡΡΡΠ° Π±ΡΠ΄Π΅Ρ Π²ΠΎΠ·Π½ΠΈΠΊΠ°ΡΡ Π½Π°ΠΏΡΡΠΆΠ΅Π½ΠΈΠ΅ Π² ΠΎΠ΄ΠΈΠ½ Π²ΠΎΠ»ΡΡ. Π Π΅Π°Π»ΡΠ½ΡΠΉ, Π½Π΅ ΡΠ²Π΅ΡΡ ΠΏΡΠΎΠ²ΠΎΠ΄ΡΡΠΈΠΉ, ΠΊΠΎΠ½ΡΡΡ ΠΎΠ±Π»Π°Π΄Π°Π΅Ρ ΠΎΠΌΠΈΡΠ΅ΡΠΊΠΈΠΌ ΡΠΎΠΏΡΠΎΡΠΈΠ²Π»Π΅Π½ΠΈΠ΅ΠΌ R, ΠΏΠΎΡΡΠΎΠΌΡ Π½Π° Π½ΡΠΌ Π±ΡΠ΄Π΅Ρ Π΄ΠΎΠΏΠΎΠ»Π½ΠΈΡΠ΅Π»ΡΠ½ΠΎ Π²ΠΎΠ·Π½ΠΈΠΊΠ°ΡΡ Π½Π°ΠΏΡΡΠΆΠ΅Π½ΠΈΠ΅ U=I*R, Π³Π΄Π΅ I β ΡΠΈΠ»Π° ΡΠΎΠΊΠ°, ΠΏΡΠΎΡΠ΅ΠΊΠ°ΡΡΠ΅Π³ΠΎ ΠΏΠΎ ΠΊΠΎΠ½ΡΡΡΡ Π² Π΄Π°Π½Π½ΠΎΠ΅ ΠΌΠ³Π½ΠΎΠ²Π΅Π½ΠΈΠ΅ Π²ΡΠ΅ΠΌΠ΅Π½ΠΈ.
Π‘ΠΈΠΌΠ²ΠΎΠ» , ΠΈΡΠΏΠΎΠ»ΡΠ·ΡΠ΅ΠΌΡΠΉ Π΄Π»Ρ ΠΎΠ±ΠΎΠ·Π½Π°ΡΠ΅Π½ΠΈΡ ΠΈΠ½Π΄ΡΠΊΡΠΈΠ²Π½ΠΎΡΡΠΈ, Π±ΡΠ» Π²Π·ΡΡ Π² ΡΠ΅ΡΡΡ ΠΠ΅Π½ΡΠ° ΠΠΌΠΈΠ»ΠΈΡ Π₯ΡΠΈΡΡΠΈΠ°Π½ΠΎΠ²ΠΈΡΠ° (Heinrich Friedrich Emil Lenz) [ ΠΈΡΡΠΎΡΠ½ΠΈΠΊ Π½Π΅ ΡΠΊΠ°Π·Π°Π½ 1017 Π΄Π½Π΅ΠΉ ] . ΠΠ΄ΠΈΠ½ΠΈΡΠ° ΠΈΠ·ΠΌΠ΅ΡΠ΅Π½ΠΈΡ ΠΈΠ½Π΄ΡΠΊΡΠΈΠ²Π½ΠΎΡΡΠΈ Π½Π°Π·Π²Π°Π½Π° Π² ΡΠ΅ΡΡΡ ΠΠΆΠΎΠ·Π΅ΡΠ° ΠΠ΅Π½ΡΠΈ (Joseph Henry) [8] . Π‘Π°ΠΌ ΡΠ΅ΡΠΌΠΈΠ½ ΠΈΠ½Π΄ΡΠΊΡΠΈΠ²Π½ΠΎΡΡΡ Π±ΡΠ» ΠΏΡΠ΅Π΄Π»ΠΎΠΆΠ΅Π½ ΠΠ»ΠΈΠ²Π΅ΡΠΎΠΌ Π₯Π΅Π²ΠΈΡΠ°ΠΉΠ΄ΠΎΠΌ (Oliver Heaviside) Π² ΡΠ΅Π²ΡΠ°Π»Π΅ 1886 Π³ΠΎΠ΄Π° [ ΠΈΡΡΠΎΡΠ½ΠΈΠΊ Π½Π΅ ΡΠΊΠ°Π·Π°Π½ 1017 Π΄Π½Π΅ΠΉ ] .
ΠΠ»Π΅ΠΊΡΡΠΈΡΠ΅ΡΠΊΠΈΠΉ ΡΠΎΠΊ, ΠΊΠΎΡΠΎΡΡΠΉ ΡΠ΅ΡΠ΅Ρ Π² Π·Π°ΠΌΠΊΠ½ΡΡΠΎΠΌ ΠΊΠΎΠ½ΡΡΡΠ΅, ΡΠΎΠ·Π΄Π°Π΅Ρ Π²ΠΎΠΊΡΡΠ³ ΡΠ΅Π±Ρ ΠΌΠ°Π³Π½ΠΈΡΠ½ΠΎΠ΅ ΠΏΠΎΠ»Π΅, ΠΈΠ½Π΄ΡΠΊΡΠΈΡ ΠΊΠΎΡΠΎΡΠΎΠ³ΠΎ, ΡΠΎΠ³Π»Π°ΡΠ½ΠΎ Π·Π°ΠΊΠΎΠ½Ρ ΠΠΈΠΎ-Π‘Π°Π²Π°ΡΠ°-ΠΠ°ΠΏΠ»Π°ΡΠ°, ΠΏΡΠΎΠΏΠΎΡΡΠΈΠΎΠ½Π°Π»ΡΠ½Π° ΡΠΎΠΊΡ. Π‘ΡΠ΅ΠΏΠ»Π΅Π½Π½ΡΠΉ Ρ ΠΊΠΎΠ½ΡΡΡΠΎΠΌ ΠΌΠ°Π³Π½ΠΈΡΠ½ΡΠΉ ΠΏΠΎΡΠΎΠΊ Π€ ΠΏΠΎΡΡΠΎΠΌΡ ΠΏΡΡΠΌΠΎ ΠΏΡΠΎΠΏΠΎΡΡΠΈΠΎΠ½Π°Π»Π΅Π½ ΡΠΎΠΊΡ I Π² ΠΊΠΎΠ½ΡΡΡΠ΅: (1) Π³Π΄Π΅ ΠΊΠΎΡΡΡΠΈΡΠΈΠ΅Π½Ρ ΠΏΡΠΎΠΏΠΎΡΡΠΈΠΎΠ½Π°Π»ΡΠ½ΠΎΡΡΠΈ L Π½Π°Π·ΡΠ²Π°Π΅ΡΡΡΠΈΠ½Π΄ΡΠΊΡΠΈΠ²Π½ΠΎΡΡΡΡ ΠΊΠΎΠ½ΡΡΡΠ°. ΠΡΠΈ ΠΈΠ·ΠΌΠ΅Π½Π΅Π½ΠΈΠΈ Π² ΠΊΠΎΠ½ΡΡΡΠ΅ ΡΠΈΠ»Ρ ΡΠΎΠΊΠ° Π±ΡΠ΄Π΅Ρ ΡΠ°ΠΊΠΆΠ΅ ΠΈΠ·ΠΌΠ΅Π½ΡΡΡΡΡ ΠΈ ΡΡΠ΅ΠΏΠ»Π΅Π½Π½ΡΠΉ Ρ Π½ΠΈΠΌ ΠΌΠ°Π³Π½ΠΈΡΠ½ΡΠΉ ΠΏΠΎΡΠΎΠΊ; Π·Π½Π°ΡΠΈΡ, Π² ΠΊΠΎΠ½ΡΡΡΠ΅ Π±ΡΠ΄Π΅Ρ ΠΈΠ½Π΄ΡΡΠΈΡΠΎΠ²Π°ΡΡΡΡ Ρ.Π΄.Ρ. ΠΠΎΠ·Π½ΠΈΠΊΠ½ΠΎΠ²Π΅Π½ΠΈΠ΅ Ρ.Π΄.Ρ. ΠΈΠ½Π΄ΡΠΊΡΠΈΠΈ Π² ΠΏΡΠΎΠ²ΠΎΠ΄ΡΡΠ΅ΠΌ ΠΊΠΎΠ½ΡΡΡΠ΅ ΠΏΡΠΈ ΠΈΠ·ΠΌΠ΅Π½Π΅Π½ΠΈΠΈ Π² Π½Π΅ΠΌ ΡΠΈΠ»Ρ ΡΠΎΠΊΠ° Π½Π°Π·ΡΠ²Π°Π΅ΡΡΡΡΠ°ΠΌΠΎΠΈΠ½Π΄ΡΠΊΡΠΈΠ΅ΠΉ. ΠΠ· Π²ΡΡΠ°ΠΆΠ΅Π½ΠΈΡ (1) Π·Π°Π΄Π°Π΅ΡΡΡ Π΅Π΄ΠΈΠ½ΠΈΡΠ° ΠΈΠ½Π΄ΡΠΊΡΠΈΠ²Π½ΠΎΡΡΠΈ Π³Π΅Π½ΡΠΈ (ΠΠ½): 1 ΠΠ½ β ΠΈΠ½Π΄ΡΠΊΡΠΈΠ²Π½ΠΎΡΡΡ ΠΊΠΎΠ½ΡΡΡΠ°, ΠΌΠ°Π³Π½ΠΈΡΠ½ΡΠΉ ΠΏΠΎΡΠΎΠΊ ΡΠ°ΠΌΠΎΠΈΠ½Π΄ΡΠΊΡΠΈΠΈ ΠΊΠΎΡΠΎΡΠΎΠ³ΠΎ ΠΏΡΠΈ ΡΠΎΠΊΠ΅ Π² 1 Π ΡΠ°Π²Π΅Π½ 1 ΠΠ±: 1 ΠΠ½ = 1 ΠΠ±/Ρ = 1 Π
ΠΡΡΠΈΡΠ»ΠΈΠΌ ΠΈΠ½Π΄ΡΠΊΡΠΈΠ²Π½ΠΎΡΡΡ Π±Π΅ΡΠΊΠΎΠ½Π΅ΡΠ½ΠΎ Π΄Π»ΠΈΠ½Π½ΠΎΠ³ΠΎ ΡΠΎΠ»Π΅Π½ΠΎΠΈΠ΄Π°. ΠΠΎΠ»Π½ΡΠΉ ΠΌΠ°Π³Π½ΠΈΡΠ½ΡΠΉ ΠΏΠΎΡΠΎΠΊ ΡΠΊΠ²ΠΎΠ·Ρ ΡΠΎΠ»Π΅Π½ΠΎΠΈΠ΄ (ΠΏΠΎΡΠΎΠΊΠΎΡΡΠ΅ΠΏΠ»Π΅Π½ΠΈΠ΅) ΡΠ°Π²Π΅Π½ ΞΌΞΌ(N 2 I/l)S . ΠΠΎΠ΄ΡΡΠ°Π²ΠΈΠ² Π² (1), Π½Π°ΠΉΠ΄Π΅ΠΌ (2) Ρ. Π΅. ΠΈΠ½Π΄ΡΠΊΡΠΈΠ²Π½ΠΎΡΡΡ ΡΠΎΠ»Π΅Π½ΠΎΠΈΠ΄Π° Π·Π°Π²ΠΈΡΠΈΡ ΠΎΡ Π΄Π»ΠΈΠ½Ρl ΡΠΎΠ»Π΅Π½ΠΈΠΎΠ΄Π°, ΡΠΈΡΠ»Π° Π΅Π³ΠΎ Π²ΠΈΡΠΊΠΎΠ² N, Π΅Π³ΠΎ , ΠΏΠ»ΠΎΡΠ°Π΄ΠΈ S ΠΈ ΠΌΠ°Π³Π½ΠΈΡΠ½ΠΎΠΉ ΠΏΡΠΎΠ½ΠΈΡΠ°Π΅ΠΌΠΎΡΡΠΈ ΞΌ Π²Π΅ΡΠ΅ΡΡΠ²Π°, ΠΈΠ· ΠΊΠΎΡΠΎΡΠΎΠ³ΠΎ ΠΈΠ·Π³ΠΎΡΠΎΠ²Π»Π΅Π½ ΡΠ΅ΡΠ΄Π΅ΡΠ½ΠΈΠΊ ΡΠΎΠ»Π΅Π½ΠΎΠΈΠ΄Π°. ΠΠΎΠΊΠ°Π·Π°Π½ΠΎ, ΡΡΠΎ ΠΈΠ½Π΄ΡΠΊΡΠΈΠ²Π½ΠΎΡΡΡ ΠΊΠΎΠ½ΡΡΡΠ° Π·Π°Π²ΠΈΡΠΈΡ Π² ΠΎΠ±ΡΠ΅ΠΌ ΡΠ»ΡΡΠ°Π΅ ΡΠΎΠ»ΡΠΊΠΎ ΠΎΡ Π³Π΅ΠΎΠΌΠ΅ΡΡΠΈΡΠ΅ΡΠΊΠΎΠΉ ΡΠΎΡΠΌΡ ΠΊΠΎΠ½ΡΡΡΠ°, Π΅Π³ΠΎ ΡΠ°Π·ΠΌΠ΅ΡΠΎΠ² ΠΈ ΠΌΠ°Π³Π½ΠΈΡΠ½ΠΎΠΉ ΠΏΡΠΎΠ½ΠΈΡΠ°Π΅ΠΌΠΎΡΡΠΈ ΡΡΠ΅Π΄Ρ, Π² ΠΊΠΎΡΠΎΡΠΎΠΉ ΠΎΠ½ ΡΠ°ΡΠΏΠΎΠ»ΠΎΠΆΠ΅Π½, ΠΈ ΠΌΠΎΠΆΠ½ΠΎ ΠΏΡΠΎΠ²Π΅ΡΡΠΈ Π°Π½Π°Π»ΠΎΠ³ ΠΈΠ½Π΄ΡΠΊΡΠΈΠ²Π½ΠΎΡΡΠΈ ΠΊΠΎΠ½ΡΡΡΠ° Ρ ΡΠ»Π΅ΠΊΡΡΠΈΡΠ΅ΡΠΊΠΎΠΉ Π΅ΠΌΠΊΠΎΡΡΡΡ ΡΠ΅Π΄ΠΈΠ½Π΅Π½Π½ΠΎΠ³ΠΎ ΠΏΡΠΎΠ²ΠΎΠ΄Π½ΠΈΠΊΠ°, ΠΊΠΎΡΠΎΡΠ°Ρ ΡΠ°ΠΊΠΆΠ΅ Π·Π°Π²ΠΈΡΠΈΡ ΡΠΎΠ»ΡΠΊΠΎ ΠΎΡ ΡΠΎΡΠΌΡ ΠΏΡΠΎΠ²ΠΎΠ΄Π½ΠΈΠΊΠ°, Π΅Π³ΠΎ ΡΠ°Π·ΠΌΠ΅ΡΠΎΠ² ΠΈ Π΄ΠΈΡΠ»Π΅ΠΊΡΡΠΈΡΠ΅ΡΠΊΠΎΠΉ ΠΏΡΠΎΠ½ΠΈΡΠ°Π΅ΠΌΠΎΡΡΠΈ ΡΡΠ΅Π΄Ρ. ΠΠ°ΠΉΠ΄Π΅ΠΌ, ΠΏΡΠΈΠΌΠ΅Π½ΡΡ ΠΊ ΡΠ²Π»Π΅Π½ΠΈΡ ΡΠ°ΠΌΠΎΠΈΠ½Π΄ΡΠΊΡΠΈΠΈ Π·Π°ΠΊΠΎΠ½ Π€Π°ΡΠ°Π΄Π΅Ρ, ΡΡΠΎ Ρ.Π΄.Ρ. ΡΠ°ΠΌΠΎΠΈΠ½Π΄ΡΠΊΡΠΈΠΈ ΡΠ°Π²Π½Π° ΠΡΠ»ΠΈ ΠΊΠΎΠ½ΡΡΡ Π½Π΅ ΠΏΡΠ΅ΡΠ΅ΡΠΏΠ΅Π²Π°Π΅Ρ Π΄Π΅ΡΠΎΡΠΌΠ°ΡΠΈΠΉ ΠΈ ΠΌΠ°Π³Π½ΠΈΡΠ½Π°Ρ ΠΏΡΠΎΠ½ΠΈΡΠ°Π΅ΠΌΠΎΡΡΡ ΡΡΠ΅Π΄Ρ ΠΎΡΡΠ°Π΅ΡΡΡ Π½Π΅ΠΈΠ·ΠΌΠ΅Π½Π½ΠΎΠΉ (Π² Π΄Π°Π»ΡΠ½Π΅ΠΉΡΠ΅ΠΌ Π±ΡΠ΄Π΅Ρ ΠΏΠΎΠΊΠ°Π·Π°Π½ΠΎ, ΡΡΠΎ ΠΏΠΎΡΠ»Π΅Π΄Π½Π΅Π΅ ΡΡΠ»ΠΎΠ²ΠΈΠ΅ Π²ΡΠΏΠΎΠ»Π½ΡΠ΅ΡΡΡ Π½Π΅ Π²ΡΠ΅Π³Π΄Π°), ΡΠΎ L = const ΠΈ(3) Π³Π΄Π΅ Π·Π½Π°ΠΊ ΠΌΠΈΠ½ΡΡ, ΠΎΠΏΡΠ΅Π΄Π΅Π»ΡΠ΅ΠΌΡΠΉ ΠΏΡΠ°Π²ΠΈΠ»ΠΎΠΌ ΠΠ΅Π½ΡΠ°, Π³ΠΎΠ²ΠΎΡΠΈΡ ΠΎ ΡΠΎΠΌ, ΡΡΠΎΠ½Π°Π»ΠΈΡΠΈΠ΅ ΠΈΠ½Π΄ΡΠΊΡΠΈΠ²Π½ΠΎΡΡΠΈ Π² ΠΊΠΎΠ½ΡΡΡΠ΅ ΠΏΡΠΈΠ²ΠΎΠ΄ΠΈΡ ΠΊ Π·Π°ΠΌΠ΅Π΄Π»Π΅Π½ΠΈΡ ΠΈΠ·ΠΌΠ΅Π½Π΅Π½ΠΈΡ ΡΠΎΠΊΠ° Π² Π½Π΅ΠΌ. ΠΡΠ»ΠΈ ΡΠΎΠΊ ΡΠΎ Π²ΡΠ΅ΠΌΠ΅Π½Π΅ΠΌ ΡΠ²Π΅Π»ΠΈΡΠΈΠ²Π°Π΅ΡΡΡ, ΡΠΎ (dI/dt 0 Ρ. Π΅. ΡΠΎΠΊ ΡΠ°ΠΌΠΎΠΈΠ½Π΄ΡΠΊΡΠΈΠΈ Π½Π°ΠΏΡΠ°Π²Π»Π΅Π½ Π½Π°Π²ΡΡΡΠ΅ΡΡ ΡΠΎΠΊΡ, ΠΎΠ±ΡΡΠ»ΠΎΠ²Π»Π΅Π½Π½ΠΎΠΌΡ Π²Π½Π΅ΡΠ½ΠΈΠΌ ΠΈΡΡΠΎΡΠ½ΠΈΠΊΠΎΠΌ, ΠΈ Π·Π°ΠΌΠ΅Π΄Π»ΡΠ΅Ρ Π΅Π³ΠΎ ΡΠ²Π΅Π»ΠΈΡΠ΅Π½ΠΈΠ΅. ΠΡΠ»ΠΈ ΡΠΎΠΊ ΡΠΎ Π²ΡΠ΅ΠΌΠ΅Π½Π΅ΠΌ ΡΠΌΠ΅Π½ΡΡΠ°Π΅ΡΡΡ, ΡΠΎ (dI/dt>0) ΠΈ ΞΎs >1), ΠΎΠ±Π»Π°Π΄Π°ΡΡΠ΅ΠΉ Π±ΠΎΠ»ΡΒΡΠΎΠΉ ΠΈΠ½Π΄ΡΠΊΡΠΈΠ²Π½ΠΎΡΡΡΡ, Ρ.Π΄.Ρ. ΡΠ°ΠΌΠΎΠΈΠ½Π΄ΡΠΊΡΠΈΠΈ ΠΌΠΎΠΆΠ΅Ρ Π²ΠΎ ΠΌΠ½ΠΎΠ³ΠΎ ΡΠ°Π· ΠΏΡΠ΅Π²ΡΡΠ°ΡΡ Ρ.Π΄.Ρ. ΠΈΡΡΠΎΡΠ½ΠΈΠΊΠ° ΡΠΎΠΊΠ°, Π²ΠΊΠ»ΡΡΠ΅Π½Π½ΠΎΠ³ΠΎ Π² ΡΠ΅ΠΏΡ. Π’Π°ΠΊΠΈΠΌ ΠΎΠ±ΡΠ°Π·ΠΎΠΌ, Π½Π΅ΠΎΠ±Ρ ΠΎΠ΄ΠΈΠΌΠΎ ΡΡΠΈΡΡΠ²Π°ΡΡ, ΡΡΠΎ ΠΊΠΎΠ½ΡΡΡ, ΡΠΎΠ΄Π΅ΡΠΆΠ°ΡΠΈΠΉ ΠΈΠ½Π΄ΡΠΊΡΠΈΠ²Π½ΠΎΡΡΡ, Π½Π΅Π»ΡΠ·Ρ ΡΠ΅Π·ΠΊΠΎ ΡΠ°Π·ΠΌΡΠΊΠ°ΡΡ, ΡΠ°ΠΊ ΠΊΠ°ΠΊ ΡΡΠΎ (Π²ΠΎΠ·Π½ΠΈΠΊΠ½ΠΎΠ²Π΅ΒΠ½ΠΈΠ΅ Π·Π½Π°ΡΠΈΡΠ΅Π»ΡΠ½ΡΡ Ρ.Π΄.Ρ. ΡΠ°ΠΌΠΎΠΈΠ½Π΄ΡΠΊΡΠΈΠΈ) ΠΌΠΎΠΆΠ΅Ρ ΠΏΡΠΈΠ²Π΅ΡΡΠΈ ΠΊ ΠΏΡΠΎΠ±ΠΎΡ ΠΈΠ·ΠΎΠ»ΡΡΠΈΠΈ ΠΈ Π²ΡΠ²ΠΎΠ΄Ρ ΠΈΠ· ΡΡΡΠΎΡ ΠΈΠ·ΠΌΠ΅ΡΠΈΡΠ΅Π»ΡΠ½ΡΡ ΠΏΡΠΈΠ±ΠΎΡΠΎΠ². ΠΡΠ»ΠΈ Π² ΠΊΠΎΠ½ΡΡΡ ΡΠΎΠΏΡΠΎΡΠΈΠ²Π»Π΅Π½ΠΈΠ΅ Π²Π²ΠΎΠ΄ΠΈΡΡ ΠΏΠΎΡΡΠ΅ΠΏΠ΅Π½Π½ΠΎ, ΡΠΎ Ρ.Π΄.Ρ. ΡΠ°ΠΌΠΎΠΈΠ½Π΄ΡΠΊΡΠΈΠΈ Π½Π΅ Π΄ΠΎΡΡΠΈΠ³Π½Π΅Ρ Π±ΠΎΠ»ΡΡΠΈΡ Π·Π½Π°ΡΠ΅Π½ΠΈΠΉ.
43. Π―Π²Π»Π΅Π½ΠΈΠ΅ Π²Π·Π°ΠΈΠΌΠ½ΠΎΠΉ ΠΈΠ½Π΄ΡΠΊΡΠΈΠΈ. Π’ΡΠ°Π½ΡΡΠΎΡΠΌΠ°ΡΠΎΡ.
Π Π°ΡΡΠΌΠΎΡΡΠΈΠΌ Π΄Π²Π° Π½Π΅ΠΏΠΎΠ΄Π²ΠΈΠΆΠ½ΡΡ ΠΊΠΎΠ½ΡΡΡΠ° (1 ΠΈ 2), ΠΊΠΎΡΠΎΡΡΠ΅ ΡΠ°ΡΠΏΠΎΠ»ΠΎΠΆΠ΅Π½Ρ Π΄ΠΎΡΡΠ°ΡΠΎΡΠ½ΠΎ Π±Π»ΠΈΠ·ΠΊΠΎ Π΄ΡΡΠ³ ΠΎΡ Π΄ΡΡΠ³Π° (ΡΠΈΡ. 1). ΠΡΠ»ΠΈ Π² ΠΊΠΎΠ½ΡΡΡΠ΅ 1 ΠΏΡΠΎΡΠ΅ΠΊΠ°Π΅Ρ ΡΠΎΠΊ I1, ΡΠΎ ΠΌΠ°Π³Π½ΠΈΡΠ½ΡΠΉ ΠΏΠΎΡΠΎΠΊ, ΠΊΠΎΡΠΎΡΡΠΉ ΡΠΎΠ·Π΄Π°Π²Π°Π΅ΡΡΡ ΡΡΠΈΠΌ ΡΠΎΠΊΠΎΠΌ (ΠΏΠΎΠ»Π΅, ΡΠΎΠ·Π΄Π°ΡΡΠ΅Π΅ ΡΡΠΎΡ ΠΏΠΎΡΠΎΠΊ, Π½Π° ΡΠΈΡΡΠ½ΠΊΠ΅ ΠΈΠ·ΠΎΠ±ΡΠ°ΠΆΠ΅Π½ΠΎ ΡΠΏΠ»ΠΎΡΠ½ΡΠΌΠΈ Π»ΠΈΠ½ΠΈΡΠΌΠΈ), ΠΏΡΡΠΌΠΎ ΠΏΡΠΎΠΏΠΎΡΡΠΈΠΎΠ½Π°Π»Π΅Π½ I1. ΠΠ±ΠΎΠ·Π½Π°ΡΠΈΠΌ ΡΠ΅ΡΠ΅Π· Π€21 ΡΠ°ΡΡΡ ΠΏΠΎΡΠΎΠΊΠ°,ΠΏΡΠΎΠ½ΠΈΠ·ΡΠ²Π°ΡΡΠ°Ρ ΠΊΠΎΠ½ΡΡΡ 2. Π’ΠΎΠ³Π΄Π° (1) Π³Π΄Π΅ L21 β ΠΊΠΎΡΡΡΠΈΡΠΈΠ΅Π½Ρ ΠΏΡΠΎΠΏΠΎΡΡΠΈΠΎΠ½Π°Π»ΡΠ½ΠΎΡΡΠΈ.
ΠΡΠ»ΠΈ ΡΠΎΠΊ I1 ΠΌΠ΅Π½ΡΠ΅Ρ ΡΠ²ΠΎΠ΅ Π·Π½Π°ΡΠ΅Π½ΠΈΠ΅, ΡΠΎ Π² ΠΊΠΎΠ½ΡΡΡΠ΅ 2 ΠΈΠ½Π΄ΡΡΠΈΡΡΠ΅ΡΡΡ Ρ.Π΄.Ρ. ΞΎi2 , ΠΊΠΎΡΠΎΡΠ°Ρ ΠΏΠΎ Π·Π°ΠΊΠΎΠ½Ρ Π€Π°ΡΠ°Π΄Π΅Ρ Π±ΡΠ΄Π΅Ρ ΡΠ°Π²Π½Π° ΠΈ ΠΏΡΠΎΡΠΈΠ²ΠΎΠΏΠΎΠ»ΠΎΠΆΠ½Π° ΠΏΠΎ Π·Π½Π°ΠΊΡ ΡΠΊΠΎΡΠΎΡΡΠΈ ΠΈΠ·ΠΌΠ΅Π½Π΅Π½ΠΈΡ ΠΌΠ°Π³Π½ΠΈΡΠ½ΠΎΠ³ΠΎ ΠΏΠΎΡΠΎΠΊΠ° Π€21, ΠΊΠΎΡΠΎΡΡΠΉ ΡΠΎΠ·Π΄Π°Π΅ΡΡΡ ΡΠΎΠΊΠΎΠΌ Π² ΠΏΠ΅ΡΠ²ΠΎΠΌ ΠΊΠΎΠ½ΡΡΡΠ΅ ΠΈ ΠΏΡΠΎΠ½ΠΈΠ·ΡΠ²Π°ΡΠ΅Ρ Π²ΡΠΎΡΠΎΠΉ: ΠΠ½Π°Π»ΠΎΠ³ΠΈΡΠ½ΡΠΌ ΠΎΠ±ΡΠ°Π·ΠΎΠΌ, ΠΏΡΠΈ ΠΏΡΠΎΡΠ΅ΠΊΠ°Π½ΠΈΠΈ Π² ΠΊΠΎΠ½ΡΡΡΠ΅ 2 ΡΠΎΠΊΠ° I2 ΠΌΠ°Π³Π½ΠΈΡΠ½ΡΠΉ ΠΏΠΎΡΠΎΠΊ (Π΅Π³ΠΎ ΠΏΠΎΠ»Π΅ ΠΈΠ·ΠΎΠ±ΡΠ°ΠΆΠ΅Π½ΠΎ Π½Π° ΡΠΈΡ. 1 ΡΡΡΠΈΡ Π°ΠΌΠΈ) ΠΏΡΠΎΠ½ΠΈΠ·ΡΠ²Π°Π΅Ρ ΠΏΠ΅ΡΠ²ΡΠΉ ΠΊΠΎΠ½ΡΡΡ. ΠΡΠ»ΠΈ Π€12 β ΡΠ°ΡΡΡ ΡΡΠΎΠ³ΠΎ ΠΏΠΎΡΠΎΠΊΠ°, ΠΊΠΎΡΠΎΡΡΠΉ ΠΏΡΠΎΠ½ΠΈΠ·ΡΠ²Π°Π΅Ρ ΠΊΠΎΠ½ΡΡΡ 1, ΡΠΎ ΠΡΠ»ΠΈ ΡΠΎΠΊ I2 ΠΌΠ΅Π½ΡΠ΅Ρ ΡΠ²ΠΎΠ΅ Π·Π½Π°ΡΠ΅Π½ΠΈΠ΅, ΡΠΎ Π² ΠΊΠΎΠ½ΡΡΡΠ΅ 1 ΠΈΠ½Π΄ΡΡΠΈΡΡΠ΅ΡΡΡ Ρ.Π΄.Ρ. ΞΎi1 , ΠΊΠΎΡΠΎΡΠ°Ρ ΡΠ°Π²Π½Π° ΠΈ ΠΏΡΠΎΡΠΈΠ²ΠΎΠΏΠΎΠ»ΠΎΠΆΠ½Π° ΠΏΠΎ Π·Π½Π°ΠΊΡ ΡΠΊΠΎΡΠΎΡΡΠΈ ΠΈΠ·ΠΌΠ΅Π½Π΅Π½ΠΈΡ ΠΌΠ°Π³Π½ΠΈΡΠ½ΠΎΠ³ΠΎ ΠΏΠΎΡΠΎΠΊΠ° Π€12, ΠΊΠΎΡΠΎΡΡΠΉ ΡΠΎΠ·Π΄Π°Π΅ΡΡΡ ΡΠΎΠΊΠΎΠΌ Π²ΠΎ Π²ΡΠΎΡΠΎΠΌ ΠΊΠΎΠ½ΡΡΡΠ΅ ΠΈ ΠΏΡΠΎΠ½ΠΈΠ·ΡΠ²Π°Π΅Ρ ΠΏΠ΅ΡΠ²ΡΠΉ: Π―Π²Π»Π΅Π½ΠΈΠ΅ Π²ΠΎΠ·Π½ΠΈΠΊΠ½ΠΎΠ²Π΅Π½ΠΈΡ Ρ.Π΄.Ρ. Π² ΠΎΠ΄Π½ΠΎΠΌ ΠΈΠ· ΠΊΠΎΠ½ΡΡΡΠΎΠ² ΠΏΡΠΈ ΠΈΠ·ΠΌΠ΅Π½Π΅Π½ΠΈΠΈ ΡΠΈΠ»Ρ ΡΠΎΠΊΠ° Π² Π΄ΡΡΠ³ΠΎΠΌ Π½Π°Π·ΡΠ²Π°Π΅ΡΡΡ Π²Π·Π°ΠΈΠΌΠ½ΠΎΠΉ ΠΈΠ½Π΄ΡΠΊΡΠΈΠ΅ΠΉ. ΠΠΎΡΡΡΠΈΡΠΈΠ΅Π½ΡΡ ΠΏΡΠΎΠΏΠΎΡΡΠΈΠΎΠ½Π°Π»ΡΠ½ΠΎΡΡΠΈ L21 ΠΈ L12 Π½Π°Π·ΡΠ²Π°ΡΡΡΡ Π²Π·Π°ΠΈΠΌΠ½ΠΎΠΉ ΠΈΠ½Π΄ΡΠΊΡΠΈΠ²Π½ΠΎΡΡΡΡ ΠΊΠΎΠ½ΡΡΡΠΎΠ². Π Π°ΡΡΠ΅ΡΡ, ΠΊΠΎΡΠΎΡΡΠ΅ ΠΏΠΎΠ΄ΡΠ²Π΅ΡΠΆΠ΄Π΅Π½Ρ ΠΎΠΏΡΡΠΎΠΌ, ΠΏΠΎΠΊΠ°Π·ΡΠ²Π°ΡΡ, ΡΡΠΎ L21 ΠΈ L12 ΡΠ°Π²Π½Ρ Π΄ΡΡΠ³ Π΄ΡΡΠ³Ρ, Ρ. Π΅. (2) ΠΠΎΡΡΡΠΈΡΠΈΠ΅Π½ΡΡ ΠΏΡΠΎΠΏΠΎΡΡΠΈΠΎΠ½Π°Π»ΡΠ½ΠΎΡΡΠΈ L12 ΠΈ L21 Π·Π°Π²ΠΈΡΡΡ ΠΎΡ ΡΠ°Π·ΠΌΠ΅ΡΠΎΠ², Π³Π΅ΠΎΠΌΠ΅ΡΡΠΈΡΠ΅ΡΠΊΠΎΠΉ ΡΠΎΡΠΌΡ, Π²Π·Π°ΠΈΠΌΠ½ΠΎΠ³ΠΎ ΡΠ°ΡΠΏΠΎΠ»ΠΎΠΆΠ΅Π½ΠΈΡ ΠΊΠΎΠ½ΡΡΡΠΎΠ² ΠΈ ΠΎΡ ΠΌΠ°Π³Π½ΠΈΡΠ½ΠΎΠΉ ΠΏΡΠΎΠ½ΠΈΡΠ°Π΅ΠΌΠΎΡΡΠΈ ΡΡΠ΅Π΄Ρ, ΠΎΠΊΡΡΠΆΠ°ΡΡΠ΅ΠΉ ΠΊΠΎΠ½ΡΡΡΡ. ΠΠ΄ΠΈΠ½ΠΈΡΠ° Π²Π·Π°ΠΈΠΌΠ½ΠΎΠΉ ΠΈΠ½Π΄ΡΠΊΡΠΈΠ²Π½ΠΎΡΡΠΈ ΡΠ° ΠΆΠ΅, ΡΡΠΎ ΠΈ Π΄Π»Ρ ΠΈΠ½Π΄ΡΠΊΡΠΈΠ²Π½ΠΎΡΡΠΈ, β Π³Π΅Π½ΡΠΈ (ΠΠ½). ΠΠ°ΠΉΠ΄Π΅ΠΌ Π²Π·Π°ΠΈΠΌΠ½ΡΡ ΠΈΠ½Π΄ΡΠΊΡΠΈΠ²Π½ΠΎΡΡΡ Π΄Π²ΡΡ ΠΊΠ°ΡΡΡΠ΅ΠΊ, ΠΊΠΎΡΠΎΡΡΠ΅ Π½Π°ΠΌΠΎΡΠ°Π½Ρ Π½Π° ΠΎΠ±ΡΠΈΠΉ ΡΠΎΡΠΎΠΈΠ΄Π°Π»ΡΠ½ΡΠΉ ΡΠ΅ΡΠ΄Π΅ΡΠ½ΠΈΠΊ. ΠΡΠΎΡ ΡΠ»ΡΡΠ°ΠΉ ΠΈΠΌΠ΅Π΅Ρ Π±ΠΎΠ»ΡΡΠΎΠ΅ ΠΏΡΠ°ΠΊΡΠΈΡΠ΅ΡΠΊΠΎΠ΅ Π·Π½Π°ΡΠ΅Π½ΠΈΠ΅ (ΡΠΈΡ. 2). ΠΠ°Π³Π½ΠΈΡΠ½Π°Ρ ΠΈΠ½Π΄ΡΠΊΡΠΈΡ ΠΏΠΎΠ»Ρ, ΠΊΠΎΡΠΎΡΠΎΠ΅ ΡΠΎΠ·Π΄Π°Π²Π°Π΅ΡΡΡ ΠΏΠ΅ΡΠ²ΠΎΠΉ ΠΊΠ°ΡΡΡΠΊΠΎΠΉ Ρ ΡΠΈΡΠ»ΠΎΠΌ Π²ΠΈΡΠΊΠΎΠ² N1, ΡΠΎΠΊΠΎΠΌ I1 ΠΈ ΠΌΠ°Π³Π½ΠΈΡΠ½ΠΎΠΉ ΠΏΡΠΎΠ½ΠΈΡΠ°Π΅ΠΌΠΎΡΡΡΡ ΞΌ ΡΠ΅ΡΠ΄Π΅ΡΠ½ΠΈΠΊΠ°, B = ΞΌΞΌ(N1I1/l) Π³Π΄Π΅ l β Π΄Π»ΠΈΠ½Π° ΡΠ΅ΡΠ΄Π΅ΡΠ½ΠΈΠΊΠ° ΠΏΠΎ ΡΡΠ΅Π΄Π½Π΅ΠΉ Π»ΠΈΠ½ΠΈΠΈ. ΠΠ°Π³Π½ΠΈΡΠ½ΡΠΉ ΠΏΠΎΡΠΎΠΊ ΡΠΊΠ²ΠΎΠ·Ρ ΠΎΠ΄ΠΈΠ½ Π²ΠΈΡΠΎΠΊ Π²ΡΠΎΡΠΎΠΉ ΠΊΠ°ΡΡΡΠΊΠΈ Π€2 = BS = ΞΌΞΌ(N1I1/l)S
ΠΠ½Π°ΡΠΈΡ, ΠΏΠΎΠ»Π½ΡΠΉ ΠΌΠ°Π³Π½ΠΈΡΠ½ΡΠΉ ΠΏΠΎΡΠΎΠΊ (ΠΏΠΎΡΠΎΠΊΠΎΡΡΠ΅ΠΏΠ»Π΅Π½ΠΈΠ΅) ΡΠΊΠ²ΠΎΠ·Ρ Π²ΡΠΎΡΠΈΡΠ½ΡΡ ΠΎΠ±ΠΌΠΎΡΠΊΡ, ΠΊΠΎΡΠΎΡΠ°Ρ ΡΠΎΠ΄Π΅ΡΠΆΠΈΡ N2 Π²ΠΈΡΠΊΠΎΠ², ΠΠΎΡΠΎΠΊ Ξ¨ ΡΠΎΠ·Π΄Π°Π΅ΡΡΡ ΡΠΎΠΊΠΎΠΌ I1, ΠΏΠΎΡΡΠΎΠΌΡ, ΠΈΡΠΏΠΎΠ»ΡΠ·ΡΡ (1), Π½Π°ΠΉΠ΄Π΅ΠΌ (3) ΠΡΠ»ΠΈ ΡΠ°ΡΡΡΠΈΡΠ°ΡΡ ΠΌΠ°Π³Π½ΠΈΡΠ½ΡΠΉ ΠΏΠΎΡΠΎΠΊ, ΠΊΠΎΡΠΎΡΡΠΉ ΡΠΎΠ·Π΄Π°Π²Π°Π΅ΡΡΡ ΠΊΠ°ΡΡΡΠΊΠΎΠΉ 2 ΡΠΊΠ²ΠΎΠ·Ρ ΠΊΠ°ΡΡΡΠΊΡ 1, ΡΠΎ Π΄Π»Ρ L12 ΠΏΠΎΠ»ΡΡΠΈΠΌ Π²ΡΡΠ°ΠΆΠ΅Π½ΠΈΠ΅ Π² ΡΠΎΠΎΡΠ²Π΅ΡΡΡΠ²ΠΈΠΈ Ρ ΡΠΎΡΠΌΡΠ»ΠΎΠΉ (3). ΠΠ½Π°ΡΠΈΡ, Π²Π·Π°ΠΈΠΌΠ½Π°Ρ ΠΈΠ½Π΄ΡΠΊΡΠΈΠ²Π½ΠΎΡΡΡ Π΄Π²ΡΡ ΠΊΠ°ΡΡΡΠ΅ΠΊ, ΠΊΠΎΡΠΎΡΡΠ΅ Π½Π°ΠΌΠΎΡΠ°Π½Ρ Π½Π° ΠΎΠ±ΡΠΈΠΉ ΡΠΎΡΠΎΠΈΠ΄Π°Π»ΡΠ½ΡΠΉ ΡΠ΅ΡΠ΄Π΅ΡΠ½ΠΈΠΊ,
Π’ΡΠ°Π½ΡΡΠΎΡΠΌΠ°ΜΡΠΎΡ (ΠΎΡ Π»Π°Ρ. transformo β ΠΏΡΠ΅ΠΎΠ±ΡΠ°Π·ΠΎΠ²ΡΠ²Π°ΡΡ) β ΡΡΠΎ ΡΡΠ°ΡΠΈΡΠ΅ΡΠΊΠΎΠ΅ ΡΠ»Π΅ΠΊΡΡΠΎΠΌΠ°Π³Π½ΠΈΡΠ½ΠΎΠ΅ ΡΡΡΡΠΎΠΉΡΡΠ²ΠΎ, ΠΈΠΌΠ΅ΡΡΠ΅Π΅ Π΄Π²Π΅ ΠΈΠ»ΠΈ Π±ΠΎΠ»Π΅Π΅ ΠΈΠ½Π΄ΡΠΊΡΠΈΠ²Π½ΠΎ ΡΠ²ΡΠ·Π°Π½Π½ΡΡ ΠΎΠ±ΠΌΠΎΡΠΎΠΊ Π½Π° ΠΊΠ°ΠΊΠΎΠΌ-Π»ΠΈΠ±ΠΎ ΠΌΠ°Π³Π½ΠΈΡΠΎΠΏΡΠΎΠ²ΠΎΠ΄Π΅ ΠΈ ΠΏΡΠ΅Π΄Π½Π°Π·Π½Π°ΡΠ΅Π½Π½ΠΎΠ΅ Π΄Π»Ρ ΠΏΡΠ΅ΠΎΠ±ΡΠ°Π·ΠΎΠ²Π°Π½ΠΈΡ ΠΏΠΎΡΡΠ΅Π΄ΡΡΠ²ΠΎΠΌΡΠ»Π΅ΠΊΡΡΠΎΠΌΠ°Π³Π½ΠΈΡΠ½ΠΎΠΉ ΠΈΠ½Π΄ΡΠΊΡΠΈΠΈ ΠΎΠ΄Π½ΠΎΠΉ ΠΈΠ»ΠΈ Π½Π΅ΡΠΊΠΎΠ»ΡΠΊΠΈΡ ΡΠΈΡΡΠ΅ΠΌ (Π½Π°ΠΏΡΡΠΆΠ΅Π½ΠΈΠΉ) ΠΏΠ΅ΡΠ΅ΠΌΠ΅Π½Π½ΠΎΠ³ΠΎ ΡΠΎΠΊΠ° Π² ΠΎΠ΄Π½Ρ ΠΈΠ»ΠΈ Π½Π΅ΡΠΊΠΎΠ»ΡΠΊΠΎ Π΄ΡΡΠ³ΠΈΡ ΡΠΈΡΡΠ΅ΠΌ (Π½Π°ΠΏΡΡΠΆΠ΅Π½ΠΈΠΉ) ΠΏΠ΅ΡΠ΅ΠΌΠ΅Π½Π½ΠΎΠ³ΠΎ ΡΠΎΠΊΠ° Π±Π΅Π· ΠΈΠ·ΠΌΠ΅Π½Π΅Π½ΠΈΡ ΡΠ°ΡΡΠΎΡΡ ΡΠΈΡΡΠ΅ΠΌΡ (Π½Π°ΠΏΡΡΠΆΠ΅Π½ΠΈΡ) ΠΏΠ΅ΡΠ΅ΠΌΠ΅Π½Π½ΠΎΠ³ΠΎ ΡΠΎΠΊΠ°
ΠΠΠ‘ ΠΈΠ½Π΄ΡΠΊΡΠΈΠΈ Π² Π΄Π²ΠΈΠΆΡΡΠ΅ΠΌΡΡ ΠΏΡΠΎΠ²ΠΎΠ΄Π½ΠΈΠΊΠ΅
ΠΠΠ‘ ΠΈΠ½Π΄ΡΠΊΡΠΈΠΈ Π² Π΄Π²ΠΈΠΆΡΡΠ΅ΠΌΡΡ ΠΏΡΠΎΠ²ΠΎΠ΄Π½ΠΈΠΊΠ΅:
ΠΠ° Π·Π°ΡΡΠΆΠ΅Π½Π½ΡΡ ΡΠ°ΡΡΠΈΡΡ, Π΄Π²ΠΈΠΆΡΡΡΡΡΡ Π² ΠΌΠ°Π³Π½ΠΈΡΠ½ΠΎΠΌ ΠΏΠΎΠ»Π΅ ΠΈΠ½Π΄ΡΠΊΡΠΈΠ΅ΠΉ
Π Π°ΡΡΠΌΠΎΡΡΠΈΠΌ ΠΏΡΠΎΠ²ΠΎΠ΄Π½ΠΈΠΊ, Π½Π°Ρ
ΠΎΠ΄ΡΡΠΈΠΉΡΡ Π² ΠΌΠ°Π³Π½ΠΈΡΠ½ΠΎΠΌ ΠΏΠΎΠ»Π΅. Π‘Π²ΠΎΠ±ΠΎΠ΄Π½ΡΠ΅ Π·Π°ΡΡΠ΄Ρ Π²Π½ΡΡΡΠΈ ΠΏΡΠΎΠ²ΠΎΠ΄Π½ΠΈΠΊΠ° ΡΡΠ°ΡΡΠ²ΡΡΡ Π² ΡΠ΅ΠΏΠ»ΠΎΠ²ΠΎΠΌ Π΄Π²ΠΈΠΆΠ΅Π½ΠΈΠΈ. ΠΡΠ»Π΅Π΄ΡΡΠ²ΠΈΠ΅ Ρ
Π°ΠΎΡΠΈΡΠ½ΠΎΡΡΠΈ
ΡΠ΅ΠΏΠ»ΠΎΠ²ΠΎΠ³ΠΎ Π΄Π²ΠΈΠΆΠ΅Π½ΠΈΡ ΡΡΠ΅Π΄Π½ΡΡ ΡΠΊΠΎΡΠΎΡΡΡ ΠΏΠ΅ΡΠ΅ΠΌΠ΅ΡΠ΅Π½ΠΈΡ ΠΈ ΡΡΠ΅Π΄Π½Π΅Π΅ ΡΠΌΠ΅ΡΠ΅Π½ΠΈΠ΅ ΡΠ°ΡΡΠΈΡ ΡΠ°Π²Π½Ρ Π½ΡΠ»Ρ. Π‘Π»Π΅Π΄ΠΎΠ²Π°ΡΠ΅Π»ΡΠ½ΠΎ, ΠΏΡΠΈ ΡΠ΅ΠΏΠ»ΠΎΠ²ΠΎΠΌ Π΄Π²ΠΈΠΆΠ΅Π½ΠΈΠΈ Π·Π°ΡΡΠΆΠ΅Π½Π½ΡΡ
ΡΠ°ΡΡΠΈΡ Π²Π½ΡΡΡΠΈ ΠΏΡΠΎΠ²ΠΎΠ΄Π½ΠΈΠΊΠ° ΡΡΠ΅Π΄Π½Π΅Π΅ Π·Π½Π°ΡΠ΅Π½ΠΈΠ΅ ΡΠΈΠ»Ρ ΠΠΎΡΠ΅Π½ΡΠ° ΡΠ°ΠΊΠΆΠ΅ ΡΠ°Π²Π½ΠΎ Π½ΡΠ»Ρ. ΠΡΠΈ ΠΏΠΎΡΡΡΠΏΠ°ΡΠ΅Π»ΡΠ½ΠΎΠΌ Π΄Π²ΠΈΠΆΠ΅Π½ΠΈΠΈ ΠΏΡΠΎΠ²ΠΎΠ΄Π½ΠΈΠΊΠ° ΠΊΠ°ΠΊ ΡΠ΅Π»ΠΎΠ³ΠΎ ΡΠΈΡΡΠ°ΡΠΈΡ ΠΌΠ΅Π½ΡΠ΅ΡΡΡ, ΠΏΠΎΡΠΊΠΎΠ»ΡΠΊΡ Π½Π°ΡΡΠ΄Ρ Ρ ΡΠ΅ΠΏΠ»ΠΎΠ²ΡΠΌ (Π±Π΅ΡΠΏΠΎΡΡΠ΄ΠΎΡΠ½ΡΠΌ) Π΄Π²ΠΈΠΆΠ΅Π½ΠΈΠ΅ΠΌ ΡΠ²ΠΎΠ±ΠΎΠ΄Π½ΡΠ΅ Π·Π°ΡΡΠ΄Ρ ΡΡΠ°ΡΡΠ²ΡΡΡ Π² Π½Π°ΠΏΡΠ°Π²Π»Π΅Π½Π½ΠΎΠΌ Π΄Π²ΠΈΠΆΠ΅Π½ΠΈΠΈ Π²ΠΌΠ΅ΡΡΠ΅ Ρ ΠΏΡΠΎΠ²ΠΎΠ΄Π½ΠΈΠΊΠΎΠΌ.
Π Π°ΡΡΠΌΠΎΡΡΠΈΠΌ Π΄Π²ΠΈΠΆΠ΅Π½ΠΈΠ΅ ΠΏΡΠΎΠ²ΠΎΠ΄Π½ΠΈΠΊΠ° Π΄Π»ΠΈΠ½ΠΎΠΉ lΒ ΡΠΎ ΡΠΊΠΎΡΠΎΡΡΡΡ ΠΏΠ΅ΡΠΏΠ΅Π½Π΄ΠΈΠΊΡΠ»ΡΡΠ½ΠΎ ΠΌΠ°Π³Π½ΠΈΡΠ½ΠΎΠΌΡ ΠΏΠΎΠ»Ρ ΠΈΠ½Π΄ΡΠΊΡΠΈΠ΅ΠΉ ΠΠΎΡΠΊΠΎΠ»ΡΠΊΡ ΡΠ²ΠΎΠ±ΠΎΠ΄Π½ΡΠ΅ ΡΠ»Π΅ΠΊΡΡΠΎΠ½Ρ Π΄Π²ΠΈΠΆΡΡΡΡ Π²ΠΌΠ΅ΡΡΠ΅ Ρ ΠΏΡΠΎΠ²ΠΎΠ΄Π½ΠΈΠΊΠΎΠΌ ΡΠΎ ΡΠΊΠΎΡΠΎΡΡΡΡ (ΡΠΈΡ. 164), ΡΠΎ ΠΏΠΎΠ΄ Π΄Π΅ΠΉΡΡΠ²ΠΈΠ΅ΠΌ ΡΠΈΠ»Ρ ΠΠΎΡΠ΅Π½ΡΠ° ΠΎΠ½ΠΈ Π±ΡΠ΄ΡΡ ΡΠΌΠ΅ΡΠ°ΡΡΡΡ Π²Π΄ΠΎΠ»Ρ ΠΏΡΠΎΠ²ΠΎΠ΄Π½ΠΈΠΊΠ° ΠΊ Π΅Π³ΠΎ ΠΊΠΎΠ½ΡΡ D. ΠΠ°ΠΏΡΠ°Π²Π»Π΅Π½ΠΈΠ΅ ΡΠΈΠ»Ρ ΠΠΎΡΠ΅Π½ΡΠ° ΠΌΠΎΠΆΠ½ΠΎ ΠΎΠΏΡΠ΅Π΄Π΅Π»ΠΈΡΡ ΠΏΠΎ ΠΏΡΠ°Π²ΠΈΠ»Ρ Π»Π΅Π²ΠΎΠΉ ΡΡΠΊΠΈ Ρ ΡΡΠ΅ΡΠΎΠΌ Π·Π½Π°ΠΊΠ° Π·Π°ΡΡΠ΄Π° ΡΠ»Π΅ΠΊΡΡΠΎΠ½Π°.
ΠΡΠ»Π΅Π΄ΡΡΠ²ΠΈΠ΅ ΡΡΠΎΠ³ΠΎ Π½Π° ΡΠΎΡΡΠ°Ρ D ΠΈ Π ΠΏΡΠΎΠ²ΠΎΠ΄Π½ΠΈΠΊΠ° ΠΏΠΎΡΠ²ΡΡΡΡ, ΡΠΎΠΎΡΠ²Π΅ΡΡΡΠ²Π΅Π½Π½ΠΎ, ΠΎΡΡΠΈΡΠ°ΡΠ΅Π»ΡΠ½ΡΠ΅ ΠΈ ΠΏΠΎΠ»ΠΎΠΆΠΈΡΠ΅Π»ΡΠ½ΡΠ΅ Π·Π°ΡΡΠ΄Ρ, ΠΊΠΎΡΠΎΡΡΠ΅ ΡΠΎΠ·Π΄Π°Π΄ΡΡ ΡΠ»Π΅ΠΊΡΡΠΈΡΠ΅ΡΠΊΠΎΠ΅ ΠΏΠΎΠ»Π΅, Π° Π·Π½Π°ΡΠΈΡ, ΠΈ Π½Π΅ΠΊΠΎΡΠΎΡΡΡ ΡΠ°Π·Π½ΠΎΡΡΡ ΠΏΠΎΡΠ΅Π½ΡΠΈΠ°Π»ΠΎΠ² ΠΌΠ΅ΠΆΠ΄Ρ ΡΠΎΡΡΠ°ΠΌΠΈ ΠΏΡΠΎΠ²ΠΎΠ΄Π½ΠΈΠΊΠ°.
ΠΡΠΎΡΠ΅ΡΡ ΡΠ°Π·Π΄Π΅Π»Π΅Π½ΠΈΡ Π·Π°ΡΡΠ΄ΠΎΠ² (ΠΏΠΎΠ»ΡΡΠΈΠ·Π°ΡΠΈΠΈ ΠΏΡΠΎΠ²ΠΎΠ΄Π½ΠΈΠΊΠ°) Π±ΡΠ΄Π΅Ρ ΠΏΡΠΎΠ΄ΠΎΠ»ΠΆΠ°ΡΡΡΡ Π΄ΠΎ ΡΠ΅Ρ ΠΏΠΎΡ, ΠΏΠΎΠΊΠ° ΡΠΈΠ»Π°, Π΄Π΅ΠΉΡΡΠ²ΡΡΡΠ°Ρ Π½Π° Π»ΡΠ±ΠΎΠΉ ΡΠ²ΠΎΠ±ΠΎΠ΄Π½ΡΠΉ ΡΠ»Π΅ΠΊΡΡΠΎΠ½ ΡΠΎ ΡΡΠΎΡΠΎΠ½Ρ ΡΠ»Π΅ΠΊΡΡΠΈΡΠ΅ΡΠΊΠΎΠ³ΠΎ ΠΏΠΎΠ»Ρ (ΡΠΌ. ΡΠΈΡ. 164), Π½Π΅ ΡΡΠ°Π²Π½ΠΎΠ²Π΅ΡΠΈΡ ΡΠΈΠ»Ρ ΠΠΎΡΠ΅Π½ΡΠ° Π΄Π΅ΠΉΡΡΠ²ΡΡΡΡΡ Π½Π° ΡΠ»Π΅ΠΊΡΡΠΎΠ½ ΡΠΎ ΡΡΠΎΡΠΎΠ½Ρ ΠΌΠ°Π³Π½ΠΈΡΠ½ΠΎΠ³ΠΎ ΠΏΠΎΠ»Ρ.
Π‘ ΡΡΠ΅ΡΠΎΠΌ Π²ΡΡΠ°ΠΆΠ΅Π½ΠΈΡ Π΄Π»Ρ ΡΠΈΠ»Ρ ΠΡΠ»ΠΎΠ½Π°, Π΄Π΅ΠΉΡΡΠ²ΡΡΡΠ΅ΠΉ Π½Π° ΠΊΠ°ΠΆΠ΄ΡΠΉ ΡΠ»Π΅ΠΊΡΡΠΎΠ½ Π³Π΄Π΅ β Π½Π°ΠΏΡΡΠΆΠ΅Π½Π½ΠΎΡΡΡ ΡΠΎΠ·Π΄Π°Π½Π½ΠΎΠ³ΠΎ ΡΠ»Π΅ΠΊΡΡΠΈΡΠ΅ΡΠΊΠΎΠ³ΠΎ ΠΏΠΎΠ»Ρ, ΠΈ ΡΠΈΠ»Ρ ΠΠΎΡΠ΅Π½ΡΠ° ΠΌΠΎΠΆΠ½ΠΎ Π·Π°ΠΏΠΈΡΠ°ΡΡ:
ΠΡΠΊΡΠ΄Π°
ΠΠΎΠ΄ Π΄Π΅ΠΉΡΡΠ²ΠΈΠ΅ΠΌ ΡΠΈΠ» ΠΡΠ»ΠΎΠ½Π° ΠΈ ΠΠΎΡΠ΅Π½ΡΠ° Π·Π°ΡΡΠ΄Ρ Π±ΡΠ΄ΡΡ Π½Π°Ρ
ΠΎΠ΄ΠΈΡΡΡΡ Π² ΡΠ°Π²Π½ΠΎΠ²Π΅ΡΠΈΠΈ, ΠΈ Π΄Π°Π»ΡΠ½Π΅ΠΉΡΠ΅Π΅ ΠΈΡ
ΡΠ°Π·Π΄Π΅Π»Π΅Π½ΠΈΠ΅ ΠΏΡΠ΅ΠΊΡΠ°ΡΠΈΡΡΡ. ΠΠΎ Π²ΡΠ΅ΠΉ Π΄Π»ΠΈΠ½Π΅ ΠΏΡΠΎΠ²ΠΎΠ΄Π½ΠΈΠΊΠ° ΡΡΡΠ°Π½ΠΎΠ²ΠΈΡΡΡ ΠΎΠ΄Π½ΠΎΡΠΎΠ΄Π½ΠΎΠ΅ ΡΠ»Π΅ΠΊΡΡΠΈΡΠ΅ΡΠΊΠΎΠ΅ ΠΏΠΎΠ»Π΅. ΠΠ΅ΠΆΠ΄Ρ ΠΊΠΎΠ½ΡΠ°ΠΌΠΈ ΠΏΡΠΎΠ²ΠΎΠ΄Π½ΠΈΠΊΠ° (Π² ΡΠ°Π·ΠΎΠΌΠΊΠ½ΡΡΠΎΠΉ ΡΠ΅ΠΏΠΈ) Π΄Π»ΠΈΠ½ΠΎΠΉ lΒ ΠΏΠΎΡΠ²ΠΈΡΡΡ ΡΠ°Π·Π½ΠΎΡΡΡ ΠΏΠΎΡΠ΅Π½ΡΠΈΠ°Π»ΠΎΠ²
Β ΠΊΠΎΡΠΎΡΠ°Ρ ΡΠ²Π»ΡΠ΅ΡΡΡ ΡΠ°ΡΡΠ½ΡΠΌ ΡΠ»ΡΡΠ°Π΅ΠΌ Π²ΠΎΠ·Π½ΠΈΠΊΠ½ΠΎΠ²Π΅Π½ΠΈΡ ΠΠΠ‘ ΡΠ»Π΅ΠΊΡΡΠΎΠΌΠ°Π³Π½ΠΈΡΠ½ΠΎΠΉ ΠΈΠ½Π΄ΡΠΊΡΠΈΠΈ.
Π’Π°ΠΊΠΈΠΌ ΠΎΠ±ΡΠ°Π·ΠΎΠΌ, ΠΏΡΠΈ Π΄Π²ΠΈΠΆΠ΅Π½ΠΈΠΈ ΠΏΡΠΎΠ²ΠΎΠ΄Π½ΠΈΠΊΠ° Π΄Π»ΠΈΠ½ΠΎΠΉ lΒ ΡΠΎ ΡΠΊΠΎΡΠΎΡΡΡΡ Π² ΠΏΠΎΡΡΠΎΡΠ½Π½ΠΎΠΌ ΠΌΠ°Π³Π½ΠΈΡΠ½ΠΎΠΌ ΠΏΠΎΠ»Π΅ ΠΈΠ½Π΄ΡΠΊΡΠΈΠ΅ΠΉ Π² Π½Π΅ΠΌ Π²ΠΎΠ·Π½ΠΈΠΊΠ°Π΅Ρ ΠΠΠ‘ ΠΈΠ½Π΄ΡΠΊΡΠΈΠΈ
ΠΡΠΈΡΠΈΠ½ΠΎΠΉ ΠΏΠΎΡΠ²Π»Π΅Π½ΠΈΡ ΡΡΠΎΠΉ ΠΠΠ‘ ΡΠ²Π»ΡΠ΅ΡΡΡ ΡΠΈΠ»Π° ΠΠΎΡΠ΅Π½ΡΠ°, Π΄Π΅ΠΉΡΡΠ²ΡΡΡΠ°Ρ Π½Π° ΡΠ²ΠΎΠ±ΠΎΠ΄Π½ΡΠ΅ ΡΠ»Π΅ΠΊΡΡΠΎΠ½Ρ Π² Π΄Π²ΠΈΠΆΡΡΠ΅ΠΌΡΡ ΠΏΡΠΎΠ²ΠΎΠ΄Π½ΠΈΠΊΠ΅.
ΠΠΎΠ΄ΡΠ΅ΡΠΊΠ½Π΅ΠΌ, ΡΡΠΎ ΠΏΠΎΠ»Π½Π°Ρ ΡΠ°Π±ΠΎΡΠ° ΡΠΈΠ»Ρ ΠΠΎΡΠ΅Π½ΡΠ° Π² Π΄Π²ΠΈΠΆΡΡΠ΅ΠΌΡΡ ΠΏΡΠΎΠ²ΠΎΠ΄Π½ΠΈΠΊΠ΅ ΡΠ°Π²Π½Π° Π½ΡΠ»Ρ. ΠΡΠΎΠ΄ΠΎΠ»ΡΠ½Π°Ρ ΡΠΎΡΡΠ°Π²Π»ΡΡΡΠ°Ρ ΡΠΈΠ»Ρ ΠΠΎΡΠ΅Π½ΡΠ° Π΄Π΅ΠΉΡΡΠ²ΡΡΡΠ°Ρ Π½Π° Π·Π°ΡΡΠ΄Ρ, ΡΠΎΠ²Π΅ΡΡΠ°Π΅Ρ ΡΠ°Π±ΠΎΡΡ ΠΏΠΎ ΠΈΡ ΡΠ°Π·Π΄Π΅Π»Π΅Π½ΠΈΡ (ΡΠΈΡ. 165).
ΠΠΎΠΏΠ΅ΡΠ΅ΡΠ½Π°Ρ ΡΠΎΡΡΠ°Π²Π»ΡΡΡΠ°Ρ Π½Π°ΠΏΡΠ°Π²Π»Π΅Π½Π½Π°Ρ ΠΏΡΠΎΡΠΈΠ²ΠΎΠΏΠΎΠ»ΠΎΠΆΠ½ΠΎ ΡΠΊΠΎΡΠΎΡΡΠΈ Π΄Π²ΠΈΠΆΠ΅Π½ΠΈΡ ΠΏΡΠΎΠ²ΠΎΠ΄Π½ΠΈΠΊΠ°, ΡΠΎΠ²Π΅ΡΡΠ°Π΅Ρ ΡΠ°Π±ΠΎΡΡ ΠΠΎΠ»Π½Π°Ρ ΡΠ°Π±ΠΎΡΠ° ΡΠΈΠ»Ρ ΠΠΎΡΠ΅Π½ΡΠ° (ΡΡΠΌΠΌΠ° ΡΡΠΈΡ ΡΠ°Π±ΠΎΡ) ΡΠ°Π²Π½Π° Π½ΡΠ»Ρ ΠΡΠ»Π΅Π΄ΡΡΠ²ΠΈΠ΅ ΡΠΎΠ³ΠΎ ΡΡΠΎ ΡΠ°Π±ΠΎΡΠ° ΠΏΠΎΠΏΠ΅ΡΠ΅ΡΠ½ΠΎΠΉ ΡΠΎΡΡΠ°Π²Π»ΡΡΡΠ΅ΠΉ ΡΠΈΠ»Ρ ΠΠΎΡΠ΅Π½ΡΠ° ΠΎΡΡΠΈΡΠ°ΡΠ΅Π»ΡΠ½Π°, ΠΏΡΠΈ Π΄Π²ΠΈΠΆΠ΅Π½ΠΈΠΈ ΠΏΡΠΎΠ²ΠΎΠ΄Π½ΠΈΠΊΠ° Π² ΠΌΠ°Π³Π½ΠΈΡΠ½ΠΎΠΌ ΠΏΠΎΠ»Π΅ ΠΏΡΠΎΠΈΡΡ ΠΎΠ΄ΠΈΡ Π΅Π³ΠΎ ΡΠΎΡΠΌΠΎΠΆΠ΅Π½ΠΈΠ΅. ΠΠΎΡΡΠΎΠΌΡ Π΄Π»Ρ Π΄Π²ΠΈΠΆΠ΅Π½ΠΈΡ ΠΏΡΠΎΠ²ΠΎΠ΄Π½ΠΈΠΊΠ° Ρ ΠΏΠΎΡΡΠΎΡΠ½Π½ΠΎΠΉ ΡΠΊΠΎΡΠΎΡΡΡΡ Π½Π° Π½Π΅Π³ΠΎ Π½Π΅ΠΎΠ±Ρ ΠΎΠ΄ΠΈΠΌΠΎ Π΄Π΅ΠΉΡΡΠ²ΠΎΠ²Π°ΡΡ ΠΏΠΎΡΡΠΎΡΠ½Π½ΠΎΠΉ Π²Π½Π΅ΡΠ½Π΅ΠΉ ΡΠΈΠ»ΠΎΠΉ, ΡΠ°Π²Π½ΠΎΠΉ ΠΏΠΎ ΠΌΠΎΠ΄ΡΠ»Ρ ΠΏΠΎΠΏΠ΅ΡΠ΅ΡΠ½ΠΎΠΉ ΡΠΎΡΡΠ°Π²Π»ΡΡΡΠ΅ΠΉ ΡΠΈΠ»Ρ ΠΠΎΡΠ΅Π½ΡΠ°.
ΠΡΠ»ΠΈ ΠΏΡΠΎΠ²ΠΎΠ΄Π½ΠΈΠΊ Π·Π°ΠΌΠΊΠ½ΡΡΡ ΡΠ°ΡΠΏΠΎΠ»ΠΎΠΆΠ΅Π½Π½ΡΠΌ Π²Π½Π΅ ΠΌΠ°Π³Π½ΠΈΡΠ½ΠΎΠ³ΠΎ ΠΏΠΎΠ»Ρ ΠΏΡΠΎΠ²ΠΎΠ΄ΠΎΠΌ (ΡΠΈΡ. 166), ΡΠΎ ΠΏΠΎ ΡΡΠΎΠΌΡ ΠΏΡΠΎΠ²ΠΎΠ΄Ρ ΠΎΡ ΡΠΎΡΠΊΠΈ D ΠΊ ΡΠΎΡΠΊΠ΅ Π ΡΠ»Π΅ΠΊΡΡΠΎΠ½Ρ Π±ΡΠ΄ΡΡ ΠΏΠ΅ΡΠ΅ΠΌΠ΅ΡΠ°ΡΡΡΡ ΠΏΠΎΠ΄ Π΄Π΅ΠΉΡΡΠ²ΠΈΠ΅ΠΌ ΠΠΠ‘ ΡΠ»Π΅ΠΊΡΡΠΎΠΌΠ°Π³Π½ΠΈΡΠ½ΠΎΠΉ ΠΈΠ½Π΄ΡΠΊΡΠΈΠΈ.
Π‘ΡΡΠ΅ΡΡΠ²ΠΎΠ²Π°Π½ΠΈΠ΅ ΠΠΠ‘ ΠΈΠ½Π΄ΡΠΊΡΠΈΠΈ ΠΏΡΠΈΠ²ΠΎΠ΄ΠΈΡ ΠΊ ΠΏΠΎΡΠ²Π»Π΅Π½ΠΈΡ Π² Π·Π°ΠΌΠΊΠ½ΡΡΠΎΠΌ ΠΊΠΎΠ½ΡΡΡΠ΅ ΠΈΠ½Π΄ΡΠΊΡΠΈΠΎΠ½Π½ΠΎΠ³ΠΎ ΡΠΎΠΊΠ°. Π’Π°ΠΊΠΈΠΌ ΠΎΠ±ΡΠ°Π·ΠΎΠΌ, ΡΠ»Π΅ΠΊΡΡΠΎΠ½Ρ Π² ΠΏΡΠΎΠ²ΠΎΠ΄Π½ΠΈΠΊΠ΅ Π΄Π²ΠΈΠΆΡΡΡΡ ΠΎΡ ΡΠΎΡΠΊΠΈ Π ΠΊ ΡΠΎΡΠΊΠ΅ D ΠΏΡΠΎΡΠΈΠ² ΡΠΈΠ» ΡΠ»Π΅ΠΊΡΡΠΈΡΠ΅ΡΠΊΠΎΠ³ΠΎ ΠΏΠΎΠ»Ρ ΠΏΠΎΠ΄ Π΄Π΅ΠΉΡΡΠ²ΠΈΠ΅ΠΌ ΡΠΈΠ»Ρ ΠΠΎΡΠ΅Π½ΡΠ°, Π° ΠΎΡ ΡΠΎΡΠΊΠΈ D ΠΊ ΡΠΎΡΠΊΠ΅ Π ΠΏΠΎ ΠΏΡΠΎΠ²ΠΎΠ΄Ρ (Π²ΠΎ Π²Π½Π΅ΡΠ½Π΅ΠΉ ΡΠ΅ΠΏΠΈ) ΠΈΡ Π΄Π²ΠΈΠΆΠ΅Π½ΠΈΠ΅ ΠΏΡΠΎΠΈΡΡ ΠΎΠ΄ΠΈΡ ΠΏΠΎΠ΄ Π΄Π΅ΠΉΡΡΠ²ΠΈΠ΅ΠΌ ΡΠΈΠ» ΡΠ»Π΅ΠΊΡΡΠΈΡΠ΅ΡΠΊΠΎΠ³ΠΎ ΠΏΠΎΠ»Ρ.
ΠΠΎΠ³Π΄Π° ΠΏΡΠΎΠ²ΠΎΠ΄Π½ΠΈΠΊ Π΄Π»ΠΈΠ½ΠΎΠΉ lΒ Ρ ΠΈΠ½Π΄ΡΠΊΡΠΈΠΎΠ½Π½ΡΠΌ ΡΠΎΠΊΠΎΠΌ ΡΠΈΠ»ΠΎΠΉ IΒ Π΄Π²ΠΈΠΆΠ΅ΡΡΡ Π² ΠΌΠ°Π³Π½ΠΈΡΠ½ΠΎΠΌ ΠΏΠΎΠ»Π΅ ΠΈΠ½Π΄ΡΠΊΡΠΈΠ΅ΠΉ ΡΠΎ Π½Π° Π½Π΅Π³ΠΎ ΡΠΎ ΡΡΠΎΡΠΎΠ½Ρ ΠΏΠΎΠ»Ρ Π΄Π΅ΠΉΡΡΠ²ΡΠ΅Ρ ΡΠΈΠ»Π° ΠΠΌΠΏΠ΅ΡΠ°, ΠΌΠΎΠ΄ΡΠ»Ρ ΠΊΠΎΡΠΎΡΠΎΠΉ ΠΠ»Ρ ΠΎΠΏΡΠ΅Π΄Π΅Π»Π΅Π½ΠΈΡ Π½Π°ΠΏΡΠ°Π²Π»Π΅Π½ΠΈΡ ΡΡΠΎΠΉ ΡΠΈΠ»Ρ ΠΏΡΠΈΠΌΠ΅Π½ΠΈΠΌ ΠΏΡΠ°Π²ΠΈΠ»ΠΎ Π»Π΅Π²ΠΎΠΉ ΡΡΠΊΠΈ (ΡΠΈΡ. 167).
ΠΠ°ΠΏΡΠ°Π²Π»Π΅Π½ΠΈΠ΅ ΡΠΈΠ»Ρ ΠΏΡΠΎΡΠΈΠ²ΠΎΠΏΠΎΠ»ΠΎΠΆΠ½ΠΎ Π½Π°ΠΏΡΠ°Π²Π»Π΅Π½ΠΈΡ Π΄Π²ΠΈΠΆΠ΅Π½ΠΈΡ ΠΏΡΠΎ
Π²ΠΎΠ΄Π½ΠΈΠΊΠ°. Π‘Π»Π΅Π΄ΠΎΠ²Π°ΡΠ΅Π»ΡΠ½ΠΎ, ΠΈΠ½Π΄ΡΠΊΡΠΈΠΎΠ½Π½ΡΠΉ ΡΠΎΠΊ, Π²Π·Π°ΠΈΠΌΠΎΠ΄Π΅ΠΉΡΡΠ²ΡΡ Ρ ΠΌΠ°Π³Π½ΠΈΡΠ½ΡΠΌ ΠΏΠΎΠ»Π΅ΠΌ, ΡΠΎΠ·Π΄Π°Π΅Ρ ΡΠΈΠ»Ρ, ΡΠΎΡΠΌΠΎΠ·ΡΡΡΡ Π΄Π²ΠΈΠΆΠ΅Π½ΠΈΠ΅ ΠΏΡΠΎΠ²ΠΎΠ΄Π½ΠΈΠΊΠ°. Π’Π°ΠΊΠΈΠΌ ΠΎΠ±ΡΠ°Π·ΠΎΠΌ, Π΄Π»Ρ Π΄Π²ΠΈΠΆΠ΅Π½ΠΈΡ ΠΏΡΠΎΠ²ΠΎΠ΄Π½ΠΈΠΊΠ° Π½Π΅ΠΎΠ±Ρ
ΠΎΠ΄ΠΈΠΌΠ° Π²Π½Π΅ΡΠ½ΡΡ ΡΠΈΠ»Π°, ΡΠΎΠ²Π΅ΡΡΠ°ΡΡΠ°Ρ ΡΠ°Π±ΠΎΡΡ ΠΏΡΠΎΡΠΈΠ² ΡΠΈΠ»Ρ ΠΠΌΠΏΠ΅ΡΠ°. ΠΠ½Π΄ΡΠΊΡΠΈΠΎΠ½Π½ΡΠΉ ΡΠΎΠΊ Π½Π°Π³ΡΠ΅Π²Π°Π΅Ρ ΠΏΡΠΎΠ²ΠΎΠ΄Π½ΠΈΠΊ, ΠΏΠΎ ΠΊΠΎΡΠΎΡΠΎΠΌΡ ΠΎΠ½ ΠΏΡΠΎΡ
ΠΎΠ΄ΠΈΡ. ΠΡΠ΄Π΅Π»ΡΡΡΠ°ΡΡΡ Π² Π²ΠΈΠ΄Π΅ ΡΠ΅ΠΏΠ»Π° ΡΠ½Π΅ΡΠ³ΠΈΡ ΠΏΠΎΡΡΡΠΏΠ°Π΅Ρ Π² ΡΠ΅ΠΏΡ Π±Π»Π°Π³ΠΎΠ΄Π°ΡΡ ΡΠ°Π±ΠΎΡΠ΅, ΠΊΠΎΡΠΎΡΡΡ ΡΠΎΠ²Π΅ΡΡΠ°Π΅Ρ Π²Π½Π΅ΡΠ½ΡΡ ΡΠΈΠ»Π°, Π΄Π²ΠΈΠ³Π°Ρ ΠΏΡΠΎΠ²ΠΎΠ΄Π½ΠΈΠΊ ΠΏΡΠΎΡΠΈΠ² ΡΠΈΠ» ΠΏΠΎΠ»Ρ.
ΠΡΠ»ΠΈ ΠΏΡΠΎΠ²ΠΎΠ΄Π½ΠΈΠΊ, ΡΠ°ΡΠΏΠΎΠ»ΠΎΠΆΠ΅Π½Π½ΡΠΉ ΠΏΠ΅ΡΠΏΠ΅Π½Π΄ΠΈΠΊΡΠ»ΡΡΠ½ΠΎ Π»ΠΈΠ½ΠΈΡΠΌ ΠΈΠ½Π΄ΡΠΊΡΠΈΠΈ ΠΌΠ°Π³Π½ΠΈΡΠ½ΠΎΠ³ΠΎ ΠΏΠΎΠ»Ρ, Π΄Π²ΠΈΠΆΠ΅ΡΡΡ ΡΠΎ ΡΠΊΠΎΡΠΎΡΡΡΡ Π½Π°ΠΏΡΠ°Π²Π»Π΅Π½Π½ΠΎΠΉ ΠΏΠΎΠ΄ ΡΠ³Π»ΠΎΠΌ ΠΊ Π»ΠΈΠ½ΠΈΡΠΌ ΠΈΠ½Π΄ΡΠΊΡΠΈΠΈ (ΡΠΈΡ. 168, Π°), ΡΠΎ Π΅Π΅ ΠΌΠΎΠΆΠ½ΠΎ ΡΠ°Π·Π»ΠΎΠΆΠΈΡΡ Π½Π° Π΄Π²Π΅ ΡΠΎΡΡΠ°Π²Π»ΡΡΡΠΈΠ΅ β ΠΏΠ°ΡΠ°Π»Π»Π΅Π»ΡΠ½ΡΡ ΠΈ ΠΏΠ΅ΡΠΏΠ΅Π½Π΄ΠΈΠΊΡΠ»ΡΡΠ½ΡΡΒ Π½Π°ΠΏΡΠ°Π²Π»Π΅Π½ΠΈΡ ΠΌΠ°Π³Π½ΠΈΡΠ½ΠΎΠ³ΠΎ ΠΏΠΎΠ»Ρ (ΡΠΈΡ. 168, Π±):
ΠΡΠΈ Π΄Π²ΠΈΠΆΠ΅Π½ΠΈΠΈ ΠΏΡΠΎΠ²ΠΎΠ΄Π½ΠΈΠΊΠ° Π²Π΄ΠΎΠ»Ρ Π»ΠΈΠ½ΠΈΠΈ ΠΈΠ½Π΄ΡΠΊΡΠΈΠΈ Π² Π½Π΅ΠΌ Π½Π΅ Π²ΠΎΠ·Π½ΠΈΠΊΠ°Π΅Ρ ΠΠΠ‘ ΠΈΠ½Π΄ΡΠΊΡΠΈΠΈ, ΠΏΠΎΡΡΠΎΠΌΡ Π²ΠΊΠ»Π°Π΄ Π²Β Π²Π½ΠΎΡΠΈΡ ΡΠΎΠ»ΡΠΊΠΎ ΠΏΠ΅ΡΠΏΠ΅Π½Π΄ΠΈΠΊΡΠ»ΡΡΠ½Π°Ρ ΡΠΎΡΡΠ°Π²Π»ΡΡΡΠ°Ρ ΡΠΊΠΎΡΠΎΡΡΠΈ ΠΡΠ»Π΅Π΄ΡΡΠ²ΠΈΠ΅ ΡΡΠΎΠ³ΠΎ Π²ΡΡΠ°ΠΆΠ΅Π½ΠΈΠ΅ Π΄Π»Ρ ΠΠΠ‘ ΠΈΠ½Π΄ΡΠΊΡΠΈΠΈ Π² ΠΎΠ±ΡΠ΅ΠΌ Π²ΠΈΠ΄Π΅ ΠΏΡΠ΅Π΄ΡΡΠ°Π²Π»ΡΠ΅ΡΡΡ ΡΠΎΠΎΡΠ½ΠΎΡΠ΅Π½ΠΈΠ΅ΠΌ
Π’Π°ΠΊΠΈΠΌ ΠΎΠ±ΡΠ°Π·ΠΎΠΌ, Π² ΠΎΠ±ΡΠ΅ΠΌ ΡΠ»ΡΡΠ°Π΅ Π½Π΅ΠΎΠ±Ρ
ΠΎΠ΄ΠΈΠΌΠΎ ΡΡΠΈΡΡΠ²Π°ΡΡ ΡΠ³ΠΎΠ» ΠΌΠ΅ΠΆΠ΄Ρ Π½Π°ΠΏΡΠ°Π²Π»Π΅Π½ΠΈΠ΅ΠΌ ΡΠΊΠΎΡΠΎΡΡΠΈ Π΄Π²ΠΈΠΆΠ΅Π½ΠΈΡ ΠΏΡΠΎΠ²ΠΎΠ΄Π½ΠΈΠΊΠ° (ΡΠΊΠΎΡΠΎΡΡΠΈ ΡΠ»Π΅ΠΊΡΡΠΎΠ½ΠΎΠ²) ΠΈ Π²Π΅ΠΊΡΠΎΡΠΎΠΌ ΠΈΠ½Π΄ΡΠΊΡΠΈΠΈ ΠΌΠ°Π³Π½ΠΈΡΠ½ΠΎΠ³ΠΎ ΠΏΠΎΠ»Ρ (ΠΊΠ°ΠΊ ΡΡΠΎ Π΄Π΅Π»Π°Π»ΠΎΡΡ Π΄Π»Ρ ΡΠΈΠ»Ρ ΠΠΎΡΠ΅Π½ΡΠ°).
ΠΠ΅ΡΠ½Π΅ΠΌΡΡ ΠΊ ΡΠ°ΡΡΠΌΠΎΡΡΠ΅Π½ΠΈΡ ΠΠΠ‘ ΡΠ»Π΅ΠΊΡΡΠΎΠΌΠ°Π³Π½ΠΈΡΠ½ΠΎΠΉ ΠΈΠ½Π΄ΡΠΊΡΠΈΠΈ, Π²ΠΎΠ·Π½ΠΈΠΊΠ°ΡΡΠ΅ΠΉ Π² ΠΏΡΠΎΠ²ΠΎΠ΄Π½ΠΈΠΊΠ΅, Π΄Π²ΠΈΠΆΡΡΠ΅ΠΌΡΡ ΡΠ°Π²Π½ΠΎΠΌΠ΅ΡΠ½ΠΎ ΠΈ ΠΏΡΡΠΌΠΎΠ»ΠΈΠ½Π΅ΠΉΠ½ΠΎ Π² ΠΌΠ°Π³Π½ΠΈΡΠ½ΠΎΠΌ ΠΏΠΎΠ»Π΅. ΠΠΎΡΠΊΠΎΠ»ΡΠΊΡ ΡΠΊΠΎΡΠΎΡΡΡ ΠΎΠΏΡΠ΅Π΄Π΅Π»ΡΠ΅ΡΡΡ, ΠΊΠ°ΠΊ ΡΠΎ Π²ΡΡΠ°ΠΆΠ΅Π½ΠΈΠ΅ Π΄Π»Ρ ΠΠΠ‘ ΠΌΠΎΠΆΠ½ΠΎ ΠΏΡΠ΅Π΄ΡΡΠ°Π²ΠΈΡΡ Π² Π²ΠΈΠ΄Π΅
Π£ΡΠΈΡΡΠ²Π°Ρ, ΡΡΠΎ β ΠΏΠ»ΠΎΡΠ°Π΄Ρ ΠΏΡΡΠΌΠΎΡΠ³ΠΎΠ»ΡΠ½ΠΈΠΊΠ° KNN’K’, ΠΎΡ
Π²Π°ΡΡΠ²Π°Π΅ΠΌΠ°Ρ ΠΏΡΠΎΠ²ΠΎΠ΄Π½ΠΈΠΊΠΎΠΌ Π·Π° ΠΏΡΠΎΠΌΠ΅ΠΆΡΡΠΎΠΊ Π²ΡΠ΅ΠΌΠ΅Π½ΠΈ (ΡΠΌ. ΡΠΈΡ. 168, Π°), ΠΌΠΎΠΆΠ΅ΠΌ Π·Π°ΠΏΠΈΡΠ°ΡΡ:
Π‘ ΡΡΠ΅ΡΠΎΠΌ ΠΎΠΏΡΠ΅Π΄Π΅Π»Π΅Π½ΠΈΡ ΠΌΠ°Π³Π½ΠΈΡΠ½ΠΎΠ³ΠΎ ΠΏΠΎΡΠΎΠΊΠ° ΠΏΠΎΠ»ΡΡΠ΅Π½Π½ΠΎΠ΅ Π²ΡΡΠ°ΠΆΠ΅Π½ΠΈΠ΅ Π΄Π»Ρ ΠΠΠ‘ ΠΈΠ½Π΄ΡΠΊΡΠΈΠΈ ΠΌΠΎΠΆΠ½ΠΎ ΠΏΡΠ΅Π΄ΡΡΠ°Π²ΠΈΡΡ Π² Π²ΠΈΠ΄Π΅ (ΡΠΌ. ΡΠΈΡ. 168, Π±):
ΠΠ΄Π΅ΡΡ β ΠΈΠ·ΠΌΠ΅Π½Π΅Π½ΠΈΠ΅ ΠΌΠ°Π³Π½ΠΈΡΠ½ΠΎΠ³ΠΎ ΠΏΠΎΡΠΎΠΊΠ° ΡΠ΅ΡΠ΅Π· ΠΊΠΎΠ½ΡΡΡ Π·Π° ΠΏΡΠΎΠΌΠ΅ΠΆΡΡΠΎΠΊ Π²ΡΠ΅ΠΌΠ΅Π½ΠΈ (ΡΠΈΡΠ»ΠΎ ΠΏΠ΅ΡΠ΅ΡΠ΅ΡΠ΅Π½Π½ΡΡ ΠΏΡΠΎΠ²ΠΎΠ΄Π½ΠΈΠΊΠΎΠΌ Π»ΠΈΠ½ΠΈΠΉ ΠΈΠ½Π΄ΡΠΊΡΠΈΠΈ ΠΌΠ°Π³Π½ΠΈΡΠ½ΠΎΠ³ΠΎ ΠΏΠΎΠ»Ρ).
ΠΠ°ΠΏΡΠ°Π²Π»Π΅Π½ΠΈΠ΅ ΠΈΠ½Π΄ΡΠΊΡΠΈΠΎΠ½Π½ΠΎΠ³ΠΎ ΡΠΎΠΊΠ° Π² ΠΊΠΎΠ½ΡΡΡΠ΅ Ρ ΠΏΠ΅ΡΠ΅ΠΌΠ΅ΡΠ°ΡΡΠΈΠΌΡΡ ΠΏΡΠΎΠ²ΠΎΠ΄Π½ΠΈΠΊΠΎΠΌ ΠΌΠΎΠΆΠ΅Ρ Π±ΡΡΡ ΡΡΡΠ°Π½ΠΎΠ²Π»Π΅Π½ΠΎ Ρ ΠΏΠΎΠΌΠΎΡΡΡ ΠΏΡΠ°Π²ΠΈΠ»Π° ΠΏΡΠ°Π²ΠΎΠΉ ΡΡΠΊΠΈ (ΡΠΈΡ. 169):
Π΅ΡΠ»ΠΈ Π»Π°Π΄ΠΎΠ½Ρ ΠΏΡΠ°Π²ΠΎΠΉ ΡΡΠΊΠΈ ΡΠ°ΡΠΏΠΎΠ»ΠΎΠΆΠΈΡΡ ΡΠ°ΠΊ, ΡΡΠΎΠ±Ρ Π²Π΅ΠΊΡΠΎΡ ΠΈΠ½Π΄ΡΠΊΡΠΈΠΈ ΠΌΠ°Π³Π½ΠΈΡΠ½ΠΎΠ³ΠΎ ΠΏΠΎΠ»Ρ Π²Ρ
ΠΎΠ΄ΠΈΠ» Π² Π»Π°Π΄ΠΎΠ½Ρ, Π° ΠΎΡΡΡΠ°Π²Π»Π΅Π½Π½ΡΠΉ Π½Π° 90Β° Π±ΠΎΠ»ΡΡΠΎΠΉ ΠΏΠ°Π»Π΅Ρ ΡΠΎΠ²ΠΏΠ°Π΄Π°Π» Ρ Π½Π°ΠΏΡΠ°Π²Π»Π΅Π½ΠΈΠ΅ΠΌ ΡΠΊΠΎΡΠΎΡΡΠΈ Π΄Π²ΠΈΠΆΠ΅Π½ΠΈΡ ΠΏΡΠΎΠ²ΠΎΠ΄Π½ΠΈΠΊΠ°, ΡΠΎ ΡΠ΅ΡΡΡΠ΅ Π²ΡΡΡΠ½ΡΡΡΡ
ΠΏΠ°Π»ΡΡΠ° ΡΠΊΠ°ΠΆΡΡ Π½Π°ΠΏΡΠ°Π²Π»Π΅Π½ΠΈΠ΅ ΠΈΠ½Π΄ΡΠΊΡΠΈΠΎΠ½Π½ΠΎΠ³ΠΎ ΡΠΎΠΊΠ°.
Π’Π°ΠΊΠΈΠΌ ΠΎΠ±ΡΠ°Π·ΠΎΠΌ, ΡΠ²Π»Π΅Π½ΠΈΠ΅ ΡΠ»Π΅ΠΊΡΡΠΎΠΌΠ°Π³Π½ΠΈΡΠ½ΠΎΠΉ ΠΈΠ½Π΄ΡΠΊΡΠΈΠΈ ΡΠ²ΡΠ·Π°Π½ΠΎ Ρ ΠΈΠ·ΠΌΠ΅Π½Π΅Π½ΠΈΠ΅ΠΌ ΠΌΠ°Π³Π½ΠΈΡΠ½ΠΎΠ³ΠΎ ΠΏΠΎΡΠΎΠΊΠ° ΡΠ΅ΡΠ΅Π· ΠΊΠΎΠ½ΡΡΡ Π²Π½Π΅ Π·Π°Π²ΠΈΡΠΈΠΌΠΎΡΡΠΈ ΠΎΡ Ρ Π°ΡΠ°ΠΊΡΠ΅ΡΠ° ΠΏΡΠΈΡΠΈΠ½, ΠΎΠ±ΡΡΠ»ΠΎΠ²Π»ΠΈΠ²Π°ΡΡΠΈΡ ΡΡΠΎ ΠΈΠ·ΠΌΠ΅Π½Π΅Π½ΠΈΠ΅. ΠΠΠ‘ ΡΠ»Π΅ΠΊΡΡΠΎΠΌΠ°Π³Π½ΠΈΡΠ½ΠΎΠΉ ΠΈΠ½Π΄ΡΠΊΡΠΈΠΈ ΠΏΡΡΠΌΠΎ ΠΏΡΠΎΠΏΠΎΡΡΠΈΠΎΠ½Π°Π»ΡΠ½Π° ΡΠΊΠΎΡΠΎΡΡΠΈ ΠΈΠ·ΠΌΠ΅Π½Π΅Π½ΠΈΡ ΠΌΠ°Π³Π½ΠΈΡΠ½ΠΎΠ³ΠΎ ΠΏΠΎΡΠΎΠΊΠ°, Ρ. Π΅. ΠΈΠ·ΠΌΠ΅Π½Π΅Π½ΠΈΡ ΠΌΠ°Π³Π½ΠΈΡΠ½ΠΎΠ³ΠΎ ΠΏΠΎΡΠΎΠΊΠ° ΡΠ΅ΡΠ΅Π· ΠΊΠΎΠ½ΡΡΡ Π² Π΅Π΄ΠΈΠ½ΠΈΡΡ Π²ΡΠ΅ΠΌΠ΅Π½ΠΈ.
ΠΠΎΠ΄ΡΠ΅ΡΠΊΠ½Π΅ΠΌ Π΅ΡΠ΅ ΡΠ°Π·, ΡΡΠΎ ΠΈΠ·ΠΌΠ΅Π½Π΅Π½ΠΈΠ΅ ΠΌΠ°Π³Π½ΠΈΡΠ½ΠΎΠ³ΠΎ ΠΏΠΎΡΠΎΠΊΠ° ΡΠ΅ΡΠ΅Π· ΠΊΠΎΠ½ΡΡΡ Π²ΠΎΠ·Π½ΠΈΠΊΠ°Π΅Ρ ΠΏΡΠΈ ΠΈΠ·ΠΌΠ΅Π½Π΅Π½ΠΈΠΈ:
- ΠΈΠ½Π΄ΡΠΊΡΠΈΠΈ ΠΌΠ°Π³Π½ΠΈΡΠ½ΠΎΠ³ΠΎ ΠΏΠΎΠ»Ρ;
- ΠΏΠ»ΠΎΡΠ°Π΄ΠΈ SΒ ΠΊΠΎΠ½ΡΡΡΠ°;
- ΠΎΡΠΈΠ΅Π½ΡΠ°ΡΠΈΠΈ ΠΊΠΎΠ½ΡΡΡΠ° Π² ΠΌΠ°Π³Π½ΠΈΡΠ½ΠΎΠΌ ΠΏΠΎΠ»Π΅ (ΡΠ³Π»Π° ).
ΡΡΡΡ ΡΠ²Π»Π΅Π½ΠΈΡ, Π·Π°ΠΊΠΎΠ½ Π€Π°ΡΠ°Π΄Π΅Ρ, ΡΠΎΡΠΌΡΠ»Ρ
ΠΠ»Π΅ΠΊΡΡΠΎΠΌΠ°Π³Π½ΠΈΡΠ½Π°Ρ ΠΈΠ½Π΄ΡΠΊΡΠΈΡ β ΡΡΠΎ ΠΎΡΠ΅Π½Ρ Π²Π°ΠΆΠ½ΠΎΠ΅ ΡΠΈΠ·ΠΈΡΠ΅ΡΠΊΠΎΠ΅ ΡΠ²Π»Π΅Π½ΠΈΠ΅, ΠΈΡΠΏΠΎΠ»ΡΠ·ΡΠ΅ΠΌΠΎΠ΅ Π² ΡΠ°Π±ΠΎΡΠ΅ ΠΌΠ½ΠΎΠ³ΠΈΡ ΡΡΡΡΠΎΠΉΡΡΠ², ΡΠ°ΠΊΠΈΡ ΠΊΠ°ΠΊ ΡΡΠ°Π½ΡΡΠΎΡΠΌΠ°ΡΠΎΡ, Π³Π΅Π½Π΅ΡΠ°ΡΠΎΡ ΠΏΠ΅ΡΠ΅ΠΌΠ΅Π½Π½ΠΎΠ³ΠΎ Π½Π°ΠΏΡΡΠΆΠ΅Π½ΠΈΡ, ΠΈΠ½Π΄ΡΠΊΡΠΈΠΎΠ½Π½Π°Ρ ΠΏΠ»ΠΈΡΠ°. ΠΠ½ΠΎ ΡΠ°ΠΊΠΆΠ΅ ΠΈΠΌΠ΅Π»ΠΎ Π±ΠΎΠ»ΡΡΠΎΠ΅ ΡΠ΅ΠΎΡΠ΅ΡΠΈΡΠ΅ΡΠΊΠΎΠ΅ Π·Π½Π°ΡΠ΅Π½ΠΈΠ΅ β ΠΏΡΠΈΠ²Π΅Π»ΠΎ ΠΊ ΠΎΡΠΊΡΡΡΠΈΡ ΡΠ»Π΅ΠΊΡΡΠΎΠΌΠ°Π³Π½ΠΈΡΠ½ΠΎΠΉ Π²ΠΎΠ»Π½Ρ.
Π€Π°ΡΠ°Π΄Π΅Ρ, ΠΏΠ΅ΡΠ²ΠΎΠΎΡΠΊΡΡΠ²Π°ΡΠ΅Π»Ρ ΡΠ²Π»Π΅Π½ΠΈΡ ΡΠ»Π΅ΠΊΡΡΠΎΠΌΠ°Π³Π½ΠΈΡΠ½ΠΎΠΉ ΠΈΠ½Π΄ΡΠΊΡΠΈΠΈ, ΠΏΠΎΡΠ΅ΡΠΈΠ» Π² ΡΠ²ΠΎΠ΅ΠΉ Π»Π°Π±ΠΎΡΠ°ΡΠΎΡΠΈΠΈ ΠΌΠΈΠ½ΠΈΡΡΡ ΡΠΈΠ½Π°Π½ΡΠΎΠ² ΠΠ΅Π»ΠΈΠΊΠΎΠ±ΡΠΈΡΠ°Π½ΠΈΠΈ ΠΈ ΡΠΏΡΠΎΡΠΈΠ»:
- β ΠΠ°ΠΊΡΡ ΠΏΠΎΠ»ΡΠ·Ρ ΡΠ΅Π»ΠΎΠ²Π΅ΡΠ΅ΡΡΠ²ΠΎ ΠΏΠΎΠ»ΡΡΠΈΡ ΠΎΡ Π²Π°ΡΠ΅Π³ΠΎ ΠΈΡΡΠ»Π΅Π΄ΠΎΠ²Π°Π½ΠΈΡ? β
Π€Π°ΡΠ°Π΄Π΅ΠΉ ΠΎΡΠ²Π΅ΡΠΈΠ»:
- β Π’ΡΡΠ΄Π½ΠΎ ΡΡΠ΄ΠΈΡΡ, Π½ΠΎ Ρ ΡΠ²Π΅ΡΠ΅Π½, ΡΡΠΎ Π²Ρ Π±ΡΠ΄Π΅ΡΠ΅ ΡΠΎΠ±ΠΈΡΠ°ΡΡ Ρ ΡΡΠΎΠ³ΠΎ Π½Π°Π»ΠΎΠ³ΠΈ. β
ΠΠ½ Π½Π΅ ΠΎΡΠΈΠ±ΡΡ β ΠΠΠ‘ Π² ΡΠΎΠΉ ΠΆΠ΅ ΠΠ΅Π»ΠΈΠΊΠΎΠ±ΡΠΈΡΠ°Π½ΠΈΠΈ Π΄ΠΎΠ±Π°Π²Π»ΡΠ΅ΡΡΡ ΠΊ ΡΠ΅Π½Π΅ ΡΠ»Π΅ΠΊΡΡΠΎΡΠ½Π΅ΡΠ³ΠΈΠΈ, ΠΏΠΎΡΡΠ°Π²Π»ΡΠ΅ΠΌΠΎΠΉ Π² Π΄ΠΎΠΌ.
ΠΡΠΈΠ²Π΅Π΄Π΅Π½Π½ΡΠΉ Π²ΡΡΠ΅ ΡΠΏΠΈΡΠΎΠΊ ΠΏΡΠΈΠΌΠ΅Π½Π΅Π½ΠΈΠΉ, Ρ ΠΎΡΡ ΠΈ Π½Π΅ΠΏΠΎΠ»Π½ΡΠΉ, Π²ΠΏΠ΅ΡΠ°ΡΠ»ΡΠ΅Ρ. ΠΠ½ΠΈ, Π±Π΅Π·ΡΡΠ»ΠΎΠ²Π½ΠΎ, ΠΏΡΠΈΡΡΡΡΡΠ²ΡΡΡ Π² Π½Π°ΡΠ΅ΠΉ ΠΆΠΈΠ·Π½ΠΈ ΠΈ ΡΠ²Π»ΡΡΡΡΡ ΠΈΠ½ΠΆΠ΅Π½Π΅ΡΠ½ΡΠΌΠΈ ΡΠ°Π·ΡΠ°Π±ΠΎΡΠΊΠ°ΠΌΠΈ ΡΠ²Π»Π΅Π½ΠΈΡ ΡΠ»Π΅ΠΊΡΡΠΎΠΌΠ°Π³Π½ΠΈΡΠ½ΠΎΠΉ ΠΈΠ½Π΄ΡΠΊΡΠΈΠΈ.
Π ΡΠ΅ΠΌ Π·Π°ΠΊΠ»ΡΡΠ°Π΅ΡΡΡ ΡΠ²Π»Π΅Π½ΠΈΠ΅ ΡΠ»Π΅ΠΊΡΡΠΎΠΌΠ°Π³Π½ΠΈΡΠ½ΠΎΠΉ ΠΈΠ½Π΄ΡΠΊΡΠΈΠΈ?
Π ΠΎΠ±ΡΠ΅ΠΌ ΡΠΌΡΡΠ»Π΅ ΡΠ²Π»Π΅Π½ΠΈΠ΅ ΡΠ»Π΅ΠΊΡΡΠΎΠΌΠ°Π³Π½ΠΈΡΠ½ΠΎΠ΅ ΠΈΠ½Π΄ΡΠΊΡΠΈΠΈ Π·Π°ΠΊΠ»ΡΡΠ°Π΅ΡΡΡ Π² Π³Π΅Π½Π΅ΡΠ°ΡΠΈΠΈ ΡΠ»Π΅ΠΊΡΡΠΈΡΠ΅ΡΠΊΠΎΠ³ΠΎ ΡΠΎΠΊΠ° Ρ ΠΏΠΎΠΌΠΎΡΡΡ ΠΌΠ°Π³Π½ΠΈΡΠ½ΠΎΠ³ΠΎ ΠΏΠΎΠ»Ρ.
Π‘ΠΊΠ°ΠΆΠ΅ΠΌ ΡΠΎΡΠ½Π΅Π΅, ΡΠ²Π»Π΅Π½ΠΈΠ΅ ΡΠ»Π΅ΠΊΡΡΠΎΠΌΠ°Π³Π½ΠΈΡΠ½ΠΎΠΉ ΠΈΠ½Π΄ΡΠΊΡΠΈΠΈ Π·Π°ΠΊΠ»ΡΡΠ°Π΅ΡΡΡ Π² ΠΎΠ±ΡΠ°Π·ΠΎΠ²Π°Π½ΠΈΠΈ ΡΠ»Π΅ΠΊΡΡΠΎΠ΄Π²ΠΈΠΆΡΡΠ΅ΠΉ ΡΠΈΠ»Ρ (ΠΠΠ‘) Π² ΠΏΡΠΎΠ²ΠΎΠ΄Π½ΠΈΠΊΠ΅ Π² ΡΠ΅Π·ΡΠ»ΡΡΠ°ΡΠ΅ ΠΈΠ·ΠΌΠ΅Π½Π΅Π½ΠΈΡ ΠΏΠΎΡΠΎΠΊΠ° ΠΌΠ°Π³Π½ΠΈΡΠ½ΠΎΠ³ΠΎ ΠΏΠΎΠ»Ρ, ΠΏΡΠΎΠ½ΠΈΠ·ΡΠ²Π°ΡΡΠ΅Π³ΠΎ ΠΏΠΎΠ²Π΅ΡΡ Π½ΠΎΡΡΡ, ΠΎΡ Π²Π°ΡΡΠ²Π°ΡΡΡΡ ΠΏΡΠΎΠ²ΠΎΠ΄Π½ΠΈΠΊ. Π Π·Π°ΠΌΠΊΠ½ΡΡΠΎΠΉ ΡΠ΅ΠΏΠΈ ΡΠ»Π΅ΠΊΡΡΠΎΠ΄Π²ΠΈΠΆΡΡΠ°Ρ ΡΠΈΠ»Π° (ΠΠΠ‘) Π²ΡΠ·ΡΠ²Π°Π΅Ρ ΠΏΡΠΎΡΠ΅ΠΊΠ°Π½ΠΈΠ΅ ΡΠ»Π΅ΠΊΡΡΠΈΡΠ΅ΡΠΊΠΎΠ³ΠΎ ΡΠΎΠΊΠ°.
Π ΠΏΡΠΈΠ²Π΅Π΄Π΅Π½Π½ΠΎΠΌ Π²ΡΡΠ΅ ΠΎΠΏΡΠ΅Π΄Π΅Π»Π΅Π½ΠΈΠΈ ΡΠ²Π»Π΅Π½ΠΈΡ ΠΌΠΎΠ³ΡΡ Π±ΡΡΡ Π½Π΅ΡΡΠ½ΡΠΌΠΈ Π΄Π²Π° ΠΏΠΎΠ½ΡΡΠΈΡ β ΠΠΠ‘ ΠΈΠ½Π΄ΡΠΊΡΠΈΠΈ ΠΈ ΠΌΠ°Π³Π½ΠΈΡΠ½ΡΠΉ ΠΏΠΎΡΠΎΠΊ.
ΠΠΠ‘ ΠΈΠ½Π΄ΡΠΊΡΠΈΠΈ.
ΠΠ±ΡΠΎΠ»ΡΡΠ½Π°Ρ Π²Π΅Π»ΠΈΡΠΈΠ½Π° ΡΠ»Π΅ΠΊΡΡΠΎΠ΄Π²ΠΈΠΆΡΡΠ΅ΠΉ ΡΠΈΠ»Ρ ( ΠΠΠ‘ ΠΈΠ½Π΄ΡΠΊΡΠΈΠΈ Ρ ΡΠΈΠΌΠ²ΠΎΠ»ΠΎΠΌ Ξ΅ΠΈΠ½Π΄ ) Π΅ΡΡΡ ΡΠ°Π±ΠΎΡΠ° Π²Π½Π΅ΡΠ½Π΅ΠΉ ΡΠΈΠ»Ρ Az, ΠΊΠΎΡΠΎΡΠ°Ρ Π²ΡΠ·ΡΠ²Π°Π΅Ρ ΠΏΠ΅ΡΠ΅ΠΌΠ΅ΡΠ΅Π½ΠΈΠ΅ Π΅Π΄ΠΈΠ½ΠΈΡΠ½ΠΎΠ³ΠΎ Π·Π°ΡΡΠ΄Π° ΠΏΠΎ ΡΠ΅ΠΏΠΈ. Π‘Π»Π΅Π΄ΠΎΠ²Π°ΡΠ΅Π»ΡΠ½ΠΎ: | Ξ΅ΠΈΠ½Π΄ | = Az / q .
ΠΠ°ΠΊ Π²ΠΈΠ΄ΠΈΡΠ΅, Π² ΠΎΠΏΡΠ΅Π΄Π΅Π»Π΅Π½ΠΈΠΈ ΠΌΡ ΠΈΡΠΏΠΎΠ»ΡΠ·ΠΎΠ²Π°Π»ΠΈ Π°Π±ΡΠΎΠ»ΡΡΠ½ΠΎΠ΅ Π·Π½Π°ΡΠ΅Π½ΠΈΠ΅ ΠΠΠ‘ ΠΈΠ½Π΄ΡΠΊΡΠΈΠΈ. ΠΡΠΎ ΠΏΠΎΡΠΎΠΌΡ, ΡΡΠΎ ΠΎΠ½ΠΎ ΠΌΠΎΠΆΠ΅Ρ Π±ΡΡΡ ΠΎΡΡΠΈΡΠ°ΡΠ΅Π»ΡΠ½ΡΠΌ, ΠΏΡΠΈ ΠΎΠΏΡΠ΅Π΄Π΅Π»Π΅Π½Π½ΡΡ ΡΠΈΡΡΠ°ΡΠΈΡΡ . Π‘ Π΄ΡΡΠ³ΠΎΠΉ ΡΡΠΎΡΠΎΠ½Ρ, ΡΠ°Π±ΠΎΡΠ° Π²Π½Π΅ΡΠ½ΠΈΡ ΡΠΈΠ», ΡΠΎΠ³Π»Π°ΡΠ½ΠΎ ΠΏΡΠΈΠ½ΡΠΈΠΏΡ ΡΠΎΡ ΡΠ°Π½Π΅Π½ΠΈΡ ΡΠ½Π΅ΡΠ³ΠΈΠΈ, Π²ΡΠ΅Π³Π΄Π°, ΠΏΡΠΈ Π³Π΅Π½Π΅ΡΠ°ΡΠΈΠΈ ΡΠ»Π΅ΠΊΡΡΠΈΡΠ΅ΡΠΊΠΎΠ³ΠΎ ΡΠΎΠΊΠ°, Π΄ΠΎΠ»ΠΆΠ½Π° Π±ΡΡΡ ΠΏΠΎΠ»ΠΎΠΆΠΈΡΠ΅Π»ΡΠ½ΠΎΠΉ.
ΠΠΏΡΠ΅Π΄Π΅Π»Π΅Π½ΠΈΠ΅ ΠΏΠΎΡΠΎΠΊΠ° ΠΌΠ°Π³Π½ΠΈΡΠ½ΠΎΠΉ ΠΈΠ½Π΄ΡΠΊΡΠΈΠΈ.
ΠΠΎΡΠΎΠΊ ΠΌΠ°Π³Π½ΠΈΡΠ½ΠΎΠΉ ΠΈΠ½Π΄ΡΠΊΡΠΈΠΈ B ΡΠ΅ΡΠ΅Π· ΠΏΠΎΠ²Π΅ΡΡ Π½ΠΎΡΡΡ S Π½Π°Π·ΡΠ²Π°Π΅ΡΡΡ ΡΠΊΠ°Π»ΡΡΠ½ΡΠΌ ΠΏΡΠΎΠΈΠ·Π²Π΅Π΄Π΅Π½ΠΈΠ΅ΠΌ Π²Π΅ΠΊΡΠΎΡΠΎΠ² B ΠΈ S: dΠ€ = B * S * cos Ξ± , Π³Π΄Π΅ Ξ± β ΡΠ³ΠΎΠ» ΠΌΠ΅ΠΆΠ΄Ρ Π΄Π²ΡΠΌΡ Π²Π΅ΠΊΡΠΎΡΠ°ΠΌΠΈ, Π° S β Π²Π΅ΠΊΡΠΎΡ, ΠΏΠ΅ΡΠΏΠ΅Π½Π΄ΠΈΠΊΡΠ»ΡΡΠ½ΡΠΉ ΠΏΠΎΠ²Π΅ΡΡ Π½ΠΎΡΡΠΈ S Ρ Π²Π΅Π»ΠΈΡΠΈΠ½ΠΎΠΉ, ΡΠ°Π²Π½ΠΎΠΉ ΠΏΠ»ΠΎΡΠ°Π΄ΠΈ ΡΡΠΎΠΉ ΠΏΠΎΠ²Π΅ΡΡ Π½ΠΎΡΡΠΈ.
ΠΠ°Π³Π½ΠΈΡΠ½ΡΠΉ ΠΏΠΎΡΠΎΠΊ Π±ΡΠ΄Π΅Ρ ΠΌΠ΅Π½ΡΡΡΡΡ ΠΏΡΠΈ ΠΈΠ·ΠΌΠ΅Π½Π΅Π½ΠΈΠΈ Π»ΡΠ±ΠΎΠΉ Π²Π΅Π»ΠΈΡΠΈΠ½Ρ, Π²Ρ ΠΎΠ΄ΡΡΠ΅ΠΉ Π² ΡΠΎΡΠΌΡΠ»Ρ β ΠΏΠ»ΠΎΡΠ°Π΄ΠΈ ΠΏΠΎΠ²Π΅ΡΡ Π½ΠΎΡΡΠΈ, Π·Π½Π°ΡΠ΅Π½ΠΈΡ ΠΌΠ°Π³Π½ΠΈΡΠ½ΠΎΠΉ ΠΈΠ½Π΄ΡΠΊΡΠΈΠΈ, ΡΠ³Π»Π° ΠΌΠ΅ΠΆΠ΄Ρ ΠΏΠ»ΠΎΡΠ°Π΄ΡΡ ΠΏΠΎΠ²Π΅ΡΡ Π½ΠΎΡΡΠΈ ΠΈ Π²Π΅ΠΊΡΠΎΡΠΎΠΌ ΠΈΠ½Π΄ΡΠΊΡΠΈΠΈ β ΠΏΡΠΈ ΡΠΎΡ ΡΠ°Π½Π΅Π½ΠΈΠΈ ΠΏΠΎΡΡΠΎΡΠ½ΡΡΠ²Π° ΠΎΡΡΠ°Π»ΡΠ½ΡΡ ΠΏΠ΅ΡΠ΅ΠΌΠ΅Π½Π½ΡΡ . ΠΠΎΠ½Π΅ΡΠ½ΠΎ, Π²ΡΠ΅ ΡΡΠΈ Π²Π΅Π»ΠΈΡΠΈΠ½Ρ ΠΌΠΎΠ³ΡΡ ΠΈΠ·ΠΌΠ΅Π½ΡΡΡΡΡ ΠΎΠ΄Π½ΠΎΠ²ΡΠ΅ΠΌΠ΅Π½Π½ΠΎ, Π½ΠΎ ΡΠ°ΠΊΠΈΠΌ ΠΎΠ±ΡΠ°Π·ΠΎΠΌ, ΡΡΠΎ ΠΈΡ ΠΏΡΠΎΠΈΠ·Π²Π΅Π΄Π΅Π½ΠΈΠ΅ Π½Π΅ ΠΎΡΡΠ°Π΅ΡΡΡ ΠΏΠΎΡΡΠΎΡΠ½Π½ΡΠΌ.
Π ΡΠΎΠΌ, ΡΡΠΎ ΡΠ»Π΅ΠΊΡΡΠΈΡΠ΅ΡΠΊΠΈΠΉ ΡΠΎΠΊ ΡΠ²Π»ΡΠ΅ΡΡΡ ΠΈΡΡΠΎΡΠ½ΠΈΠΊΠΎΠΌ ΠΌΠ°Π³Π½ΠΈΡΠ½ΠΎΠ³ΠΎ ΠΏΠΎΠ»Ρ, Π±ΡΠ»ΠΎ ΠΈΠ·Π²Π΅ΡΡΠ½ΠΎ Ρ 1820 Π³ΠΎΠ΄Π° (ΡΠ°Π±ΠΎΡΠ° ΠΡΡΡΠ΅Π΄Π°). Π€Π°ΡΠ°Π΄Π΅ΠΉ Π·Π°Π΄Π°Π»ΡΡ Π²ΠΎΠΏΡΠΎΡΠΎΠΌ, Π½Π΅ Π²Π΅ΡΠ½ΠΎ Π»ΠΈ ΠΈ ΠΎΠ±ΡΠ°ΡΠ½ΠΎΠ΅ β Π½Π΅ ΠΌΠΎΠΆΠ΅Ρ Π»ΠΈ ΠΌΠ°Π³Π½ΠΈΡΠ½ΠΎΠ΅ ΠΏΠΎΠ»Π΅ Π±ΡΡΡ ΠΈΡΡΠΎΡΠ½ΠΈΠΊΠΎΠΌ (ΠΏΡΠΈΡΠΈΠ½ΠΎΠΉ) ΡΠ»Π΅ΠΊΡΡΠΈΡΠ΅ΡΠΊΠΎΠ³ΠΎ ΡΠΎΠΊΠ°. ΠΠ΄Π½Π°ΠΊΠΎ Π΄Π΅Π»ΠΎ ΠΎΠΊΠ°Π·Π°Π»ΠΎΡΡ Π½Π΅ ΡΠ°ΠΊΠΈΠΌ ΠΏΡΠΎΡΡΡΠΌ. Π’ΠΎΠ»ΡΠΊΠΎ Π² 1831 Π³ΠΎΠ΄Ρ ΡΡΠ΅Π½ΡΠΉ Π½Π°Π±Π»ΡΠ΄Π°Π» ΡΡΠΎ ΡΠ²Π»Π΅Π½ΠΈΠ΅ ΠΏΡΠΈ ΠΎΠΏΡΠ΅Π΄Π΅Π»Π΅Π½Π½ΡΡ ΠΎΡΠΎΠ±ΡΡ ΠΎΠ±ΡΡΠΎΡΡΠ΅Π»ΡΡΡΠ²Π°Ρ . ΠΠΊΠ°Π·Π°Π»ΠΎΡΡ, ΡΡΠΎ ΠΏΡΠΈ ΡΡΠ°Π±ΠΈΠ»ΡΠ½ΡΡ ΡΡΠ»ΠΎΠ²ΠΈΡΡ ΡΠ»Π΅ΠΊΡΡΠΈΡΠ΅ΡΠΊΠΈΠΉ ΡΠΎΠΊ Π½Π΅ Π²ΠΎΠ·Π½ΠΈΠΊΠ°Π΅Ρ.
ΠΠΎΡΠ΅ΠΌΡ ΡΡΠΎ ΠΏΡΠΎΠΈΡΡ ΠΎΠ΄ΠΈΡ? ΠΠ°ΠΆΠ΅ Π² ΠΎΡΠ΅Π½Ρ ΡΠΈΠ»ΡΠ½ΠΎΠΌ, Π½ΠΎ ΠΏΠΎΡΡΠΎΡΠ½Π½ΠΎΠΌ Π²ΠΎ Π²ΡΠ΅ΠΌΠ΅Π½ΠΈ ΠΌΠ°Π³Π½ΠΈΡΠ½ΠΎΠΌ ΠΏΠΎΠ»Π΅ ΡΠ»Π΅ΠΊΡΡΠΈΡΠ΅ΡΠΊΠΈΠΉ ΡΠΎΠΊ Π½Π΅ Π±ΡΠ΄Π΅Ρ ΡΠ΅ΡΡ Π² Π·Π°ΠΌΠΊΠ½ΡΡΠΎΠΉ ΡΠ΅ΠΏΠΈ βΡΠ°ΠΌ ΠΏΠΎ ΡΠ΅Π±Π΅β. ΠΠ½ ΡΠ΅ΡΠ΅Ρ ΡΠΎΠ»ΡΠΊΠΎ ΡΠΎΠ³Π΄Π°, ΠΊΠΎΠ³Π΄Π° ΠΌΡ ΡΠΎΠΎΡΠ²Π΅ΡΡΡΠ²ΡΡΡΠΈΠΌ ΠΎΠ±ΡΠ°Π·ΠΎΠΌ ΠΏΠ΅ΡΠ΅ΠΌΠ΅ΡΠ°Π΅ΠΌ ΠΊΠΎΠ½ΡΡΡ ΠΈΠ»ΠΈ ΠΈΠ·ΠΌΠ΅Π½ΡΠ΅ΠΌ ΠΌΠ°Π³Π½ΠΈΡΠ½ΠΎΠ΅ ΠΏΠΎΠ»Π΅, Π² ΠΊΠΎΡΠΎΡΠΎΠΌ Π½Π°Ρ ΠΎΠ΄ΠΈΡΡΡ ΠΊΠΎΠ½ΡΡΡ.
ΠΠΎΠ³Π΄Π° Π€Π°ΡΠ°Π΄Π΅ΠΉ ΠΎΠ±ΡΠ°ΡΠΈΠ» Π²Π½ΠΈΠΌΠ°Π½ΠΈΠ΅ Π½Π° ΡΡΠ»ΠΎΠ²ΠΈΡ, ΠΏΡΠΈ ΠΊΠΎΡΠΎΡΡΡ Π² ΠΏΡΠΈΡΡΡΡΡΠ²ΠΈΠΈ ΠΌΠ°Π³Π½ΠΈΡΠ½ΠΎΠ³ΠΎ ΠΏΠΎΠ»Ρ Π²ΠΎΠ·Π½ΠΈΠΊΠ°Π΅Ρ ΡΠ»Π΅ΠΊΡΡΠΈΡΠ΅ΡΠΊΠΈΠΉ ΡΠΎΠΊ, ΠΎΠ½ ΠΏΡΠΎΠ²Π΅Π» Π΄Π΅ΡΡΡΠΊΠΈ ΡΠΊΡΠΏΠ΅ΡΠΈΠΌΠ΅Π½ΡΠΎΠ², ΠΊΠΎΡΠΎΡΡΠ΅ ΠΎΠ±ΠΎΠ±ΡΠΈΠ» ΠΈ ΠΈΠ· ΠΊΠΎΡΠΎΡΡΡ ΡΠ΄Π΅Π»Π°Π» ΠΊΠΎΠ»ΠΈΡΠ΅ΡΡΠ²Π΅Π½Π½ΡΠ΅ Π²ΡΠ²ΠΎΠ΄Ρ Π² Π²ΠΈΠ΄Π΅ Π·Π°ΠΊΠΎΠ½Π° ΡΠ»Π΅ΠΊΡΡΠΎΠΌΠ°Π³Π½ΠΈΡΠ½ΠΎΠΉ ΠΈΠ½Π΄ΡΠΊΡΠΈΠΈ. ΠΡ Π½Π΅ Π±ΡΠ΄Π΅ΠΌ Π·Π΄Π΅ΡΡ Π³ΠΎΠ²ΠΎΡΠΈΡΡ ΠΎΠ± ΡΡΠΎΠΌ Π·Π°ΠΊΠΎΠ½Π΅, Π° ΡΠΎΡΡΠ΅Π΄ΠΎΡΠΎΡΠΈΠΌΡΡ ΡΠΎΠ»ΡΠΊΠΎ Π½Π° ΡΡΡΠΈ ΡΠ²Π»Π΅Π½ΠΈΡ ΡΠ»Π΅ΠΊΡΡΠΎΠΌΠ°Π³Π½ΠΈΡΠ½ΠΎΠΉ ΠΈΠ½Π΄ΡΠΊΡΠΈΠΈ. ΠΡ ΠΏΠΎΠΏΡΡΠ°Π΅ΠΌΡΡ ΡΠ²ΠΈΠ΄Π΅ΡΡ Π΄Π²ΠΎΠΉΡΡΠ²Π΅Π½Π½ΠΎΡΡΡ ΡΡΠΎΠ³ΠΎ ΡΠ²Π»Π΅Π½ΠΈΡ, Ρ.Π΅. ΡΠΎ, ΡΡΠΎ ΠΎΠ½ΠΎ ΠΈΠΌΠ΅Π΅Ρ Π΄Π²Π΅ ΡΠ°Π·Π½ΠΎΠ²ΠΈΠ΄Π½ΠΎΡΡΠΈ, ΠΈ ΠΎΡΠ²Π΅ΡΠΈΡΡ Π½Π° Π²ΠΎΠΏΡΠΎΡ, ΠΏΠΎΡΠ΅ΠΌΡ ΡΠ»Π΅ΠΊΡΡΠΈΡΠ΅ΡΠΊΠΈΠΉ ΡΠΎΠΊ ΡΠ΅ΡΠ΅Ρ ΠΏΡΠΈ ΠΎΠΏΡΠ΅Π΄Π΅Π»Π΅Π½Π½ΡΡ ΡΡΠ»ΠΎΠ²ΠΈΡΡ .
ΠΡ ΡΠ°ΡΡΠΌΠΎΡΡΠΈΠΌ, ΠΊΠ°ΠΊΠΈΠ΅ ΡΠΈΠ»Ρ Π²ΡΠ·ΡΠ²Π°ΡΡ ΠΈΠ½Π΄ΡΠΊΡΠΈΠΎΠ½Π½ΡΠΉ ΡΠΎΠΊ, Ρ.Π΅. ΠΊΠ°ΠΊΠΈΠ΅ ΡΠΈΠ»Ρ Π΄Π΅ΠΉΡΡΠ²ΡΡΡ Π½Π° ΡΠ²ΠΎΠ±ΠΎΠ΄Π½ΡΠ΅ Π·Π°ΡΡΠ΄Ρ Π² ΠΏΡΠΎΠ²ΠΎΠ΄Π½ΠΈΠΊΠ΅, Π·Π°ΡΡΠ°Π²Π»ΡΡ ΠΈΡ Π΄Π²ΠΈΠ³Π°ΡΡΡΡ.
ΠΠΊΡΠΏΠ΅ΡΠΈΠΌΠ΅Π½Ρ Π€Π°ΡΠ°Π΄Π΅Ρ 1831 Π³ΠΎΠ΄Π°, Π΄Π΅ΠΌΠΎΠ½ΡΡΡΠΈΡΡΡΡΠΈΠΉ ΡΠ»Π΅ΠΊΡΡΠΎΠΌΠ°Π³Π½ΠΈΡΠ½ΡΡ ΠΈΠ½Π΄ΡΠΊΡΠΈΡ ΠΌΠ΅ΠΆΠ΄Ρ Π΄Π²ΡΠΌΡ ΠΊΠ°ΡΡΡΠΊΠ°ΠΌΠΈ (ΡΠΌ. ΡΠΈΡΡΠ½ΠΎΠΊ 1).
Π‘ΠΏΡΠ°Π²Π° Π½Π°Ρ ΠΎΠ΄ΠΈΡΡΡ Π°ΠΊΠΊΡΠΌΡΠ»ΡΡΠΎΡ, ΠΏΠΈΡΠ°ΡΡΠΈΠΉ ΠΌΠ΅Π½ΡΡΡΡ ΠΈΠ· Π΄Π²ΡΡ ΠΊΠ°ΡΡΡΠ΅ΠΊ (A), ΠΊΠΎΡΠΎΡΠ°Ρ ΡΠΎΠ·Π΄Π°Π΅Ρ ΠΌΠ°Π³Π½ΠΈΡΠ½ΠΎΠ΅ ΠΏΠΎΠ»Π΅. ΠΠΎΠ³Π΄Π° ΡΡΠ° ΠΊΠ°ΡΡΡΠΊΠ° Π½Π°Ρ ΠΎΠ΄ΠΈΡΡΡ Π² ΡΠΎΡΡΠΎΡΠ½ΠΈΠΈ ΠΏΠΎΠΊΠΎΡ, ΠΈΠ½Π΄ΡΠΊΡΠΈΠΎΠ½Π½ΡΠΉ ΡΠΎΠΊ Π½Π΅ Π½Π°Π±Π»ΡΠ΄Π°Π΅ΡΡΡ. ΠΠ΄Π½Π°ΠΊΠΎ Π΅ΡΠ»ΠΈ ΠΏΠ΅ΡΠ΅ΠΌΠ΅ΡΡΠΈΡΡ Π΅Π³ΠΎ Π²Π½ΡΡΡΡ Π±ΠΎΠ»ΡΡΠ΅ΠΉ ΠΊΠ°ΡΡΡΠΊΠΈ (B), ΠΏΠ΅ΡΠ΅ΠΌΠ΅Π½Π½ΡΠΉ ΠΌΠ°Π³Π½ΠΈΡΠ½ΡΠΉ ΠΏΠΎΡΠΎΠΊ ΠΈΠ½Π΄ΡΡΠΈΡΡΠ΅Ρ Π² Π½Π΅ΠΉ ΡΠΎΠΊ. ΠΡ ΠΎΠ±Π½Π°ΡΡΠΆΠΈΠ²Π°Π΅ΠΌ ΡΡΠΎ, Π½Π°Π±Π»ΡΠ΄Π°Ρ Π·Π° ΠΊΠΎΠ»Π΅Π±Π°Π½ΠΈΡΠΌΠΈ ΡΡΡΠ΅Π»ΠΊΠΈ Π³Π°Π»ΡΠ²Π°Π½ΠΎΠΌΠ΅ΡΡΠ° (G) ΡΠ»Π΅Π²Π°.
Π ΠΈΡ. 1. ΠΠΊΡΠΏΠ΅ΡΠΈΠΌΠ΅Π½Ρ Π€Π°ΡΠ°Π΄Π΅Ρ 1831 Π³ΠΎΠ΄Π°, Π΄Π΅ΠΌΠΎΠ½ΡΡΡΠΈΡΡΡΡΠΈΠΉ ΡΠ»Π΅ΠΊΡΡΠΎΠΌΠ°Π³Π½ΠΈΡΠ½ΡΡ ΠΈΠ½Π΄ΡΠΊΡΠΈΡ ΠΌΠ΅ΠΆΠ΄Ρ Π΄Π²ΡΠΌΡ ΠΊΠ°ΡΡΡΠΊΠ°ΠΌΠΈ (ΡΠΌ. ΡΠΈΡΡΠ½ΠΎΠΊ 1). ΠΡΡΠΎΡΠ½ΠΈΠΊ: J. Lambert [Public domain], Wikimedia Commons)ΠΠ°ΠΊΠΎΠ½ ΡΠ»Π΅ΠΊΡΡΠΎΠΌΠ°Π³Π½ΠΈΡΠ½ΠΎΠΉ ΠΈΠ½Π΄ΡΠΊΡΠΈΠΈ Π€Π°ΡΠ°Π΄Π΅Ρ
Π―Π²Π»Π΅Π½ΠΈΠ΅ ΡΠ»Π΅ΠΊΡΡΠΎΠΌΠ°Π³Π½ΠΈΡΠ½ΠΎΠΉ ΠΈΠ½Π΄ΡΠΊΡΠΈΠΈ ΠΎΠΏΠΈΡΡΠ²Π°Π΅ΡΡΡ Π·Π°ΠΊΠΎΠ½ΠΎΠΌ Π€Π°ΡΠ°Π΄Π΅Ρ, ΠΏΠ΅ΡΠ²ΠΎΠΎΡΠΊΡΡΠ²Π°ΡΠ΅Π»Ρ ΠΈ ΠΈΡΡΠ»Π΅Π΄ΠΎΠ²Π°ΡΠ΅Π»Ρ ΡΡΠΎΠ³ΠΎ ΡΠ²Π»Π΅Π½ΠΈΡ.
ΠΡΠ΅Π΄ΡΡΠ°Π²ΡΡΠ΅ ΡΠ΅Π±Π΅ ΠΏΡΠΎΡΡΠ΅ΠΉΡΠΈΠΉ ΠΊΠΎΠ½ΡΡΡ Ρ ΠΏΠΎΠ΄Π²ΠΈΠΆΠ½ΠΎΠΉ ΡΡΠΎΡΠΎΠ½ΠΎΠΉ, ΠΏΠΎΠΌΠ΅ΡΠ΅Π½Π½ΡΠΉ Π² ΠΌΠ°Π³Π½ΠΈΡΠ½ΠΎΠ΅ ΠΏΠΎΠ»Π΅ ΡΠ°ΠΊ, ΡΡΠΎΠ±Ρ ΠΏΠΎΠ²Π΅ΡΡ Π½ΠΎΡΡΡ ΠΊΠΎΠ½ΡΡΡΠ° Π±ΡΠ»Π° ΠΏΠ΅ΡΠΏΠ΅Π½Π΄ΠΈΠΊΡΠ»ΡΡΠ½Π° Π»ΠΈΠ½ΠΈΡΠΌ ΠΌΠ°Π³Π½ΠΈΡΠ½ΠΎΠ³ΠΎ ΠΏΠΎΠ»Ρ (ΡΠΈΡ. 2.).
Π ΠΈΡ. 2. ΠΠΎΠ½ΡΡΡ Ρ ΠΏΠΎΠ΄Π²ΠΈΠΆΠ½ΠΎΠΉ ΡΡΠΎΡΠΎΠ½ΠΎΠΉ (ΠΏΠ΅ΡΠ΅ΠΊΠ»Π°Π΄ΠΈΠ½ΠΎΠΉ)ΠΡ ΠΏΠ΅ΡΠ΅ΠΌΠ΅ΡΠ°Π΅ΠΌ ΠΊΠΎΠ½ΡΡΡ ΡΠΎ ΡΠΊΠΎΡΠΎΡΡΡΡ v Π²ΠΏΡΠ°Π²ΠΎ. ΠΡΠΎ ΠΈΠ·ΠΌΠ΅Π½ΡΠ΅Ρ ΠΏΠΎΡΠΎΠΊ ΠΌΠ°Π³Π½ΠΈΡΠ½ΠΎΠΉ ΠΈΠ½Π΄ΡΠΊΡΠΈΠΈ, ΠΏΡΠΎΠ½ΠΈΠ·ΡΠ²Π°ΡΡΠΈΠΉ ΠΏΠΎΠ²Π΅ΡΡ Π½ΠΎΡΡΡ, ΠΎΡ Π²Π°ΡΡΠ²Π°Π΅ΠΌΡΡ ΠΊΠΎΠ½ΡΡΡΠΎΠΌ, ΠΎΠ±ΠΎΠ·Π½Π°ΡΠ΅Π½Π½ΡΠΌ Π½Π° ΡΠΈΡΡΠ½ΠΊΠ΅ Π±ΠΎΠ»Π΅Π΅ ΡΠ΅ΠΌΠ½ΡΠΌ ΡΠ²Π΅ΡΠΎΠΌ.
ΠΡΠΏΠΎΠΌΠΈΠ½Π°Ρ ΠΎΠΏΡΠ΅Π΄Π΅Π»Π΅Π½ΠΈΠ΅ ΠΌΠ°Π³Π½ΠΈΡΠ½ΠΎΠ³ΠΎ ΠΏΠΎΡΠΎΠΊΠ° ΠΈΠ½Π΄ΡΠΊΡΠΈΠΈ, ΠΌΡ ΠΌΠΎΠΆΠ΅ΠΌ ΠΏΠΎΠ½ΡΡΡ, ΠΏΠΎΡΠ΅ΠΌΡ ΠΈΠ·ΠΌΠ΅Π½ΡΠ΅ΡΡΡ ΠΏΠΎΡΠΎΠΊ Π€B (ΡΠΈΡ. 2) β ΠΏΠΎΡΠΎΠΌΡ ΡΡΠΎ, Π·Π½Π°ΡΠ΅Π½ΠΈΠ΅ ΠΏΠ»ΠΎΡΠ°Π΄ΠΈ S ΠΏΠΎΠ²Π΅ΡΡ Π½ΠΎΡΡΠΈ ΡΠ²Π΅Π»ΠΈΡΠΈΠ²Π°Π΅ΡΡΡ .
ΠΡΠ»Π΅Π΄ΡΡΠ²ΠΈΠ΅ ΠΈΠ·ΠΌΠ΅Π½Π΅Π½ΠΈΡ ΠΏΠΎΡΠΎΠΊΠ° ΠΌΠ°Π³Π½ΠΈΡΠ½ΠΎΠΉ ΠΈΠ½Π΄ΡΠΊΡΠΈΠΈ Π² ΡΠ°ΡΡΠΌΠ°ΡΡΠΈΠ²Π°Π΅ΠΌΠΎΠΉ ΡΠ΅ΠΏΠΈ Π²ΠΎΠ·Π½ΠΈΠΊΠ½Π΅Ρ ΡΠ»Π΅ΠΊΡΡΠΎΠ΄Π²ΠΈΠΆΡΡΠ°Ρ ΡΠΈΠ»Π° ΠΈΠ½Π΄ΡΠΊΡΠΈΠΈ ΠΈ, ΡΠ»Π΅Π΄ΠΎΠ²Π°ΡΠ΅Π»ΡΠ½ΠΎ, ΠΏΠΎΡΠ΅ΡΠ΅Ρ ΡΠ»Π΅ΠΊΡΡΠΈΡΠ΅ΡΠΊΠΈΠΉ ΡΠΎΠΊ.
Π ΠΈΡ. 3. ΠΠ½Π΅ΡΠ½ΡΡ ΡΠΈΠ»Π° Fz ΡΡΠ°Π²Π½ΠΎΠ²Π΅ΡΠΈΠ²Π°Π΅Ρ ΡΠ»Π΅ΠΊΡΡΠΎΠ΄ΠΈΠ½Π°ΠΌΠΈΡΠ΅ΡΠΊΡΡ ΡΠΈΠ»Ρ Fed , Π΄Π΅ΠΉΡΡΠ²ΡΡΡΡΡ Π½Π° ΠΊΠΎΠ½ΡΡΡ, Π΄Π²ΠΈΠΆΡΡΠΈΠΉΡΡ Ρ ΠΏΠΎΡΡΠΎΡΠ½Π½ΠΎΠΉ ΡΠΊΠΎΡΠΎΡΡΡΡ vΠ ΡΠ°ΡΡΠΌΠ°ΡΡΠΈΠ²Π°Π΅ΠΌΠΎΠΌ Π½Π°ΠΌΠΈ ΡΠ»ΡΡΠ°Π΅ Π»Π΅Π³ΠΊΠΎ Π²ΡΡΠΈΡΠ»ΠΈΡΡ ΡΠ°Π±ΠΎΡΡ Π²Π½Π΅ΡΠ½Π΅ΠΉ ΡΠΈΠ»Ρ, ΠΏΡΠ΅Π΄ΠΏΠΎΠ»Π°Π³Π°Ρ ΠΏΠΎΡΡΠΎΡΠ½Π½ΡΡ ΡΠΊΠΎΡΠΎΡΡΡ Π΄Π²ΠΈΠΆΠ΅Π½ΠΈΡ ΠΊΠΎΠ½ΡΡΡΠ°. ΠΠ½Π΅ΡΠ½ΡΡ ΡΠΈΠ»Π° Fz Π΄Π΅ΠΉΡΡΠ²ΡΠ΅Ρ Π² ΡΠΎΠΎΡΠ²Π΅ΡΡΡΠ²ΠΈΠΈ ΡΠΎ ΡΠΌΠ΅ΡΠ΅Π½ΠΈΠ΅ΠΌ ΠΊΠΎΠ½ΡΡΡΠ° (ΠΈ Π²Π΅ΠΊΡΠΎΡΠΎΠΌ ΡΠΊΠΎΡΠΎΡΡΠΈ) ΠΈ Π² Π»ΡΠ±ΠΎΠΉ ΠΌΠΎΠΌΠ΅Π½Ρ ΡΡΠ°Π²Π½ΠΎΠ²Π΅ΡΠΈΠ²Π°Π΅Ρ ΡΠ»Π΅ΠΊΡΡΠΎΠ΄ΠΈΠ½Π°ΠΌΠΈΡΠ΅ΡΠΊΡΡ ΡΠΈΠ»Ρ (ΡΠΈΠ»Ρ ΠΠΌΠΏΠ΅ΡΠ°) Fed, Π΄Π΅ΠΉΡΡΠ²ΡΡΡΡΡ Π² ΠΏΡΠΎΡΠΈΠ²ΠΎΠΏΠΎΠ»ΠΎΠΆΠ½ΠΎΠΌ Π½Π°ΠΏΡΠ°Π²Π»Π΅Π½ΠΈΠΈ (ΡΠΈΡ. 3.). Π‘ΠΎΠ³Π»Π°ΡΠ½ΠΎ ΠΎΠΏΡΠ΅Π΄Π΅Π»Π΅Π½ΠΈΡ ΡΠ°Π±ΠΎΡΡ Az = F * Ξx Π³Π΄Π΅ Ξx β ΡΠΌΠ΅ΡΠ΅Π½ΠΈΠ΅ ΠΊΠΎΠ½ΡΡΡΠ° Π²ΠΎ Π²ΡΠ΅ΠΌΠ΅Π½ΠΈ Ξt.
ΠΠ΅Π»ΠΈΡΠΈΠ½Π° ΡΠΈΠ»Ρ Fz ΡΠ°Π²Π½Π° Π²Π΅Π»ΠΈΡΠΈΠ½Π΅ ΡΠ»Π΅ΠΊΡΡΠΎΠ΄ΠΈΠ½Π°ΠΌΠΈΡΠ΅ΡΠΊΠΎΠΉ ΡΠΈΠ»Ρ (ΡΠΈΠ»Π΅ ΠΠΌΠΏΠ΅ΡΠ°) Fed, Π΄Π΅ΠΉΡΡΠ²ΡΡΡΠ΅ΠΉ Π½Π° ΠΊΠΎΠ½ΡΡΡ. ΠΠΎΡΡΠΎΠΌΡ Az = I * L * B * Ξx, Π³Π΄Π΅ β I ΡΠΈΠ»Π° ΠΈΠ½Π΄ΡΠΊΡΠΈΠΎΠ½Π½ΠΎΠ³ΠΎ ΡΠΎΠΊΠ°, ΠΏΡΠΎΡΠ΅ΠΊΠ°ΡΡΠ΅Π³ΠΎ Π² ΡΠ΅ΠΏΠΈ (ΠΈ Π² ΠΊΠΎΠ½ΡΡΡΠ΅), L β Π΄Π»ΠΈΠ½Π° ΠΊΠΎΠ½ΡΡΡΠ° (ΡΠΎΠΉ ΡΠ°ΡΡΠΈ, Π³Π΄Π΅ ΠΏΡΠΎΡΠ΅ΠΊΠ°Π΅Ρ ΡΠ»Π΅ΠΊΡΡΠΈΡΠ΅ΡΠΊΠΈΠΉ ΡΠΎΠΊ), B β Π²Π΅Π»ΠΈΡΠΈΠ½Π° ΠΌΠ°Π³Π½ΠΈΡΠ½ΠΎΠΉ ΠΈΠ½Π΄ΡΠΊΡΠΈΠΈ. ΠΠ°Π²Π°ΠΉΡΠ΅ Π²Π²Π΅Π΄Π΅ΠΌ Π½Π°ΡΠ΅ Π²ΡΡΠ°ΠΆΠ΅Π½ΠΈΠ΅ Π² ΠΎΠΏΡΠ΅Π΄Π΅Π»Π΅Π½ΠΈΠ΅ ΠΠΠ‘ ΠΈΠ½Π΄ΡΠΊΡΠΈΠΈ. ΠΠ½Π°Ρ, ΡΡΠΎ q = I * Ξt, ΠΏΠΎΠ»ΡΡΠ°Π΅ΠΌ:
| Ξ΅ΠΈΠ½Π΄ | = Az / q = I * L * B * Ξx / I * Ξt = B * L * Ξx / Ξt = B * ΞS / Ξt = dΠ€B / dt.
ΠΡ ΠΏΠΎΠ»ΡΡΠΈΠ»ΠΈ ΠΈΠ½ΡΠ΅ΡΠ΅ΡΠ½ΡΠΉ ΡΠ΅Π·ΡΠ»ΡΡΠ°Ρ. ΠΠ±ΡΠΎΠ»ΡΡΠ½ΠΎΠ΅ Π·Π½Π°ΡΠ΅Π½ΠΈΠ΅ ΠΠΠ‘ ΠΈΠ½Π΄ΡΠΊΡΠΈΠΈ ΡΠ°Π²Π½ΠΎ ΡΠΊΠΎΡΠΎΡΡΠΈ ΠΈΠ·ΠΌΠ΅Π½Π΅Π½ΠΈΡ ΠΏΠΎΡΠΎΠΊΠ° ΠΌΠ°Π³Π½ΠΈΡΠ½ΠΎΠΉ ΠΈΠ½Π΄ΡΠΊΡΠΈΠΈ.
Π ΡΠ°ΡΡΠΌΠ°ΡΡΠΈΠ²Π°Π΅ΠΌΠΎΠΌ Π·Π΄Π΅ΡΡ ΡΠ»ΡΡΠ°Π΅ ΠΏΠΎΡΠΎΠΊ ΠΌΠ°Π³Π½ΠΈΡΠ½ΠΎΠΉ ΠΈΠ½Π΄ΡΠΊΡΠΈΠΈ ΠΈΠ·ΠΌΠ΅Π½ΡΠ΅ΡΡΡ ΡΠ°Π²Π½ΠΎΠΌΠ΅ΡΠ½ΠΎ Π²ΠΎ Π²ΡΠ΅ΠΌΠ΅Π½ΠΈ. Π ΠΎΠ±ΡΠ΅ΠΌ ΡΠ»ΡΡΠ°Π΅ ΡΡΠΎ ΡΠΎΠ²ΡΠ΅ΠΌ Π½Π΅ ΠΎΠ±ΡΠ·Π°ΡΠ΅Π»ΡΠ½ΠΎ. ΠΠΎΡ ΠΏΠΎΡΠ΅ΠΌΡ ΠΌΡ ΠΏΠΈΡΠ΅ΠΌ: Ξ΅ΠΈΠ½Π΄ = ΞΠ€B / Ξt , Π³Π΄Π΅ Ξt β 0, ΠΊΠΎΡΠΎΡΡΠΉ Π² ΡΠΎΠΊΡΠ°ΡΠ΅Π½Π½ΠΎΠΌ Π²ΠΈΠ΄Π΅ Π·Π°ΠΏΠΈΡΡΠ²Π°Π΅ΡΡΡ ΠΊΠ°ΠΊ dΠ€B / dt . ΠΡΠΎ ΠΏΡΠΎΠΈΠ·Π²ΠΎΠ΄Π½Π°Ρ ΠΌΠ°Π³Π½ΠΈΡΠ½ΠΎΠ³ΠΎ ΠΏΠΎΡΠΎΠΊΠ° ΠΏΠΎ Π²ΡΠ΅ΠΌΠ΅Π½ΠΈ.
Π₯ΠΎΡΡ Π½Π°Ρ Π²ΡΠ²ΠΎΠ΄ ΡΠΎΡΠΌΡΠ»Ρ ΠΎΡΠ½ΠΎΡΠΈΡΡΡ ΠΊ ΠΎΠ΄Π½ΠΎΠΌΡ ΠΏΡΠΈΠΌΠ΅ΡΡ, ΠΎΠΊΠ°Π·ΡΠ²Π°Π΅ΡΡΡ, ΡΡΠΎ Π²ΡΠ²Π΅Π΄Π΅Π½Π½ΠΎΠ΅ ΠΎΡΠ½ΠΎΡΠ΅Π½ΠΈΠ΅ ΡΠ²Π»ΡΠ΅ΡΡΡ ΠΎΠ±ΡΠΈΠΌ. ΠΠ΅ΠΎΠ±Ρ ΠΎΠ΄ΠΈΠΌΠΎ ΡΠ΄Π΅Π»Π°ΡΡ Π»ΠΈΡΡ Π½Π΅Π±ΠΎΠ»ΡΡΡΡ ΠΏΠΎΠΏΡΠ°Π²ΠΊΡ. ΠΡΠΎ Π·Π½Π°ΠΊ ΠΌΠΈΠ½ΡΡ, ΠΊΠΎΡΠΎΡΡΠΉ ΡΠ²ΡΠ·Π°Π½ Ρ ΠΎΠΏΡΠ΅Π΄Π΅Π»Π΅Π½Π½ΠΎΠΉ ΡΡΠ»ΠΎΠ²Π½ΠΎΡΡΡΡ ΠΈ ΠΏΡΠΈΠ½ΡΠΈΠΏΠΎΠΌ ΡΠΎΡ ΡΠ°Π½Π΅Π½ΠΈΡ ΡΠ½Π΅ΡΠ³ΠΈΠΈ.
Π’Π°ΠΊΠΈΠΌ ΠΎΠ±ΡΠ°Π·ΠΎΠΌ, Π·Π°ΠΊΠΎΠ½ ΡΠ»Π΅ΠΊΡΡΠΎΠΌΠ°Π³Π½ΠΈΡΠ½ΠΎΠΉ ΠΈΠ½Π΄ΡΠΊΡΠΈΠΈ Π€Π°ΡΠ°Π΄Π΅Ρ Π·Π°ΠΏΠΈΡΡΠ²Π°Π΅ΡΡΡ ΡΠ»Π΅Π΄ΡΡΡΠΈΠΌ ΠΎΠ±ΡΠ°Π·ΠΎΠΌ: Ξ΅ΠΈΠ½Π΄ = β dΠ€B / dt ΠΈ ΡΠΎΡΠΌΡΠ»ΠΈΡΡΠ΅ΡΡΡ ΡΠ°ΠΊ:
ΠΠ»Ρ Π»ΡΠ±ΠΎΠ³ΠΎ ΠΊΠΎΠ½ΡΡΡΠ° ΠΈΠ½Π΄ΡΡΠΈΡΠΎΠ²Π°Π½Π½Π°ΡΒ ΡΠ»Π΅ΠΊΡΡΠΎΠ΄Π²ΠΈΠΆΡΡΠ°Ρ ΡΠΈΠ»Π°Β (ΠΠΠ‘) ΡΠ°Π²Π½Π° ΡΠΊΠΎΡΠΎΡΡΠΈ ΠΈΠ·ΠΌΠ΅Π½Π΅Π½ΠΈΡΒ ΠΌΠ°Π³Π½ΠΈΡΠ½ΠΎΠ³ΠΎ ΠΏΠΎΡΠΎΠΊΠ°, ΠΏΡΠΎΡ ΠΎΠ΄ΡΡΠ΅Π³ΠΎ ΡΠ΅ΡΠ΅Π· ΡΡΠΎΡ ΠΊΠΎΠ½ΡΡΡ, Π²Π·ΡΡΠΎΠΉ ΡΠΎ Π·Π½Π°ΠΊΠΎΠΌ ΠΌΠΈΠ½ΡΡ.
ΠΠΈΠΊΠΈΠΏΠ΅Π΄ΠΈΡ
ΠΠ½Π°ΠΊ βΠΌΠΈΠ½ΡΡβ ΠΎΠ·Π½Π°ΡΠ°Π΅Ρ, ΡΡΠΎ ΠΠΠ‘ ΠΈΠ½Π΄ΡΠΊΡΠΈΠΈ Π΄Π΅ΠΉΡΡΠ²ΡΠ΅Ρ ΡΠ°ΠΊ, ΡΡΠΎ ΠΈΠ½Π΄ΡΠΊΡΠΈΠΎΠ½Π½ΡΠΉ ΡΠΎΠΊ ΠΏΡΠ΅ΠΏΡΡΡΡΠ²ΡΠ΅Ρ ΠΈΠ·ΠΌΠ΅Π½Π΅Π½ΠΈΡ ΠΏΠΎΡΠΎΠΊΠ°. ΠΡΠΎΡ ΡΠ°ΠΊΡ ΠΎΡΡΠ°ΠΆΡΠ½ Π² ΠΏΡΠ°Π²ΠΈΠ»Π΅ ΠΠ΅Π½ΡΠ°.
ΠΡΠΎΡ Π·Π°ΠΊΠΎΠ½ Π²Π΅ΡΠ΅Π½ Π½Π΅Π·Π°Π²ΠΈΡΠΈΠΌΠΎ ΠΎΡ ΡΠΎΠ³ΠΎ, ΠΊΠ°ΠΊ ΠΈΠ·ΠΌΠ΅Π½ΡΠ΅ΡΡΡ ΠΏΠΎΡΠΎΠΊ ΠΌΠ°Π³Π½ΠΈΡΠ½ΠΎΠ³ΠΎ ΠΏΠΎΠ»Ρ; ΠΊΠΎΠ³Π΄Π° ΠΈΠ·ΠΌΠ΅Π½Π΅Π½ΠΈΠ΅ Π²ΡΠ·Π²Π°Π½ΠΎ ΠΎΡΠ½ΠΎΡΠΈΡΠ΅Π»ΡΠ½ΡΠΌ Π΄Π²ΠΈΠΆΠ΅Π½ΠΈΠ΅ΠΌ ΠΈΡΡΠΎΡΠ½ΠΈΠΊΠ° ΠΌΠ°Π³Π½ΠΈΡΠ½ΠΎΠ³ΠΎ ΠΏΠΎΠ»Ρ ΠΈ ΠΊΠΎΠ½ΡΡΡΠ°, ΠΈΠ»ΠΈ ΠΊΠΎΠ³Π΄Π° Π΄Π²ΠΈΠΆΠ΅Π½ΠΈΡ Π²ΠΎΠΎΠ±ΡΠ΅ Π½Π΅Ρ, Π½ΠΎ Π·Π½Π°ΡΠ΅Π½ΠΈΠ΅ ΠΌΠ°Π³Π½ΠΈΡΠ½ΠΎΠΉ ΠΈΠ½Π΄ΡΠΊΡΠΈΠΈ ΠΌΠ΅Π½ΡΠ΅ΡΡΡ.
ΠΠ°ΠΊΠΎΠ½ Π€Π°ΡΠ°Π΄Π΅Ρ β ΡΡΠΎ ΡΠ½ΠΈΠ²Π΅ΡΡΠ°Π»ΡΠ½ΡΠΉ, Π²ΡΠ΅ΠΎΠ±ΡΠ΅ΠΌΠ»ΡΡΠΈΠΉ ΠΈ ΠΏΠΎΠ»Π½ΡΠΉ ΠΌΠ°ΡΠ΅ΠΌΠ°ΡΠΈΡΠ΅ΡΠΊΠΈΠΉ ΠΎΡΡΠ΅Ρ ΠΎ ΡΠ²Π»Π΅Π½ΠΈΠΈ ΡΠ»Π΅ΠΊΡΡΠΎΠΌΠ°Π³Π½ΠΈΡΠ½ΠΎΠΉ ΠΈΠ½Π΄ΡΠΊΡΠΈΠΈ.
ΠΠ΅ΡΠ½Π΅ΠΌΡΡ Π½Π° ΠΌΠ³Π½ΠΎΠ²Π΅Π½ΠΈΠ΅ ΠΊ Π½Π°ΡΠ΅ΠΌΡ ΠΏΡΠΈΠΌΠ΅ΡΡ ΠΈ ΠΎΡΠΌΠ΅ΡΠΈΠΌ, ΡΡΠΎ ΡΠΊΠΎΡΠΎΡΡΡ ΠΈΠ·ΠΌΠ΅Π½Π΅Π½ΠΈΡ ΠΏΠΎΡΠΎΠΊΠ°, Π° Π·Π½Π°ΡΠΈΡ ΠΈ Π°Π±ΡΠΎΠ»ΡΡΠ½ΠΎΠ΅ Π·Π½Π°ΡΠ΅Π½ΠΈΠ΅ ΠΠΠ‘ ΠΈΠ½Π΄ΡΠΊΡΠΈΠΈ, Π² Π΄Π°Π½Π½ΠΎΠΌ ΡΠ»ΡΡΠ°Π΅ ΡΠ°Π²Π½Π° ΠΏΡΠΎΠΈΠ·Π²Π΅Π΄Π΅Π½ΠΈΡ B*L*v. ΠΡΠΎ ΡΠ»Π΅Π΄ΡΠ΅Ρ ΠΈΠ· ΡΠ°Π½Π΅Π΅ Π½Π°ΠΏΠΈΡΠ°Π½Π½ΡΡ ΡΠΎΠΎΡΠ½ΠΎΡΠ΅Π½ΠΈΠΉ, Π° ΠΈΠΌΠ΅Π½Π½ΠΎ:
| Ξ΅ΠΈΠ½Π΄ | = Az / q = I * L * B * Ξx / I * Ξt = B * L * Ξx / Ξt = B * L * ( Ξx / Ξt ) = B * L * v .
ΠΡΠ°Π²ΠΈΠ»ΠΎ ΠΠ΅Π½ΡΠ°.
ΠΡΠ°Π²ΠΈΠ»ΠΎ ΠΠ΅Π½ΡΠ° ΠΏΠΎΠ·Π²ΠΎΠ»ΡΠ΅Ρ Π±ΡΡΡΡΠΎ ΠΈ Π»Π΅Π³ΠΊΠΎ ΠΎΠΏΡΠ΅Π΄Π΅Π»ΠΈΡΡ Π½Π°ΠΏΡΠ°Π²Π»Π΅Π½ΠΈΠ΅ ΠΈΠ½Π΄ΡΠΊΡΠΈΠΎΠ½Π½ΠΎΠ³ΠΎ ΡΠΎΠΊΠ°. ΠΡΠΎ Π΄Π΅ΠΉΡΡΠ²ΠΈΡΠ΅Π»ΡΠ½ΠΎ ΠΎΠ΄Π½Π° ΠΈΠ· ΡΠΎΡΠΌ ΠΏΡΠΈΠ½ΡΠΈΠΏΠ° ΡΠΎΡ ΡΠ°Π½Π΅Π½ΠΈΡ ΡΠ½Π΅ΡΠ³ΠΈΠΈ. ΠΡΠ°Π²ΠΈΠ»ΠΎ Π³Π»Π°ΡΠΈΡ, ΡΡΠΎ ΠΈΠ½Π΄ΡΠΊΡΠΈΠΎΠ½Π½ΡΠΉ ΡΠΎΠΊ, Π½Π°Π²Π΅Π΄Π΅Π½Π½ΡΠΉ Π² ΠΏΡΠΎΠ²ΠΎΠ΄Π½ΠΈΠΊΠ΅ ΠΏΠΎΠ΄ Π΄Π΅ΠΉΡΡΠ²ΠΈΠ΅ΠΌ ΠΏΠ΅ΡΠ΅ΠΌΠ΅Π½Π½ΠΎΠ³ΠΎ ΠΏΠΎΡΠΎΠΊΠ° ΠΌΠ°Π³Π½ΠΈΡΠ½ΠΎΠΉ ΠΈΠ½Π΄ΡΠΊΡΠΈΠΈ, Π²ΡΠ΅Π³Π΄Π° ΠΈΠΌΠ΅Π΅Ρ ΡΠ°ΠΊΠΎΠ΅ Π½Π°ΠΏΡΠ°Π²Π»Π΅Π½ΠΈΠ΅, ΡΡΠΎ ΠΌΠ°Π³Π½ΠΈΡΠ½ΠΎΠ΅ ΠΏΠΎΠ»Π΅, ΡΠΎΠ·Π΄Π°Π²Π°Π΅ΠΌΠΎΠ΅ ΡΡΠΈΠΌ ΠΈΠ½Π΄ΡΠΊΡΠΈΠΎΠ½Π½ΡΠΌ ΡΠΎΠΊΠΎΠΌ, ΠΏΡΠΎΡΠΈΠ²ΠΎΠ΄Π΅ΠΉΡΡΠ²ΡΠ΅Ρ ΠΏΡΠΈΡΠΈΠ½Π΅ (Ρ.Π΅. ΠΈΠ·ΠΌΠ΅Π½Π΅Π½ΠΈΡ ΠΏΠΎΡΠΎΠΊΠ° ΠΌΠ°Π³Π½ΠΈΡΠ½ΠΎΠ³ΠΎ ΠΏΠΎΠ»Ρ), ΠΊΠΎΡΠΎΡΠ°Ρ Π΅Π³ΠΎ Π²ΡΠ·Π²Π°Π»Π°.
ΠΡΠΈΠΌΠ΅Ρ Π·Π°Π΄Π°ΡΠΈ
ΠΠ°Π½ΠΎ:
ΠΠΎΠ½ΡΡΡ Π² ΡΠΎΡΠΌΠ΅ ΠΊΠ²Π°Π΄ΡΠ°ΡΠ° ΡΠΎ ΡΡΠΎΡΠΎΠ½ΠΎΠΉ d = 0,5 ΠΌ βΠ²ΡΡΠ³ΠΈΠ²Π°Π΅ΡΡΡβ Ρ ΠΏΠΎΡΡΠΎΡΠ½Π½ΠΎΠΉ ΡΠΊΠΎΡΠΎΡΡΡΡ v = 4 ΠΌ/Ρ Π² ΠΎΠ±Π»Π°ΡΡΡ ΠΎΠ΄Π½ΠΎΡΠΎΠ΄Π½ΠΎΠ³ΠΎ ΠΌΠ°Π³Π½ΠΈΡΠ½ΠΎΠ³ΠΎ ΠΏΠΎΠ»Ρ, Π²Π΅Π»ΠΈΡΠΈΠ½Π° ΠΈΠ½Π΄ΡΠΊΡΠΈΠΈ ΠΊΠΎΡΠΎΡΠΎΠ³ΠΎ B = 1 Π’Π» (ΡΠΌ. ΡΠΈΡ. 4). ΠΠ»Π΅ΠΊΡΡΠΈΡΠ΅ΡΠΊΠΎΠ΅ ΡΠΎΠΏΡΠΎΡΠΈΠ²Π»Π΅Π½ΠΈΠ΅ ΡΠ΅ΠΏΠΈ ΡΠ°Π²Π½ΠΎ R = 2 ΠΠΌ.
Π ΠΈΡ. 4. ΠΡΠΈΠΌΠ΅Ρ Π·Π°Π΄Π°ΡΠΈ ΠΏΠΎ ΡΠ»Π΅ΠΊΡΡΠΎΡΡΠ°ΡΠΈΡΠ΅ΡΠΊΠΎΠΉ ΠΈΠ½Π΄ΡΠΊΡΠΈΠΈΠΠ°ΠΌ Π½ΡΠΆΠ½ΠΎ Π½Π°ΠΉΡΠΈ ΠΎΡΠ²Π΅ΡΡ Π½Π° ΡΠ»Π΅Π΄ΡΡΡΠΈΠ΅ Π²ΠΎΠΏΡΠΎΡΡ:
a) ΠΠΎΠ³Π΄Π° (Π² ΠΊΠ°ΠΊΠΎΠΉ ΠΌΠΎΠΌΠ΅Π½Ρ/ΠΌΠΎΠΌΠ΅Π½ΡΡ) Π² ΡΠ°ΠΌΠΊΠ΅ Π±ΡΠ΄Π΅Ρ ΠΏΡΠΎΡΠ΅ΠΊΠ°ΡΡ ΡΠ»Π΅ΠΊΡΡΠΈΡΠ΅ΡΠΊΠΈΠΉ ΡΠΎΠΊ?
Π±) ΠΠΏΡΠ΅Π΄Π΅Π»ΠΈΡΠ΅ Π½Π°ΠΏΡΠ°Π²Π»Π΅Π½ΠΈΠ΅ ΡΡΠΎΠ³ΠΎ ΡΠ»Π΅ΠΊΡΡΠΈΡΠ΅ΡΠΊΠΎΠ³ΠΎ ΡΠΎΠΊΠ°.
(Π²) ΠΡΡΠΈΡΠ»ΠΈΡΠ΅ Π·Π½Π°ΡΠ΅Π½ΠΈΠ΅ ΡΠΈΠ»Ρ, Π΄Π΅ΠΉΡΡΠ²ΡΡΡΠ΅ΠΉ Π½Π° ΡΠ°ΠΌΠΊΡ ΠΏΡΠΈ Π΅Π΅ ΠΏΠ΅ΡΠ΅ΠΌΠ΅ΡΠ΅Π½ΠΈΠΈ Π² ΡΠΎΠΎΡΠ²Π΅ΡΡΡΠ²ΠΈΠΈ Ρ Π½Π°ΠΏΡΠ°Π²Π»Π΅Π½ΠΈΠ΅ΠΌ Π²Π΅ΠΊΡΠΎΡΠ° ΡΠΊΠΎΡΠΎΡΡΠΈ. ΠΡΠ΅Π΄ΠΏΠΎΠ»ΠΎΠΆΠΈΡΠ΅ ΠΎΡΡΡΡΡΡΠ²ΠΈΠ΅ ΠΌΠ΅Ρ Π°Π½ΠΈΡΠ΅ΡΠΊΠΎΠ³ΠΎ ΡΠΎΠΏΡΠΎΡΠΈΠ²Π»Π΅Π½ΠΈΡ Π΄Π²ΠΈΠΆΠ΅Π½ΠΈΡ.
Π Π΅ΡΠ΅Π½ΠΈΠ΅.
(a) ΠΠ½Π΄ΡΠΊΡΠΈΠΎΠ½Π½ΡΠΉ ΡΠΎΠΊ ΠΏΡΠΎΡΠ΅ΠΊΠ°Π΅Ρ ΠΏΡΠΈ ΠΈΠ·ΠΌΠ΅Π½Π΅Π½ΠΈΠΈ ΠΏΠΎΡΠΎΠΊΠ° ΠΌΠ°Π³Π½ΠΈΡΠ½ΠΎΠΉ ΠΈΠ½Π΄ΡΠΊΡΠΈΠΈ ΡΠ΅ΡΠ΅Π· ΠΏΠΎΠ²Π΅ΡΡ Π½ΠΎΡΡΡ, ΠΎΡ Π²Π°ΡΠ΅Π½Π½ΡΡ ΠΊΠΎΠ½ΡΡΡΠΎΠΌ. Π ΡΠΈΡΡΠ°ΡΠΈΠΈ, ΠΏΠΎΠΊΠ°Π·Π°Π½Π½ΠΎΠΉ Π½Π° ΡΠΈΡΡΠ½ΠΊΠ΅ 4, ΠΌΠ°Π³Π½ΠΈΡΠ½ΡΠΉ ΠΏΠΎΡΠΎΠΊ ΡΠ°Π²Π΅Π½ Π½ΡΠ»Ρ ΠΈ Π±ΡΠ΄Π΅Ρ ΠΎΡΡΠ°Π²Π°ΡΡΡΡ ΡΠ°ΠΊΠΎΠ²ΡΠΌ Π΄ΠΎ ΡΠ΅Ρ ΠΏΠΎΡ, ΠΏΠΎΠΊΠ° ΠΏΡΠ°Π²ΡΠΉ ΠΊΡΠ°ΠΉ ΠΊΠΎΠ½ΡΡΡΠ° Π½Π΅ ΠΊΠΎΡΠ½Π΅ΡΡΡ Π³ΡΠ°Π½ΠΈΡΡ ΠΎΠ±Π»Π°ΡΡΠΈ ΠΌΠ°Π³Π½ΠΈΡΠ½ΠΎΠ³ΠΎ ΠΏΠΎΠ»Ρ. ΠΠ°ΡΠ΅ΠΌ, ΠΏΠΎ ΠΌΠ΅ΡΠ΅ Π΄Π²ΠΈΠΆΠ΅Π½ΠΈΡ ΠΊΠΎΠ½ΡΡΡΠ°, ΠΎΠ½ Π±ΡΠ΄Π΅Ρ Π²ΡΠ΅ Π±ΠΎΠ»ΡΡΠ΅ ΠΈ Π±ΠΎΠ»ΡΡΠ΅ Π·Π°ΠΏΠΎΠ»Π½ΡΡΡΡΡ ΠΌΠ°Π³Π½ΠΈΡΠ½ΡΠΌ ΠΏΠΎΠ»Π΅ΠΌ β ΠΌΠ°Π³Π½ΠΈΡΠ½ΡΠΉ ΠΏΠΎΡΠΎΠΊ Π±ΡΠ΄Π΅Ρ ΡΠ²Π΅Π»ΠΈΡΠΈΠ²Π°ΡΡΡΡ. ΠΠΎΡΡΠΎΠΌΡ Π²ΡΠΏΠΎΠ»Π½ΡΠ΅ΡΡΡ ΡΡΠ»ΠΎΠ²ΠΈΠ΅ ΡΠ»Π΅ΠΊΡΡΠΎΠΌΠ°Π³Π½ΠΈΡΠ½ΠΎΠΉ ΠΈΠ½Π΄ΡΠΊΡΠΈΠΈ, Ρ.Π΅. Π½Π°ΡΠΈΠ½Π°Π΅Ρ ΠΏΡΠΎΡΠ΅ΠΊΠ°ΡΡ ΠΈΠ½Π΄ΡΠΊΡΠΈΠΎΠ½Π½ΡΠΉ ΡΠΎΠΊ. ΠΠ°ΠΊ Π΄ΠΎΠ»Π³ΠΎ? ΠΡΠΎ Π»Π΅Π³ΠΊΠΎ Π²ΡΡΠΈΡΠ»ΠΈΡΡ, ΠΏΠΎΡΠΊΠΎΠ»ΡΠΊΡ Π΄Π²ΠΈΠΆΠ΅Π½ΠΈΠ΅ ΡΠ°ΠΌΡ ΡΠ°Π²Π½ΠΎΠΌΠ΅ΡΠ½ΠΎ:
t = d / v = 0,5 / 2 = 0,25 ΡΠ΅ΠΊΡΠ½Π΄
Π’ΠΎΠΊ Π±ΡΠ΄Π΅Ρ ΡΠ΅ΡΡ Π΄ΠΎ ΡΠ΅Ρ ΠΏΠΎΡ, ΠΏΠΎΠΊΠ° Π²Π΅ΡΡ ΠΊΠ²Π°Π΄ΡΠ°Ρ Π½Π΅ Π²ΠΎΠΉΠ΄Π΅Ρ Π² ΠΌΠ°Π³Π½ΠΈΡΠ½ΠΎΠ΅ ΠΏΠΎΠ»Π΅. Π’ΠΎΠ³Π΄Π° ΠΏΠΎΡΠΎΠΊ Π±ΡΠ΄Π΅Ρ Π½Π΅Π½ΡΠ»Π΅Π²ΡΠΌ, Π½ΠΎ Π±ΠΎΠ»ΡΡΠ΅ Π½Π΅ Π±ΡΠ΄Π΅Ρ ΠΌΠ΅Π½ΡΡΡΡΡ.
Π±) ΠΠΎΡΠΏΠΎΠ»ΡΠ·ΡΠ΅ΠΌΡΡ ΠΏΡΠ°Π²ΠΈΠ»ΠΎΠΌ ΠΠ΅Π½ΡΠ°. ΠΡ ΡΠΆΠ΅ Π·Π°ΠΌΠ΅ΡΠΈΠ»ΠΈ, ΡΡΠΎ ΠΏΠΎΡΠΎΠΊ ΠΌΠ°Π³Π½ΠΈΡΠ½ΠΎΠΉ ΠΈΠ½Π΄ΡΠΊΡΠΈΠΈ ΠΏΡΠΈ βΠ²ΡΡΠ³ΠΈΠ²Π°Π½ΠΈΠΈβ ΠΊΠΎΠ½ΡΡΡΠ° Π² ΠΌΠ°Π³Π½ΠΈΡΠ½ΠΎΠ΅ ΠΏΠΎΠ»Π΅ ΡΠ²Π΅Π»ΠΈΡΠΈΠ²Π°Π΅ΡΡΡ. ΠΠΎΡΡΠΎΠΌΡ ΠΈΠ½Π΄ΡΠΊΡΠΈΠΎΠ½Π½ΡΠΉ ΡΠΎΠΊ Π±ΡΠ΄Π΅Ρ ΠΏΡΠΎΡΠ΅ΠΊΠ°ΡΡ Π² ΡΠ°ΠΊΠΎΠΌ Π½Π°ΠΏΡΠ°Π²Π»Π΅Π½ΠΈΠΈ, ΡΡΠΎΠ±Ρ ΠΏΡΠΎΡΠΈΠ²ΠΎΠ΄Π΅ΠΉΡΡΠ²ΠΎΠ²Π°ΡΡ ΡΠ²Π΅Π»ΠΈΡΠ΅Π½ΠΈΡ ΠΏΠΎΡΠΎΠΊΠ°.
ΠΠ°Π³Π½ΠΈΡΠ½ΠΎΠ΅ ΠΏΠΎΠ»Π΅, ΡΠΎΠ·Π΄Π°Π²Π°Π΅ΠΌΠΎΠ΅ ΠΈΠ½Π΄ΡΠΊΡΠΈΠΎΠ½Π½ΡΠΌ ΡΠΎΠΊΠΎΠΌ Ρ Π²Π΅ΠΊΡΠΎΡΠΎΠΌ ΠΈΠ½Π΄ΡΠΊΡΠΈΠΈ BΠΈΠ½Π΄, Π±ΡΠ΄Π΅Ρ ΠΏΡΠΎΡΠΈΠ²ΠΎΠΏΠΎΠ»ΠΎΠΆΠ½ΠΎ Π²Π΅ΠΊΡΠΎΡΡ B.
Π’Π°ΠΊΠΈΠΌ ΠΎΠ±ΡΠ°Π·ΠΎΠΌ, Π²Π΅ΠΊΡΠΎΡ BΠΈΠ½Π΄ Π½Π°ΠΏΡΠ°Π²Π»Π΅Π½ Π² Π½Π°ΡΡ ΡΡΠΎΡΠΎΠ½Ρ. ΠΡΠ»ΠΈ ΡΠ°ΡΠΏΠΎΠ»ΠΎΠΆΠΈΡΡ ΡΠ°ΠΊΠΈΠΌ ΠΎΠ±ΡΠ°Π·ΠΎΠΌ Π±ΠΎΠ»ΡΡΠΎΠΉ ΠΏΠ°Π»Π΅Ρ ΠΏΡΠ°Π²ΠΎΠΉ ΡΡΠΊΠΈ, ΠΎΡΡΠ°Π»ΡΠ½ΡΠ΅ ΡΠΎΠ³Π½ΡΡΡΠ΅ ΠΏΠ°Π»ΡΡΡ ΠΏΠΎΠΊΠ°ΠΆΡΡ Π½Π°ΠΏΡΠ°Π²Π»Π΅Π½ΠΈΠ΅ ΠΈΠ½Π΄ΡΠΊΡΠΈΠΎΠ½Π½ΠΎΠ³ΠΎ ΡΠΎΠΊΠ°. Π’ΠΎΠΊ Π±ΡΠ΄Π΅Ρ ΡΠ΅ΡΡ ΠΏΡΠΎΡΠΈΠ² ΡΠ°ΡΠΎΠ²ΠΎΠΉ ΡΡΡΠ΅Π»ΠΊΠΈ.
(Π²) Π‘Π½ΠΎΠ²Π° Π²ΠΎΡΠΏΠΎΠ»ΡΠ·ΡΠ΅ΠΌΡΡ ΡΠ°Π²Π½ΠΎΠΌΠ΅ΡΠ½ΠΎΡΡΡΡ Π΄Π²ΠΈΠΆΠ΅Π½ΠΈΡ ΡΠ°ΠΌΡ. ΠΠ±ΡΠ°ΡΠΈΡΠ΅ Π²Π½ΠΈΠΌΠ°Π½ΠΈΠ΅, ΡΡΠΎ ΡΠΈΠ»Π°, ΠΊΠΎΡΠΎΡΠ°Ρ Π΄Π΅ΠΉΡΡΠ²ΡΠ΅Ρ Π½Π° ΡΠ°ΠΌΠΊΡ ΠΏΡΠΈ Π΅Π΅ ΠΏΠ΅ΡΠ΅ΠΌΠ΅ΡΠ΅Π½ΠΈΠΈ ΠΏΠΎ Π²Π΅ΠΊΡΠΎΡΡ ΡΠΊΠΎΡΠΎΡΡΠΈ (Π½Π°ΠΏΡΠΈΠΌΠ΅Ρ, ΡΠΈΠ»Π° ΠΌΠΎΠ΅ΠΉ ΡΡΠΊΠΈ), Π½Π΅ ΠΌΠΎΠΆΠ΅Ρ Π±ΡΡΡ Π΅Π΄ΠΈΠ½ΡΡΠ²Π΅Π½Π½ΠΎΠΉ ΡΠΈΠ»ΠΎΠΉ, Π΄Π΅ΠΉΡΡΠ²ΡΡΡΠ΅ΠΉ Π½Π° ΠΊΠ²Π°Π΄ΡΠ°Ρ. ΠΡΠ»ΠΈ Π±Ρ ΡΡΠΎ Π±ΡΠ»ΠΎ ΡΠ°ΠΊ, ΠΎΠ½ Π±Ρ Π΄Π²ΠΈΠ³Π°Π»ΡΡ Ρ ΡΡΠΊΠΎΡΠ΅Π½ΠΈΠ΅ΠΌ. ΠΠΎΡΠΊΠΎΠ»ΡΠΊΡ Π΄Π²ΠΈΠΆΠ΅Π½ΠΈΠ΅ ΡΠ°Π²Π½ΠΎΠΌΠ΅ΡΠ½ΠΎΠ΅, ΡΡΠΎ ΠΎΠ·Π½Π°ΡΠ°Π΅Ρ, ΡΡΠΎ Π² ΠΊΠ°ΠΆΠ΄ΡΠΉ ΠΌΠΎΠΌΠ΅Π½Ρ Π²ΡΠ΅ΠΌΠ΅Π½ΠΈ ΡΡΡΠ΅ΡΡΠ²ΡΠ΅Ρ ΡΠΈΠ»Π°, ΠΊΠΎΡΠΎΡΠ°Ρ ΡΡΠ°Π²Π½ΠΎΠ²Π΅ΡΠΈΠ²Π°Π΅Ρ ΡΠΈΠ»Ρ ΠΌΠΎΠ΅ΠΉ ΡΡΠΊΠΈ. ΠΡΠΎ ΠΈ Π΅ΡΡΡ ΡΠ»Π΅ΠΊΡΡΠΎΠ΄ΠΈΠ½Π°ΠΌΠΈΡΠ΅ΡΠΊΠ°Ρ ΡΠΈΠ»Π°. ΠΠ΅Π΄Ρ ΡΠ΅ΠΏΠ΅ΡΡ Π² ΡΠ°ΠΌΠΊΠ΅ ΡΠ΅ΡΠ΅Ρ ΡΠΎΠΊ, ΠΈ ΡΠ°ΡΡΡ Π΅Π³ΠΎ ΠΏΡΠΎΡΠ΅ΠΊΠ°Π΅Ρ Π² ΠΌΠ°Π³Π½ΠΈΡΠ½ΠΎΠΌ ΠΏΠΎΠ»Π΅ (ΡΠΌ. ΡΠΈΡ. 5).
Π ΠΈΡ. 5ΠΡΠ°ΡΠ½Π°Ρ ΡΡΡΠ΅Π»ΠΊΠ° ΠΏΠΎΠΊΠ°Π·ΡΠ²Π°Π΅Ρ Π½Π°ΠΏΡΠ°Π²Π»Π΅Π½ΠΈΠ΅ ΡΠ»Π΅ΠΊΡΡΠΈΡΠ΅ΡΠΊΠΎΠ³ΠΎ ΡΠΎΠΊΠ°. ΠΠ»Π΅ΠΊΡΡΠΎΠ΄ΠΈΠ½Π°ΠΌΠΈΡΠ΅ΡΠΊΠ°Ρ ΡΠΈΠ»Π° (ΡΠΈΠ»Π° ΠΠΌΠΏΠ΅ΡΠ°) Π΄Π΅ΠΉΡΡΠ²ΡΠ΅Ρ ΡΠ»Π΅Π²Π° (Ρ ΠΎΠΏΡΠ΅Π΄Π΅Π»ΠΈΠ» Π΅Π΅ Ρ ΠΏΠΎΠΌΠΎΡΡΡ ΠΏΡΠ°Π²ΠΈΠ»Π° ΡΡΠ΅Ρ ΠΏΠ°Π»ΡΡΠ΅Π²). ΠΠ° Π²Π΅ΡΡ Π½ΡΡ ΡΠ°ΡΡΡ ΡΠ°ΠΌΠΊΠΈ ΠΈ Π½ΠΈΠΆΠ½ΡΡ ΡΠ°ΡΡΡ ΡΠ°ΠΊΠΆΠ΅ Π΄Π΅ΠΉΡΡΠ²ΡΡΡ ΡΠ»Π΅ΠΊΡΡΠΎΠ΄ΠΈΠ½Π°ΠΌΠΈΡΠ΅ΡΠΊΠΈΠ΅ ΡΠΈΠ»Ρ, Π½ΠΎ ΠΎΠ½ΠΈ Π°Π½Π½ΡΠ»ΠΈΡΡΡΡ Π΄ΡΡΠ³ Π΄ΡΡΠ³Π°.
ΠΠΎΠ΄Π²Π΅Π΄Π΅ΠΌ ΠΈΡΠΎΠ³: ΡΠ»Π΅ΠΊΡΡΠΎΠ΄ΠΈΠ½Π°ΠΌΠΈΡΠ΅ΡΠΊΠ°Ρ ΡΠΈΠ»Π° ΡΡΠ°Π²Π½ΠΎΠ²Π΅ΡΠΈΠ²Π°Π΅Ρ ΡΠΈΠ»Ρ ΠΌΠΎΠ΅ΠΉ ΡΡΠΊΠΈ. Π’Π°ΠΊΠΈΠΌ ΠΎΠ±ΡΠ°Π·ΠΎΠΌ, Ρ ΠΌΠΎΠ³Ρ ΡΡΠ°Π²Π½ΠΈΡΡ Π·Π½Π°ΡΠ΅Π½ΠΈΡ ΠΎΠ±Π΅ΠΈΡ ΡΠΈΠ», ΡΠΎ Π΅ΡΡΡ F = Fed = B * I * d, Π³Π΄Π΅ I β ΡΠΈΠ»Π° ΠΈΠ½Π΄ΡΠΊΡΠΈΠΎΠ½Π½ΠΎΠ³ΠΎ ΡΠΎΠΊΠ°. Π’Π΅ΠΏΠ΅ΡΡ Π΄ΠΎΡΡΠ°ΡΠΎΡΠ½ΠΎ ΡΠ°ΡΡΡΠΈΡΠ°ΡΡ Π·Π½Π°ΡΠ΅Π½ΠΈΠ΅ ΡΠΈΠ»Ρ ΡΡΠΎΠ³ΠΎ ΡΠΎΠΊΠ°. ΠΡ Π±ΡΠ΄Π΅ΠΌ ΠΈΡΠΏΠΎΠ»ΡΠ·ΠΎΠ²Π°ΡΡ Π·Π°ΠΊΠΎΠ½ Π€Π°ΡΠ°Π΄Π΅Ρ ΠΈ Π·Π°ΠΊΠΎΠ½ ΠΠΌΠ° Π΄Π»Ρ ΡΡΠ°ΡΡΠΊΠ° ΡΠ΅ΠΏΠΈ. ΠΠ°Π²Π°ΠΉΡΠ΅ Π½Π°ΡΠ½Π΅ΠΌ Ρ ΠΏΠΎΡΠ»Π΅Π΄Π½Π΅Π³ΠΎ: ΠΏΠΎΡΠΊΠΎΠ»ΡΠΊΡ Π½Π°Ρ ΠΈΠ½ΡΠ΅ΡΠ΅ΡΡΠ΅Ρ ΡΠΎΠ»ΡΠΊΠΎ Π·Π½Π°ΡΠ΅Π½ΠΈΠ΅ I, ΠΌΡ Π½Π°ΠΏΠΈΡΠ΅ΠΌ
I = Ξ΅ΠΈΠ½Π΄ / R .
| Ξ΅ΠΈΠ½Π΄ | = ΞΠ€B / Ξt = Ξx * d * B / Ξt = ( Ξx / Ξt ) * d * B = v * d * B .
ΠΠΎΡΠ»Π΅ ΠΏΠΎΠ΄ΡΡΠ°Π½ΠΎΠ²ΠΊΠΈ Π² I ΠΏΠΎΠ»ΡΡΠ°Π΅ΠΌ: I = Ξ΅ΠΈΠ½Π΄ / R = v * d * B / R .
Π ΠΊΠΎΠ½Π΅ΡΠ½ΠΎΠΌ ΠΈΡΠΎΠ³Π΅ ΠΈΡΠΊΠΎΠΌΠΎΠ΅ Π·Π½Π°ΡΠ΅Π½ΠΈΠ΅ ΡΠΈΠ»Ρ Π±ΡΠ΄Π΅Ρ Π²ΡΡΠ°ΠΆΠ΅Π½ΠΎ ΡΠ΅ΡΠ΅Π·: Fed = B * I * d = ( B * d * v * d * B ) / R = ( B2 * d2 * v ) / R .
ΠΠΎΠ΄ΡΡΠ°Π²Π»ΡΡ ΡΠΈΡΠ»Π΅Π½Π½ΡΠ΅ Π·Π½Π°ΡΠ΅Π½ΠΈΡ ΠΏΠΎΠ»ΡΡΠΈΠΌ: Fed = F = ( 12 * 0,52 * 4 ) / 2 = 0,5 Π .
Π‘ΠΏΠΈΡΠΎΠΊ ΠΈΡΠΏΠΎΠ»ΡΠ·ΠΎΠ²Π°Π½Π½ΠΎΠΉ Π»ΠΈΡΠ΅ΡΠ°ΡΡΡΡ
- ΠΠΈΠ»Π»Π΅Ρ Π. Π., ΠΠ΅ΡΠΌΠΈΡΠΈΠ½ Π. Π. ΠΠ»Π΅ΠΊΡΡΠΎΠΌΠ°Π³Π½ΠΈΡΠ½Π°Ρ ΠΈΠ½Π΄ΡΠΊΡΠΈΡ // Π€ΠΈΠ·ΠΈΡΠ΅ΡΠΊΠ°Ρ ΡΠ½ΡΠΈΠΊΠ»ΠΎΠΏΠ΅Π΄ΠΈΡ : [Π² 5 Ρ.] / ΠΠ». ΡΠ΅Π΄. Π. Π. ΠΡΠΎΡ ΠΎΡΠΎΠ². β Π.: ΠΠΎΠ»ΡΡΠ°Ρ ΡΠΎΡΡΠΈΠΉΡΠΊΠ°Ρ ΡΠ½ΡΠΈΠΊΠ»ΠΎΠΏΠ΅Π΄ΠΈΡ, 1999. β Π’. 5: Π‘ΡΡΠΎΠ±ΠΎΡΠΊΠΎΠΏΠΈΡΠ΅ΡΠΊΠΈΠ΅ ΠΏΡΠΈΠ±ΠΎΡΡ β Π―ΡΠΊΠΎΡΡΡ. β Π‘. 537β538. β 692 Ρ. β 20 000 ΡΠΊΠ·. β ISBN 5-85270-101-7.
- Π. ΠΠΈΠ²ΡΠΈΡ. ΠΠ°ΠΊΠΎΠ½ ΡΠ»Π΅ΠΊΡΡΠΎΠΌΠ°Π³Π½ΠΈΡΠ½ΠΎΠΉ ΠΈΠ½Π΄ΡΠΊΡΠΈΠΈ ΠΈΠ»ΠΈ Β«ΠΏΡΠ°Π²ΠΈΠ»ΠΎ ΠΏΠΎΡΠΎΠΊΠ°Β»? // ΠΠ²Π°Π½Ρ. β 1998. β β 3. β Π‘. 37β38.
- Π€ΠΈΠ·ΠΈΠΊΠ°, Π±Π°Π·ΠΎΠ²ΡΠΉ ΡΡΠΎΠ²Π΅Π½Ρ, 11 ΠΊΠ»Π°ΡΡ, ΡΡΠ΅Π±Π½ΠΈΠΊ β ΠΡΡΡΡΠ΅Π²Π° Π.Π‘., ΠΠ°ΠΆΠ΅Π΅Π²ΡΠΊΠ°Ρ Π.Π., ΠΡΠ°Π΅Π² Π.Π., Π§Π°ΡΡΠ³ΠΈΠ½ Π.Π
8.1 ΠΠΠ‘ ΠΈΠ½Π΄ΡΠΊΡΠΈΠΈ ΠΈ ΠΌΠ°Π³Π½ΠΈΡΠ½ΡΠΉ ΠΏΠΎΡΠΎΠΊ β ΠΠΎΠ»Π»Π΅Π΄ΠΆ ΠΡΠ³Π»Π°ΡΠ°, ΡΠΈΠ·ΠΈΠΊΠ° 1207
ΠΠ»Π°Π²Π° 8 ΠΠ»Π΅ΠΊΡΡΠΎΠΌΠ°Π³Π½ΠΈΡΠ½Π°Ρ ΠΈΠ½Π΄ΡΠΊΡΠΈΡ, ΡΠ΅ΠΏΠΈ ΠΏΠ΅ΡΠ΅ΠΌΠ΅Π½Π½ΠΎΠ³ΠΎ ΡΠΎΠΊΠ° ΠΈ ΡΠ»Π΅ΠΊΡΡΠΈΡΠ΅ΡΠΊΠΈΠ΅ ΡΠ΅Ρ Π½ΠΎΠ»ΠΎΠ³ΠΈΠΈ
Π‘Π²ΠΎΠ΄ΠΊΠ°
- Π Π°ΡΡΡΠΈΡΠ°ΠΉΡΠ΅ ΠΏΠΎΡΠΎΠΊ ΠΎΠ΄Π½ΠΎΡΠΎΠ΄Π½ΠΎΠ³ΠΎ ΠΌΠ°Π³Π½ΠΈΡΠ½ΠΎΠ³ΠΎ ΠΏΠΎΠ»Ρ ΡΠ΅ΡΠ΅Π· ΠΏΠ΅ΡΠ»Ρ ΠΏΡΠΎΠΈΠ·Π²ΠΎΠ»ΡΠ½ΠΎΠΉ ΠΎΡΠΈΠ΅Π½ΡΠ°ΡΠΈΠΈ.
- ΠΠΏΠΈΡΠ°ΡΡ ΠΌΠ΅ΡΠΎΠ΄Ρ ΡΠΎΠ·Π΄Π°Π½ΠΈΡ ΡΠ»Π΅ΠΊΡΡΠΎΠ΄Π²ΠΈΠΆΡΡΠ΅ΠΉ ΡΠΈΠ»Ρ (ΠΠΠ‘) Ρ ΠΏΠΎΠΌΠΎΡΡΡ ΠΌΠ°Π³Π½ΠΈΡΠ½ΠΎΠ³ΠΎ ΠΏΠΎΠ»Ρ ΠΈΠ»ΠΈ ΠΌΠ°Π³Π½ΠΈΡΠ° ΠΈ ΠΏΡΠΎΠ²ΠΎΠ»ΠΎΡΠ½ΠΎΠΉ ΠΏΠ΅ΡΠ»ΠΈ.
Π£ΡΡΡΠΎΠΉΡΡΠ²ΠΎ, ΠΈΡΠΏΠΎΠ»ΡΠ·ΠΎΠ²Π°Π½Π½ΠΎΠ΅ Π€Π°ΡΠ°Π΄Π΅Π΅ΠΌ Π΄Π»Ρ Π΄Π΅ΠΌΠΎΠ½ΡΡΡΠ°ΡΠΈΠΈ ΡΠΎΠ³ΠΎ, ΡΡΠΎ ΠΌΠ°Π³Π½ΠΈΡΠ½ΡΠ΅ ΠΏΠΎΠ»Ρ ΠΌΠΎΠ³ΡΡ ΡΠΎΠ·Π΄Π°Π²Π°ΡΡ ΡΠΎΠΊΠΈ, ΠΏΠΎΠΊΠ°Π·Π°Π½ΠΎ Π½Π° ΡΠΈΡΡΠ½ΠΊΠ΅ 1. ΠΠΎΠ³Π΄Π° ΠΏΠ΅ΡΠ΅ΠΊΠ»ΡΡΠ°ΡΠ΅Π»Ρ Π·Π°ΠΌΠΊΠ½ΡΡ, ΠΌΠ°Π³Π½ΠΈΡΠ½ΠΎΠ΅ ΠΏΠΎΠ»Π΅ ΡΠΎΠ·Π΄Π°Π΅ΡΡΡ Π² ΠΊΠ°ΡΡΡΠΊΠ΅ Π² Π²Π΅ΡΡ Π½Π΅ΠΉ ΡΠ°ΡΡΠΈ ΠΆΠ΅Π»Π΅Π·Π½ΠΎΠ³ΠΎ ΠΊΠΎΠ»ΡΡΠ° ΠΈ ΠΏΠ΅ΡΠ΅Π΄Π°Π΅ΡΡΡ Π½Π° ΠΊΠ°ΡΡΡΠΊΡ Π² Π½ΠΈΠΆΠ½Π΅ΠΉ ΡΠ°ΡΡΠΈ. ΡΠ°ΡΡΡ ΠΊΠΎΠ»ΡΡΠ°. ΠΠ°Π»ΡΠ²Π°Π½ΠΎΠΌΠ΅ΡΡ ΠΈΡΠΏΠΎΠ»ΡΠ·ΡΠ΅ΡΡΡ Π΄Π»Ρ ΠΎΠ±Π½Π°ΡΡΠΆΠ΅Π½ΠΈΡ Π»ΡΠ±ΠΎΠ³ΠΎ ΡΠΎΠΊΠ°, Π½Π°Π²Π΅Π΄Π΅Π½Π½ΠΎΠ³ΠΎ Π² ΠΊΠ°ΡΡΡΠΊΠ΅ Π½Π° Π΄Π½Π΅. ΠΡΠ»ΠΎ ΠΎΠ±Π½Π°ΡΡΠΆΠ΅Π½ΠΎ, ΡΡΠΎ ΠΊΠ°ΠΆΠ΄ΡΠΉ ΡΠ°Π·, ΠΊΠΎΠ³Π΄Π° ΠΏΠ΅ΡΠ΅ΠΊΠ»ΡΡΠ°ΡΠ΅Π»Ρ Π·Π°ΠΌΡΠΊΠ°Π΅ΡΡΡ, Π³Π°Π»ΡΠ²Π°Π½ΠΎΠΌΠ΅ΡΡ ΡΠ΅Π³ΠΈΡΡΡΠΈΡΡΠ΅Ρ ΡΠΎΠΊ Π² ΠΎΠ΄Π½ΠΎΠΌ Π½Π°ΠΏΡΠ°Π²Π»Π΅Π½ΠΈΠΈ Π² ΠΊΠ°ΡΡΡΠΊΠ΅ Π½Π° Π΄Π½Π΅. (ΠΡ ΡΠ°ΠΊΠΆΠ΅ ΠΌΠΎΠΆΠ΅ΡΠ΅ Π½Π°Π±Π»ΡΠ΄Π°ΡΡ ΡΡΠΎ Π² ΡΠΈΠ·ΠΈΡΠ΅ΡΠΊΠΎΠΉ Π»Π°Π±ΠΎΡΠ°ΡΠΎΡΠΈΠΈ.) ΠΠ°ΠΆΠ΄ΡΠΉ ΡΠ°Π·, ΠΊΠΎΠ³Π΄Π° ΠΏΠ΅ΡΠ΅ΠΊΠ»ΡΡΠ°ΡΠ΅Π»Ρ ΡΠ°Π·ΠΌΡΠΊΠ°Π΅ΡΡΡ, Π³Π°Π»ΡΠ²Π°Π½ΠΎΠΌΠ΅ΡΡ ΠΎΠ±Π½Π°ΡΡΠΆΠΈΠ²Π°Π΅Ρ ΡΠΎΠΊ Π² ΠΏΡΠΎΡΠΈΠ²ΠΎΠΏΠΎΠ»ΠΎΠΆΠ½ΠΎΠΌ Π½Π°ΠΏΡΠ°Π²Π»Π΅Π½ΠΈΠΈ. ΠΠ½ΡΠ΅ΡΠ΅ΡΠ½ΠΎ, ΡΡΠΎ Π΅ΡΠ»ΠΈ ΠΏΠ΅ΡΠ΅ΠΊΠ»ΡΡΠ°ΡΠ΅Π»Ρ ΠΎΡΡΠ°Π΅ΡΡΡ Π·Π°ΠΌΠΊΠ½ΡΡΡΠΌ ΠΈΠ»ΠΈ ΡΠ°Π·ΠΎΠΌΠΊΠ½ΡΡΡΠΌ ΠΊΠ°ΠΊΠΎΠ΅-ΡΠΎ Π²ΡΠ΅ΠΌΡ, ΡΠΎΠΊ ΡΠ΅ΡΠ΅Π· Π³Π°Π»ΡΠ²Π°Π½ΠΎΠΌΠ΅ΡΡ ΠΎΡΡΡΡΡΡΠ²ΡΠ΅Ρ. ΠΠ°ΠΌΡΠΊΠ°Π½ΠΈΠ΅ ΠΈ ΡΠ°Π·ΠΌΡΠΊΠ°Π½ΠΈΠ΅ ΠΏΠ΅ΡΠ΅ΠΊΠ»ΡΡΠ°ΡΠ΅Π»Ρ ΠΈΠ½Π΄ΡΡΠΈΡΡΠ΅Ρ ΡΠΎΠΊ. ΠΡΠΎ ΠΈΠ·ΠΌΠ΅Π½Π΅Π½ΠΈΠ΅ ΠΌΠ°Π³Π½ΠΈΡΠ½ΠΎΠ³ΠΎ ΠΏΠΎΠ»Ρ ΡΠΎΠ·Π΄Π°Π΅Ρ ΡΠΎΠΊ. ΠΠΎΠ»Π΅Π΅ ΠΏΡΠΎΡΡΡΠΌ, ΡΠ΅ΠΌ ΡΠ΅ΠΊΡΡΠΈΠΉ ΡΠΎΠΊ, ΡΠ²Π»ΡΠ΅ΡΡΡ ΠΠΠ‘ , ΠΊΠΎΡΠΎΡΠ°Ρ Π΅Π³ΠΎ Π²ΡΠ·ΡΠ²Π°Π΅Ρ. Π’ΠΎΠΊ ΡΠ²Π»ΡΠ΅ΡΡΡ ΡΠ΅Π·ΡΠ»ΡΡΠ°ΡΠΎΠΌ ΠΠΠ‘ , ΠΈΠ½Π΄ΡΡΠΈΡΠΎΠ²Π°Π½Π½ΠΎΠΉ ΠΈΠ·ΠΌΠ΅Π½ΡΡΡΠΈΠΌΡΡ ΠΌΠ°Π³Π½ΠΈΡΠ½ΡΠΌ ΠΏΠΎΠ»Π΅ΠΌ , Π½Π΅Π·Π°Π²ΠΈΡΠΈΠΌΠΎ ΠΎΡ ΡΠΎΠ³ΠΎ, Π΅ΡΡΡ Π»ΠΈ ΠΏΡΡΡ Π΄Π»Ρ ΠΏΡΠΎΡΠ΅ΠΊΠ°Π½ΠΈΡ ΡΠΎΠΊΠ°.
Π ΠΈΡ. 1. ΠΠΏΠΏΠ°ΡΠ°Ρ Π€Π°ΡΠ°Π΄Π΅Ρ Π΄Π»Ρ Π΄Π΅ΠΌΠΎΠ½ΡΡΡΠ°ΡΠΈΠΈ ΡΠΎΠ³ΠΎ, ΡΡΠΎ ΠΌΠ°Π³Π½ΠΈΡΠ½ΠΎΠ΅ ΠΏΠΎΠ»Π΅ ΠΌΠΎΠΆΠ΅Ρ ΠΏΡΠΎΠΈΠ·Π²ΠΎΠ΄ΠΈΡΡ ΡΠΎΠΊ. ΠΠ·ΠΌΠ΅Π½Π΅Π½ΠΈΠ΅ ΠΏΠΎΠ»Ρ, ΡΠΎΠ·Π΄Π°Π²Π°Π΅ΠΌΠΎΠ³ΠΎ Π²Π΅ΡΡ Π½Π΅ΠΉ ΠΊΠ°ΡΡΡΠΊΠΎΠΉ, ΠΈΠ½Π΄ΡΡΠΈΡΡΠ΅Ρ ΠΠΠ‘ ΠΈ, ΡΠ»Π΅Π΄ΠΎΠ²Π°ΡΠ΅Π»ΡΠ½ΠΎ, ΡΠΎΠΊ Π² Π½ΠΈΠΆΠ½Π΅ΠΉ ΠΊΠ°ΡΡΡΠΊΠ΅. ΠΠΎΠ³Π΄Π° ΠΏΠ΅ΡΠ΅ΠΊΠ»ΡΡΠ°ΡΠ΅Π»Ρ ΡΠ°Π·ΠΌΡΠΊΠ°Π΅ΡΡΡ ΠΈ Π·Π°ΠΌΡΠΊΠ°Π΅ΡΡΡ, Π³Π°Π»ΡΠ²Π°Π½ΠΎΠΌΠ΅ΡΡ ΡΠ΅Π³ΠΈΡΡΡΠΈΡΡΠ΅Ρ ΡΠΎΠΊΠΈ Π² ΠΏΡΠΎΡΠΈΠ²ΠΎΠΏΠΎΠ»ΠΎΠΆΠ½ΡΡ Π½Π°ΠΏΡΠ°Π²Π»Π΅Π½ΠΈΡΡ . Π§Π΅ΡΠ΅Π· Π³Π°Π»ΡΠ²Π°Π½ΠΎΠΌΠ΅ΡΡ Π½Π΅ ΠΏΡΠΎΡΠ΅ΠΊΠ°Π΅Ρ ΡΠΎΠΊ, ΠΊΠΎΠ³Π΄Π° ΠΏΠ΅ΡΠ΅ΠΊΠ»ΡΡΠ°ΡΠ΅Π»Ρ ΠΎΡΡΠ°Π΅ΡΡΡ Π·Π°ΠΌΠΊΠ½ΡΡΡΠΌ ΠΈΠ»ΠΈ ΡΠ°Π·ΠΎΠΌΠΊΠ½ΡΡΡΠΌ.ΠΠΊΡΠΏΠ΅ΡΠΈΠΌΠ΅Π½Ρ, ΠΊΠΎΡΠΎΡΡΠΉ Π»Π΅Π³ΠΊΠΎ Π²ΡΠΏΠΎΠ»Π½ΠΈΡΡ ΠΈ ΡΠ°ΡΡΠΎ ΠΏΡΠΎΠ²ΠΎΠ΄ΡΡ Π² ΡΠΈΠ·ΠΈΡΠ΅ΡΠΊΠΈΡ Π»Π°Π±ΠΎΡΠ°ΡΠΎΡΠΈΡΡ , ΠΏΠΎΠΊΠ°Π·Π°Π½ Π½Π° ΡΠΈΡΡΠ½ΠΊΠ΅ 2. ΠΠΠ‘ ΠΈΠ½Π΄ΡΡΠΈΡΡΠ΅ΡΡΡ Π² ΠΊΠ°ΡΡΡΠΊΠ΅, ΠΊΠΎΠ³Π΄Π° ΡΡΠ΅ΡΠΆΠ½Π΅Π²ΠΎΠΉ ΠΌΠ°Π³Π½ΠΈΡ Π²ΡΠ°Π»ΠΊΠΈΠ²Π°Π΅ΡΡΡ ΠΈ Π²ΡΡΠ°Π»ΠΊΠΈΠ²Π°Π΅ΡΡΡ ΠΈΠ· Π½Π΅Π΅. ΠΠΠ‘ ΡΠ°Π·Π½ΡΡ Π·Π½Π°ΠΊΠΎΠ² ΡΠΎΠ·Π΄Π°ΡΡΡΡ Π΄Π²ΠΈΠΆΠ΅Π½ΠΈΠ΅ΠΌ Π² ΠΏΡΠΎΡΠΈΠ²ΠΎΠΏΠΎΠ»ΠΎΠΆΠ½ΡΡ Π½Π°ΠΏΡΠ°Π²Π»Π΅Π½ΠΈΡΡ , Π° ΡΠ°ΠΊΠΆΠ΅ ΠΈΠ·ΠΌΠ΅Π½Π΅Π½ΠΈΠ΅ΠΌ ΠΏΠΎΠ»ΡΡΠ½ΠΎΡΡΠΈ ΠΠΠ‘ Π½Π° ΠΏΡΠΎΡΠΈΠ²ΠΎΠΏΠΎΠ»ΠΎΠΆΠ½ΠΎΠ΅. Π’Π΅ ΠΆΠ΅ ΡΠ΅Π·ΡΠ»ΡΡΠ°ΡΡ ΠΏΠΎΠ»ΡΡΠ°ΡΡΡΡ, Π΅ΡΠ»ΠΈ ΠΏΠ΅ΡΠ΅ΠΌΠ΅ΡΠ°ΡΡ ΠΊΠ°ΡΡΡΠΊΡ, Π° Π½Π΅ ΠΌΠ°Π³Π½ΠΈΡ β Π²Π°ΠΆΠ½ΠΎ ΠΎΡΠ½ΠΎΡΠΈΡΠ΅Π»ΡΠ½ΠΎΠ΅ Π΄Π²ΠΈΠΆΠ΅Π½ΠΈΠ΅. Π§Π΅ΠΌ Π±ΡΡΡΡΠ΅Π΅ Π΄Π²ΠΈΠΆΠ΅Π½ΠΈΠ΅, ΡΠ΅ΠΌ Π±ΠΎΠ»ΡΡΠ΅ ΠΠΠ‘, Π° ΠΊΠΎΠ³Π΄Π° ΠΌΠ°Π³Π½ΠΈΡ Π½Π΅ΠΏΠΎΠ΄Π²ΠΈΠΆΠ΅Π½ ΠΎΡΠ½ΠΎΡΠΈΡΠ΅Π»ΡΠ½ΠΎ ΠΊΠ°ΡΡΡΠΊΠΈ, ΠΠΠ‘ ΠΎΡΡΡΡΡΡΠ²ΡΠ΅Ρ.
Π ΠΈΡ. 2. ΠΠ²ΠΈΠΆΠ΅Π½ΠΈΠ΅ ΠΌΠ°Π³Π½ΠΈΡΠ° ΠΎΡΠ½ΠΎΡΠΈΡΠ΅Π»ΡΠ½ΠΎ ΠΊΠ°ΡΡΡΠΊΠΈ ΡΠΎΠ·Π΄Π°Π΅Ρ ΠΠΠ‘, ΠΊΠ°ΠΊ ΠΏΠΎΠΊΠ°Π·Π°Π½ΠΎ Π½Π° ΡΠΈΡΡΠ½ΠΊΠ΅. Π’Π°ΠΊΠΈΠ΅ ΠΆΠ΅ ΠΠΠ‘ Π²ΠΎΠ·Π½ΠΈΠΊΠ°ΡΡ, Π΅ΡΠ»ΠΈ ΠΊΠ°ΡΡΡΠΊΡ ΠΏΠ΅ΡΠ΅ΠΌΠ΅ΡΠ°ΡΡ ΠΎΡΠ½ΠΎΡΠΈΡΠ΅Π»ΡΠ½ΠΎ ΠΌΠ°Π³Π½ΠΈΡΠ°. Π§Π΅ΠΌ Π±ΠΎΠ»ΡΡΠ΅ ΡΠΊΠΎΡΠΎΡΡΡ, ΡΠ΅ΠΌ Π±ΠΎΠ»ΡΡΠ΅ Π²Π΅Π»ΠΈΡΠΈΠ½Π° ΠΠΠ‘, Π° ΠΠΠ‘ ΡΠ°Π²Π½Π° Π½ΡΠ»Ρ, ΠΊΠΎΠ³Π΄Π° Π½Π΅Ρ Π΄Π²ΠΈΠΆΠ΅Π½ΠΈΡ.ΠΠ΅ΡΠΎΠ΄ ΠΈΠ½Π΄ΡΠΊΡΠΈΠΈ ΠΠΠ‘, ΠΈΡΠΏΠΎΠ»ΡΠ·ΡΠ΅ΠΌΡΠΉ Π² Π±ΠΎΠ»ΡΡΠΈΠ½ΡΡΠ²Π΅ ΡΠ»Π΅ΠΊΡΡΠΈΡΠ΅ΡΠΊΠΈΡ Π³Π΅Π½Π΅ΡΠ°ΡΠΎΡΠΎΠ², ΠΏΠΎΠΊΠ°Π·Π°Π½ Π½Π° ΡΠΈΡΡΠ½ΠΊΠ΅ 3. ΠΠ°ΡΡΡΠΊΠ° Π²ΡΠ°ΡΠ°Π΅ΡΡΡ Π² ΠΌΠ°Π³Π½ΠΈΡΠ½ΠΎΠΌ ΠΏΠΎΠ»Π΅, ΡΠΎΠ·Π΄Π°Π²Π°Ρ ΠΠΠ‘ ΠΏΠ΅ΡΠ΅ΠΌΠ΅Π½Π½ΠΎΠ³ΠΎ ΡΠΎΠΊΠ°, ΠΊΠΎΡΠΎΡΠ°Ρ Π·Π°Π²ΠΈΡΠΈΡ ΠΎΡ ΡΠΊΠΎΡΠΎΡΡΠΈ Π²ΡΠ°ΡΠ΅Π½ΠΈΡ ΠΈ Π΄ΡΡΠ³ΠΈΡ ΡΠ°ΠΊΡΠΎΡΠΎΠ², ΠΊΠΎΡΠΎΡΡΠ΅ Π±ΡΠ΄ΡΡ ΠΈΡΡΠ»Π΅Π΄ΠΎΠ²Π°Π½Ρ Π² ΠΏΠΎΡΠ»Π΅Π΄ΡΡΡΠΈΡ ΡΠ°Π·Π΄Π΅Π»Π°Ρ . ΠΠ±ΡΠ°ΡΠΈΡΠ΅ Π²Π½ΠΈΠΌΠ°Π½ΠΈΠ΅, ΡΡΠΎ Π³Π΅Π½Π΅ΡΠ°ΡΠΎΡ ΠΎΡΠ΅Π½Ρ ΠΏΠΎΡ ΠΎΠΆ ΠΏΠΎ ΠΊΠΎΠ½ΡΡΡΡΠΊΡΠΈΠΈ Π½Π° Π΄Π²ΠΈΠ³Π°ΡΠ΅Π»Ρ (Π΅ΡΠ΅ ΠΎΠ΄Π½Π° ΡΠΈΠΌΠΌΠ΅ΡΡΠΈΡ).
Π ΠΈΡ. 3. ΠΡΠ°ΡΠ΅Π½ΠΈΠ΅ ΠΊΠ°ΡΡΡΠΊΠΈ Π² ΠΌΠ°Π³Π½ΠΈΡΠ½ΠΎΠΌ ΠΏΠΎΠ»Π΅ ΡΠΎΠ·Π΄Π°Π΅Ρ ΠΠΠ‘. ΠΡΠΎ ΠΎΡΠ½ΠΎΠ²Π½Π°Ρ ΠΊΠΎΠ½ΡΡΡΡΠΊΡΠΈΡ Π³Π΅Π½Π΅ΡΠ°ΡΠΎΡΠ°, Π² ΠΊΠΎΡΠΎΡΠΎΠΌ ΡΠ°Π±ΠΎΡΠ° ΠΏΠΎ Π²ΡΠ°ΡΠ΅Π½ΠΈΡ ΠΊΠ°ΡΡΡΠΊΠΈ ΠΏΡΠ΅ΠΎΠ±ΡΠ°Π·ΡΠ΅ΡΡΡ Π² ΡΠ»Π΅ΠΊΡΡΠΈΡΠ΅ΡΠΊΡΡ ΡΠ½Π΅ΡΠ³ΠΈΡ. ΠΠ±ΡΠ°ΡΠΈΡΠ΅ Π²Π½ΠΈΠΌΠ°Π½ΠΈΠ΅, Π³Π΅Π½Π΅ΡΠ°ΡΠΎΡ ΠΎΡΠ΅Π½Ρ ΠΏΠΎΡ ΠΎΠΆ ΠΏΠΎ ΠΊΠΎΠ½ΡΡΡΡΠΊΡΠΈΠΈ Π½Π° Π΄Π²ΠΈΠ³Π°ΡΠ΅Π»Ρ.ΠΡΠ°ΠΊ, ΠΌΡ Π²ΠΈΠ΄ΠΈΠΌ, ΡΡΠΎ ΠΈΠ·ΠΌΠ΅Π½Π΅Π½ΠΈΠ΅ Π²Π΅Π»ΠΈΡΠΈΠ½Ρ ΠΈΠ»ΠΈ Π½Π°ΠΏΡΠ°Π²Π»Π΅Π½ΠΈΡ ΠΌΠ°Π³Π½ΠΈΡΠ½ΠΎΠ³ΠΎ ΠΏΠΎΠ»Ρ ΡΠΎΠ·Π΄Π°Π΅Ρ ΠΠΠ‘. ΠΠΊΡΠΏΠ΅ΡΠΈΠΌΠ΅Π½ΡΡ ΠΏΠΎΠΊΠ°Π·Π°Π»ΠΈ, ΡΡΠΎ ΡΡΡΠ΅ΡΡΠ²ΡΠ΅Ρ ΡΠ΅ΡΠ°ΡΡΠ°Ρ Π²Π΅Π»ΠΈΡΠΈΠ½Π°, Π½Π°Π·ΡΠ²Π°Π΅ΠΌΠ°Ρ ΠΌΠ°Π³Π½ΠΈΡΠ½ΡΠΌ ΠΏΠΎΡΠΎΠΊΠΎΠΌ, ΠΎΠΏΡΠ΅Π΄Π΅Π»ΡΠ΅ΠΌΠ°Ρ ΠΊΠ°ΠΊ
Ο = BA cos ΞΈ
, Π³Π΄Π΅ B β Π½Π°ΠΏΡΡΠΆΠ΅Π½Π½ΠΎΡΡΡ ΠΌΠ°Π³Π½ΠΈΡΠ½ΠΎΠ³ΠΎ ΠΏΠΎΠ»Ρ Π½Π° ΠΏΠ»ΠΎΡΠ°Π΄ΠΈ A ΠΏΠΎΠ΄ ΡΠ³Π»ΠΎΠΌ ΞΈ , ΠΊΠ°ΠΊ ΠΏΠΎΠΊΠ°Π·Π°Π½ΠΎ Π½Π° ΡΠΈΡΡΠ½ΠΊΠ΅ 4. ΠΡΠ±ΠΎΠ΅ ΠΈΠ·ΠΌΠ΅Π½Π΅Π½ΠΈΠ΅ ΠΌΠ°Π³Π½ΠΈΡΠ½ΠΎΠ³ΠΎ ΠΏΠΎΡΠΎΠΊΠ° Ο ΠΈΠ½Π΄ΡΡΠΈΡΡΠ΅Ρ ΠΠΠ‘. ΠΡΠΎΡ ΠΏΡΠΎΡΠ΅ΡΡ ΠΎΠΏΡΠ΅Π΄Π΅Π»ΡΠ΅ΡΡΡ ΠΊΠ°ΠΊ ΡΠ»Π΅ΠΊΡΡΠΎΠΌΠ°Π³Π½ΠΈΡΠ½Π°Ρ ΠΈΠ½Π΄ΡΠΊΡΠΈΡ. ΠΠ΄ΠΈΠ½ΠΈΡΡ ΠΌΠ°Π³Π½ΠΈΡΠ½ΠΎΠ³ΠΎ ΠΏΠΎΡΠΎΠΊΠ° Ο ΡΠ°Π²Π½Ρ Π’Π»Β·ΠΌ 2 . ΠΠ°ΠΊ Π²ΠΈΠ΄Π½ΠΎ Π½Π° ΡΠΈΡΡΠ½ΠΊΠ΅ 4, , ΠΊΠΎΡΠΎΡΡΠΉ ΡΠ²Π»ΡΠ΅ΡΡΡ ΠΊΠΎΠΌΠΏΠΎΠ½Π΅Π½ΡΠΎΠΌ B , ΠΏΠ΅ΡΠΏΠ΅Π½Π΄ΠΈΠΊΡΠ»ΡΡΠ½ΡΠΌ ΠΏΠ»ΠΎΡΠ°Π΄ΠΈ A . Π’Π°ΠΊΠΈΠΌ ΠΎΠ±ΡΠ°Π·ΠΎΠΌ, ΠΌΠ°Π³Π½ΠΈΡΠ½ΡΠΉ ΠΏΠΎΡΠΎΠΊ ΡΠ°Π²Π΅Π½ , ΠΏΡΠΎΠΈΠ·Π²Π΅Π΄Π΅Π½ΠΈΡ ΠΏΠ»ΠΎΡΠ°Π΄ΠΈ ΠΈ ΡΠΎΡΡΠ°Π²Π»ΡΡΡΠ΅ΠΉ ΠΌΠ°Π³Π½ΠΈΡΠ½ΠΎΠ³ΠΎ ΠΏΠΎΠ»Ρ, ΠΏΠ΅ΡΠΏΠ΅Π½Π΄ΠΈΠΊΡΠ»ΡΡΠ½ΠΎΠΉ Π΅ΠΉ.
Π ΠΈΡΡΠ½ΠΎΠΊ 4. ΠΠ°Π³Π½ΠΈΡΠ½ΡΠΉ ΠΏΠΎΡΠΎΠΊ Ξ¦ ΡΠ²ΡΠ·Π°Π½ Ρ ΠΌΠ°Π³Π½ΠΈΡΠ½ΡΠΌ ΠΏΠΎΠ»Π΅ΠΌ ΠΈ ΠΏΠ»ΠΎΡΠ°Π΄ΡΡ, Π½Π°Π΄ ΠΊΠΎΡΠΎΡΠΎΠΉ ΠΎΠ½ΠΎ ΡΡΡΠ΅ΡΡΠ²ΡΠ΅Ρ. ΠΠΎΡΠΎΠΊ Ξ¦ = BA cos ΞΈ ΡΠ²ΡΠ·Π°Π½ Ρ ΠΈΠ½Π΄ΡΠΊΡΠΈΠ΅ΠΉ; Π»ΡΠ±ΠΎΠ΅ ΠΈΠ·ΠΌΠ΅Π½Π΅Π½ΠΈΠ΅ Ξ¦ ΠΈΠ½Π΄ΡΡΠΈΡΡΠ΅Ρ ΠΠΠ‘.ΠΡΡ ΠΈΠ½Π΄ΡΠΊΡΠΈΡ, Π²ΠΊΠ»ΡΡΠ°Ρ ΠΏΡΠΈΠΌΠ΅ΡΡ, ΠΏΡΠΈΠ²Π΅Π΄Π΅Π½Π½ΡΠ΅ Π΄ΠΎ ΡΠΈΡ ΠΏΠΎΡ, Π²ΠΎΠ·Π½ΠΈΠΊΠ°Π΅Ρ ΠΈΠ·-Π·Π° Π½Π΅ΠΊΠΎΡΠΎΡΠΎΠ³ΠΎ ΠΈΠ·ΠΌΠ΅Π½Π΅Π½ΠΈΡ ΠΌΠ°Π³Π½ΠΈΡΠ½ΠΎΠ³ΠΎ ΠΏΠΎΡΠΎΠΊΠ° Ο . ΠΠ°ΠΏΡΠΈΠΌΠ΅Ρ, Π€Π°ΡΠ°Π΄Π΅ΠΉ ΠΌΠ΅Π½ΡΠ» Π½Π° ΠΈ, ΡΠ»Π΅Π΄ΠΎΠ²Π°ΡΠ΅Π»ΡΠ½ΠΎ, Π½Π° ΠΏΡΠΈ ΡΠ°Π·ΠΌΡΠΊΠ°Π½ΠΈΠΈ ΠΈ Π·Π°ΠΌΡΠΊΠ°Π½ΠΈΠΈ ΠΏΠ΅ΡΠ΅ΠΊΠ»ΡΡΠ°ΡΠ΅Π»Ρ Π² ΡΠ²ΠΎΠ΅ΠΌ Π°ΠΏΠΏΠ°ΡΠ°ΡΠ΅ (ΠΏΠΎΠΊΠ°Π·Π°Π½Π½ΠΎΠΌ Π½Π° ΡΠΈΡ. 1). ΠΡΠΎ ΡΠ°ΠΊΠΆΠ΅ Π²Π΅ΡΠ½ΠΎ Π΄Π»Ρ ΡΡΠ΅ΡΠΆΠ½Π΅Π²ΠΎΠ³ΠΎ ΠΌΠ°Π³Π½ΠΈΡΠ° ΠΈ ΠΊΠ°ΡΡΡΠΊΠΈ, ΠΏΠΎΠΊΠ°Π·Π°Π½Π½ΡΡ Π½Π° ΡΠΈΡΡΠ½ΠΊΠ΅ 2. ΠΡΠΈ Π²ΡΠ°ΡΠ΅Π½ΠΈΠΈ ΠΊΠ°ΡΡΡΠΊΠΈ Π³Π΅Π½Π΅ΡΠ°ΡΠΎΡΠ° ΡΠ³ΠΎΠ» ΞΈ ΠΈ, ΡΠ»Π΅Π΄ΠΎΠ²Π°ΡΠ΅Π»ΡΠ½ΠΎ, Ο ΠΈΠ·ΠΌΠ΅Π½ΡΠ΅ΡΡΡ. Π’ΠΎ, Π½Π°ΡΠΊΠΎΠ»ΡΠΊΠΎ Π²Π΅Π»ΠΈΠΊΠ° ΠΠΠ‘ ΠΈ ΠΊΠ°ΠΊΠΎΠ΅ Π½Π°ΠΏΡΠ°Π²Π»Π΅Π½ΠΈΠ΅ ΠΎΠ½Π° ΠΏΡΠΈΠ½ΠΈΠΌΠ°Π΅Ρ, Π·Π°Π²ΠΈΡΠΈΡ ΠΎΡ ΠΈΠ·ΠΌΠ΅Π½Π΅Π½ΠΈΡ Ο ΠΈ ΠΎΡ ΡΠΎΠ³ΠΎ, Π½Π°ΡΠΊΠΎΠ»ΡΠΊΠΎ Π±ΡΡΡΡΠΎ ΠΏΡΠΎΠΈΡΡ ΠΎΠ΄ΠΈΡ ΡΡΠΎ ΠΈΠ·ΠΌΠ΅Π½Π΅Π½ΠΈΠ΅, ΠΊΠ°ΠΊ Π±ΡΠ΄Π΅Ρ ΡΠ°ΡΡΠΌΠΎΡΡΠ΅Π½ΠΎ Π² ΡΠ»Π΅Π΄ΡΡΡΠ΅ΠΌ ΡΠ°Π·Π΄Π΅Π»Π΅.
- Π Π΅ΡΠ°ΡΡΠ΅ΠΉ Π²Π΅Π»ΠΈΡΠΈΠ½ΠΎΠΉ Π² ΠΈΠ½Π΄ΡΠΊΡΠΈΠΈ ΡΠ²Π»ΡΠ΅ΡΡΡ ΠΌΠ°Π³Π½ΠΈΡΠ½ΡΠΉ ΠΏΠΎΡΠΎΠΊ Ο , ΠΎΠΏΡΠ΅Π΄Π΅Π»ΡΠ΅ΠΌΡΠΉ ΠΊΠ°ΠΊ Ο = BA cosΞΈ, , Π³Π΄Π΅ B β Π½Π°ΠΏΡΡΠΆΠ΅Π½Π½ΠΎΡΡΡ ΠΌΠ°Π³Π½ΠΈΡΠ½ΠΎΠ³ΠΎ ΠΏΠΎΠ»Ρ Π½Π° ΠΏΠ»ΠΎΡΠ°Π΄ΠΈ A ΠΏΠΎΠ΄ ΡΠ³Π»ΠΎΠΌ ΞΈ ΠΊ ΠΏΠ΅ΡΠΏΠ΅Π½Π΄ΠΈΠΊΡΠ»ΡΡΡ ΠΊ ΠΏΠ»ΠΎΡΠ°Π΄ΠΈ .
- ΠΠ΄ΠΈΠ½ΠΈΡΡ ΠΌΠ°Π³Π½ΠΈΡΠ½ΠΎΠ³ΠΎ ΠΏΠΎΡΠΎΠΊΠ° Ο ΡΠ²Π»ΡΡΡΡΡ Tβ’m 2 .
- ΠΡΠ±ΠΎΠ΅ ΠΈΠ·ΠΌΠ΅Π½Π΅Π½ΠΈΠ΅ ΠΌΠ°Π³Π½ΠΈΡΠ½ΠΎΠ³ΠΎ ΠΏΠΎΡΠΎΠΊΠ° Ο ΠΈΠ½Π΄ΡΡΠΈΡΡΠ΅Ρ ΠΠΠ‘ β ΠΏΡΠΎΡΠ΅ΡΡ ΠΎΠΏΡΠ΅Π΄Π΅Π»ΡΠ΅ΡΡΡ ΠΊΠ°ΠΊ ΡΠ»Π΅ΠΊΡΡΠΎΠΌΠ°Π³Π½ΠΈΡΠ½Π°Ρ ΠΈΠ½Π΄ΡΠΊΡΠΈΡ.
- ΠΌΠ°Π³Π½ΠΈΡΠ½ΡΠΉ ΠΏΠΎΡΠΎΠΊ
- Π²Π΅Π»ΠΈΡΠΈΠ½Π° ΠΌΠ°Π³Π½ΠΈΡΠ½ΠΎΠ³ΠΎ ΠΏΠΎΠ»Ρ, ΠΏΡΠΎΡ ΠΎΠ΄ΡΡΠ΅Π³ΠΎ ΡΠ΅ΡΠ΅Π· ΠΎΠΏΡΠ΅Π΄Π΅Π»Π΅Π½Π½ΡΡ ΠΏΠ»ΠΎΡΠ°Π΄Ρ, ΡΠ°ΡΡΡΠΈΡΠ°Π½Π½Π°Ρ ΠΏΠΎ ΡΠΎΡΠΌΡΠ»Π΅ Ο = BA cosΞΈ, Π³Π΄Π΅ B β Π½Π°ΠΏΡΡΠΆΠ΅Π½Π½ΠΎΡΡΡ ΠΌΠ°Π³Π½ΠΈΡΠ½ΠΎΠ³ΠΎ ΠΏΠΎΠ»Ρ Π½Π° ΠΏΠ»ΠΎΡΠ°Π΄ΠΈ A ΠΏΠΎΠ΄ ΡΠ³Π»ΠΎΠΌ ΞΈ ΠΊ ΠΏΠ΅ΡΠΏΠ΅Π½Π΄ΠΈΠΊΡΠ»ΡΡΡ ΠΊ ΠΏΠ»ΠΎΡΠ°Π΄ΠΈ
- ΡΠ»Π΅ΠΊΡΡΠΎΠΌΠ°Π³Π½ΠΈΡΠ½Π°Ρ ΠΈΠ½Π΄ΡΠΊΡΠΈΡ
- ΠΏΡΠΎΡΠ΅ΡΡ Π½Π°Π²Π΅Π΄Π΅Π½ΠΈΡ ΠΠΠ‘ (Π½Π°ΠΏΡΡΠΆΠ΅Π½ΠΈΡ) ΠΏΡΠΈ ΠΈΠ·ΠΌΠ΅Π½Π΅Π½ΠΈΠΈ ΠΌΠ°Π³Π½ΠΈΡΠ½ΠΎΠ³ΠΎ ΠΏΠΎΡΠΎΠΊΠ°
ΡΠ»Π΅ΠΊΡΡΠΎΠΌΠ°Π³Π½Π΅ΡΠΈΠ·ΠΌ — Π§ΡΠΎ ΡΠ°ΠΊΠΎΠ΅ ΠΎΠΏΡΠ΅Π΄Π΅Π»Π΅Π½ΠΈΠ΅ Π½Π°ΠΏΡΠ°Π²Π»Π΅Π½ΠΈΡ ΠΠΠ‘ ΠΈΠ½Π΄ΡΠΊΡΠΈΠΈ?
$\begingroup$
ΠΡΠΎΡ ΠΏΠΎΡΡ ΡΠΎΡΡΠΎΠΈΡ ΠΈΠ· Π΄Π²ΡΡ Π²ΠΎΠΏΡΠΎΡΠΎΠ², ΠΊΠ°ΠΆΠ΄ΡΠΉ ΠΈΠ· ΠΊΠΎΡΠΎΡΡΡ ΡΠ°ΠΊ ΠΈΠ»ΠΈ ΠΈΠ½Π°ΡΠ΅ ΠΎΡΠ½ΠΎΡΠΈΡΡΡ ΠΊ Π°ΡΠΏΠ΅ΠΊΡΠ°ΠΌ Π½Π°ΠΏΡΠ°Π²Π»Π΅Π½Π½ΠΎΡΡΠΈ ΠΠΠ‘. Π― Π½Π΅ ΡΠΌΠΎΠ³ Π²ΠΊΠ»ΡΡΠΈΡΡ ΠΎΠ±Π° Π²ΠΎΠΏΡΠΎΡΠ° Π² Π·Π°Π³ΠΎΠ»ΠΎΠ²ΠΎΠΊ, ΠΏΠΎΡΡΠΎΠΌΡ Π²ΡΠ±ΡΠ°Π» ΡΠ°ΠΌΡΠΉ Π²ΠΎΠ»Π½ΡΡΡΠΈΠΉ.
ΠΠ²Π΅Π΄Π΅Π½ΠΈΠ΅: Π§ΡΠΎΠ±Ρ ΠΏΠΎΠ½ΡΡΡ Π²ΠΎΠΏΡΠΎΡΡ, Π½Π°ΠΌ Π½ΡΠΆΠ½ΠΎ ΠΏΠΎΠ½ΡΡΡ Π·Π½Π°ΡΠ΅Π½ΠΈΠ΅ ΠΠΠ‘ ΠΈΠ½Π΄ΡΠΊΡΠΈΠΈ Π΄Π»Ρ ΠΏΠ΅ΡΠ»Ρ Π² ΡΠΎΡΡΠΎΡΠ½ΠΈΠΈ ΠΏΠΎΠΊΠΎΡ. ΠΠ»Ρ ΡΠ°ΠΊΠΎΠ³ΠΎ ΠΊΠΎΠ½ΡΡΡΠ° ΠΠΠ‘ ΠΈΠ½Π΄ΡΠΊΡΠΈΠΈ ΠΎΠΏΡΠ΅Π΄Π΅Π»ΡΠ΅ΡΡΡ ΠΊΠ°ΠΊ ΡΠ°Π±ΠΎΡΠ°, ΡΠΎΠ²Π΅ΡΡΠ°Π΅ΠΌΠ°Ρ ΡΠΈΡΡΡΠΌ ΠΠΠΊΠΎΠ½ΡΠ΅ΡΠ²Π°ΡΠΈΠ²Π½ΡΠΌ ΡΠ»Π΅ΠΊΡΡΠΈΡΠ΅ΡΠΊΠΈΠΌ ΠΏΠΎΠ»Π΅ΠΌ Π½Π°Π΄ Π·Π°ΡΡΠ΄ΠΎΠΌ, Π΄Π΅Π»Π΅Π½Π½Π°Ρ Π½Π° Π·Π°ΡΡΠ΄. Π‘Π»Π΅Π΄ΠΎΠ²Π°ΡΠ΅Π»ΡΠ½ΠΎ, ΡΡΠΎ ΡΠΊΠ°Π»ΡΡΠ½Π°Ρ Π²Π΅Π»ΠΈΡΠΈΠ½Π°. ΠΠ°ΡΠ΅ΠΌΠ°ΡΠΈΡΠ΅ΡΠΊΠΈ: $\epsilon =\oint_C\vec{E}\cdot d\vec{l}.$ ΠΠΎ ΡΡΠΎ ΡΠ°ΠΊΠΎΠ΅ $d\vec{l}?$ ΠΡΠΎ Π²Π΅ΠΊΡΠΎΡ, ΠΏΡΠ΅Π΄ΡΡΠ°Π²Π»ΡΡΡΠΈΠΉ ΡΠ»Π΅ΠΌΠ΅Π½Ρ Π½Π° ΠΏΠ΅ΡΠ»Π΅ Π΄Π»ΠΈΠ½Ρ $dl$ ΠΈ Π½Π°ΠΏΡΠ°Π²Π»Π΅Π½ΠΈΠ΅ Π΅Π³ΠΎ Π½Π°ΠΏΡΠ°Π²Π»Π΅Π½ΠΈΡ ΠΊΠ°ΡΠ°ΡΠ΅Π»ΡΠ½ΠΎ ΠΊ ΠΊΡΠΈΠ²ΠΎΠΉ. ΠΠΎ ΡΠ°ΠΊΠΈΡ Π½Π°ΠΏΡΠ°Π²Π»Π΅Π½ΠΈΠΉ Π΄Π²Π°, ΠΎΠ±Π° ΠΏΡΠΎΡΠΈΠ²ΠΎΠΏΠΎΠ»ΠΎΠΆΠ½ΡΠ΅ Π΄ΡΡΠ³ Π΄ΡΡΠ³Ρ (ΡΠΌ. ΡΠΈΡ.). Π‘Π»Π΅Π΄ΠΎΠ²Π°ΡΠ΅Π»ΡΠ½ΠΎ, ΠΌΡ ΠΌΠΎΠΆΠ΅ΠΌ Π²ΡΡΠΈΡΠ»ΠΈΡΡ ΠΠΠ‘ $\epsilon$ Π΄Π²ΡΠΌΡ ΡΠΏΠΎΡΠΎΠ±Π°ΠΌΠΈ; ΠΎΠ΄ΠΈΠ½ ΠΏΡΠΎΡΠΈΠ² ΡΠ°ΡΠΎΠ²ΠΎΠΉ ΡΡΡΠ΅Π»ΠΊΠΈ, Π° Π΄ΡΡΠ³ΠΎΠΉ ΠΏΡΠΎΡΠΈΠ² ΡΠ°ΡΠΎΠ²ΠΎΠΉ ΡΡΡΠ΅Π»ΠΊΠΈ. 9{nd}$ question: ΠΠΎ ΠΌΠ½ΠΎΠ³ΠΈΡ ΠΊΠ½ΠΈΠ³Π°Ρ ΠΈ ΡΡΠ°ΡΡΡΡ ΠΈΡΠΏΠΎΠ»ΡΠ·ΡΠ΅ΡΡΡ «Π½Π°ΠΏΡΠ°Π²Π»Π΅Π½ΠΈΠ΅ ΠΠΠ‘ ΠΈΠ½Π΄ΡΠΊΡΠΈΠΈ». Π§ΡΠΎ ΠΎΠ·Π½Π°ΡΠ°Π΅Ρ Π½Π°ΠΏΡΠ°Π²Π»Π΅Π½ΠΈΠ΅ ΠΠΠ‘ ΠΈΠ½Π΄ΡΠΊΡΠΈΠΈ? ΠΡΠΎ ΡΠΊΠ°Π»ΡΡΠ½Π°Ρ Π²Π΅Π»ΠΈΡΠΈΠ½Π° ΠΈ, ΡΠ»Π΅Π΄ΠΎΠ²Π°ΡΠ΅Π»ΡΠ½ΠΎ, Π½Π΅ ΠΈΠΌΠ΅Π΅Ρ Π½Π°ΠΏΡΠ°Π²Π»Π΅Π½ΠΈΡ Π² ΡΠΎΠΌ ΡΠΌΡΡΠ»Π΅, Π² ΠΊΠ°ΠΊΠΎΠΌ ΠΎΠ½ΠΎ Π΅ΡΡΡ Ρ Π²Π΅ΠΊΡΠΎΡΠΎΠ². Π§ΡΠΎ Π·Π΄Π΅ΡΡ ΠΎΠ·Π½Π°ΡΠ°Π΅Ρ Π½Π°ΠΏΡΠ°Π²Π»Π΅Π½ΠΈΠ΅? Π§ΡΠΎ ΡΡΠΎ ΠΎΠ·Π½Π°ΡΠ°Π΅Ρ Ρ ΡΠΈΠ·ΠΈΡΠ΅ΡΠΊΠΎΠΉ ΡΠΎΡΠΊΠΈ Π·ΡΠ΅Π½ΠΈΡ, ΠΊΠΎΠ³Π΄Π° ΠΌΡ Π³ΠΎΠ²ΠΎΡΠΈΠΌ, ΡΡΠΎ ΠΠΠ‘ ΠΈΠ½Π΄ΡΠΊΡΠΈΠΈ Π½Π°ΠΏΡΠ°Π²Π»Π΅Π½Π° ββΠΏΠΎ ΡΠ°ΡΠΎΠ²ΠΎΠΉ ΡΡΡΠ΅Π»ΠΊΠ΅, Π΅ΡΠ»ΠΈ ΡΠΌΠΎΡΡΠ΅ΡΡ ΠΈΠ· ΡΠΎΡΠΊΠΈ ΠΏΠ΅ΡΠ»ΠΈ? ΠΠ·Π½Π°ΡΠ°Π΅Ρ Π»ΠΈ ΠΠΠ‘ ΠΈΠ½Π΄ΡΠΊΡΠΈΠΈ Π·Π΄Π΅ΡΡ ΡΠΎΠΊ?
ΠΡΠΈΠΌΠ΅ΡΠ°Π½ΠΈΠ΅. ΠΠ΅ΡΠ²ΠΎΠ½Π°ΡΠ°Π»ΡΠ½ΠΎ ΡΡΠΎΡ Π²ΠΎΠΏΡΠΎΡ ΡΠ°ΠΊΠΆΠ΅ ΡΠΎΡΡΠΎΡΠ» ΠΈΠ· ΡΡΠΎΠ³ΠΎ Π²ΠΎΠΏΡΠΎΡΠ°, Π½ΠΎ Ρ ΠΎΠ±Π½Π°ΡΡΠΆΠΈΠ», ΡΡΠΎ ΠΎΠ½ Π΄Π΅Π»Π°Π΅Ρ ΠΌΠΎΠΉ Π²ΠΎΠΏΡΠΎΡ ΡΠ»ΠΈΡΠΊΠΎΠΌ Π΄Π»ΠΈΠ½Π½ΡΠΌ ΠΈ Π·Π°ΠΏΡΡΠ°Π½Π½ΡΠΌ.
- ΡΠ»Π΅ΠΊΡΡΠΎΠΌΠ°Π³Π½Π΅ΡΠΈΠ·ΠΌ
- ΠΏΠΎΡΠ΅Π½ΡΠΈΠ°Π»
- Π½Π°ΠΏΡΡΠΆΠ΅Π½ΠΈΠ΅
- ΠΎΠΏΡΠ΅Π΄Π΅Π»Π΅Π½ΠΈΠ΅
- ΡΠ»Π΅ΠΊΡΡΠΎΠΌΠ°Π³Π½ΠΈΡΠ½Π°Ρ ΠΈΠ½Π΄ΡΠΊΡΠΈΡ
$\endgroup$
0
$\begingroup$
ΠΡ ΠΌΠΎΠΆΠ΅ΡΠ΅ ΠΏΠΎΠ»ΡΡΠΈΡΡ ΠΠΠ‘ ΠΈΠ½Π΄ΡΠΊΡΠΈΠΈ Π±Π΅Π· ΡΠ΅Π·ΡΠ»ΡΡΠΈΡΡΡΡΠ΅Π³ΠΎ ΠΈΠ½Π΄ΡΡΠΈΡΠΎΠ²Π°Π½Π½ΠΎΠ³ΠΎ ΡΠΎΠΊΠ° ΠΈ, ΡΠ»Π΅Π΄ΠΎΠ²Π°ΡΠ΅Π»ΡΠ½ΠΎ, Π±Π΅Π· Β«ΠΏΡΠΎΡΠΈΠ²ΠΎΠ΄Π΅ΠΉΡΡΠ²ΠΈΡΒ», Π½ΠΎ ΡΠΎΠ³Π΄Π° ΡΠ°Π±ΠΎΡΠ° Π½Π΅ Π±ΡΠ΄Π΅Ρ Π²ΡΠΏΠΎΠ»Π½Π΅Π½Π°. Π’Π°ΠΊΠΈΠΌ ΠΎΠ±ΡΠ°Π·ΠΎΠΌ, Π»ΡΠ±ΠΎΠ΅ ΡΠΏΠΎΠΌΠΈΠ½Π°Π½ΠΈΠ΅ ΠΎΠ± ΠΈΠ½Π΄ΡΡΠΈΡΠΎΠ²Π°Π½Π½ΠΎΠΌ ΡΠΎΠΊΠ΅ ΡΠ»Π΅Π΄ΡΠ΅Ρ ΡΠ°ΡΡΠΌΠ°ΡΡΠΈΠ²Π°ΡΡ ΠΊΠ°ΠΊ ΡΠ°ΡΡΠΌΠΎΡΡΠ΅Π½ΠΈΠ΅ ΡΠΎΠ³ΠΎ, ΡΡΠΎ ΠΌΠΎΠΆΠ΅Ρ ΠΏΡΠΎΠΈΠ·ΠΎΠΉΡΠΈ, Π΅ΡΠ»ΠΈ ΠΏΠΎΠ·Π²ΠΎΠ»ΠΈΡΡ ΠΏΡΠΎΡΠ΅ΠΊΠ°ΡΡ ΠΈΠ½Π΄ΡΡΠΈΡΠΎΠ²Π°Π½Π½ΠΎΠΌΡ ΡΠΎΠΊΡ.
ΠΠ°Ρ Π²ΠΎΠΏΡΠΎΡ ΠΎ Π½Π°ΠΏΡΠ°Π²Π»Π΅Π½ΠΈΠΈ ΡΠ΅ΡΠ΅Π½, Π΅ΡΠ»ΠΈ Π²Ρ Π±ΡΠ΄Π΅ΡΠ΅ ΡΠ»Π΅Π΄ΠΎΠ²Π°ΡΡ ΠΏΡΠ°Π²ΠΈΠ»Ρ ΠΏΡΠ°Π²ΠΎΠΉ ΡΡΠΊΠΈ, ΠΊΠ°ΠΊ ΠΏΠΎΠΊΠ°Π·Π°Π½ΠΎ Π½Π° Π΄ΠΈΠ°Π³ΡΠ°ΠΌΠΌΠ΅ Π½ΠΈΠΆΠ΅.
ΠΠΏΡΠ΅Π΄Π΅Π»ΠΈΡΡ Π½Π°ΠΏΡΠ°Π²Π»Π΅Π½ΠΈΠ΅ Π΅Π΄ΠΈΠ½ΠΈΡΠ½ΠΎΠ³ΠΎ Π²Π΅ΠΊΡΠΎΡΠ° Π½ΠΎΡΠΌΠ°Π»ΠΈ ΠΊ ΠΏΠ»ΠΎΡΠ°Π΄ΠΈ $\hat n$, Π° Π·Π°ΡΠ΅ΠΌ ΠΎΠΏΡΠ΅Π΄Π΅Π»ΠΈΡΡ ΠΏΠΎΠ»ΠΎΠΆΠΈΡΠ΅Π»ΡΠ½ΠΎΠ΅ Π½Π°ΠΏΡΠ°Π²Π»Π΅Π½ΠΈΠ΅ Π»ΠΈΠ½Π΅ΠΉΠ½ΠΎΠ³ΠΎ ΠΈΠ½ΡΠ΅Π³ΡΠ°Π»Π° Π²ΠΎΠΊΡΡΠ³ ΠΏΠ΅ΡΠ»ΠΈ $C$ ΠΏΠΎ ΠΏΡΠ°Π²ΠΈΠ»Ρ ΠΏΡΠ°Π²ΠΎΠΉ ΡΡΠΊΠΈ, Π½Π°ΠΏΡΠΈΠΌΠ΅Ρ, Π±ΠΎΠ»ΡΡΠΈΠΌ ΠΏΠ°Π»ΡΡΠ΅ΠΌ ΠΏΡΠ°Π²ΠΎΠΉ ΡΡΠΊΠΈ Π² Π½Π°ΠΏΡΠ°Π²Π»Π΅Π½ΠΈΠΈ Π΅Π΄ΠΈΠ½ΠΈΡΠ½ΠΎΠ³ΠΎ Π²Π΅ΠΊΡΠΎΡΠ° Π½ΠΎΡΠΌΠ°Π»ΠΈ, Π° ΡΠΎΠ³Π½ΡΡΡΠ΅ ΠΏΠ°Π»ΡΡΡ ΠΏΡΠ°Π²ΠΎΠΉ ΡΡΠΊΠΈ Π΄Π°ΡΡ ΠΏΠΎΠ»ΠΎΠΆΠΈΡΠ΅Π»ΡΠ½ΠΎΠ΅ Π½Π°ΠΏΡΠ°Π²Π»Π΅Π½ΠΈΠ΅ Π»ΠΈΠ½Π΅ΠΉΠ½ΠΎΠ³ΠΎ ΠΈΠ½ΡΠ΅Π³ΡΠ°Π»Π°.
ΠΡΠ΅Π΄ΠΏΠΎΠ»ΠΎΠΆΠΈΠΌ, ΡΡΠΎ ΠΌΠ°Π³Π½ΠΈΡΠ½ΠΎΠ΅ ΠΏΠΎΠ»Π΅ Π½Π°ΠΏΡΠ°Π²Π»Π΅Π½ΠΎ Π² ΡΡΠΎΡΠΎΠ½Ρ $\hat n$ ΠΈ ΡΠ²Π΅Π»ΠΈΡΠΈΠ²Π°Π΅ΡΡΡ, ΡΠΎΠ³Π΄Π° $\vec B\cdot \hat n$ ΠΏΠΎΠ»ΠΎΠΆΠΈΡΠ΅Π»ΡΠ½ΠΎ, ΠΏΠΎΡΡΠΎΠΌΡ ΠΏΡΠ°Π²Π°Ρ ΡΠ°ΡΡΡ Π·Π°ΠΊΠΎΠ½Π° Π€Π°ΡΠ°Π΄Π΅Ρ, $-\frac {d\ phi}{dt}$ Π±ΡΠ΄Π΅Ρ ΠΎΡΡΠΈΡΠ°ΡΠ΅Π»ΡΠ½ΡΠΌ.
ΠΠΈΠ½Π΅ΠΉΠ½ΡΠΉ ΠΈΠ½ΡΠ΅Π³ΡΠ°Π» Π² Π»Π΅Π²ΠΎΠΉ ΡΠ°ΡΡΠΈ, ΠΠΠ‘ $(\displaystyle \oint _{\rm C} \vec E\cdot d \vec l)$, ΡΠ°ΠΊΠΈΠΌ ΠΎΠ±ΡΠ°Π·ΠΎΠΌ, Π±ΡΠ΄Π΅Ρ ΠΎΡΡΠΈΡΠ°ΡΠ΅Π»ΡΠ½ΡΠΌ, Ρ.Π΅. ΠΏΠΎ ΡΠ°ΡΠΎΠ²ΠΎΠΉ ΡΡΡΠ΅Π»ΠΊΠ΅, Π΅ΡΠ»ΠΈ ΡΠΌΠΎΡΡΠ΅ΡΡ ΡΠ²Π΅ΡΡ
Ρ Π² Π΄ΠΈΠ°Π³ΡΠ°ΠΌΠΌΠ°.
ΠΡΠΎ ΡΠ°ΠΊΠΆΠ΅ Π±ΡΠ΄Π΅Ρ Π½Π°ΠΏΡΠ°Π²Π»Π΅Π½ΠΈΠ΅ ΠΈΠ½Π΄ΡΡΠΈΡΠΎΠ²Π°Π½Π½ΠΎΠ³ΠΎ ΡΠΎΠΊΠ°, ΠΊΠΎΡΠΎΡΠΎΠ΅ ΡΠΎΠ³Π»Π°ΡΡΠ΅ΡΡΡ Ρ ΠΠ΅Π½ΡΠ΅ΠΌ.
$\displaystyle \oint_{\rm C} \vec E\cdot d \vec l$ — ΡΠ°Π±ΠΎΡΠ°, ΡΠΎΠ²Π΅ΡΡΠ°Π΅ΠΌΠ°Ρ ΡΠ»Π΅ΠΊΡΡΠΈΡΠ΅ΡΠΊΠΈΠΌ ΠΏΠΎΠ»Π΅ΠΌ Π½Π°Π΄ Π΅Π΄ΠΈΠ½ΠΈΡΠ½ΡΠΌ ΠΏΠΎΠ»ΠΎΠΆΠΈΡΠ΅Π»ΡΠ½ΡΠΌ Π·Π°ΡΡΠ΄ΠΎΠΌ, ΠΊΠΎΠ³Π΄Π° Π·Π°ΡΡΠ΄ ΠΏΡΠΎΡ
ΠΎΠ΄ΠΈΡ ΠΏΠΎΠ»Π½ΡΠΉ ΠΊΠΎΠ½ΡΡΡ, Π½Π°Ρ
ΠΎΠ΄ΡΡΠΈΠΉΡΡ Π² ΡΠΎΡΡΠΎΡΠ½ΠΈΠΈ ΠΏΠΎΠΊΠΎΡ.
Π ΠΌΠΎΠ΅ΠΌ ΠΏΡΠΈΠΌΠ΅ΡΠ΅ Π½Π°ΠΏΡΠ°Π²Π»Π΅Π½ΠΈΠ΅ ΡΠ»Π΅ΠΊΡΡΠΈΡΠ΅ΡΠΊΠΎΠ³ΠΎ ΠΏΠΎΠ»Ρ ΠΏΠΎ ΡΠ°ΡΠΎΠ²ΠΎΠΉ ΡΡΡΠ΅Π»ΠΊΠ΅, ΡΡΠΎ ΠΎΠ·Π½Π°ΡΠ°Π΅Ρ, ΡΡΠΎ ΠΈΠ½Π΄ΡΡΠΈΡΠΎΠ²Π°Π½Π½ΡΠΉ ΡΠΎΠΊ (Π΄Π²ΠΈΠΆΠ΅Π½ΠΈΠ΅ ΠΏΠΎΠ»ΠΎΠΆΠΈΡΠ΅Π»ΡΠ½ΡΡ
Π·Π°ΡΡΠ΄ΠΎΠ²) Π½Π°ΠΏΡΠ°Π²Π»Π΅Π½ ΠΏΠΎ ΡΠ°ΡΠΎΠ²ΠΎΠΉ ΡΡΡΠ΅Π»ΠΊΠ΅.
ΠΡΠΎ Π²ΡΠ·Π²Π°Π½ΠΎ ΠΠΠ‘ ΠΈΠ½Π΄ΡΠΊΡΠΈΠΈ, ΠΊΠΎΡΠΎΡΠ°Ρ ΡΠ°ΠΊΠΆΠ΅ Π½Π°ΠΏΡΠ°Π²Π»Π΅Π½Π° ββΠΏΠΎ ΡΠ°ΡΠΎΠ²ΠΎΠΉ ΡΡΡΠ΅Π»ΠΊΠ΅, ΠΏΠΎΡΠΊΠΎΠ»ΡΠΊΡ ΠΎΠ½Π° ΠΏΡΠΈΠ²ΠΎΠ΄ΠΈΡ Π² Π΄Π²ΠΈΠΆΠ΅Π½ΠΈΠ΅ ΠΏΠΎΠ»ΠΎΠΆΠΈΡΠ΅Π»ΡΠ½ΡΠ΅ Π·Π°ΡΡΠ΄Ρ ΡΠ°ΠΊΠΈΠΌ ΠΎΠ±ΡΠ°Π·ΠΎΠΌ, Ρ.Π΅. Π² Π½Π°ΠΏΡΠ°Π²Π»Π΅Π½ΠΈΠΈ, ΠΏΡΠΎΡΠΈΠ²ΠΎΠΏΠΎΠ»ΠΎΠΆΠ½ΠΎΠΌ ΡΡΡΠ΅Π»ΠΊΠ΅ Ρ $C$ ΡΡΠ΄ΠΎΠΌ Ρ Π½Π΅ΠΉ.
$\endgroup$
2
$\begingroup$
ΠΠ±Π° ΠΌΠ΅ΡΠΎΠ΄Π°/Π·Π°ΠΊΠΎΠ½Π° ΠΏΡΠΈΠΌΠ΅Π½ΠΈΠΌΡ Π΄Π»Ρ ΠΎΠΏΡΠ΅Π΄Π΅Π»Π΅Π½ΠΈΡ Π½Π°ΠΏΡΠ°Π²Π»Π΅Π½ΠΈΡ. ΠΠΎ ΠΌΠ½Π΅, Π΅ΡΡΡ ΡΠΎΠ»ΡΠΊΠΎ ΠΎΠ΄Π½ΠΎ ΠΎΡΠ»ΠΈΡΠΈΠ΅. ΠΠ΅ΡΠ²ΡΠΉ ΠΌΠ΅ΡΠΎΠ΄, ΠΈΡΠΏΠΎΠ»ΡΠ·ΡΡΡΠΈΠΉ ΠΏΡΠ°Π²ΠΈΠ»ΠΎ Π»Π΅Π²ΠΎΠΉ ΡΡΠΊΠΈ, ΠΌΠΎΠΆΠ½ΠΎ Π½Π°Π·Π²Π°ΡΡ Β«ΡΠ»ΠΎΠ²ΠΊΠΎΠΉΒ», ΠΏΠΎΡΠΊΠΎΠ»ΡΠΊΡ ΠΎΠ½ Π½Π΅ Π΄Π°Π΅Ρ ΠΎΠ±ΡΡΡΠ½Π΅Π½ΠΈΡ, ΠΏΠΎΡΠ΅ΠΌΡ ΠΎΠ½ ΡΠ°Π±ΠΎΡΠ°Π΅Ρ. Π‘ Π΄ΡΡΠ³ΠΎΠΉ ΡΡΠΎΡΠΎΠ½Ρ, Π·Π°ΠΊΠΎΠ½ ΠΠ΅Π½ΡΠ° ΠΎΠ±ΡΡΡΠ½ΡΠ΅Ρ, ΠΏΠΎΡΠ΅ΠΌΡ ΠΈΠ½Π΄ΡΡΠΈΡΠΎΠ²Π°Π½Π½ΡΠΉ ΡΠΎΠΊ Π΄ΠΎΠ»ΠΆΠ΅Π½ ΠΏΡΠ΅ΠΏΡΡΡΡΠ²ΠΎΠ²Π°ΡΡ ΠΈΠ·ΠΌΠ΅Π½Π΅Π½ΠΈΡ ΠΌΠ°Π³Π½ΠΈΡΠ½ΠΎΠ³ΠΎ ΠΏΠΎΠ»Ρ (ΡΠΎΡ ΡΠ°Π½Π΅Π½ΠΈΠ΅ ΡΠ½Π΅ΡΠ³ΠΈΠΈ).
ΠΠ°ΠΏΡΠ°Π²Π»Π΅Π½ΠΈΠ΅ ΠΠΠ‘ ΡΠΈΠ·ΠΈΡΠ΅ΡΠΊΠΈ ΡΠ²ΡΠ·Π°Π½ΠΎ Ρ Π½Π°ΠΏΡΠ°Π²Π»Π΅Π½ΠΈΠ΅ΠΌ ΠΈΠ½Π΄ΡΠΊΡΠΈΠΎΠ½Π½ΠΎΠ³ΠΎ ΡΠΎΠΊΠ°. Π― Π½Π΅ ΠΏΠΎΠ½ΠΈΠΌΠ°Ρ, Π² ΡΠ΅ΠΌ Π·Π°ΠΊΠ»ΡΡΠ°Π΅ΡΡΡ Π²Π°Ρ Π²ΠΎΠΏΡΠΎΡ, ΠΏΠΎΡΡΠΎΠΌΡ Ρ Π½Π΅ Π·Π½Π°Ρ, ΠΎΡΠ²Π΅ΡΠ°Π΅Ρ Π»ΠΈ ΡΡΠΎ Π½Π° Π²Π°Ρ Π²ΠΎΠΏΡΠΎΡ.
ΠΡ ΠΌΠΎΠΆΠ΅ΡΠ΅ ΠΏΡΠΈΠ½ΡΡΡ Π½Π°ΠΏΡΠ°Π²Π»Π΅Π½ΠΈΠ΅ $d \vec l$ Π² Π»ΡΠ±ΠΎΠΌ ΠΈΠ· Π΄Π²ΡΡ Π½Π°ΠΏΡΠ°Π²Π»Π΅Π½ΠΈΠΉ. ΠΡΠ»ΠΈ Π²Ρ Π²ΡΠ±Π΅ΡΠ΅ΡΠ΅ ΠΎΠΏΡΠ΅Π΄Π΅Π»Π΅Π½Π½ΠΎΠ΅ Π½Π°ΠΏΡΠ°Π²Π»Π΅Π½ΠΈΠ΅, ΡΠΎ Π²Ρ Π±ΡΠ΄Π΅ΡΠ΅ Π·Π½Π°ΡΡ Π½Π°ΠΏΡΠ°Π²Π»Π΅Π½ΠΈΠ΅ $d\vec A$ ΠΏΡΠΈ ΡΠ°ΡΡΠ΅ΡΠ΅ ΠΌΠ°Π³Π½ΠΈΡΠ½ΠΎΠ³ΠΎ ΠΏΠΎΡΠΎΠΊΠ° ΠΏΠΎ ΠΏΡΠ°Π²ΠΈΠ»Ρ ΠΏΡΠ°Π²ΠΎΠΉ ΡΡΠΊΠΈ. ΠΡΠΎ Π±ΡΠ΄Π΅Ρ Π²Π΄ΠΎΠ»Ρ Π±ΠΎΠ»ΡΡΠΎΠ³ΠΎ ΠΏΠ°Π»ΡΡΠ°, Π΅ΡΠ»ΠΈ ΡΠΎΠ³Π½ΡΡΡ ΠΏΠ°Π»ΡΡΡ Π²Π΄ΠΎΠ»Ρ ΠΏΠ΅ΡΠ»ΠΈ Π² Π½Π°ΠΏΡΠ°Π²Π»Π΅Π½ΠΈΠΈ $d \vec l$. Π’Π΅ΠΏΠ΅ΡΡ Π²Ρ ΠΌΠΎΠΆΠ΅ΡΠ΅ Π²ΡΡΠΈΡΠ»ΠΈΡΡ ΡΠΊΠ°Π»ΡΡΠ½ΡΠ΅ ΠΏΡΠΎΠΈΠ·Π²Π΅Π΄Π΅Π½ΠΈΡ Π² $\oint_C\vec{E}\cdot d\vec{l}=-\frac{d\phi_m}{dt}$, ΠΈ ΠΎΠ±Π° ΡΠΏΠΎΡΠΎΠ±Π° ΠΎΠ±Π΅ΡΠΏΠ΅ΡΠ°Ρ ΠΏΡΠ°Π²ΠΈΠ»ΡΠ½ΠΎΠ΅ Π½Π°ΠΏΡΠ°Π²Π»Π΅Π½ΠΈΠ΅.
$\endgroup$
3
$\begingroup$
- ΠΠ±Π° Π·Π°ΠΊΠΎΠ½Π° Π²Π΅ΡΠ½Ρ. ΠΠ½Π°ΠΊ -ve Π² Π·Π°ΠΊΠΎΠ½Π΅ Π€Π°ΡΠ°Π΄Π΅Ρ ΡΠΊΠ°Π·ΡΠ²Π°Π΅Ρ Π½Π°ΠΏΡΠ°Π²Π»Π΅Π½ΠΈΠ΅ ΠΠΠ‘. Π ΡΠΎ Π²ΡΠ΅ΠΌΡ ΠΊΠ°ΠΊ ΠΠ΅Π½ΡΠ° ΡΡΠ²Π΅ΡΠΆΠ΄Π°Π΅Ρ, ΡΡΠΎ Π½Π°ΠΏΡΠ°Π²Π»Π΅Π½ΠΈΠ΅ ΠΈΠ½Π΄ΡΡΠΈΡΡΠ΅ΠΌΠΎΠΉ ΠΠΠ‘ ΡΠ°ΠΊΠΎΠ²ΠΎ, ΡΡΠΎ ΠΎΠ½ΠΎ ΠΏΡΠΎΡΠΈΠ²ΠΎΡΡΠΎΠΈΡ ΠΏΡΠΈΡΠΈΠ½Π΅, Π²ΡΠ·Π²Π°Π²ΡΠ΅ΠΉ ΡΠ΄Ρ. ΠΡΠ°ΠΊ, Π·Π°ΠΊΠΎΠ½ ΠΠ΅Π½ΡΠ° Π΄Π°Π΅Ρ Π½Π°ΠΌ Π½Π°ΠΏΡΠ°Π²Π»Π΅Π½ΠΈΠ΅ ΠΈΠ½Π΄ΡΡΠΈΡΠΎΠ²Π°Π½Π½ΠΎΠΉ ΠΠΠ‘.
ΠΠ°ΠΏΡΠΈΠΌΠ΅Ρ, ΠΏΡΠ΅Π΄ΠΏΠΎΠ»ΠΎΠΆΠΈΠΌ, ΡΡΠΎ ΡΠΎΠΊ Π² ΠΏΡΠΎΠ²ΠΎΠ΄ΡΡΠ΅ΠΉ ΠΏΠ΅ΡΠ»Π΅ ΡΠ²Π΅Π»ΠΈΡΠΈΠ²Π°Π΅ΡΡΡ ΡΠΎ Π²ΡΠ΅ΠΌΠ΅Π½Π΅ΠΌ. Π’Π°ΠΊΠΈΠΌ ΠΎΠ±ΡΠ°Π·ΠΎΠΌ, ΠΌΠ°Π³Π½ΠΈΡΠ½ΡΠΉ ΠΏΠΎΡΠΎΠΊ ΡΠ΅ΡΠ΅Π· ΠΏΠ΅ΡΠ»Ρ Π±ΡΠ΄Π΅Ρ ΡΠ²Π΅Π»ΠΈΡΠΈΠ²Π°ΡΡΡΡ, ΠΈ, ΡΠ»Π΅Π΄ΠΎΠ²Π°ΡΠ΅Π»ΡΠ½ΠΎ, Π² ΠΏΠ΅ΡΠ»Π΅ Π±ΡΠ΄Π΅Ρ ΠΈΠ½Π΄ΡΡΠΈΡΠΎΠ²Π°ΡΡΡΡ ΠΠΠ‘. Π’ΠΎΠ³Π΄Π° ΠΏΠΎ Π·Π°ΠΊΠΎΠ½Ρ ΠΠ΅Π½ΡΠ° Π½Π°ΠΏΡΠ°Π²Π»Π΅Π½ΠΈΠ΅ ΠΠΠ‘ Π±ΡΠ΄Π΅Ρ ΠΏΡΠΎΡΠΈΠ²ΠΎΠ΄Π΅ΠΉΡΡΠ²ΠΎΠ²Π°ΡΡ ΡΠ²Π΅Π»ΠΈΡΠ΅Π½ΠΈΡ ΡΠΎΠΊΠ°. Π’Π°ΠΊΠΈΠΌ ΠΎΠ±ΡΠ°Π·ΠΎΠΌ, ΡΠΎΠΊ, ΡΠΎΠ·Π΄Π°Π²Π°Π΅ΠΌΡΠΉ (ΠΈΠ½Π΄ΡΡΠΈΡΠΎΠ²Π°Π½Π½ΡΠΉ ΡΠΎΠΊ) ΠΈΠ·-Π·Π° ΠΈΠ½Π΄ΡΡΠΈΡΠΎΠ²Π°Π½Π½ΠΎΠΉ ΠΠΠ‘, Π±ΡΠ΄Π΅Ρ ΠΏΡΠΎΡΠΈΠ²ΠΎΠΏΠΎΠ»ΠΎΠΆΠ΅Π½ ΡΠ°ΠΊΡΠΈΡΠ΅ΡΠΊΠΎΠΌΡ Π½Π°ΠΏΡΠ°Π²Π»Π΅Π½ΠΈΡ ΡΠΎΠΊΠ°. Π§ΡΠΎ Π² ΡΠ΅Π·ΡΠ»ΡΡΠ°ΡΠ΅ ΡΠΎΠ·Π΄Π°Π΅Ρ ΠΌΠ°Π³Π½ΠΈΡΠ½ΠΎΠ΅ ΠΏΠΎΠ»Π΅, ΠΏΡΠΎΡΠΈΠ²ΠΎΠΏΠΎΠ»ΠΎΠΆΠ½ΠΎΠ΅ ΡΠ²Π΅Π»ΠΈΡΠΈΠ²Π°ΡΡΠ΅ΠΌΡΡΡ ΠΌΠ°Π³Π½ΠΈΡΠ½ΠΎΠΌΡ ΠΏΠΎΠ»Ρ, ΡΠΎΠ·Π΄Π°Π²Π°Π΅ΠΌΠΎΠΌΡ ΠΈΠ·-Π·Π° ΡΠ°ΠΊΡΠΈΡΠ΅ΡΠΊΠΎΠ³ΠΎ ΡΠ²Π΅Π»ΠΈΡΠ΅Π½ΠΈΡ ΡΠΎΠΊΠ°. Π’Π°ΠΊΠΈΠΌ ΠΎΠ±ΡΠ°Π·ΠΎΠΌ, ΠΎΠ±Π° Π·Π°ΠΊΠΎΠ½Π° ΡΠΊΠ°Π·ΡΠ²Π°ΡΡ ΠΎΠ΄Π½ΠΎ ΠΈ ΡΠΎ ΠΆΠ΅ Π½Π°ΠΏΡΠ°Π²Π»Π΅Π½ΠΈΠ΅, Π½ΠΎ Π·Π°ΠΊΠΎΠ½ Π€Π°ΡΠ°Π΄Π΅Ρ ΡΠ°ΠΊΠΆΠ΅ Π΄Π°Π΅Ρ Π²Π΅Π»ΠΈΡΠΈΠ½Ρ ΠΈΠ½Π΄ΡΡΠΈΡΡΠ΅ΠΌΠΎΠΉ ΠΠΠ‘.
ΠΠΎΠ΄ Π½Π°ΠΏΡΠ°Π²Π»Π΅Π½ΠΈΠ΅ΠΌ ΠΌΡ ΠΏΡΠΎΡΡΠΎ ΠΏΠΎΠ΄ΡΠ°Π·ΡΠΌΠ΅Π²Π°Π΅ΠΌ ΠΏΠΎΠ»ΡΡΠ½ΠΎΡΡΡ ΠΈΠ½Π΄ΡΡΠΈΡΠΎΠ²Π°Π½Π½ΠΎΠ³ΠΎ ΠΠΠ‘. ΠΡΠ»ΠΈ ΠΌΡ ΠΏΡΠ΅Π΄ΠΏΠΎΠ»ΠΎΠΆΠΈΠΌ, ΡΡΠΎ ΠΠΠ‘ ΠΈΠ½Π΄ΡΠΊΡΠΈΠΈ ΠΏΠΎΡ ΠΎΠΆΠ° Π½Π° Π±Π°ΡΠ°ΡΠ΅Ρ, ΡΠΎ Π΅Π΅ Π±ΠΎΠ»Π΅Π΅ Π²ΡΡΠΎΠΊΠ°Ρ ΠΏΠΎΠ»ΡΡΠ½ΠΎΡΡΡ Π±ΡΠ΄Π΅Ρ Π² Π½Π°ΠΏΡΠ°Π²Π»Π΅Π½ΠΈΠΈ ΡΠΎΠΊΠ°, ΠΈΠ½Π΄ΡΡΠΈΡΠΎΠ²Π°Π½Π½ΠΎΠ³ΠΎ Π΄Π»Ρ ΠΏΡΠΎΡΠΈΠ²ΠΎΠ΄Π΅ΠΉΡΡΠ²ΠΈΡ ΠΈΠ·ΠΌΠ΅Π½Π΅Π½ΠΈΡ ΠΏΠΎΡΠΎΠΊΠ°. ΠΠΎ Π²ΡΠ΅Π³Π΄Π° ΠΏΠΎΠΌΠ½ΠΈΡΠ΅, ΡΡΠΎ ΠΠΠ ΡΠΎΠ»ΡΠΊΠΎ ΠΏΠΎΠΏΡΡΠ°Π΅ΡΡΡ ΠΏΡΠΎΡΠΈΠ²ΠΎΡΡΠΎΡΡΡ, Π½ΠΎ Π½Π΅ Π±ΡΠ΄Π΅Ρ ΠΏΠΎΠ»Π½ΠΎΡΡΡΡ ΠΏΡΠΎΡΠΈΠ²ΠΎΠ΄Π΅ΠΉΡΡΠ²ΠΎΠ²Π°ΡΡ Π΄Π΅Π»Ρ. ΠΠΎ ΠΠΠ‘, Π½Π°Π²Π΅Π΄Π΅Π½Π½Π°Ρ Π² ΡΠ²Π΅ΡΡ ΠΏΡΠΎΠ²ΠΎΠ΄Π½ΠΈΠΊΠ΅, ΡΠΌΠΎΠΆΠ΅Ρ ΡΡΠΎ ΡΠ΄Π΅Π»Π°ΡΡ.
ΠΡΠΎ Π²ΠΎΠΏΡΠΎΡ ΠΌΠ°ΡΠ΅ΠΌΠ°ΡΠΈΠΊΠΈ. ΠΡ Π²ΡΠ΅Π³Π΄Π° Π±Π΅ΡΠ΅ΠΌ ΠΌΠ°Π»Π΅Π½ΡΠΊΠΈΠΉ ΡΠ»Π΅ΠΌΠ΅Π½Ρ Π²ΠΌΠ΅ΡΡΠ΅ Ρ ΠΈΠ·ΠΌΠ΅Π½Π΅Π½ΠΈΠ΅ΠΌ. Π’Π°ΠΊ ΠΊΠ°ΠΊ Π·Π΄Π΅ΡΡ ΠΌΡ ΠΈΠ½ΡΠ΅Π³ΡΠΈΡΡΠ΅ΠΌ ΠΏΠΎ ΠΏΠ΅ΡΠ»Π΅, ΡΠΎ ΠΈΠ·ΠΌΠ΅Π½Π΅Π½ΠΈΠ΅ Π΄Π»ΠΈΠ½Ρ ΠΌΡ Π²ΡΠ΅Π³Π΄Π° Π±Π΅ΡΠ΅ΠΌ ΠΏΠΎ ΡΠ»Π΅ΠΊΡΡΠΈΡΠ΅ΡΠΊΠΎΠΌΡ ΠΏΠΎΠ»Ρ ΠΈΠ»ΠΈ ΠΏΠΎ ΠΊΠΎΠΌΠΏΠΎΠ½Π΅Π½ΡΠ΅ ΡΠ»Π΅ΠΊΡΡΠΈΡΠ΅ΡΠΊΠΎΠ³ΠΎ ΠΏΠΎΠ»Ρ.
$\endgroup$
Π’Π²ΠΎΠΉ ΠΎΡΠ²Π΅Ρ
ΠΠ°ΡΠ΅Π³ΠΈΡΡΡΠΈΡΡΠΉΡΠ΅ΡΡ ΠΈΠ»ΠΈ Π²ΠΎΠΉΠ΄ΠΈΡΠ΅ Π² ΡΠΈΡΡΠ΅ΠΌΡ
ΠΠ°ΡΠ΅Π³ΠΈΡΡΡΠΈΡΡΠΉΡΠ΅ΡΡ Ρ ΠΏΠΎΠΌΠΎΡΡΡ Google
ΠΠ°ΡΠ΅Π³ΠΈΡΡΡΠΈΡΠΎΠ²Π°ΡΡΡΡ ΡΠ΅ΡΠ΅Π· Facebook
ΠΠ°ΡΠ΅Π³ΠΈΡΡΡΠΈΡΡΠΉΡΠ΅ΡΡ, ΠΈΡΠΏΠΎΠ»ΡΠ·ΡΡ Π°Π΄ΡΠ΅Ρ ΡΠ»Π΅ΠΊΡΡΠΎΠ½Π½ΠΎΠΉ ΠΏΠΎΡΡΡ ΠΈ ΠΏΠ°ΡΠΎΠ»Ρ
ΠΠΏΡΠ±Π»ΠΈΠΊΠΎΠ²Π°ΡΡ ΠΊΠ°ΠΊ Π³ΠΎΡΡΡ
ΠΠ»Π΅ΠΊΡΡΠΎΠ½Π½Π°Ρ ΠΏΠΎΡΡΠ°
Π’ΡΠ΅Π±ΡΠ΅ΡΡΡ, Π½ΠΎ Π½Π΅ ΠΎΡΠΎΠ±ΡΠ°ΠΆΠ°Π΅ΡΡΡ
ΠΠΏΡΠ±Π»ΠΈΠΊΠΎΠ²Π°ΡΡ ΠΊΠ°ΠΊ Π³ΠΎΡΡΡ
ΠΠ»Π΅ΠΊΡΡΠΎΠ½Π½Π°Ρ ΠΏΠΎΡΡΠ°
Π’ΡΠ΅Π±ΡΠ΅ΡΡΡ, Π½ΠΎ Π½Π΅ ΠΎΡΠΎΠ±ΡΠ°ΠΆΠ°Π΅ΡΡΡ
ΠΠ°ΠΆΠΈΠΌΠ°Ρ Β«ΠΠΏΡΠ±Π»ΠΈΠΊΠΎΠ²Π°ΡΡ ΡΠ²ΠΎΠΉ ΠΎΡΠ²Π΅ΡΒ», Π²Ρ ΡΠΎΠ³Π»Π°ΡΠ°Π΅ΡΠ΅ΡΡ Ρ Π½Π°ΡΠΈΠΌΠΈ ΡΡΠ»ΠΎΠ²ΠΈΡΠΌΠΈ ΠΎΠ±ΡΠ»ΡΠΆΠΈΠ²Π°Π½ΠΈΡ, ΠΏΠΎΠ»ΠΈΡΠΈΠΊΠΎΠΉ ΠΊΠΎΠ½ΡΠΈΠ΄Π΅Π½ΡΠΈΠ°Π»ΡΠ½ΠΎΡΡΠΈ ΠΈ ΠΏΠΎΠ»ΠΈΡΠΈΠΊΠΎΠΉ ΠΈΡΠΏΠΎΠ»ΡΠ·ΠΎΠ²Π°Π½ΠΈΡ ΡΠ°ΠΉΠ»ΠΎΠ² cookie
.ΠΏΡΠΈΠ»ΠΎΠΆΠ΅Π½ΠΈΠΉ ΠΌΠ°Π³Π½ΠΈΡΠ½ΠΎΠΉ ΠΈΠ½Π΄ΡΠΊΡΠΈΠΈ β ΡΠΈΠ·ΠΈΠΊΠ°
ΠΠ½Π΄ΡΡΠΈΡΠΎΠ²Π°Π½Π½Π°Ρ ΠΠΠ‘ ΠΈ ΠΌΠ°Π³Π½ΠΈΡΠ½ΡΠΉ ΠΏΠΎΡΠΎΠΊ
ΠΠ°ΠΊΠΎΠ½ ΠΈΠ½Π΄ΡΠΊΡΠΈΠΈ Π€Π°ΡΠ°Π΄Π΅Ρ Π³Π»Π°ΡΠΈΡ, ΡΡΠΎ ΡΠ»Π΅ΠΊΡΡΠΎΠ΄Π²ΠΈΠΆΡΡΠ°Ρ ΡΠΈΠ»Π° ΠΈΠ½Π΄ΡΡΠΈΡΡΠ΅ΡΡΡ ΠΈΠ·ΠΌΠ΅Π½Π΅Π½ΠΈΠ΅ΠΌ ΠΌΠ°Π³Π½ΠΈΡΠ½ΠΎΠ³ΠΎ ΠΏΠΎΡΠΎΠΊΠ°.
Π¦Π΅Π»ΠΈ ΠΎΠ±ΡΡΠ΅Π½ΠΈΡ
ΠΠ±ΡΡΡΠ½ΠΈΡΡ Π²Π·Π°ΠΈΠΌΠΎΡΠ²ΡΠ·Ρ ΠΌΠ΅ΠΆΠ΄Ρ ΠΌΠ°Π³Π½ΠΈΡΠ½ΡΠΌ ΠΏΠΎΠ»Π΅ΠΌ ΠΈ ΡΠ»Π΅ΠΊΡΡΠΎΠ΄Π²ΠΈΠΆΡΡΠ΅ΠΉ ΡΠΈΠ»ΠΎΠΉ
ΠΠ»ΡΡΠ΅Π²ΡΠ΅ Π²ΡΠ²ΠΎΠ΄Ρ
ΠΠ»ΡΡΠ΅Π²ΡΠ΅ ΠΌΠΎΠΌΠ΅Π½ΡΡ
- ΠΡΠΎ ΠΈΠ·ΠΌΠ΅Π½Π΅Π½ΠΈΠ΅ ΠΏΠΎΡΠΎΠΊΠ° ΠΌΠ°Π³Π½ΠΈΡΠ½ΠΎΠ³ΠΎ ΠΏΠΎΠ»Ρ, ΠΊΠΎΡΠΎΡΠΎΠ΅ ΠΏΡΠΈΠ²ΠΎΠ΄ΠΈΡ ΠΊ Π²ΠΎΠ·Π½ΠΈΠΊΠ½ΠΎΠ²Π΅Π½ΠΈΡ ΡΠ»Π΅ΠΊΡΡΠΎΠ΄Π²ΠΈΠΆΡΡΠ΅ΠΉ ΡΠΈΠ»Ρ (ΠΈΠ»ΠΈ Π½Π°ΠΏΡΡΠΆΠ΅Π½ΠΈΡ).
- ΠΠ°Π³Π½ΠΈΡΠ½ΡΠΉ ΠΏΠΎΡΠΎΠΊ (ΡΠ°ΡΡΠΎ ΠΎΠ±ΠΎΠ·Π½Π°ΡΠ°Π΅ΠΌΡΠΉ Ξ¦ ΠΈΠ»ΠΈ Ξ¦ B ) ΡΠ΅ΡΠ΅Π· ΠΏΠΎΠ²Π΅ΡΡ Π½ΠΎΡΡΡ β ΡΠΎΡΡΠ°Π²Π»ΡΡΡΠ°Ρ ΠΌΠ°Π³Π½ΠΈΡΠ½ΠΎΠ³ΠΎ ΠΏΠΎΠ»Ρ, ΠΏΡΠΎΡ ΠΎΠ΄ΡΡΠ°Ρ ΡΠ΅ΡΠ΅Π· ΡΡΡ ΠΏΠΎΠ²Π΅ΡΡ Π½ΠΎΡΡΡ.
- Π Π½Π°ΠΈΠ±ΠΎΠ»Π΅Π΅ ΠΎΠ±ΡΠ΅ΠΌ Π²ΠΈΠ΄Π΅ ΠΌΠ°Π³Π½ΠΈΡΠ½ΡΠΉ ΠΏΠΎΡΠΎΠΊ ΠΎΠΏΡΠ΅Π΄Π΅Π»ΡΠ΅ΡΡΡ ΠΊΠ°ΠΊ [Π»Π°ΡΠ΅ΠΊΡ]\Phi_{\text{B}} = \iint_{\text{A}} \mathbf{\text{B}} \cdot \text{d} \mathbf {\text{A}}[/Π»Π°ΡΠ΅ΠΊΡ]. ΠΡΠΎ ΠΈΠ½ΡΠ΅Π³ΡΠ°Π» (ΡΡΠΌΠΌΠ°) Π²ΡΠ΅Π³ΠΎ ΠΌΠ°Π³Π½ΠΈΡΠ½ΠΎΠ³ΠΎ ΠΏΠΎΠ»Ρ, ΠΏΡΠΎΡ ΠΎΠ΄ΡΡΠ΅Π³ΠΎ ΡΠ΅ΡΠ΅Π· Π±Π΅ΡΠΊΠΎΠ½Π΅ΡΠ½ΠΎ ΠΌΠ°Π»ΡΠ΅ ΡΠ»Π΅ΠΌΠ΅Π½ΡΡ ΠΏΠ»ΠΎΡΠ°Π΄ΠΈ dA.
ΠΠ»ΡΡΠ΅Π²ΡΠ΅ ΡΠ΅ΡΠΌΠΈΠ½Ρ
- ΠΏΠ»ΠΎΡΠ°Π΄Ρ Π²Π΅ΠΊΡΠΎΡΠ° : ΠΠ΅ΠΊΡΠΎΡ, Π²Π΅Π»ΠΈΡΠΈΠ½Π° ΠΊΠΎΡΠΎΡΠΎΠ³ΠΎ ΠΏΡΠ΅Π΄ΡΡΠ°Π²Π»ΡΠ΅Ρ ΡΠΎΠ±ΠΎΠΉ ΡΠ°ΡΡΠΌΠ°ΡΡΠΈΠ²Π°Π΅ΠΌΡΡ ΠΏΠ»ΠΎΡΠ°Π΄Ρ ΠΈ Π½Π°ΠΏΡΠ°Π²Π»Π΅Π½ΠΈΠ΅ ΠΊΠΎΡΠΎΡΠΎΠ³ΠΎ ΠΏΠ΅ΡΠΏΠ΅Π½Π΄ΠΈΠΊΡΠ»ΡΡΠ½ΠΎ ΠΏΠ»ΠΎΡΠ°Π΄ΠΈ ΠΏΠΎΠ²Π΅ΡΡ Π½ΠΎΡΡΠΈ.
- Π³Π°Π»ΡΠ²Π°Π½ΠΎΠΌΠ΅ΡΡ : ΠΠ½Π°Π»ΠΎΠ³ΠΎΠ²ΡΠΉ ΠΈΠ·ΠΌΠ΅ΡΠΈΡΠ΅Π»ΡΠ½ΡΠΉ ΠΏΡΠΈΠ±ΠΎΡ, ΠΎΠ±ΠΎΠ·Π½Π°ΡΠ°Π΅ΠΌΡΠΉ Π±ΡΠΊΠ²ΠΎΠΉ G, ΠΊΠΎΡΠΎΡΡΠΉ ΠΈΠ·ΠΌΠ΅ΡΡΠ΅Ρ ΡΠΎΠΊ Ρ ΠΏΠΎΠΌΠΎΡΡΡ ΠΎΡΠΊΠ»ΠΎΠ½Π΅Π½ΠΈΡ ΡΡΡΠ΅Π»ΠΊΠΈ, Π²ΡΠ·Π²Π°Π½Π½ΠΎΠ³ΠΎ ΡΠΈΠ»ΠΎΠΉ ΠΌΠ°Π³Π½ΠΈΡΠ½ΠΎΠ³ΠΎ ΠΏΠΎΠ»Ρ, Π΄Π΅ΠΉΡΡΠ²ΡΡΡΠ΅ΠΉ Π½Π° ΠΏΡΠΎΠ²ΠΎΠ΄Π½ΠΈΠΊ Ρ ΡΠΎΠΊΠΎΠΌ.
ΠΠ½Π΄ΡΡΠΈΡΠΎΠ²Π°Π½Π½Π°Ρ ΠΠΠ‘
Π£ΡΡΡΠΎΠΉΡΡΠ²ΠΎ, ΠΈΡΠΏΠΎΠ»ΡΠ·ΠΎΠ²Π°Π½Π½ΠΎΠ΅ Π€Π°ΡΠ°Π΄Π΅Π΅ΠΌ Π΄Π»Ρ Π΄Π΅ΠΌΠΎΠ½ΡΡΡΠ°ΡΠΈΠΈ ΡΠΎΠ³ΠΎ, ΡΡΠΎ ΠΌΠ°Π³Π½ΠΈΡΠ½ΡΠ΅ ΠΏΠΎΠ»Ρ ΠΌΠΎΠ³ΡΡ ΡΠΎΠ·Π΄Π°Π²Π°ΡΡ ΡΠΎΠΊΠΈ, ΠΏΠΎΠΊΠ°Π·Π°Π½ΠΎ Π½Π° ΡΠ»Π΅Π΄ΡΡΡΠ΅ΠΌ ΡΠΈΡΡΠ½ΠΊΠ΅. ΠΠΎΠ³Π΄Π° ΠΏΠ΅ΡΠ΅ΠΊΠ»ΡΡΠ°ΡΠ΅Π»Ρ Π·Π°ΠΌΠΊΠ½ΡΡ, ΠΌΠ°Π³Π½ΠΈΡΠ½ΠΎΠ΅ ΠΏΠΎΠ»Π΅ ΡΠΎΠ·Π΄Π°Π΅ΡΡΡ Π² ΠΊΠ°ΡΡΡΠΊΠ΅ Π² Π²Π΅ΡΡ Π½Π΅ΠΉ ΡΠ°ΡΡΠΈ ΠΆΠ΅Π»Π΅Π·Π½ΠΎΠ³ΠΎ ΠΊΠΎΠ»ΡΡΠ° ΠΈ ΠΏΠ΅ΡΠ΅Π΄Π°Π΅ΡΡΡ (ΠΈΠ»ΠΈ Π½Π°ΠΏΡΠ°Π²Π»ΡΠ΅ΡΡΡ) Π½Π° ΠΊΠ°ΡΡΡΠΊΡ Π² Π½ΠΈΠΆΠ½Π΅ΠΉ ΡΠ°ΡΡΠΈ ΠΊΠΎΠ»ΡΡΠ°. ΠΠ°Π»ΡΠ²Π°Π½ΠΎΠΌΠ΅ΡΡ ΠΈΡΠΏΠΎΠ»ΡΠ·ΡΠ΅ΡΡΡ Π΄Π»Ρ ΠΎΠ±Π½Π°ΡΡΠΆΠ΅Π½ΠΈΡ Π»ΡΠ±ΠΎΠ³ΠΎ ΡΠΎΠΊΠ°, Π½Π°Π²Π΅Π΄Π΅Π½Π½ΠΎΠ³ΠΎ Π² ΠΎΡΠ΄Π΅Π»ΡΠ½ΠΎΠΉ ΠΊΠ°ΡΡΡΠΊΠ΅ Π½Π° Π΄Π½Π΅.
ΠΠΏΠΏΠ°ΡΠ°Ρ Π€Π°ΡΠ°Π΄Π΅Ρ : ΠΡΠΎ Π°ΠΏΠΏΠ°ΡΠ°Ρ Π€Π°ΡΠ°Π΄Π΅Ρ Π΄Π»Ρ Π΄Π΅ΠΌΠΎΠ½ΡΡΡΠ°ΡΠΈΠΈ ΡΠΎΠ³ΠΎ, ΡΡΠΎ ΠΌΠ°Π³Π½ΠΈΡΠ½ΠΎΠ΅ ΠΏΠΎΠ»Π΅ ΠΌΠΎΠΆΠ΅Ρ ΠΏΡΠΎΠΈΠ·Π²ΠΎΠ΄ΠΈΡΡ ΡΠΎΠΊ. ΠΠ·ΠΌΠ΅Π½Π΅Π½ΠΈΠ΅ ΠΏΠΎΠ»Ρ, ΡΠΎΠ·Π΄Π°Π²Π°Π΅ΠΌΠΎΠ³ΠΎ Π²Π΅ΡΡ Π½Π΅ΠΉ ΠΊΠ°ΡΡΡΠΊΠΎΠΉ, ΠΈΠ½Π΄ΡΡΠΈΡΡΠ΅Ρ ΠΠΠ‘ ΠΈ, ΡΠ»Π΅Π΄ΠΎΠ²Π°ΡΠ΅Π»ΡΠ½ΠΎ, ΡΠΎΠΊ Π² Π½ΠΈΠΆΠ½Π΅ΠΉ ΠΊΠ°ΡΡΡΠΊΠ΅. ΠΠΎΠ³Π΄Π° ΠΏΠ΅ΡΠ΅ΠΊΠ»ΡΡΠ°ΡΠ΅Π»Ρ ΡΠ°Π·ΠΌΡΠΊΠ°Π΅ΡΡΡ ΠΈ Π·Π°ΠΌΡΠΊΠ°Π΅ΡΡΡ, Π³Π°Π»ΡΠ²Π°Π½ΠΎΠΌΠ΅ΡΡ ΡΠ΅Π³ΠΈΡΡΡΠΈΡΡΠ΅Ρ ΡΠΎΠΊΠΈ Π² ΠΏΡΠΎΡΠΈΠ²ΠΎΠΏΠΎΠ»ΠΎΠΆΠ½ΡΡ Π½Π°ΠΏΡΠ°Π²Π»Π΅Π½ΠΈΡΡ . Π§Π΅ΡΠ΅Π· Π³Π°Π»ΡΠ²Π°Π½ΠΎΠΌΠ΅ΡΡ Π½Π΅ ΠΏΡΠΎΡΠ΅ΠΊΠ°Π΅Ρ ΡΠΎΠΊ, ΠΊΠΎΠ³Π΄Π° ΠΏΠ΅ΡΠ΅ΠΊΠ»ΡΡΠ°ΡΠ΅Π»Ρ ΠΎΡΡΠ°Π΅ΡΡΡ Π·Π°ΠΌΠΊΠ½ΡΡΡΠΌ ΠΈΠ»ΠΈ ΡΠ°Π·ΠΎΠΌΠΊΠ½ΡΡΡΠΌ.
ΠΡΠ»ΠΎ ΠΎΠ±Π½Π°ΡΡΠΆΠ΅Π½ΠΎ, ΡΡΠΎ ΠΊΠ°ΠΆΠ΄ΡΠΉ ΡΠ°Π·, ΠΊΠΎΠ³Π΄Π° ΠΏΠ΅ΡΠ΅ΠΊΠ»ΡΡΠ°ΡΠ΅Π»Ρ Π·Π°ΠΌΡΠΊΠ°Π΅ΡΡΡ, Π³Π°Π»ΡΠ²Π°Π½ΠΎΠΌΠ΅ΡΡ ΡΠ΅Π³ΠΈΡΡΡΠΈΡΡΠ΅Ρ ΡΠΎΠΊ Π² ΠΎΠ΄Π½ΠΎΠΌ Π½Π°ΠΏΡΠ°Π²Π»Π΅Π½ΠΈΠΈ Π² ΠΊΠ°ΡΡΡΠΊΠ΅ Π½Π° Π΄Π½Π΅. ΠΠ°ΠΆΠ΄ΡΠΉ ΡΠ°Π·, ΠΊΠΎΠ³Π΄Π° ΠΏΠ΅ΡΠ΅ΠΊΠ»ΡΡΠ°ΡΠ΅Π»Ρ ΡΠ°Π·ΠΌΡΠΊΠ°Π΅ΡΡΡ, Π³Π°Π»ΡΠ²Π°Π½ΠΎΠΌΠ΅ΡΡ ΡΠ΅Π³ΠΈΡΡΡΠΈΡΡΠ΅Ρ ΡΠΎΠΊ Π² ΠΏΡΠΎΡΠΈΠ²ΠΎΠΏΠΎΠ»ΠΎΠΆΠ½ΠΎΠΌ Π½Π°ΠΏΡΠ°Π²Π»Π΅Π½ΠΈΠΈ. ΠΠ½ΡΠ΅ΡΠ΅ΡΠ½ΠΎ, ΡΡΠΎ Π΅ΡΠ»ΠΈ ΠΏΠ΅ΡΠ΅ΠΊΠ»ΡΡΠ°ΡΠ΅Π»Ρ ΠΎΡΡΠ°Π΅ΡΡΡ Π·Π°ΠΌΠΊΠ½ΡΡΡΠΌ ΠΈΠ»ΠΈ ΡΠ°Π·ΠΎΠΌΠΊΠ½ΡΡΡΠΌ ΠΊΠ°ΠΊΠΎΠ΅-ΡΠΎ Π²ΡΠ΅ΠΌΡ, ΡΠΎΠΊ ΡΠ΅ΡΠ΅Π· Π³Π°Π»ΡΠ²Π°Π½ΠΎΠΌΠ΅ΡΡ ΠΎΡΡΡΡΡΡΠ²ΡΠ΅Ρ. ΠΠ°ΠΌΡΠΊΠ°Π½ΠΈΠ΅ ΠΈ ΡΠ°Π·ΠΌΡΠΊΠ°Π½ΠΈΠ΅ ΠΏΠ΅ΡΠ΅ΠΊΠ»ΡΡΠ°ΡΠ΅Π»Ρ ΠΈΠ½Π΄ΡΡΠΈΡΡΠ΅Ρ ΡΠΎΠΊ. ΠΠΌΠ΅Π½Π½ΠΎ ΠΈΠ·ΠΌΠ΅Π½Π΅Π½ΠΈΠ΅ ΠΌΠ°Π³Π½ΠΈΡΠ½ΠΎΠ³ΠΎ ΠΏΠΎΠ»Ρ ΡΠΎΠ·Π΄Π°Π΅Ρ ΡΠΎΠΊ. ΠΠΎΠ»Π΅Π΅ Π²Π°ΠΆΠ½ΡΠΌ, ΡΠ΅ΠΌ ΡΠ΅ΠΊΡΡΠΈΠΉ ΡΠΎΠΊ, ΡΠ²Π»ΡΠ΅ΡΡΡ ΡΠ»Π΅ΠΊΡΡΠΎΠ΄Π²ΠΈΠΆΡΡΠ°Ρ ΡΠΈΠ»Π° (ΠΠΠ‘), ΠΊΠΎΡΠΎΡΠ°Ρ Π΅Π³ΠΎ Π²ΡΠ·ΡΠ²Π°Π΅Ρ. Π’ΠΎΠΊ ΡΠ²Π»ΡΠ΅ΡΡΡ ΡΠ΅Π·ΡΠ»ΡΡΠ°ΡΠΎΠΌ ΠΠΠ‘, ΠΈΠ½Π΄ΡΡΠΈΡΠΎΠ²Π°Π½Π½ΠΎΠΉ ΠΈΠ·ΠΌΠ΅Π½ΡΡΡΠΈΠΌΡΡ ΠΌΠ°Π³Π½ΠΈΡΠ½ΡΠΌ ΠΏΠΎΠ»Π΅ΠΌ, Π½Π΅Π·Π°Π²ΠΈΡΠΈΠΌΠΎ ΠΎΡ ΡΠΎΠ³ΠΎ, Π΅ΡΡΡ Π»ΠΈ ΠΏΡΡΡ Π΄Π»Ρ ΠΏΡΠΎΡΠ΅ΠΊΠ°Π½ΠΈΡ ΡΠΎΠΊΠ°.
ΠΠ°Π³Π½ΠΈΡΠ½ΡΠΉ ΠΏΠΎΡΠΎΠΊ
ΠΠ°Π³Π½ΠΈΡΠ½ΡΠΉ ΠΏΠΎΡΠΎΠΊ (ΡΠ°ΡΡΠΎ ΠΎΠ±ΠΎΠ·Π½Π°ΡΠ°Π΅ΠΌΡΠΉ Ξ¦ ΠΈΠ»ΠΈ Ξ¦ B ) ΡΠ΅ΡΠ΅Π· ΠΏΠΎΠ²Π΅ΡΡ Π½ΠΎΡΡΡ ΠΏΡΠ΅Π΄ΡΡΠ°Π²Π»ΡΠ΅Ρ ΡΠΎΠ±ΠΎΠΉ ΠΊΠΎΠΌΠΏΠΎΠ½Π΅Π½Ρ ΠΌΠ°Π³Π½ΠΈΡΠ½ΠΎΠ³ΠΎ ΠΏΠΎΠ»Ρ, ΠΏΡΠΎΡ ΠΎΠ΄ΡΡΠ΅Π³ΠΎ ΡΠ΅ΡΠ΅Π· ΡΡΡ ΠΏΠΎΠ²Π΅ΡΡ Π½ΠΎΡΡΡ. ΠΠ°Π³Π½ΠΈΡΠ½ΡΠΉ ΠΏΠΎΡΠΎΠΊ ΡΠ΅ΡΠ΅Π· Π½Π΅ΠΊΠΎΡΠΎΡΡΡ ΠΏΠΎΠ²Π΅ΡΡ Π½ΠΎΡΡΡ ΠΏΡΠΎΠΏΠΎΡΡΠΈΠΎΠ½Π°Π»Π΅Π½ ΡΠΈΡΠ»Ρ ΡΠΈΠ»ΠΎΠ²ΡΡ Π»ΠΈΠ½ΠΈΠΉ, ΠΏΡΠΎΡ ΠΎΠ΄ΡΡΠΈΡ ΡΠ΅ΡΠ΅Π· ΡΡΡ ΠΏΠΎΠ²Π΅ΡΡ Π½ΠΎΡΡΡ. ΠΠ°Π³Π½ΠΈΡΠ½ΡΠΉ ΠΏΠΎΡΠΎΠΊ, ΠΏΡΠΎΡ ΠΎΠ΄ΡΡΠΈΠΉ ΡΠ΅ΡΠ΅Π· ΠΏΠΎΠ²Π΅ΡΡ Π½ΠΎΡΡΡ Π²Π΅ΠΊΡΠΎΡΠ½ΠΎΠΉ ΠΏΠ»ΠΎΡΠ°Π΄ΠΈ A, ΡΠ°Π²Π΅Π½
[Π»Π°ΡΠ΅ΠΊΡ]\Phi_\text{B} = \mathbf{\text{B}} \cdot \mathbf{\text{A}} = \text{BA } \cos \theta[/latex],
, Π³Π΄Π΅ B β Π²Π΅Π»ΠΈΡΠΈΠ½Π° ΠΌΠ°Π³Π½ΠΈΡΠ½ΠΎΠ³ΠΎ ΠΏΠΎΠ»Ρ (Π² Π΅Π΄ΠΈΠ½ΠΈΡΠ°Ρ Π’Π΅ΡΠ»Π°, Π’Π»), A β ΠΏΠ»ΠΎΡΠ°Π΄Ρ ΠΏΠΎΠ²Π΅ΡΡ Π½ΠΎΡΡΠΈ, Π° ΞΈ β ΡΠ³ΠΎΠ» ΠΌΠ΅ΠΆΠ΄Ρ ΡΠΈΠ»ΠΎΠ²ΡΠΌΠΈ Π»ΠΈΠ½ΠΈΡΠΌΠΈ ΠΌΠ°Π³Π½ΠΈΡΠ½ΠΎΠ³ΠΎ ΠΏΠΎΠ»Ρ ΠΈ Π½ΠΎΡΠΌΠ°Π»Ρ (ΠΏΠ΅ΡΠΏΠ΅Π½Π΄ΠΈΠΊΡΠ»ΡΡ) ΠΊ A.
ΠΠ»Ρ ΠΏΠ΅ΡΠ΅ΠΌΠ΅Π½Π½ΠΎΠ³ΠΎ ΠΌΠ°Π³Π½ΠΈΡΠ½ΠΎΠ³ΠΎ ΠΏΠΎΠ»Ρ ΡΠ½Π°ΡΠ°Π»Π° ΡΠ°ΡΡΠΌΠΎΡΡΠΈΠΌ ΠΌΠ°Π³Π½ΠΈΡΠ½ΡΠΉ ΠΏΠΎΡΠΎΠΊ [Π»Π°ΡΠ΅ΠΊΡ]\ΡΠ΅ΠΊΡΡ{d}\Phi _\text{B}[/Π»Π°ΡΠ΅ΠΊΡ] ΡΠ΅ΡΠ΅Π· ΡΠ»Π΅ΠΌΠ΅Π½Ρ Π±Π΅ΡΠΊΠΎΠ½Π΅ΡΠ½ΠΎ ΠΌΠ°Π»ΠΎΠΉ ΠΏΠ»ΠΎΡΠ°Π΄ΠΈ dA, Π³Π΄Π΅ ΠΌΡ ΠΌΠΎΠΆΠ΅ΠΌ ΡΡΠΈΡΠ°ΡΡ ΠΏΠΎΠ»Π΅ ΠΏΠΎΡΡΠΎΡΠ½Π½ΡΠΌ :
ΠΠ΅ΡΠ΅ΠΌΠ΅Π½Π½ΠΎΠ΅ ΠΌΠ°Π³Π½ΠΈΡΠ½ΠΎΠ΅ ΠΏΠΎΠ»Π΅ : ΠΠ°ΠΆΠ΄Π°Ρ ΡΠΎΡΠΊΠ° Π½Π° ΠΏΠΎΠ²Π΅ΡΡ Π½ΠΎΡΡΠΈ ΡΠ²ΡΠ·Π°Π½Π° Ρ Π½Π°ΠΏΡΠ°Π²Π»Π΅Π½ΠΈΠ΅ΠΌ, Π½Π°Π·ΡΠ²Π°Π΅ΠΌΡΠΌ Π½ΠΎΡΠΌΠ°Π»ΡΡ ΠΊ ΠΏΠΎΠ²Π΅ΡΡ Π½ΠΎΡΡΠΈ; ΡΠΎΠ³Π΄Π° ΠΌΠ°Π³Π½ΠΈΡΠ½ΡΠΉ ΠΏΠΎΡΠΎΠΊ ΡΠ΅ΡΠ΅Π· ΡΠΎΡΠΊΡ ΡΠ²Π»ΡΠ΅ΡΡΡ ΡΠΎΡΡΠ°Π²Π»ΡΡΡΠ΅ΠΉ ΠΌΠ°Π³Π½ΠΈΡΠ½ΠΎΠ³ΠΎ ΠΏΠΎΠ»Ρ Π²Π΄ΠΎΠ»Ρ ΡΡΠΎΠ³ΠΎ Π½ΠΎΡΠΌΠ°Π»ΡΠ½ΠΎΠ³ΠΎ Π½Π°ΠΏΡΠ°Π²Π»Π΅Π½ΠΈΡ.
[Π»Π°ΡΠ΅ΠΊΡ]\text{d}\Phi_\text{B} = \mathbf{\text{B}} \cdot \text{d}\mathbf{\text{A}}[/latex]
ΠΠ±ΡΠ°Ρ ΠΏΠΎΠ²Π΅ΡΡ Π½ΠΎΡΡΡ A Π·Π°ΡΠ΅ΠΌ ΠΌΠΎΠΆΠ΅Ρ Π±ΡΡΡ ΡΠ°Π·Π±ΠΈΡΠ° Π½Π° Π±Π΅ΡΠΊΠΎΠ½Π΅ΡΠ½ΠΎ ΠΌΠ°Π»ΡΠ΅ ΡΠ»Π΅ΠΌΠ΅Π½ΡΡ, ΠΈ ΡΠΎΠ³Π΄Π° ΠΎΠ±ΡΠΈΠΉ ΠΌΠ°Π³Π½ΠΈΡΠ½ΡΠΉ ΠΏΠΎΡΠΎΠΊ ΡΠ΅ΡΠ΅Π· ΠΏΠΎΠ²Π΅ΡΡ Π½ΠΎΡΡΡ ΠΏΡΠ΅Π΄ΡΡΠ°Π²Π»ΡΠ΅Ρ ΡΠΎΠ±ΠΎΠΉ ΠΏΠΎΠ²Π΅ΡΡ Π½ΠΎΡΡΠ½ΡΠΉ ΠΈΠ½ΡΠ΅Π³ΡΠ°Π»
[latex]\Phi_\text{B} = \iint_\text{A} \mathbf {\text{B}} \cdot \text{d}\mathbf {\text{A}}[/latex].
ΠΠ°ΠΊΠΎΠ½ ΠΈΠ½Π΄ΡΠΊΡΠΈΠΈ Π€Π°ΡΠ°Π΄Π΅Ρ ΠΈ Π·Π°ΠΊΠΎΠ½ ΠΠ΅Π½ΡΠ°
ΠΠ°ΠΊΠΎΠ½ ΠΈΠ½Π΄ΡΠΊΡΠΈΠΈ Π€Π°ΡΠ°Π΄Π΅Ρ ΡΡΠ²Π΅ΡΠΆΠ΄Π°Π΅Ρ, ΡΡΠΎ ΠΠΠ‘, ΠΈΠ½Π΄ΡΡΠΈΡΠΎΠ²Π°Π½Π½Π°Ρ ΠΈΠ·ΠΌΠ΅Π½Π΅Π½ΠΈΠ΅ΠΌ ΠΌΠ°Π³Π½ΠΈΡΠ½ΠΎΠ³ΠΎ ΠΏΠΎΡΠΎΠΊΠ°, ΡΠ°Π²Π½Π° [Π»Π°ΡΠ΅ΠΊΡ]\text{ΠΠΠ‘} = -\text{N}\frac{\Delta \Phi} {\Delta \text{t}}[/latex], ΠΊΠΎΠ³Π΄Π° ΠΏΠΎΡΠΎΠΊ ΠΈΠ·ΠΌΠ΅Π½ΡΠ΅ΡΡΡ Π½Π° Ξ Π·Π° Π²ΡΠ΅ΠΌΡ Ξt.
Π¦Π΅Π»ΠΈ ΠΎΠ±ΡΡΠ΅Π½ΠΈΡ
ΠΡΡΠ°Π·ΠΈΡΠ΅ Π·Π°ΠΊΠΎΠ½ ΠΈΠ½Π΄ΡΠΊΡΠΈΠΈ Π€Π°ΡΠ°Π΄Π΅Ρ Π² ΡΠΎΡΠΌΠ΅ ΡΡΠ°Π²Π½Π΅Π½ΠΈΡ
ΠΠ»ΡΡΠ΅Π²ΡΠ΅ Π²ΡΠ²ΠΎΠ΄Ρ
ΠΠ»ΡΡΠ΅Π²ΡΠ΅ ΠΌΠΎΠΌΠ΅Π½ΡΡ
- ΠΠΈΠ½ΡΡ Π² Π·Π°ΠΊΠΎΠ½Π΅ Π€Π°ΡΠ°Π΄Π΅Ρ ΠΎΠ·Π½Π°ΡΠ°Π΅Ρ, ΡΡΠΎ ΠΠΠ‘ ΡΠΎΠ·Π΄Π°Π΅Ρ ΡΠΎΠΊ I ΠΈ ΠΌΠ°Π³Π½ΠΈΡΠ½ΠΎΠ΅ ΠΏΠΎΠ»Π΅ B, ΠΊΠΎΡΠΎΡΡΠ΅ ΠΏΡΠ΅ΠΏΡΡΡΡΠ²ΡΡΡ ΠΈΠ·ΠΌΠ΅Π½Π΅Π½ΠΈΡ Π² ΠΏΠΎΡΠΎΠΊΠ΅ Ξ ΡΡΠΎ ΠΈΠ·Π²Π΅ΡΡΠ½ΠΎ ΠΊΠ°ΠΊ Π·Π°ΠΊΠΎΠ½ ΠΠ΅Π½ΡΠ°.
- ΠΠ°ΠΊΠΎΠ½ ΠΈΠ½Π΄ΡΠΊΡΠΈΠΈ Π€Π°ΡΠ°Π΄Π΅Ρ ΡΠ²Π»ΡΠ΅ΡΡΡ ΡΡΠ½Π΄Π°ΠΌΠ΅Π½ΡΠ°Π»ΡΠ½ΡΠΌ ΠΏΡΠΈΠ½ΡΠΈΠΏΠΎΠΌ ΡΠ°Π±ΠΎΡΡ ΡΡΠ°Π½ΡΡΠΎΡΠΌΠ°ΡΠΎΡΠΎΠ², ΠΊΠ°ΡΡΡΠ΅ΠΊ ΠΈΠ½Π΄ΡΠΊΡΠΈΠ²Π½ΠΎΡΡΠΈ ΠΈ ΠΌΠ½ΠΎΠ³ΠΈΡ ΡΠΈΠΏΠΎΠ² ΡΠ»Π΅ΠΊΡΡΠΎΠ΄Π²ΠΈΠ³Π°ΡΠ΅Π»Π΅ΠΉ, Π³Π΅Π½Π΅ΡΠ°ΡΠΎΡΠΎΠ² ΠΈ ΡΠΎΠ»Π΅Π½ΠΎΠΈΠ΄ΠΎΠ².
- ΠΠ°ΠΊΠΎΠ½ Π€Π°ΡΠ°Π΄Π΅Ρ Π³Π»Π°ΡΠΈΡ, ΡΡΠΎ ΠΠΠ‘, ΠΈΠ½Π΄ΡΡΠΈΡΠΎΠ²Π°Π½Π½Π°Ρ ΠΈΠ·ΠΌΠ΅Π½Π΅Π½ΠΈΠ΅ΠΌ ΠΌΠ°Π³Π½ΠΈΡΠ½ΠΎΠ³ΠΎ ΠΏΠΎΡΠΎΠΊΠ°, Π·Π°Π²ΠΈΡΠΈΡ ΠΎΡ ΠΈΠ·ΠΌΠ΅Π½Π΅Π½ΠΈΡ ΠΏΠΎΡΠΎΠΊΠ° Ξ, Π²ΡΠ΅ΠΌΠ΅Π½ΠΈ Ξt ΠΈ ΡΠΈΡΠ»Π° Π²ΠΈΡΠΊΠΎΠ² ΠΊΠ°ΡΡΡΠΊΠΈ.
ΠΡΠ½ΠΎΠ²Π½ΡΠ΅ ΡΠ΅ΡΠΌΠΈΠ½Ρ
- ΡΠ»Π΅ΠΊΡΡΠΎΠ΄Π²ΠΈΠΆΡΡΠ°Ρ ΡΠΈΠ»Π° : (ΠΠΠ‘) β Π½Π°ΠΏΡΡΠΆΠ΅Π½ΠΈΠ΅, ΡΠΎΠ·Π΄Π°Π²Π°Π΅ΠΌΠΎΠ΅ Π±Π°ΡΠ°ΡΠ΅Π΅ΠΉ ΠΈΠ»ΠΈ ΠΌΠ°Π³Π½ΠΈΡΠ½ΠΎΠΉ ΡΠΈΠ»ΠΎΠΉ Π² ΡΠΎΠΎΡΠ²Π΅ΡΡΡΠ²ΠΈΠΈ Ρ Π·Π°ΠΊΠΎΠ½ΠΎΠΌ Π€Π°ΡΠ°Π΄Π΅Ρ. ΠΠ½Π° ΠΈΠ·ΠΌΠ΅ΡΡΠ΅ΡΡΡ Π² Π²ΠΎΠ»ΡΡΠ°Ρ , Π° Π½Π΅ Π² Π½ΡΡΡΠΎΠ½Π°Ρ , ΠΈ, ΡΠ»Π΅Π΄ΠΎΠ²Π°ΡΠ΅Π»ΡΠ½ΠΎ, Π½Π° ΡΠ°ΠΌΠΎΠΌ Π΄Π΅Π»Π΅ Π½Π΅ ΡΠ²Π»ΡΠ΅ΡΡΡ ΡΠΈΠ»ΠΎΠΉ.
- ΡΠΎΠ»Π΅Π½ΠΎΠΈΠ΄ : ΠΠ°ΡΡΡΠΊΠ° ΠΏΡΠΎΠ²ΠΎΠ»ΠΎΠΊΠΈ, ΠΊΠΎΡΠΎΡΠ°Ρ Π΄Π΅ΠΉΡΡΠ²ΡΠ΅Ρ ΠΊΠ°ΠΊ ΠΌΠ°Π³Π½ΠΈΡ, ΠΊΠΎΠ³Π΄Π° ΡΠ΅ΡΠ΅Π· Π½Π΅Π΅ ΠΏΡΠΎΡ ΠΎΠ΄ΠΈΡ ΡΠ»Π΅ΠΊΡΡΠΈΡΠ΅ΡΠΊΠΈΠΉ ΡΠΎΠΊ.
- ΠΏΠΎΡΠΎΠΊ : Π‘ΠΊΠΎΡΠΎΡΡΡ ΠΏΠ΅ΡΠ΅Π΄Π°ΡΠΈ ΡΠ½Π΅ΡΠ³ΠΈΠΈ (ΠΈΠ»ΠΈ Π΄ΡΡΠ³ΠΎΠΉ ΡΠΈΠ·ΠΈΡΠ΅ΡΠΊΠΎΠΉ Π²Π΅Π»ΠΈΡΠΈΠ½Ρ) ΡΠ΅ΡΠ΅Π· Π΄Π°Π½Π½ΡΡ ΠΏΠΎΠ²Π΅ΡΡ Π½ΠΎΡΡΡ, Π² ΡΠ°ΡΡΠ½ΠΎΡΡΠΈ, ΡΠ»Π΅ΠΊΡΡΠΈΡΠ΅ΡΠΊΠΈΠΉ ΠΏΠΎΡΠΎΠΊ ΠΈΠ»ΠΈ ΠΌΠ°Π³Π½ΠΈΡΠ½ΡΠΉ ΠΏΠΎΡΠΎΠΊ.
ΠΠ°ΠΊΠΎΠ½ ΠΈΠ½Π΄ΡΠΊΡΠΈΠΈ Π€Π°ΡΠ°Π΄Π΅Ρ
ΠΠ°ΠΊΠΎΠ½ ΠΈΠ½Π΄ΡΠΊΡΠΈΠΈ Π€Π°ΡΠ°Π΄Π΅Ρ β ΡΡΠΎ ΠΎΡΠ½ΠΎΠ²Π½ΠΎΠΉ Π·Π°ΠΊΠΎΠ½ ΡΠ»Π΅ΠΊΡΡΠΎΠΌΠ°Π³Π½Π΅ΡΠΈΠ·ΠΌΠ°, ΠΊΠΎΡΠΎΡΡΠΉ ΠΏΡΠ΅Π΄ΡΠΊΠ°Π·ΡΠ²Π°Π΅Ρ, ΠΊΠ°ΠΊ ΠΌΠ°Π³Π½ΠΈΡΠ½ΠΎΠ΅ ΠΏΠΎΠ»Π΅ Π±ΡΠ΄Π΅Ρ Π²Π·Π°ΠΈΠΌΠΎΠ΄Π΅ΠΉΡΡΠ²ΠΎΠ²Π°ΡΡ Ρ ΡΠ»Π΅ΠΊΡΡΠΈΡΠ΅ΡΠΊΠΎΠΉ ΡΠ΅ΠΏΡΡ, ΡΠΎΠ·Π΄Π°Π²Π°Ρ ΡΠ»Π΅ΠΊΡΡΠΎΠ΄Π²ΠΈΠΆΡΡΡΡ ΡΠΈΠ»Ρ (ΠΠΠ‘). ΠΡΠΎ ΠΎΡΠ½ΠΎΠ²Π½ΠΎΠΉ ΠΏΡΠΈΠ½ΡΠΈΠΏ ΡΠ°Π±ΠΎΡΡ ΡΡΠ°Π½ΡΡΠΎΡΠΌΠ°ΡΠΎΡΠΎΠ², ΠΊΠ°ΡΡΡΠ΅ΠΊ ΠΈΠ½Π΄ΡΠΊΡΠΈΠ²Π½ΠΎΡΡΠΈ ΠΈ ΠΌΠ½ΠΎΠ³ΠΈΡ ΡΠΈΠΏΠΎΠ² ΡΠ»Π΅ΠΊΡΡΠΎΠ΄Π²ΠΈΠ³Π°ΡΠ΅Π»Π΅ΠΉ, Π³Π΅Π½Π΅ΡΠ°ΡΠΎΡΠΎΠ² ΠΈ ΡΠΎΠ»Π΅Π½ΠΎΠΈΠ΄ΠΎΠ².
ΠΠΊΡΠΏΠ΅ΡΠΈΠΌΠ΅Π½ΡΡ Π€Π°ΡΠ°Π΄Π΅Ρ ΠΏΠΎΠΊΠ°Π·Π°Π»ΠΈ, ΡΡΠΎ ΠΠΠ‘, Π²ΡΠ·Π²Π°Π½Π½Π°Ρ ΠΈΠ·ΠΌΠ΅Π½Π΅Π½ΠΈΠ΅ΠΌ ΠΌΠ°Π³Π½ΠΈΡΠ½ΠΎΠ³ΠΎ ΠΏΠΎΡΠΎΠΊΠ°, Π·Π°Π²ΠΈΡΠΈΡ Π»ΠΈΡΡ ΠΎΡ Π½Π΅ΡΠΊΠΎΠ»ΡΠΊΠΈΡ ΡΠ°ΠΊΡΠΎΡΠΎΠ². ΠΠΎ-ΠΏΠ΅ΡΠ²ΡΡ , ΠΠΠ‘ ΠΏΡΡΠΌΠΎ ΠΏΡΠΎΠΏΠΎΡΡΠΈΠΎΠ½Π°Π»ΡΠ½Π° ΠΈΠ·ΠΌΠ΅Π½Π΅Π½ΠΈΡ ΠΏΠΎΡΠΎΠΊΠ° Ξ. ΠΠΎ-Π²ΡΠΎΡΡΡ , ΠΠΠ‘ ΠΌΠ°ΠΊΡΠΈΠΌΠ°Π»ΡΠ½Π°, ΠΊΠΎΠ³Π΄Π° ΠΈΠ·ΠΌΠ΅Π½Π΅Π½ΠΈΠ΅ Π²ΡΠ΅ΠΌΠ΅Π½ΠΈ Ξt Π½Π°ΠΈΠΌΠ΅Π½ΡΡΠ΅Π΅, ΡΠΎ Π΅ΡΡΡ ΠΠΠ‘ ΠΎΠ±ΡΠ°ΡΠ½ΠΎ ΠΏΡΠΎΠΏΠΎΡΡΠΈΠΎΠ½Π°Π»ΡΠ½Π° Ξt. ΠΠ°ΠΊΠΎΠ½Π΅Ρ, Π΅ΡΠ»ΠΈ ΠΊΠ°ΡΡΡΠΊΠ° ΠΈΠΌΠ΅Π΅Ρ N Π²ΠΈΡΠΊΠΎΠ², Π±ΡΠ΄Π΅Ρ ΡΠΎΠ·Π΄Π°Π²Π°ΡΡΡΡ ΠΠΠ‘, ΠΊΠΎΡΠΎΡΠ°Ρ Π² N ΡΠ°Π· Π±ΠΎΠ»ΡΡΠ΅, ΡΠ΅ΠΌ Π΄Π»Ρ ΠΎΠ΄ΠΈΠ½ΠΎΡΠ½ΠΎΠΉ ΠΊΠ°ΡΡΡΠΊΠΈ, ΡΠ°ΠΊ ΡΡΠΎ ΠΠΠ‘ ΠΏΡΡΠΌΠΎ ΠΏΡΠΎΠΏΠΎΡΡΠΈΠΎΠ½Π°Π»ΡΠ½Π° N. Π£ΡΠ°Π²Π½Π΅Π½ΠΈΠ΅ Π΄Π»Ρ ΠΠΠ‘, ΠΈΠ½Π΄ΡΡΠΈΡΠΎΠ²Π°Π½Π½ΠΎΠΉ ΠΈΠ·ΠΌΠ΅Π½Π΅Π½ΠΈΠ΅ΠΌ ΠΌΠ°Π³Π½ΠΈΡΠ½ΠΎΠ³ΠΎ ΠΏΠΎΡΠΎΠΊΠ°, ΠΈΠΌΠ΅Π΅Ρ Π²ΠΈΠ΄ 9. 0005
[Π»Π°ΡΠ΅ΠΊΡ]\ΡΠ΅ΠΊΡΡ{ΠΠΠ‘} = -\ΡΠ΅ΠΊΡΡ{N}\frac{\Delta \Phi}{\Delta \text{t}}[/latex].
ΠΡΠΎ ΡΠΎΠΎΡΠ½ΠΎΡΠ΅Π½ΠΈΠ΅ ΠΈΠ·Π²Π΅ΡΡΠ½ΠΎ ΠΊΠ°ΠΊ Π·Π°ΠΊΠΎΠ½ ΠΈΠ½Π΄ΡΠΊΡΠΈΠΈ Π€Π°ΡΠ°Π΄Π΅Ρ. ΠΠ΄ΠΈΠ½ΠΈΡΠ°ΠΌΠΈ ΠΠΠ‘, ΠΊΠ°ΠΊ ΠΎΠ±ΡΡΠ½ΠΎ, ΡΠ²Π»ΡΡΡΡΡ Π²ΠΎΠ»ΡΡΡ.
ΠΠ°ΠΊΠΎΠ½ ΠΠ΅Π½ΡΠ°
ΠΠ½Π°ΠΊ ΠΌΠΈΠ½ΡΡ Π² Π·Π°ΠΊΠΎΠ½Π΅ ΠΈΠ½Π΄ΡΠΊΡΠΈΠΈ Π€Π°ΡΠ°Π΄Π΅Ρ ΠΎΡΠ΅Π½Ρ Π²Π°ΠΆΠ΅Π½. ΠΠΈΠ½ΡΡ ΠΎΠ·Π½Π°ΡΠ°Π΅Ρ, ΡΡΠΎ ΠΠΠ‘ ΡΠΎΠ·Π΄Π°Π΅Ρ ΡΠΎΠΊ I ΠΈ ΠΌΠ°Π³Π½ΠΈΡΠ½ΠΎΠ΅ ΠΏΠΎΠ»Π΅ B, ΠΏΡΠΎΡΠΈΠ²ΠΎΠ΄Π΅ΠΉΡΡΠ²ΡΡΡΠΈΠ΅ ΠΈΠ·ΠΌΠ΅Π½Π΅Π½ΠΈΡ ΠΏΠΎΡΠΎΠΊΠ° Ξ, ΡΡΠΎ ΠΈΠ·Π²Π΅ΡΡΠ½ΠΎ ΠΊΠ°ΠΊ Π·Π°ΠΊΠΎΠ½ ΠΠ΅Π½ΡΠ°. ΠΠ°ΠΏΡΠ°Π²Π»Π΅Π½ΠΈΠ΅ (Π·Π°Π΄Π°Π½Π½ΠΎΠ΅ Π·Π½Π°ΠΊΠΎΠΌ ΠΌΠΈΠ½ΡΡ) ΠΠΠ‘ Π½Π°ΡΡΠΎΠ»ΡΠΊΠΎ Π²Π°ΠΆΠ½ΠΎ, ΡΡΠΎ Π΅Π³ΠΎ Π½Π°Π·ΡΠ²Π°ΡΡ Π·Π°ΠΊΠΎΠ½ΠΎΠΌ ΠΠ΅Π½ΡΠ° Π² ΡΠ΅ΡΡΡ ΡΡΡΡΠΊΠΎΠ³ΠΎ ΠΠ΅Π½ΡΠΈΡ Π° ΠΠ΅Π½ΡΠ° (1804β1865), ΠΊΠΎΡΠΎΡΡΠΉ, ΠΏΠΎΠ΄ΠΎΠ±Π½ΠΎ Π€Π°ΡΠ°Π΄Π΅Ρ ΠΈ ΠΠ΅Π½ΡΠΈ, Π½Π΅Π·Π°Π²ΠΈΡΠΈΠΌΠΎ ΠΈΡΡΠ»Π΅Π΄ΠΎΠ²Π°Π» Π°ΡΠΏΠ΅ΠΊΡΡ ΠΈΠ½Π΄ΡΠΊΡΠΈΠΈ. Π€Π°ΡΠ°Π΄Π΅ΠΉ Π·Π½Π°Π» ΠΎΠ± ΡΡΠΎΠΌ Π½Π°ΠΏΡΠ°Π²Π»Π΅Π½ΠΈΠΈ, Π½ΠΎ ΠΠ΅Π½Ρ ΡΠΊΠ°Π·Π°Π» Π΅Π³ΠΎ, ΠΏΠΎΡΡΠΎΠΌΡ Π΅ΠΌΡ ΠΏΡΠΈΠΏΠΈΡΡΠ²Π°ΡΡ Π΅Π³ΠΎ ΠΎΡΠΊΡΡΡΠΈΠ΅.
ΠΠ°ΠΊΠΎΠ½ ΠΠ΅Π½ΡΠ° : (a) ΠΠΎΠ³Π΄Π° ΡΡΠΎΡ ΡΡΠ΅ΡΠΆΠ½Π΅Π²ΠΎΠΉ ΠΌΠ°Π³Π½ΠΈΡ Π²ΡΠ°Π»ΠΊΠΈΠ²Π°Π΅ΡΡΡ Π² ΠΊΠ°ΡΡΡΠΊΡ, Π½Π°ΠΏΡΡΠΆΠ΅Π½Π½ΠΎΡΡΡ ΠΌΠ°Π³Π½ΠΈΡΠ½ΠΎΠ³ΠΎ ΠΏΠΎΠ»Ρ Π² ΠΊΠ°ΡΡΡΠΊΠ΅ ΡΠ²Π΅Π»ΠΈΡΠΈΠ²Π°Π΅ΡΡΡ. Π’ΠΎΠΊ, Π½Π°Π²Π΅Π΄Π΅Π½Π½ΡΠΉ Π² ΠΊΠ°ΡΡΡΠΊΠ΅, ΡΠΎΠ·Π΄Π°Π΅Ρ Π΄ΡΡΠ³ΠΎΠ΅ ΠΏΠΎΠ»Π΅ Π² Π½Π°ΠΏΡΠ°Π²Π»Π΅Π½ΠΈΠΈ, ΠΏΡΠΎΡΠΈΠ²ΠΎΠΏΠΎΠ»ΠΎΠΆΠ½ΠΎΠΌ Π½Π°ΠΏΡΠ°Π²Π»Π΅Π½ΠΈΡ ΡΡΠ΅ΡΠΆΠ½Π΅Π²ΠΎΠ³ΠΎ ΠΌΠ°Π³Π½ΠΈΡΠ°, ΡΡΠΎΠ±Ρ ΠΏΡΠΎΡΠΈΠ²ΠΎΡΡΠΎΡΡΡ ΡΠ²Π΅Π»ΠΈΡΠ΅Π½ΠΈΡ. ΠΡΠΎ ΠΎΠ΄ΠΈΠ½ ΠΈΠ· Π°ΡΠΏΠ΅ΠΊΡΠΎΠ² Π·Π°ΠΊΠΎΠ½Π° ΠΠ΅Π½ΡΠ°: ΠΈΠ½Π΄ΡΠΊΡΠΈΡ ΠΏΡΠ΅ΠΏΡΡΡΡΠ²ΡΠ΅Ρ Π»ΡΠ±ΠΎΠΌΡ ΠΈΠ·ΠΌΠ΅Π½Π΅Π½ΠΈΡ ΠΏΠΎΡΠΎΠΊΠ°. (b) ΠΈ (c) β Π΄Π²Π΅ Π΄ΡΡΠ³ΠΈΠ΅ ΡΠΈΡΡΠ°ΡΠΈΠΈ. Π£Π±Π΅Π΄ΠΈΡΠ΅ΡΡ ΡΠ°ΠΌΠΈ, ΡΡΠΎ ΠΏΠΎΠΊΠ°Π·Π°Π½Π½ΠΎΠ΅ Π½Π°ΠΏΡΠ°Π²Π»Π΅Π½ΠΈΠ΅ ΠΈΠ½Π΄ΡΡΠΈΡΠΎΠ²Π°Π½Π½ΠΎΠΉ ΠΊΠ°ΡΡΡΠΊΠΈ B Π΄Π΅ΠΉΡΡΠ²ΠΈΡΠ΅Π»ΡΠ½ΠΎ ΠΏΡΠΎΡΠΈΠ²ΠΎΡΡΠΎΠΈΡ ΠΈΠ·ΠΌΠ΅Π½Π΅Π½ΠΈΡ ΠΏΠΎΡΠΎΠΊΠ° ΠΈ ΡΡΠΎ ΠΏΠΎΠΊΠ°Π·Π°Π½Π½ΠΎΠ΅ Π½Π°ΠΏΡΠ°Π²Π»Π΅Π½ΠΈΠ΅ ΡΠΎΠΊΠ° ΡΠΎΠ³Π»Π°ΡΡΠ΅ΡΡΡ Ρ ΠΏΡΠ°Π²ΠΈΠ»ΠΎΠΌ ΠΏΡΠ°Π²ΠΎΠΉ ΡΡΠΊΠΈ.
Π‘ΠΎΡ ΡΠ°Π½Π΅Π½ΠΈΠ΅ ΡΠ½Π΅ΡΠ³ΠΈΠΈ
ΠΠ°ΠΊΠΎΠ½ ΠΠ΅Π½ΡΠ° ΡΠ²Π»ΡΠ΅ΡΡΡ ΠΏΡΠΎΡΠ²Π»Π΅Π½ΠΈΠ΅ΠΌ ΡΠΎΡ ΡΠ°Π½Π΅Π½ΠΈΡ ΡΠ½Π΅ΡΠ³ΠΈΠΈ. ΠΠ½Π΄ΡΡΠΈΡΠΎΠ²Π°Π½Π½Π°Ρ ΠΠΠ‘ ΡΠΎΠ·Π΄Π°Π΅Ρ ΡΠΎΠΊ, ΠΏΡΠΎΡΠΈΠ²ΠΎΠ΄Π΅ΠΉΡΡΠ²ΡΡΡΠΈΠΉ ΠΈΠ·ΠΌΠ΅Π½Π΅Π½ΠΈΡ ΠΏΠΎΡΠΎΠΊΠ°, ΠΏΠΎΡΠΎΠΌΡ ΡΡΠΎ ΠΈΠ·ΠΌΠ΅Π½Π΅Π½ΠΈΠ΅ ΠΏΠΎΡΠΎΠΊΠ° ΠΎΠ·Π½Π°ΡΠ°Π΅Ρ ΠΈΠ·ΠΌΠ΅Π½Π΅Π½ΠΈΠ΅ ΡΠ½Π΅ΡΠ³ΠΈΠΈ. ΠΠ½Π΅ΡΠ³ΠΈΡ ΠΌΠΎΠΆΠ΅Ρ Π²ΠΎΠΉΡΠΈ ΠΈΠ»ΠΈ ΡΠΉΡΠΈ, Π½ΠΎ Π½Π΅ ΠΌΠ³Π½ΠΎΠ²Π΅Π½Π½ΠΎ. ΠΠ°ΠΊΠΎΠ½ ΠΠ΅Π½ΡΠ° ΡΠ²Π»ΡΠ΅ΡΡΡ ΡΠ»Π΅Π΄ΡΡΠ²ΠΈΠ΅ΠΌ. ΠΠΎΠ³Π΄Π° ΠΈΠ·ΠΌΠ΅Π½Π΅Π½ΠΈΠ΅ Π½Π°ΡΠΈΠ½Π°Π΅ΡΡΡ, Π·Π°ΠΊΠΎΠ½ Π³ΠΎΠ²ΠΎΡΠΈΡ, ΡΡΠΎ ΠΈΠ½Π΄ΡΠΊΡΠΈΡ ΠΏΡΠΎΡΠΈΠ²ΠΎΠ΄Π΅ΠΉΡΡΠ²ΡΠ΅Ρ ΠΈ, ΡΠ°ΠΊΠΈΠΌ ΠΎΠ±ΡΠ°Π·ΠΎΠΌ, Π·Π°ΠΌΠ΅Π΄Π»ΡΠ΅Ρ ΠΈΠ·ΠΌΠ΅Π½Π΅Π½ΠΈΠ΅. ΠΠ° ΡΠ°ΠΌΠΎΠΌ Π΄Π΅Π»Π΅, Π΅ΡΠ»ΠΈ Π±Ρ ΠΈΠ½Π΄ΡΡΠΈΡΠΎΠ²Π°Π½Π½Π°Ρ ΠΠΠ‘ Π±ΡΠ»Π° Π½Π°ΠΏΡΠ°Π²Π»Π΅Π½Π° ββΠ² ΡΠΎΠΌ ΠΆΠ΅ Π½Π°ΠΏΡΠ°Π²Π»Π΅Π½ΠΈΠΈ, ΡΡΠΎ ΠΈ ΠΈΠ·ΠΌΠ΅Π½Π΅Π½ΠΈΠ΅ ΠΏΠΎΡΠΎΠΊΠ°, ΡΡΡΠ΅ΡΡΠ²ΠΎΠ²Π°Π»Π° Π±Ρ ΠΏΠΎΠ»ΠΎΠΆΠΈΡΠ΅Π»ΡΠ½Π°Ρ ΠΎΠ±ΡΠ°ΡΠ½Π°Ρ ΡΠ²ΡΠ·Ρ, ΠΊΠΎΡΠΎΡΠ°Ρ Π΄Π°Π²Π°Π»Π° Π±Ρ Π½Π°ΠΌ ΡΠ²ΠΎΠ±ΠΎΠ΄Π½ΡΡ ΡΠ½Π΅ΡΠ³ΠΈΡ Π±Π΅Π· Π²ΠΈΠ΄ΠΈΠΌΠΎΠ³ΠΎ ΠΈΡΡΠΎΡΠ½ΠΈΠΊΠ° β Π·Π°ΠΊΠΎΠ½ ΡΠΎΡ ΡΠ°Π½Π΅Π½ΠΈΡ ΡΠ½Π΅ΡΠ³ΠΈΠΈ Π±ΡΠ» Π±Ρ Π½Π°ΡΡΡΠ΅Π½.
ΠΠΠ‘ Π΄Π²ΠΈΠΆΠ΅Π½ΠΈΡ
ΠΠ²ΠΈΠΆΠ΅Π½ΠΈΠ΅ Π² ΠΌΠ°Π³Π½ΠΈΡΠ½ΠΎΠΌ ΠΏΠΎΠ»Π΅, ΡΡΠ°ΡΠΈΠΎΠ½Π°ΡΠ½ΠΎΠΌ ΠΎΡΠ½ΠΎΡΠΈΡΠ΅Π»ΡΠ½ΠΎ ΠΠ΅ΠΌΠ»ΠΈ, ΠΈΠ½Π΄ΡΡΠΈΡΡΠ΅Ρ ΠΠΠ‘ Π΄Π²ΠΈΠΆΠ΅Π½ΠΈΡ (ΡΠ»Π΅ΠΊΡΡΠΎΠ΄Π²ΠΈΠΆΡΡΠ°Ρ ΡΠΈΠ»Π°).
Π¦Π΅Π»ΠΈ ΠΎΠ±ΡΡΠ΅Π½ΠΈΡ
ΠΠΏΡΠ΅Π΄Π΅Π»ΠΈΡΡ ΠΏΡΠΎΡΠ΅ΡΡ, Π²ΡΠ·ΡΠ²Π°ΡΡΠΈΠΉ Π΄Π²ΠΈΠΆΡΡΡΡ ΡΠ»Π΅ΠΊΡΡΠΎΠ΄Π²ΠΈΠΆΡΡΡΡ ΡΠΈΠ»Ρ ΡΠΈΡΡΠ΅ΠΌΠ°.
ΠΡΠ½ΠΎΠ²Π½ΡΠ΅ ΡΠ΅ΡΠΌΠΈΠ½Ρ
- ΡΠ»Π΅ΠΊΡΡΠΎΠ΄Π²ΠΈΠΆΡΡΠ°Ρ ΡΠΈΠ»Π° : (ΠΠΠ‘) β Π½Π°ΠΏΡΡΠΆΠ΅Π½ΠΈΠ΅, ΡΠΎΠ·Π΄Π°Π²Π°Π΅ΠΌΠΎΠ΅ Π±Π°ΡΠ°ΡΠ΅Π΅ΠΉ ΠΈΠ»ΠΈ ΠΌΠ°Π³Π½ΠΈΡΠ½ΠΎΠΉ ΡΠΈΠ»ΠΎΠΉ Π² ΡΠΎΠΎΡΠ²Π΅ΡΡΡΠ²ΠΈΠΈ Ρ Π·Π°ΠΊΠΎΠ½ΠΎΠΌ Π€Π°ΡΠ°Π΄Π΅Ρ. ΠΠ½Π° ΠΈΠ·ΠΌΠ΅ΡΡΠ΅ΡΡΡ Π² Π²ΠΎΠ»ΡΡΠ°Ρ , Π° Π½Π΅ Π² Π½ΡΡΡΠΎΠ½Π°Ρ , ΠΈ, ΡΠ»Π΅Π΄ΠΎΠ²Π°ΡΠ΅Π»ΡΠ½ΠΎ, Π½Π° ΡΠ°ΠΌΠΎΠΌ Π΄Π΅Π»Π΅ Π½Π΅ ΡΠ²Π»ΡΠ΅ΡΡΡ ΡΠΈΠ»ΠΎΠΉ.
- ΠΌΠ°Π³Π½ΠΈΡΠ½ΡΠΉ ΠΏΠΎΡΠΎΠΊ : ΠΠ΅ΡΠ° ΡΠΈΠ»Ρ ΠΌΠ°Π³Π½ΠΈΡΠ½ΠΎΠ³ΠΎ ΠΏΠΎΠ»Ρ Π² Π΄Π°Π½Π½ΠΎΠΉ ΠΎΠ±Π»Π°ΡΡΠΈ.
- ΠΈΠ½Π΄ΡΠΊΡΠΈΡ : ΠΠ΅Π½Π΅ΡΠ°ΡΠΈΡ ΡΠ»Π΅ΠΊΡΡΠΈΡΠ΅ΡΠΊΠΎΠ³ΠΎ ΡΠΎΠΊΠ° ΠΏΠ΅ΡΠ΅ΠΌΠ΅Π½Π½ΡΠΌ ΠΌΠ°Π³Π½ΠΈΡΠ½ΡΠΌ ΠΏΠΎΠ»Π΅ΠΌ.
ΠΠ°ΠΊ Π²ΠΈΠ΄Π½ΠΎ ΠΈΠ· ΠΏΡΠ΅Π΄ΡΠ΄ΡΡΠΈΡ ΠΡΠΎΠΌΠΎΠ², Π»ΡΠ±ΠΎΠ΅ ΠΈΠ·ΠΌΠ΅Π½Π΅Π½ΠΈΠ΅ ΠΌΠ°Π³Π½ΠΈΡΠ½ΠΎΠ³ΠΎ ΠΏΠΎΡΠΎΠΊΠ° ΠΈΠ½Π΄ΡΡΠΈΡΡΠ΅Ρ ΡΠ»Π΅ΠΊΡΡΠΎΠ΄Π²ΠΈΠΆΡΡΡΡ ΡΠΈΠ»Ρ (ΠΠΠ‘), ΠΏΡΠΎΡΠΈΠ²ΠΎΠ΄Π΅ΠΉΡΡΠ²ΡΡΡΡΡ ΡΡΠΎΠΌΡ ΠΈΠ·ΠΌΠ΅Π½Π΅Π½ΠΈΡ β ΠΏΡΠΎΡΠ΅ΡΡ, ΠΈΠ·Π²Π΅ΡΡΠ½ΡΠΉ ΠΊΠ°ΠΊ ΠΈΠ½Π΄ΡΠΊΡΠΈΡ. ΠΠ²ΠΈΠΆΠ΅Π½ΠΈΠ΅ ΡΠ²Π»ΡΠ΅ΡΡΡ ΠΎΠ΄Π½ΠΎΠΉ ΠΈΠ· ΠΎΡΠ½ΠΎΠ²Π½ΡΡ ΠΏΡΠΈΡΠΈΠ½ ΠΈΠ½Π΄ΡΠΊΡΠΈΠΈ. ΠΠ°ΠΏΡΠΈΠΌΠ΅Ρ, ΠΌΠ°Π³Π½ΠΈΡ, ΠΏΠ΅ΡΠ΅ΠΌΠ΅ΡΠ°Π΅ΠΌΡΠΉ ΠΊ ΠΊΠ°ΡΡΡΠΊΠ΅, ΠΈΠ½Π΄ΡΡΠΈΡΡΠ΅Ρ ΠΠΠ‘, Π° ΠΊΠ°ΡΡΡΠΊΠ°, ΠΏΠ΅ΡΠ΅ΠΌΠ΅ΡΠ°Π΅ΠΌΠ°Ρ ΠΊ ΠΌΠ°Π³Π½ΠΈΡΡ, ΡΠΎΠ·Π΄Π°Π΅Ρ Π°Π½Π°Π»ΠΎΠ³ΠΈΡΠ½ΡΡ ΠΠΠ‘. Π ΡΡΠΎΠΌ ΠΡΠΎΠΌΠ΅ ΠΌΡ ΠΊΠΎΠ½ΡΠ΅Π½ΡΡΠΈΡΡΠ΅ΠΌΡΡ Π½Π° Π΄Π²ΠΈΠΆΠ΅Π½ΠΈΠΈ Π² ΠΌΠ°Π³Π½ΠΈΡΠ½ΠΎΠΌ ΠΏΠΎΠ»Π΅, ΡΡΠ°ΡΠΈΠΎΠ½Π°ΡΠ½ΠΎΠΌ ΠΎΡΠ½ΠΎΡΠΈΡΠ΅Π»ΡΠ½ΠΎ ΠΠ΅ΠΌΠ»ΠΈ, ΠΏΡΠΎΠΈΠ·Π²ΠΎΠ΄Ρ ΡΠΎ, ΡΡΠΎ ΠΌΠΎΠΆΠ½ΠΎ ΡΡΠ»ΠΎΠ²Π½ΠΎ Π½Π°Π·Π²Π°ΡΡ ΠΠΠ‘ Π΄Π²ΠΈΠΆΠ΅Π½ΠΈΡ.
ΠΠΠ‘ Π΄Π²ΠΈΠΆΠ΅Π½ΠΈΡ
Π Π°ΡΡΠΌΠΎΡΡΠΈΠΌ ΡΠΈΡΡΠ°ΡΠΈΡ, ΠΏΠΎΠΊΠ°Π·Π°Π½Π½ΡΡ Π½Π° ΡΠΈΡ. Π‘ΡΠ΅ΡΠΆΠ΅Π½Ρ Π΄Π²ΠΈΠΆΠ΅ΡΡΡ ΡΠΎ ΡΠΊΠΎΡΠΎΡΡΡΡ v ΠΏΠΎ ΠΏΠ°ΡΠ΅ ΠΏΡΠΎΠ²ΠΎΠ΄ΡΡΠΈΡ ΡΠ΅Π»ΡΡΠΎΠ², ΡΠ°Π·Π΄Π΅Π»Π΅Π½Π½ΡΡ ΡΠ°ΡΡΡΠΎΡΠ½ΠΈΠ΅ΠΌ β Π² ΠΎΠ΄Π½ΠΎΡΠΎΠ΄Π½ΠΎΠΌ ΠΌΠ°Π³Π½ΠΈΡΠ½ΠΎΠΌ ΠΏΠΎΠ»Π΅ B. Π Π΅Π»ΡΡΡ Π½Π΅ΠΏΠΎΠ΄Π²ΠΈΠΆΠ½Ρ ΠΎΡΠ½ΠΎΡΠΈΡΠ΅Π»ΡΠ½ΠΎ B ΠΈ ΡΠΎΠ΅Π΄ΠΈΠ½Π΅Π½Ρ Ρ ΠΏΠΎΡΡΠΎΡΠ½Π½ΡΠΉ ΡΠ΅Π·ΠΈΡΡΠΎΡ R (ΡΠ΅Π·ΠΈΡΡΠΎΡ ΠΌΠΎΠΆΠ΅Ρ Π±ΡΡΡ Π»ΡΠ±ΡΠΌ, ΠΎΡ Π»Π°ΠΌΠΏΠΎΡΠΊΠΈ Π΄ΠΎ Π²ΠΎΠ»ΡΡΠΌΠ΅ΡΡΠ°). Π Π°ΡΡΠΌΠΎΡΡΠΈΠΌ ΠΎΠ±Π»Π°ΡΡΡ, ΠΎΠΊΡΡΠΆΠ΅Π½Π½ΡΡ Π΄Π²ΠΈΠΆΡΡΠΈΠΌΡΡ ΡΡΠ΅ΡΠΆΠ½Π΅ΠΌ, ΡΠ΅Π»ΡΡΠ°ΠΌΠΈ ΠΈ ΡΠ΅Π·ΠΈΡΡΠΎΡΠΎΠΌ. B ΠΏΠ΅ΡΠΏΠ΅Π½Π΄ΠΈΠΊΡΠ»ΡΡΠ½Π° ΡΡΠΎΠΉ ΠΏΠ»ΠΎΡΠ°Π΄ΠΈ, ΠΈ ΠΏΠ»ΠΎΡΠ°Π΄Ρ ΡΠ²Π΅Π»ΠΈΡΠΈΠ²Π°Π΅ΡΡΡ ΠΏΠΎ ΠΌΠ΅ΡΠ΅ Π΄Π²ΠΈΠΆΠ΅Π½ΠΈΡ ΡΡΠ΅ΡΠΆΠ½Ρ. Π’Π°ΠΊΠΈΠΌ ΠΎΠ±ΡΠ°Π·ΠΎΠΌ, ΠΌΠ°Π³Π½ΠΈΡΠ½ΡΠΉ ΠΏΠΎΡΠΎΠΊ, Π·Π°ΠΊΠ»ΡΡΠ΅Π½Π½ΡΠΉ ΠΌΠ΅ΠΆΠ΄Ρ ΡΠ΅Π»ΡΡΠ°ΠΌΠΈ, ΡΡΠ΅ΡΠΆΠ½Π΅ΠΌ ΠΈ ΡΠ΅Π·ΠΈΡΡΠΎΡΠΎΠΌ, ΡΠ²Π΅Π»ΠΈΡΠΈΠ²Π°Π΅ΡΡΡ. ΠΡΠΈ ΠΈΠ·ΠΌΠ΅Π½Π΅Π½ΠΈΠΈ ΠΏΠΎΡΠΎΠΊΠ° ΠΈΠ½Π΄ΡΡΠΈΡΡΠ΅ΡΡΡ ΠΠΠ‘ Π² ΡΠΎΠΎΡΠ²Π΅ΡΡΡΠ²ΠΈΠΈ Ρ Π·Π°ΠΊΠΎΠ½ΠΎΠΌ ΠΈΠ½Π΄ΡΠΊΡΠΈΠΈ Π€Π°ΡΠ°Π΄Π΅Ρ.
ΠΠΠ‘ Π΄Π²ΠΈΠΆΠ΅Π½ΠΈΡ : (a) ΠΠΠ‘ Π΄Π²ΠΈΠΆΠ΅Π½ΠΈΡ = Bβv ΠΈΠ½Π΄ΡΡΠΈΡΡΠ΅ΡΡΡ ΠΌΠ΅ΠΆΠ΄Ρ ΡΠ΅Π»ΡΡΠ°ΠΌΠΈ, ΠΊΠΎΠ³Π΄Π° ΡΡΠΎΡ ΡΡΠ΅ΡΠΆΠ΅Π½Ρ Π΄Π²ΠΈΠΆΠ΅ΡΡΡ Π²ΠΏΡΠ°Π²ΠΎ Π² ΠΎΠ΄Π½ΠΎΡΠΎΠ΄Π½ΠΎΠΌ ΠΌΠ°Π³Π½ΠΈΡΠ½ΠΎΠΌ ΠΏΠΎΠ»Π΅. ΠΠ°Π³Π½ΠΈΡΠ½ΠΎΠ΅ ΠΏΠΎΠ»Π΅ B Π½Π°ΠΏΡΠ°Π²Π»Π΅Π½ΠΎ Π²Π½ΡΡΡΡ ΡΡΡΠ°Π½ΠΈΡΡ, ΠΏΠ΅ΡΠΏΠ΅Π½Π΄ΠΈΠΊΡΠ»ΡΡΠ½ΠΎ Π΄Π²ΠΈΠΆΡΡΠΈΠΌΡΡ ΡΡΠ΅ΡΠΆΠ½Ρ ΠΈ ΡΠ΅Π»ΡΡΠ°ΠΌ ΠΈ, ΡΠ»Π΅Π΄ΠΎΠ²Π°ΡΠ΅Π»ΡΠ½ΠΎ, ΠΎΠ³ΡΠ°Π½ΠΈΡΠ΅Π½Π½ΠΎΠΉ ΠΈΠΌΠΈ ΠΎΠ±Π»Π°ΡΡΠΈ. (b) ΠΠ°ΠΊΠΎΠ½ ΠΠ΅Π½ΡΠ° Π΄Π°Π΅Ρ Π½Π°ΠΏΡΠ°Π²Π»Π΅Π½ΠΈΡ ΠΈΠ½Π΄ΡΡΠΈΡΠΎΠ²Π°Π½Π½ΠΎΠ³ΠΎ ΠΏΠΎΠ»Ρ ΠΈ ΡΠΎΠΊΠ°, Π° ΡΠ°ΠΊΠΆΠ΅ ΠΏΠΎΠ»ΡΡΠ½ΠΎΡΡΡ ΠΈΠ½Π΄ΡΡΠΈΡΠΎΠ²Π°Π½Π½ΠΎΠΉ ΠΠΠ‘. ΠΠΎΡΠΊΠΎΠ»ΡΠΊΡ ΠΏΠΎΡΠΎΠΊ ΡΠ²Π΅Π»ΠΈΡΠΈΠ²Π°Π΅ΡΡΡ, ΠΈΠ½Π΄ΡΡΠΈΡΠΎΠ²Π°Π½Π½ΠΎΠ΅ ΠΏΠΎΠ»Π΅ Π½Π°ΠΏΡΠ°Π²Π»Π΅Π½ΠΎ Π² ΠΏΡΠΎΡΠΈΠ²ΠΎΠΏΠΎΠ»ΠΎΠΆΠ½ΠΎΠΌ Π½Π°ΠΏΡΠ°Π²Π»Π΅Π½ΠΈΠΈ ΠΈΠ»ΠΈ Π²ΡΡ ΠΎΠ΄ΠΈΡ Π·Π° ΠΏΡΠ΅Π΄Π΅Π»Ρ ΡΡΡΠ°Π½ΠΈΡΡ. ΠΡΠ°Π²ΠΈΠ»ΠΎ ΠΏΡΠ°Π²ΠΎΠΉ ΡΡΠΊΠΈ Π΄Π°Π΅Ρ ΠΏΠΎΠΊΠ°Π·Π°Π½Π½ΠΎΠ΅ Π½Π°ΠΏΡΠ°Π²Π»Π΅Π½ΠΈΠ΅ ΡΠΎΠΊΠ°, ΠΈ ΠΏΠΎΠ»ΡΡΠ½ΠΎΡΡΡ ΡΡΠ΅ΡΠΆΠ½Ρ Π±ΡΠ΄Π΅Ρ ΡΠΏΡΠ°Π²Π»ΡΡΡ ΡΠ°ΠΊΠΈΠΌ ΡΠΎΠΊΠΎΠΌ.
Π§ΡΠΎΠ±Ρ Π½Π°ΠΉΡΠΈ Π²Π΅Π»ΠΈΡΠΈΠ½Ρ ΠΠΠ‘, ΠΈΠ½Π΄ΡΡΠΈΡΠΎΠ²Π°Π½Π½ΠΎΠΉ Π²Π΄ΠΎΠ»Ρ Π΄Π²ΠΈΠΆΡΡΠ΅Π³ΠΎΡΡ ΡΡΠ΅ΡΠΆΠ½Ρ, Π²ΠΎΡΠΏΠΎΠ»ΡΠ·ΡΠ΅ΠΌΡΡ Π·Π°ΠΊΠΎΠ½ΠΎΠΌ ΠΈΠ½Π΄ΡΠΊΡΠΈΠΈ Π€Π°ΡΠ°Π΄Π΅Ρ Π±Π΅Π· Π·Π½Π°ΠΊΠ°:
[Π»Π°ΡΠ΅ΠΊΡ]\text{ΠΠΠ‘} = \text{N}\frac{\Delta \Phi}{\ ΠΠ΅Π»ΡΡΠ° \text{t}}[/latex].
Π ΡΡΠΎΠΌ ΡΡΠ°Π²Π½Π΅Π½ΠΈΠΈ N=1 ΠΈ ΠΏΠΎΡΠΎΠΊ Ξ¦=BAcosΞΈ. ΠΠΌΠ΅Π΅ΠΌ ΞΈ=0ΒΊ ΠΈ cosΞΈ=1, ΡΠ°ΠΊ ΠΊΠ°ΠΊ B ΠΏΠ΅ΡΠΏΠ΅Π½Π΄ΠΈΠΊΡΠ»ΡΡΠ½ΠΎ A. Π’Π΅ΠΏΠ΅ΡΡ Ξ=Ξ(BA)=BΞA, ΡΠ°ΠΊ ΠΊΠ°ΠΊ B ΠΎΠ΄Π½ΠΎΡΠΎΠ΄Π½ΠΎ. ΠΠ±ΡΠ°ΡΠΈΡΠ΅ Π²Π½ΠΈΠΌΠ°Π½ΠΈΠ΅, ΡΡΠΎ ΠΏΠ»ΠΎΡΠ°Π΄Ρ, Π·Π°ΠΌΠ΅ΡΠ°Π΅ΠΌΠ°Ρ ΡΡΠ΅ΡΠΆΠ½Π΅ΠΌ, ΡΠ°Π²Π½Π° ΞA=βx. ΠΠ²ΠΎΠ΄ ΡΡΠΈΡ Π²Π΅Π»ΠΈΡΠΈΠ½ Π² Π²ΡΡΠ°ΠΆΠ΅Π½ΠΈΠ΅ Π΄Π»Ρ ΠΠΠ‘ Π΄Π°Π΅Ρ:
[Π»Π°ΡΠ΅ΠΊΡ]\ΡΠ΅ΠΊΡΡ{ΠΠΠ‘} = \frac{\text{B}\Delta \text{A}}{\Delta \text{t}} = \text{ B} \frac{\text{l} \Delta \text{x}}{\Delta \text{t}} = \text{Blv}[/latex].
Π§ΡΠΎΠ±Ρ Π½Π°ΠΉΡΠΈ Π½Π°ΠΏΡΠ°Π²Π»Π΅Π½ΠΈΠ΅ ΠΈΠ½Π΄ΡΡΠΈΡΠΎΠ²Π°Π½Π½ΠΎΠ³ΠΎ ΠΏΠΎΠ»Ρ, Π½Π°ΠΏΡΠ°Π²Π»Π΅Π½ΠΈΠ΅ ΡΠΎΠΊΠ° ΠΈ ΠΏΠΎΠ»ΡΡΠ½ΠΎΡΡΡ ΠΈΠ½Π΄ΡΡΠΈΡΠΎΠ²Π°Π½Π½ΠΎΠΉ ΠΠΠ‘, ΠΌΡ ΠΏΡΠΈΠΌΠ΅Π½ΡΠ΅ΠΌ Π·Π°ΠΊΠΎΠ½ ΠΠ΅Π½ΡΠ°, ΠΊΠ°ΠΊ ΠΎΠΏΠΈΡΠ°Π½ΠΎ Π² ΠΠ°ΠΊΠΎΠ½Π΅ Π€Π°ΡΠ°Π΄Π΅Ρ ΠΎΠ± ΠΈΠ½Π΄ΡΠΊΡΠΈΠΈ: ΠΠ°ΠΊΠΎΠ½ ΠΠ΅Π½ΡΠ°. ΠΠ°ΠΊ Π²ΠΈΠ΄Π½ΠΎ Π½Π° ΡΠΈΡ. 1 (b), ΠΏΠΎΡΠΎΠΊ ΡΠ²Π΅Π»ΠΈΡΠΈΠ²Π°Π΅ΡΡΡ, ΡΠ°ΠΊ ΠΊΠ°ΠΊ ΡΠ²Π΅Π»ΠΈΡΠΈΠ²Π°Π΅ΡΡΡ ΠΏΠ»ΠΎΡΠ°Π΄Ρ ΠΎΡ Π²Π°ΡΠ°. Π’Π°ΠΊΠΈΠΌ ΠΎΠ±ΡΠ°Π·ΠΎΠΌ, ΠΈΠ½Π΄ΡΡΠΈΡΠΎΠ²Π°Π½Π½ΠΎΠ΅ ΠΏΠΎΠ»Π΅ Π΄ΠΎΠ»ΠΆΠ½ΠΎ ΠΏΡΠΎΡΠΈΠ²ΠΎΡΡΠΎΡΡΡ ΡΡΡΠ΅ΡΡΠ²ΡΡΡΠ΅ΠΌΡ ΠΈ Π±ΡΡΡ Π·Π° ΠΏΡΠ΅Π΄Π΅Π»Π°ΠΌΠΈ ΡΡΡΠ°Π½ΠΈΡΡ. (ΠΡΠ°Π²ΠΈΠ»ΠΎ ΠΏΡΠ°Π²ΠΎΠΉ ΡΡΠΊΠΈ ΡΡΠ΅Π±ΡΠ΅Ρ, ΡΡΠΎΠ±Ρ Ρ Π²ΡΠ°ΡΠ°Π»ΡΡ ΠΏΡΠΎΡΠΈΠ² ΡΠ°ΡΠΎΠ²ΠΎΠΉ ΡΡΡΠ΅Π»ΠΊΠΈ, ΡΡΠΎ, Π² ΡΠ²ΠΎΡ ΠΎΡΠ΅ΡΠ΅Π΄Ρ, ΠΎΠ·Π½Π°ΡΠ°Π΅Ρ, ΡΡΠΎ Π²Π΅ΡΡΠΈΠ½Π° ΡΡΠ΅ΡΠΆΠ½Ρ ΠΏΠΎΠ»ΠΎΠΆΠΈΡΠ΅Π»ΡΠ½Π°, ΠΊΠ°ΠΊ ΠΏΠΎΠΊΠ°Π·Π°Π½ΠΎ Π½Π° ΡΠΈΡΡΠ½ΠΊΠ΅.)
ΠΠ»Π΅ΠΊΡΡΠΈΡΠ΅ΡΠΊΠΎΠ΅ ΠΏΠΎΠ»Π΅ ΠΏΡΠΎΡΠΈΠ² ΠΌΠ°Π³Π½ΠΈΡΠ½ΠΎΠ³ΠΎ ΠΏΠΎΠ»Ρ
Π‘ΡΡΠ΅ΡΡΠ²ΡΠ΅Ρ ΠΌΠ½ΠΎΠ³ΠΎ ΡΠ²ΡΠ·Π΅ΠΉ ΠΌΠ΅ΠΆΠ΄Ρ ΡΠ»Π΅ΠΊΡΡΠΈΡΠ΅ΡΠΊΠΎΠΉ ΠΈ ΠΌΠ°Π³Π½ΠΈΡΠ½ΠΎΠΉ ΡΠΈΠ»Π°ΠΌΠΈ. Π’ΠΎ, ΡΡΠΎ Π΄Π²ΠΈΠΆΡΡΠ΅Π΅ΡΡ ΠΌΠ°Π³Π½ΠΈΡΠ½ΠΎΠ΅ ΠΏΠΎΠ»Π΅ ΡΠΎΠ·Π΄Π°Π΅Ρ ΡΠ»Π΅ΠΊΡΡΠΈΡΠ΅ΡΠΊΠΎΠ΅ ΠΏΠΎΠ»Π΅ (ΠΈ Π½Π°ΠΎΠ±ΠΎΡΠΎΡ, ΡΡΠΎ Π΄Π²ΠΈΠΆΡΡΠ΅Π΅ΡΡ ΡΠ»Π΅ΠΊΡΡΠΈΡΠ΅ΡΠΊΠΎΠ΅ ΠΏΠΎΠ»Π΅ ΡΠΎΠ·Π΄Π°Π΅Ρ ΠΌΠ°Π³Π½ΠΈΡΠ½ΠΎΠ΅ ΠΏΠΎΠ»Π΅), ΡΠ²Π»ΡΠ΅ΡΡΡ ΠΎΠ΄Π½ΠΎΠΉ ΠΈΠ· ΠΏΡΠΈΡΠΈΠ½, ΠΏΠΎ ΠΊΠΎΡΠΎΡΠΎΠΉ ΡΠ»Π΅ΠΊΡΡΠΈΡΠ΅ΡΠΊΠΈΠ΅ ΠΈ ΠΌΠ°Π³Π½ΠΈΡΠ½ΡΠ΅ ΡΠΈΠ»Ρ ΡΠ΅ΠΏΠ΅ΡΡ ΡΠ°ΡΡΠΌΠ°ΡΡΠΈΠ²Π°ΡΡΡΡ ΠΊΠ°ΠΊ ΡΠ°Π·Π»ΠΈΡΠ½ΡΡ ΠΏΡΠΎΡΠ²Π»Π΅Π½ΠΈΠΉ ΠΎΠ΄Π½ΠΎΠΉ ΠΈ ΡΠΎΠΉ ΠΆΠ΅ ΡΠΈΠ»Ρ (Π²ΠΏΠ΅ΡΠ²ΡΠ΅ Π·Π°ΠΌΠ΅ΡΠ΅Π½Π½ΠΎΠΉ ΠΠ»ΡΠ±Π΅ΡΡΠΎΠΌ ΠΠΉΠ½ΡΡΠ΅ΠΉΠ½ΠΎΠΌ). . ΠΡΠΎ ΠΊΠ»Π°ΡΡΠΈΡΠ΅ΡΠΊΠΎΠ΅ ΠΎΠ±ΡΠ΅Π΄ΠΈΠ½Π΅Π½ΠΈΠ΅ ΡΠ»Π΅ΠΊΡΡΠΈΡΠ΅ΡΠΊΠΈΡ ΠΈ ΠΌΠ°Π³Π½ΠΈΡΠ½ΡΡ ΡΠΈΠ» Π² ΡΠΎ, ΡΡΠΎ Π½Π°Π·ΡΠ²Π°Π΅ΡΡΡ ΡΠ»Π΅ΠΊΡΡΠΎΠΌΠ°Π³Π½ΠΈΡΠ½ΠΎΠΉ ΡΠΈΠ»ΠΎΠΉ , ΡΠ²Π»ΡΠ΅ΡΡΡ ΠΈΡΡΠΎΡΠ½ΠΈΠΊΠΎΠΌ Π²Π΄ΠΎΡ Π½ΠΎΠ²Π΅Π½ΠΈΡ Π΄Π»Ρ ΡΠΎΠ²ΡΠ΅ΠΌΠ΅Π½Π½ΡΡ ΡΡΠΈΠ»ΠΈΠΉ ΠΏΠΎ ΠΎΠ±ΡΠ΅Π΄ΠΈΠ½Π΅Π½ΠΈΡ Π΄ΡΡΠ³ΠΈΡ Π±Π°Π·ΠΎΠ²ΡΡ ΡΠΈΠ».
ΠΠ±ΡΠ°ΡΠ½Π°Ρ ΠΠΠ‘, Π²ΠΈΡ ΡΠ΅Π²ΡΠ΅ ΡΠΎΠΊΠΈ ΠΈ ΠΌΠ°Π³Π½ΠΈΡΠ½ΠΎΠ΅ Π΄Π΅ΠΌΠΏΡΠΈΡΠΎΠ²Π°Π½ΠΈΠ΅
ΠΠ±ΡΠ°ΡΠ½Π°Ρ ΠΠΠ‘, Π²ΠΈΡ ΡΠ΅Π²ΡΠ΅ ΡΠΎΠΊΠΈ ΠΈ ΠΌΠ°Π³Π½ΠΈΡΠ½ΠΎΠ΅ Π΄Π΅ΠΌΠΏΡΠΈΡΠΎΠ²Π°Π½ΠΈΠ΅ Π²ΠΎΠ·Π½ΠΈΠΊΠ°ΡΡ ΠΈΠ·-Π·Π° Π½Π°Π²Π΅Π΄Π΅Π½Π½ΠΎΠΉ ΠΠΠ‘ ΠΈ ΠΌΠΎΠ³ΡΡ Π±ΡΡΡ ΠΎΠ±ΡΡΡΠ½Π΅Π½Ρ Π·Π°ΠΊΠΎΠ½ΠΎΠΌ ΠΈΠ½Π΄ΡΠΊΡΠΈΠΈ Π€Π°ΡΠ°Π΄Π΅Ρ.
Π¦Π΅Π»ΠΈ ΠΎΠ±ΡΡΠ΅Π½ΠΈΡ
ΠΠ±ΡΡΡΠ½ΠΈΡΡ Π²Π·Π°ΠΈΠΌΠΎΡΠ²ΡΠ·Ρ ΠΌΠ΅ΠΆΠ΄Ρ Π΄Π²ΠΈΠΆΡΡΠ΅ΠΉ ΡΠ»Π΅ΠΊΡΡΠΎΠ΄Π²ΠΈΠΆΡΡΠ΅ΠΉ ΡΠΈΠ»ΠΎΠΉ, Π²ΠΈΡ ΡΠ΅Π²ΡΠΌΠΈ ΡΠΎΠΊΠ°ΠΌΠΈ ΠΈ ΠΌΠ°Π³Π½ΠΈΡΠ½ΡΠΌ Π΄Π΅ΠΌΠΏΡΠΈΡΠΎΠ²Π°Π½ΠΈΠ΅ΠΌ ΠΏΡΠΎΡΠΈΠ²ΠΎ-ΠΠΠ‘ Π΄Π²ΠΈΠ³Π°ΡΠ΅Π»Ρ.
ΠΡΠ½ΠΎΠ²Π½ΡΠ΅ ΡΠ΅ΡΠΌΠΈΠ½Ρ
- ΡΠ»Π΅ΠΊΡΡΠΎΠ΄Π²ΠΈΠΆΡΡΠ°Ρ ΡΠΈΠ»Π° : (ΠΠΠ‘) β Π½Π°ΠΏΡΡΠΆΠ΅Π½ΠΈΠ΅, ΡΠΎΠ·Π΄Π°Π²Π°Π΅ΠΌΠΎΠ΅ Π±Π°ΡΠ°ΡΠ΅Π΅ΠΉ ΠΈΠ»ΠΈ ΠΌΠ°Π³Π½ΠΈΡΠ½ΠΎΠΉ ΡΠΈΠ»ΠΎΠΉ Π² ΡΠΎΠΎΡΠ²Π΅ΡΡΡΠ²ΠΈΠΈ Ρ Π·Π°ΠΊΠΎΠ½ΠΎΠΌ Π€Π°ΡΠ°Π΄Π΅Ρ. ΠΠ½Π° ΠΈΠ·ΠΌΠ΅ΡΡΠ΅ΡΡΡ Π² Π²ΠΎΠ»ΡΡΠ°Ρ , Π° Π½Π΅ Π² Π½ΡΡΡΠΎΠ½Π°Ρ , ΠΈ, ΡΠ»Π΅Π΄ΠΎΠ²Π°ΡΠ΅Π»ΡΠ½ΠΎ, Π½Π° ΡΠ°ΠΌΠΎΠΌ Π΄Π΅Π»Π΅ Π½Π΅ ΡΠ²Π»ΡΠ΅ΡΡΡ ΡΠΈΠ»ΠΎΠΉ.
- ΠΠ°ΠΊΠΎΠ½ ΠΈΠ½Π΄ΡΠΊΡΠΈΠΈ Π€Π°ΡΠ°Π΄Π΅Ρ : ΠΡΠ½ΠΎΠ²Π½ΠΎΠΉ Π·Π°ΠΊΠΎΠ½ ΡΠ»Π΅ΠΊΡΡΠΎΠΌΠ°Π³Π½Π΅ΡΠΈΠ·ΠΌΠ°, ΠΊΠΎΡΠΎΡΡΠΉ ΠΏΡΠ΅Π΄ΡΠΊΠ°Π·ΡΠ²Π°Π΅Ρ, ΠΊΠ°ΠΊ ΠΌΠ°Π³Π½ΠΈΡΠ½ΠΎΠ΅ ΠΏΠΎΠ»Π΅ Π±ΡΠ΄Π΅Ρ Π²Π·Π°ΠΈΠΌΠΎΠ΄Π΅ΠΉΡΡΠ²ΠΎΠ²Π°ΡΡ Ρ ΡΠ»Π΅ΠΊΡΡΠΈΡΠ΅ΡΠΊΠΎΠΉ ΡΠ΅ΠΏΡΡ, ΡΠΎΠ·Π΄Π°Π²Π°Ρ ΡΠ»Π΅ΠΊΡΡΠΎΠ΄Π²ΠΈΠΆΡΡΡΡ ΡΠΈΠ»Ρ (ΠΠΠ‘).
ΠΠ±ΡΠ°ΡΠ½Π°Ρ ΠΠΠ‘
ΠΠ²ΠΈΠ³Π°ΡΠ΅Π»ΠΈ ΠΈ Π³Π΅Π½Π΅ΡΠ°ΡΠΎΡΡ ΠΎΡΠ΅Π½Ρ ΠΏΠΎΡ ΠΎΠΆΠΈ. (Π§ΠΈΡΠ°ΠΉΡΠ΅ Π½Π°ΡΠΈ Π°ΡΠΎΠΌΡ Π² ΡΠ°Π·Π΄Π΅Π»Π΅ Β«ΠΠ»Π΅ΠΊΡΡΠΈΡΠ΅ΡΠΊΠΈΠ΅ Π³Π΅Π½Π΅ΡΠ°ΡΠΎΡΡΒ» ΠΈ Β«ΠΠ»Π΅ΠΊΡΡΠΎΠ΄Π²ΠΈΠ³Π°ΡΠ΅Π»ΠΈΒ».) ΠΠ΅Π½Π΅ΡΠ°ΡΠΎΡΡ ΠΏΡΠ΅ΠΎΠ±ΡΠ°Π·ΡΡΡ ΠΌΠ΅Ρ Π°Π½ΠΈΡΠ΅ΡΠΊΡΡ ΡΠ½Π΅ΡΠ³ΠΈΡ Π² ΡΠ»Π΅ΠΊΡΡΠΈΡΠ΅ΡΠΊΡΡ, ΡΠΎΠ³Π΄Π° ΠΊΠ°ΠΊ Π΄Π²ΠΈΠ³Π°ΡΠ΅Π»ΠΈ ΠΏΡΠ΅ΠΎΠ±ΡΠ°Π·ΡΡΡ ΡΠ»Π΅ΠΊΡΡΠΈΡΠ΅ΡΠΊΡΡ ΡΠ½Π΅ΡΠ³ΠΈΡ Π² ΠΌΠ΅Ρ Π°Π½ΠΈΡΠ΅ΡΠΊΡΡ. ΠΡΠΎΠΌΠ΅ ΡΠΎΠ³ΠΎ, Π΄Π²ΠΈΠ³Π°ΡΠ΅Π»ΠΈ ΠΈ Π³Π΅Π½Π΅ΡΠ°ΡΠΎΡΡ ΠΈΠΌΠ΅ΡΡ ΠΎΠ΄ΠΈΠ½Π°ΠΊΠΎΠ²ΡΡ ΠΊΠΎΠ½ΡΡΡΡΠΊΡΠΈΡ. ΠΠΎΠ³Π΄Π° ΠΊΠ°ΡΡΡΠΊΠ° Π΄Π²ΠΈΠ³Π°ΡΠ΅Π»Ρ ΠΏΠΎΠ²ΠΎΡΠ°ΡΠΈΠ²Π°Π΅ΡΡΡ, ΠΌΠ°Π³Π½ΠΈΡΠ½ΡΠΉ ΠΏΠΎΡΠΎΠΊ ΠΈΠ·ΠΌΠ΅Π½ΡΠ΅ΡΡΡ, ΠΈ ΠΈΠ½Π΄ΡΡΠΈΡΡΠ΅ΡΡΡ ΡΠ»Π΅ΠΊΡΡΠΎΠ΄Π²ΠΈΠΆΡΡΠ°Ρ ΡΠΈΠ»Π° (ΠΠΠ‘), ΡΠΎΠΎΡΠ²Π΅ΡΡΡΠ²ΡΡΡΠ°Ρ Π·Π°ΠΊΠΎΠ½Ρ ΠΈΠ½Π΄ΡΠΊΡΠΈΠΈ Π€Π°ΡΠ°Π΄Π΅Ρ. Π’Π°ΠΊΠΈΠΌ ΠΎΠ±ΡΠ°Π·ΠΎΠΌ, Π΄Π²ΠΈΠ³Π°ΡΠ΅Π»Ρ Π΄Π΅ΠΉΡΡΠ²ΡΠ΅Ρ ΠΊΠ°ΠΊ Π³Π΅Π½Π΅ΡΠ°ΡΠΎΡ Π²ΡΡΠΊΠΈΠΉ ΡΠ°Π·, ΠΊΠΎΠ³Π΄Π° Π΅Π³ΠΎ ΠΊΠ°ΡΡΡΠΊΠ° Π²ΡΠ°ΡΠ°Π΅ΡΡΡ. ΠΡΠΎ ΠΏΡΠΎΠΈΠ·ΠΎΠΉΠ΄Π΅Ρ Π½Π΅Π·Π°Π²ΠΈΡΠΈΠΌΠΎ ΠΎΡ ΡΠΎΠ³ΠΎ, Π²ΡΠ°ΡΠ°Π΅ΡΡΡ Π»ΠΈ Π²Π°Π» Ρ ΠΏΠΎΠΌΠΎΡΡΡ Π²Π½Π΅ΡΠ½Π΅Π³ΠΎ ΠΈΡΡΠΎΡΠ½ΠΈΠΊΠ°, ΡΠ°ΠΊΠΎΠ³ΠΎ ΠΊΠ°ΠΊ ΡΠ΅ΠΌΠ΅Π½Π½Π°Ρ ΠΏΠ΅ΡΠ΅Π΄Π°ΡΠ°, ΠΈΠ»ΠΈ ΠΏΠΎΠ΄ Π΄Π΅ΠΉΡΡΠ²ΠΈΠ΅ΠΌ ΡΠ°ΠΌΠΎΠ³ΠΎ Π΄Π²ΠΈΠ³Π°ΡΠ΅Π»Ρ. Π’ΠΎ Π΅ΡΡΡ, ΠΊΠΎΠ³Π΄Π° Π΄Π²ΠΈΠ³Π°ΡΠ΅Π»Ρ ΡΠΎΠ²Π΅ΡΡΠ°Π΅Ρ ΡΠ°Π±ΠΎΡΡ ΠΈ Π΅Π³ΠΎ Π²Π°Π» Π²ΡΠ°ΡΠ°Π΅ΡΡΡ, Π²ΠΎΠ·Π½ΠΈΠΊΠ°Π΅Ρ ΠΠΠ‘. ΠΠ°ΠΊΠΎΠ½ ΠΠ΅Π½ΡΠ° Π³ΠΎΠ²ΠΎΡΠΈΡ Π½Π°ΠΌ, ΡΡΠΎ ΠΠΠ‘ ΠΈΠ½Π΄ΡΠΊΡΠΈΠΈ ΠΏΡΠΎΡΠΈΠ²ΠΎΡΡΠΎΠΈΡ Π»ΡΠ±ΡΠΌ ΠΈΠ·ΠΌΠ΅Π½Π΅Π½ΠΈΡΠΌ, ΠΏΠΎΡΡΠΎΠΌΡ Π²Ρ ΠΎΠ΄Π½ΠΎΠΉ ΠΠΠ‘, ΠΏΠΈΡΠ°ΡΡΠ΅ΠΉ Π΄Π²ΠΈΠ³Π°ΡΠ΅Π»Ρ, Π±ΡΠ΄Π΅Ρ ΠΏΡΠΎΡΠΈΠ²ΠΎΠ΄Π΅ΠΉΡΡΠ²ΠΎΠ²Π°ΡΡ ΠΠΠ‘, ΡΠΎΠ·Π΄Π°Π²Π°Π΅ΠΌΠ°Ρ ΡΠ°ΠΌΠΈΠΌ Π΄Π²ΠΈΠ³Π°ΡΠ΅Π»Π΅ΠΌ, Π½Π°Π·ΡΠ²Π°Π΅ΠΌΠ°Ρ ΠΎΠ±ΡΠ°ΡΠ½ΠΎΠΉ ΠΠΠ‘ Π΄Π²ΠΈΠ³Π°ΡΠ΅Π»Ρ.
ΠΠΈΡ ΡΠ΅Π²ΠΎΠΉ ΡΠΎΠΊ
ΠΠ°ΠΊ ΠΎΠ±ΡΡΠΆΠ΄Π°Π»ΠΎΡΡ Π² ΡΠ°Π·Π΄Π΅Π»Π΅ Β«ΠΠΠ‘ Π΄Π²ΠΈΠΆΠ΅Π½ΠΈΡΒ», ΠΠΠ‘ Π΄Π²ΠΈΠΆΠ΅Π½ΠΈΡ ΠΈΠ½Π΄ΡΡΠΈΡΡΠ΅ΡΡΡ, ΠΊΠΎΠ³Π΄Π° ΠΏΡΠΎΠ²ΠΎΠ΄Π½ΠΈΠΊ Π΄Π²ΠΈΠΆΠ΅ΡΡΡ Π² ΠΌΠ°Π³Π½ΠΈΡΠ½ΠΎΠΌ ΠΏΠΎΠ»Π΅ ΠΈΠ»ΠΈ ΠΊΠΎΠ³Π΄Π° ΠΌΠ°Π³Π½ΠΈΡΠ½ΠΎΠ΅ ΠΏΠΎΠ»Π΅ Π΄Π²ΠΈΠΆΠ΅ΡΡΡ ΠΎΡΠ½ΠΎΡΠΈΡΠ΅Π»ΡΠ½ΠΎ ΠΏΡΠΎΠ²ΠΎΠ΄Π½ΠΈΠΊΠ°. ΠΡΠ»ΠΈ ΠΠΠ‘ Π΄Π²ΠΈΠΆΠ΅Π½ΠΈΡ ΠΌΠΎΠΆΠ΅Ρ Π²ΡΠ·Π²Π°ΡΡ ΠΏΠ΅ΡΠ»Ρ Ρ ΡΠΎΠΊΠΎΠΌ Π² ΠΏΡΠΎΠ²ΠΎΠ΄Π½ΠΈΠΊΠ΅, ΠΌΡ Π½Π°Π·ΡΠ²Π°Π΅ΠΌ ΡΡΠΎΡ ΡΠΎΠΊ Π²ΠΈΡ ΡΠ΅Π²ΡΠΌ ΡΠΎΠΊΠΎΠΌ. ΠΠΈΡ ΡΠ΅Π²ΡΠ΅ ΡΠΎΠΊΠΈ ΠΌΠΎΠ³ΡΡ ΡΠΎΠ·Π΄Π°Π²Π°ΡΡ Π·Π½Π°ΡΠΈΡΠ΅Π»ΡΠ½ΠΎΠ΅ ΡΠΎΠΏΡΠΎΡΠΈΠ²Π»Π΅Π½ΠΈΠ΅, Π½Π°Π·ΡΠ²Π°Π΅ΠΌΠΎΠ΅ ΠΌΠ°Π³Π½ΠΈΡΠ½ΡΠΌ Π΄Π΅ΠΌΠΏΡΠΈΡΠΎΠ²Π°Π½ΠΈΠ΅ΠΌ, ΠΏΡΠΈ ΡΠΎΠΎΡΠ²Π΅ΡΡΡΠ²ΡΡΡΠ΅ΠΌ Π΄Π²ΠΈΠΆΠ΅Π½ΠΈΠΈ.
Π Π°ΡΡΠΌΠΎΡΡΠΈΠΌ ΡΡΡΡΠΎΠΉΡΡΠ²ΠΎ, ΠΏΠΎΠΊΠ°Π·Π°Π½Π½ΠΎΠ΅ Π½Π° ΡΠΈΡ., ΠΊΠΎΡΠΎΡΠΎΠ΅ ΡΠ°ΡΠΊΠ°ΡΠΈΠ²Π°Π΅Ρ ΠΌΠ°ΡΡΠ½ΠΈΠΊ ΠΌΠ΅ΠΆΠ΄Ρ ΠΏΠΎΠ»ΡΡΠ°ΠΌΠΈ ΡΠΈΠ»ΡΠ½ΠΎΠ³ΠΎ ΠΌΠ°Π³Π½ΠΈΡΠ°. ΠΡΠ»ΠΈ ΠΏΠΎΠΏΠ»Π°Π²ΠΎΠΊ ΠΌΠ΅ΡΠ°Π»Π»ΠΈΡΠ΅ΡΠΊΠΈΠΉ, ΠΏΡΠΈ Π²Ρ ΠΎΠ΄Π΅ Π² ΠΏΠΎΠ»Π΅ ΠΈ Π²ΡΡ ΠΎΠ΄Π΅ ΠΈΠ· Π½Π΅Π³ΠΎ Π²ΠΎΠ·Π½ΠΈΠΊΠ°Π΅Ρ Π·Π½Π°ΡΠΈΡΠ΅Π»ΡΠ½ΠΎΠ΅ ΡΠΎΠΏΡΠΎΡΠΈΠ²Π»Π΅Π½ΠΈΠ΅, ΡΡΠΎ Π±ΡΡΡΡΠΎ Π³Π°ΡΠΈΡ Π΄Π²ΠΈΠΆΠ΅Π½ΠΈΠ΅. ΠΡΠ»ΠΈ, ΠΎΠ΄Π½Π°ΠΊΠΎ, Π³ΡΡΠ· ΠΏΡΠ΅Π΄ΡΡΠ°Π²Π»ΡΠ΅Ρ ΡΠΎΠ±ΠΎΠΉ ΠΌΠ΅ΡΠ°Π»Π»ΠΈΡΠ΅ΡΠΊΡΡ ΠΏΠ»Π°ΡΡΠΈΠ½Ρ Ρ ΠΏΡΠΎΡΠ΅Π·ΡΠΌΠΈ, ΠΊΠ°ΠΊ ΠΏΠΎΠΊΠ°Π·Π°Π½ΠΎ Π½Π° (b), ΡΡΡΠ΅ΠΊΡ ΠΌΠ°Π³Π½ΠΈΡΠ° Π±ΡΠ΄Π΅Ρ Π½Π°ΠΌΠ½ΠΎΠ³ΠΎ ΠΌΠ΅Π½ΡΡΠ΅. ΠΠΈΠΊΠ°ΠΊΠΎΠ³ΠΎ Π·Π°ΠΌΠ΅ΡΠ½ΠΎΠ³ΠΎ Π²Π»ΠΈΡΠ½ΠΈΡ Π½Π° Π±ΠΎΠ±, ΡΠ΄Π΅Π»Π°Π½Π½ΡΠΉ ΠΈΠ· ΠΈΠ·ΠΎΠ»ΡΡΠΎΡΠ°, Π½Π΅ Π½Π°Π±Π»ΡΠ΄Π°Π΅ΡΡΡ.
Π£ΡΡΡΠΎΠΉΡΡΠ²ΠΎ Π΄Π»Ρ ΠΈΠ·ΡΡΠ΅Π½ΠΈΡ Π²ΠΈΡ ΡΠ΅Π²ΡΡ ΡΠΎΠΊΠΎΠ² ΠΈ ΠΌΠ°Π³Π½ΠΈΡΠ½ΠΎΠ³ΠΎ Π΄Π΅ΠΌΠΏΡΠΈΡΠΎΠ²Π°Π½ΠΈΡ : ΠΠ±ΡΡΠ½ΠΎΠ΅ ΡΠΈΠ·ΠΈΡΠ΅ΡΠΊΠΎΠ΅ Π΄Π΅ΠΌΠΎΠ½ΡΡΡΠ°ΡΠΈΠΎΠ½Π½ΠΎΠ΅ ΡΡΡΡΠΎΠΉΡΡΠ²ΠΎ Π΄Π»Ρ ΠΈΠ·ΡΡΠ΅Π½ΠΈΡ Π²ΠΈΡ ΡΠ΅Π²ΡΡ ΡΠΎΠΊΠΎΠ² ΠΈ ΠΌΠ°Π³Π½ΠΈΡΠ½ΠΎΠ³ΠΎ Π΄Π΅ΠΌΠΏΡΠΈΡΠΎΠ²Π°Π½ΠΈΡ. Π°) ΠΠ²ΠΈΠΆΠ΅Π½ΠΈΠ΅ ΠΌΠ΅ΡΠ°Π»Π»ΠΈΡΠ΅ΡΠΊΠΎΠ³ΠΎ ΠΌΠ°ΡΡΠ½ΠΈΠΊΠ°, ΠΊΠ°ΡΠ°ΡΡΠ΅Π³ΠΎΡΡ ΠΌΠ΅ΠΆΠ΄Ρ ΠΏΠΎΠ»ΡΡΠ°ΠΌΠΈ ΠΌΠ°Π³Π½ΠΈΡΠ°, Π±ΡΡΡΡΠΎ Π·Π°ΡΡΡ Π°Π΅Ρ ΠΏΠΎΠ΄ Π΄Π΅ΠΉΡΡΠ²ΠΈΠ΅ΠΌ Π²ΠΈΡ ΡΠ΅Π²ΡΡ ΡΠΎΠΊΠΎΠ². (b) ΠΠ° Π΄Π²ΠΈΠΆΠ΅Π½ΠΈΠ΅ ΠΌΠ΅ΡΠ°Π»Π»ΠΈΡΠ΅ΡΠΊΠΎΠ³ΠΎ ΠΏΠΎΠΏΠ»Π°Π²ΠΊΠ° Ρ ΠΏΡΠΎΡΠ΅Π·ΡΠΌΠΈ ΠΌΠ°Π»ΠΎ Π²Π»ΠΈΡΠ΅Ρ, ΡΡΠΎ ΠΎΠ·Π½Π°ΡΠ°Π΅Ρ, ΡΡΠΎ Π²ΠΈΡ ΡΠ΅Π²ΡΠ΅ ΡΠΎΠΊΠΈ ΡΡΠ°Π½ΠΎΠ²ΡΡΡΡ ΠΌΠ΅Π½Π΅Π΅ ΡΡΡΠ΅ΠΊΡΠΈΠ²Π½ΡΠΌΠΈ. (c) ΠΠ° Π½Π΅ΠΏΡΠΎΠ²ΠΎΠ΄ΡΡΠ΅ΠΌ Π³ΡΡΠ·Π΅ ΡΠ°ΠΊΠΆΠ΅ Π½Π΅Ρ ΠΌΠ°Π³Π½ΠΈΡΠ½ΠΎΠ³ΠΎ Π΄Π΅ΠΌΠΏΡΠΈΡΠΎΠ²Π°Π½ΠΈΡ, ΠΏΠΎΡΠΊΠΎΠ»ΡΠΊΡ Π²ΠΈΡ ΡΠ΅Π²ΡΠ΅ ΡΠΎΠΊΠΈ ΡΡΠ΅Π·Π²ΡΡΠ°ΠΉΠ½ΠΎ ΠΌΠ°Π»Ρ.
ΠΏΠΎΠΊΠ°Π·ΡΠ²Π°Π΅Ρ, ΡΡΠΎ ΠΏΡΠΎΠΈΡΡ ΠΎΠ΄ΠΈΡ Ρ ΠΌΠ΅ΡΠ°Π»Π»ΠΈΡΠ΅ΡΠΊΠΎΠΉ ΠΏΠ»Π°ΡΡΠΈΠ½ΠΎΠΉ, ΠΊΠΎΠ³Π΄Π° ΠΎΠ½Π° Π²Ρ ΠΎΠ΄ΠΈΡ ΠΈ Π²ΡΡ ΠΎΠ΄ΠΈΡ ΠΈΠ· ΠΌΠ°Π³Π½ΠΈΡΠ½ΠΎΠ³ΠΎ ΠΏΠΎΠ»Ρ. Π ΠΎΠ±ΠΎΠΈΡ ΡΠ»ΡΡΠ°ΡΡ Π½Π° Π½Π΅Π³ΠΎ Π΄Π΅ΠΉΡΡΠ²ΡΠ΅Ρ ΡΠΈΠ»Π°, ΠΏΡΠΎΡΠΈΠ²ΠΎΠ΄Π΅ΠΉΡΡΠ²ΡΡΡΠ°Ρ Π΅Π³ΠΎ Π΄Π²ΠΈΠΆΠ΅Π½ΠΈΡ. ΠΠΎΠ³Π΄Π° ΠΎΠ½ Π²Ρ ΠΎΠ΄ΠΈΡ ΡΠ»Π΅Π²Π°, ΠΏΠΎΡΠΎΠΊ ΡΠ²Π΅Π»ΠΈΡΠΈΠ²Π°Π΅ΡΡΡ, ΠΏΠΎΡΡΠΎΠΌΡ Π²ΠΎΠ·Π½ΠΈΠΊΠ°Π΅Ρ Π²ΠΈΡ ΡΠ΅Π²ΠΎΠΉ ΡΠΎΠΊ (Π·Π°ΠΊΠΎΠ½ Π€Π°ΡΠ°Π΄Π΅Ρ) Π² Π½Π°ΠΏΡΠ°Π²Π»Π΅Π½ΠΈΠΈ ΠΏΡΠΎΡΠΈΠ² ΡΠ°ΡΠΎΠ²ΠΎΠΉ ΡΡΡΠ΅Π»ΠΊΠΈ (Π·Π°ΠΊΠΎΠ½ ΠΠ΅Π½ΡΠ°), ΠΊΠ°ΠΊ ΠΏΠΎΠΊΠ°Π·Π°Π½ΠΎ. Π ΠΏΠΎΠ»Π΅ Π½Π°Ρ ΠΎΠ΄ΠΈΡΡΡ ΡΠΎΠ»ΡΠΊΠΎ ΠΏΡΠ°Π²Π°Ρ ΡΡΠΎΡΠΎΠ½Π° ΠΊΠΎΠ½ΡΡΡΠ° ΡΠΎΠΊΠ°, ΡΠ°ΠΊ ΡΡΠΎ ΡΠ»Π΅Π²Π° Π½Π° Π½Π΅Π³ΠΎ Π΄Π΅ΠΉΡΡΠ²ΡΠ΅Ρ Π½Π΅ΠΏΡΠΎΡΠΈΠ²ΠΎΡΠ΅ΡΠΈΠ²Π°Ρ ΡΠΈΠ»Π° (ΠΏΡΠ°Π²ΠΈΠ»ΠΎ ΠΏΡΠ°Π²ΠΎΠΉ ΡΡΠΊΠΈ). ΠΠΎΠ³Π΄Π° ΠΌΠ΅ΡΠ°Π»Π»ΠΈΡΠ΅ΡΠΊΠ°Ρ ΠΏΠ»Π°ΡΡΠΈΠ½Π° Π½Π°Ρ ΠΎΠ΄ΠΈΡΡΡ ΠΏΠΎΠ»Π½ΠΎΡΡΡΡ Π²Π½ΡΡΡΠΈ ΠΏΠΎΠ»Ρ, Π²ΠΈΡ ΡΠ΅Π²ΠΎΠΉ ΡΠΎΠΊ ΠΎΡΡΡΡΡΡΠ²ΡΠ΅Ρ, Π΅ΡΠ»ΠΈ ΠΏΠΎΠ»Π΅ ΠΎΠ΄Π½ΠΎΡΠΎΠ΄Π½ΠΎ, ΡΠ°ΠΊ ΠΊΠ°ΠΊ ΠΏΠΎΡΠΎΠΊ Π² ΡΡΠΎΠΉ ΠΎΠ±Π»Π°ΡΡΠΈ ΠΎΡΡΠ°Π΅ΡΡΡ ΠΏΠΎΡΡΠΎΡΠ½Π½ΡΠΌ. ΠΠΎ ΠΊΠΎΠ³Π΄Π° ΠΏΠ»Π°ΡΡΠΈΠ½Π° ΠΏΠΎΠΊΠΈΠ΄Π°Π΅Ρ ΠΏΠΎΠ»Π΅ ΡΠΏΡΠ°Π²Π°, ΠΏΠΎΡΠΎΠΊ ΡΠΌΠ΅Π½ΡΡΠ°Π΅ΡΡΡ, Π²ΡΠ·ΡΠ²Π°Ρ Π²ΠΈΡ ΡΠ΅Π²ΠΎΠΉ ΡΠΎΠΊ Π² Π½Π°ΠΏΡΠ°Π²Π»Π΅Π½ΠΈΠΈ ΠΏΠΎ ΡΠ°ΡΠΎΠ²ΠΎΠΉ ΡΡΡΠ΅Π»ΠΊΠ΅, ΠΊΠΎΡΠΎΡΡΠΉ, ΠΎΠΏΡΡΡ ΠΆΠ΅, ΠΈΡΠΏΡΡΡΠ²Π°Π΅Ρ ΡΠΈΠ»Ρ Π²Π»Π΅Π²ΠΎ, Π΅ΡΠ΅ Π±ΠΎΠ»ΡΡΠ΅ Π·Π°ΠΌΠ΅Π΄Π»ΡΡ Π΄Π²ΠΈΠΆΠ΅Π½ΠΈΠ΅. ΠΠ½Π°Π»ΠΎΠ³ΠΈΡΠ½ΡΠΉ Π°Π½Π°Π»ΠΈΠ· ΡΠΎΠ³ΠΎ, ΡΡΠΎ ΠΏΡΠΎΠΈΡΡ ΠΎΠ΄ΠΈΡ, ΠΊΠΎΠ³Π΄Π° ΠΏΠ»Π°ΡΡΠΈΠ½Π° ΠΊΠ°ΡΠ°Π΅ΡΡΡ ΡΠΏΡΠ°Π²Π° Π½Π°Π»Π΅Π²ΠΎ, ΠΏΠΎΠΊΠ°Π·ΡΠ²Π°Π΅Ρ, ΡΡΠΎ Π΅Π΅ Π΄Π²ΠΈΠΆΠ΅Π½ΠΈΠ΅ ΡΠ°ΠΊΠΆΠ΅ Π·Π°ΡΡΡ Π°Π΅Ρ ΠΏΡΠΈ Π²Ρ ΠΎΠ΄Π΅ Π² ΠΏΠΎΠ»Π΅ ΠΈ Π²ΡΡ ΠΎΠ΄Π΅ ΠΈΠ· Π½Π΅Π³ΠΎ.
ΠΡΠΎΠ²ΠΎΠ΄ΡΡΠ°Ρ ΠΏΠ»Π°ΡΡΠΈΠ½Π°, ΠΏΡΠΎΡ ΠΎΠ΄ΡΡΠ°Ρ ΠΌΠ΅ΠΆΠ΄Ρ ΠΏΠΎΠ»ΡΡΠ°ΠΌΠΈ ΠΌΠ°Π³Π½ΠΈΡΠ° : ΠΠΎΠ»Π΅Π΅ ΠΏΠΎΠ΄ΡΠΎΠ±Π½ΡΠΉ Π²ΠΈΠ΄ Π½Π° ΠΏΡΠΎΠ²ΠΎΠ΄ΡΡΡΡ ΠΏΠ»Π°ΡΡΠΈΠ½Ρ, ΠΏΡΠΎΡ ΠΎΠ΄ΡΡΡΡ ΠΌΠ΅ΠΆΠ΄Ρ ΠΏΠΎΠ»ΡΡΠ°ΠΌΠΈ ΠΌΠ°Π³Π½ΠΈΡΠ°. ΠΠΎΠ³Π΄Π° ΠΎΠ½ Π²Ρ ΠΎΠ΄ΠΈΡ Π² ΠΏΠΎΠ»Π΅ ΠΈ Π²ΡΡ ΠΎΠ΄ΠΈΡ ΠΈΠ· Π½Π΅Π³ΠΎ, ΠΈΠ·ΠΌΠ΅Π½Π΅Π½ΠΈΠ΅ ΠΏΠΎΡΠΎΠΊΠ° Π²ΡΠ·ΡΠ²Π°Π΅Ρ Π²ΠΈΡ ΡΠ΅Π²ΠΎΠΉ ΡΠΎΠΊ. ΠΠ°Π³Π½ΠΈΡΠ½Π°Ρ ΡΠΈΠ»Π° Π½Π° ΡΠΎΠΊΠΎΠ²ΠΎΠΉ ΠΏΠ΅ΡΠ»Π΅ ΠΏΡΠΎΡΠΈΠ²ΠΎΠ΄Π΅ΠΉΡΡΠ²ΡΠ΅Ρ Π΄Π²ΠΈΠΆΠ΅Π½ΠΈΡ. ΠΠΎΠ³Π΄Π° ΠΏΠ»Π°ΡΡΠΈΠ½Π° ΠΏΠΎΠ»Π½ΠΎΡΡΡΡ Π½Π°Ρ ΠΎΠ΄ΠΈΡΡΡ Π²Π½ΡΡΡΠΈ ΠΎΠ΄Π½ΠΎΡΠΎΠ΄Π½ΠΎΠ³ΠΎ ΠΏΠΎΠ»Ρ, ΡΠΎΠΊ ΠΈ ΠΌΠ°Π³Π½ΠΈΡΠ½ΠΎΠ΅ ΡΠΎΠΏΡΠΎΡΠΈΠ²Π»Π΅Π½ΠΈΠ΅ ΠΎΡΡΡΡΡΡΠ²ΡΡΡ.
ΠΠΎΠ³Π΄Π° ΠΌΠ΅ΡΠ°Π»Π»ΠΈΡΠ΅ΡΠΊΠ°Ρ ΠΏΠ»Π°ΡΡΠΈΠ½Π° Ρ ΠΏΡΠΎΡΠ΅Π·ΡΠΌΠΈ Π²Ρ ΠΎΠ΄ΠΈΡ Π² ΠΏΠΎΠ»Π΅, ΠΊΠ°ΠΊ ΠΏΠΎΠΊΠ°Π·Π°Π½ΠΎ Π½Π° ΡΠΈΡΡΠ½ΠΊΠ΅, ΠΠΠ‘ ΠΈΠ½Π΄ΡΡΠΈΡΡΠ΅ΡΡΡ ΠΈΠ·ΠΌΠ΅Π½Π΅Π½ΠΈΠ΅ΠΌ ΠΏΠΎΡΠΎΠΊΠ°, Π½ΠΎ ΠΎΠ½Π° ΠΌΠ΅Π½Π΅Π΅ ΡΡΡΠ΅ΠΊΡΠΈΠ²Π½Π°, ΠΏΠΎΡΠΊΠΎΠ»ΡΠΊΡ ΠΏΡΠΎΡΠ΅Π·ΠΈ ΠΎΠ³ΡΠ°Π½ΠΈΡΠΈΠ²Π°ΡΡ ΡΠ°Π·ΠΌΠ΅Ρ ΡΠΎΠΊΠΎΠ²ΡΡ ΠΏΠ΅ΡΠ΅Π»Ρ. ΠΠΎΠ»Π΅Π΅ ΡΠΎΠ³ΠΎ, Π² ΡΠΎΡΠ΅Π΄Π½ΠΈΡ ΠΏΠ΅ΡΠ»ΡΡ Π΅ΡΡΡ ΡΠΎΠΊΠΈ Π² ΠΏΡΠΎΡΠΈΠ²ΠΎΠΏΠΎΠ»ΠΎΠΆΠ½ΡΡ Π½Π°ΠΏΡΠ°Π²Π»Π΅Π½ΠΈΡΡ , ΠΈ ΠΈΡ ΡΡΡΠ΅ΠΊΡΡ ΠΊΠΎΠΌΠΏΠ΅Π½ΡΠΈΡΡΡΡΡΡ. ΠΡΠΈ ΠΈΡΠΏΠΎΠ»ΡΠ·ΠΎΠ²Π°Π½ΠΈΠΈ ΠΈΠ·ΠΎΠ»ΡΡΠΈΠΎΠ½Π½ΠΎΠ³ΠΎ ΠΌΠ°ΡΠ΅ΡΠΈΠ°Π»Π° Π²ΠΈΡ ΡΠ΅Π²ΡΠ΅ ΡΠΎΠΊΠΈ ΡΡΠ΅Π·Π²ΡΡΠ°ΠΉΠ½ΠΎ ΠΌΠ°Π»Ρ, ΠΏΠΎΡΡΠΎΠΌΡ ΠΌΠ°Π³Π½ΠΈΡΠ½ΡΠΌ Π΄Π΅ΠΌΠΏΡΠΈΡΠΎΠ²Π°Π½ΠΈΠ΅ΠΌ Π½Π° ΠΈΠ·ΠΎΠ»ΡΡΠΎΡΠ°Ρ ΠΌΠΎΠΆΠ½ΠΎ ΠΏΡΠ΅Π½Π΅Π±ΡΠ΅ΡΡ. ΠΡΠ»ΠΈ Π½Π΅ΠΎΠ±Ρ ΠΎΠ΄ΠΈΠΌΠΎ ΠΈΠ·Π±Π΅ΠΆΠ°ΡΡ Π²ΠΈΡ ΡΠ΅Π²ΡΡ ΡΠΎΠΊΠΎΠ² Π² ΠΏΡΠΎΠ²ΠΎΠ΄Π½ΠΈΠΊΠ°Ρ , ΡΠΎ ΠΎΠ½ΠΈ ΠΌΠΎΠ³ΡΡ Π±ΡΡΡ ΡΠ΅Π»Π΅Π²ΡΠΌΠΈ ΠΈΠ»ΠΈ ΠΈΠ·Π³ΠΎΡΠΎΠ²Π»Π΅Π½Ρ ΠΈΠ· ΡΠΎΠ½ΠΊΠΈΡ ΡΠ»ΠΎΠ΅Π² ΠΏΡΠΎΠ²ΠΎΠ΄ΡΡΠ΅Π³ΠΎ ΠΌΠ°ΡΠ΅ΡΠΈΠ°Π»Π°, ΡΠ°Π·Π΄Π΅Π»Π΅Π½Π½ΡΡ ΠΈΠ·ΠΎΠ»ΡΡΠΈΠΎΠ½Π½ΡΠΌΠΈ Π»ΠΈΡΡΠ°ΠΌΠΈ.
ΠΠΈΡ ΡΠ΅Π²ΡΠ΅ ΡΠΎΠΊΠΈ, ΠΈΠ½Π΄ΡΡΠΈΡΠΎΠ²Π°Π½Π½ΡΠ΅ Π² ΠΌΠ΅ΡΠ°Π»Π»ΠΈΡΠ΅ΡΠΊΠΎΠΉ ΠΏΠ»Π°ΡΡΠΈΠ½Π΅ Ρ ΠΏΡΠΎΡΠ΅Π·ΡΠΌΠΈ : ΠΠΈΡ ΡΠ΅Π²ΡΠ΅ ΡΠΎΠΊΠΈ, ΠΈΠ½Π΄ΡΡΠΈΡΠΎΠ²Π°Π½Π½ΡΠ΅ Π² ΠΌΠ΅ΡΠ°Π»Π»ΠΈΡΠ΅ΡΠΊΠΎΠΉ ΠΏΠ»Π°ΡΡΠΈΠ½Π΅ Ρ ΠΏΡΠΎΡΠ΅Π·ΡΠΌΠΈ, ΠΏΠΎΠΏΠ°Π΄Π°Ρ Π² ΠΌΠ°Π³Π½ΠΈΡΠ½ΠΎΠ΅ ΠΏΠΎΠ»Π΅, ΠΎΠ±ΡΠ°Π·ΡΡΡ Π½Π΅Π±ΠΎΠ»ΡΡΠΈΠ΅ ΠΏΠ΅ΡΠ»ΠΈ, ΠΈ Π΄Π΅ΠΉΡΡΠ²ΡΡΡΠΈΠ΅ Π½Π° Π½ΠΈΡ ΡΠΈΠ»Ρ ΠΈΠΌΠ΅ΡΡ ΡΠ΅Π½Π΄Π΅Π½ΡΠΈΡ ΠΊΠΎΠΌΠΏΠ΅Π½ΡΠΈΡΠΎΠ²Π°ΡΡΡΡ, ΡΡΠΎ Π΄Π΅Π»Π°Π΅Ρ ΠΌΠ°Π³Π½ΠΈΡΠ½ΠΎΠ΅ ΡΠΎΠΏΡΠΎΡΠΈΠ²Π»Π΅Π½ΠΈΠ΅ ΠΏΠΎΡΡΠΈ Π½ΡΠ»Π΅Π²ΡΠΌ.
ΠΠ·ΠΌΠ΅Π½Π΅Π½ΠΈΠ΅ ΠΌΠ°Π³Π½ΠΈΡΠ½ΠΎΠ³ΠΎ ΠΏΠΎΡΠΎΠΊΠ° ΡΠΎΠ·Π΄Π°Π΅Ρ ΡΠ»Π΅ΠΊΡΡΠΈΡΠ΅ΡΠΊΠΎΠ΅ ΠΏΠΎΠ»Π΅
ΠΠ°ΠΊΠΎΠ½ ΠΈΠ½Π΄ΡΠΊΡΠΈΠΈ Π€Π°ΡΠ°Π΄Π΅Ρ Π³Π»Π°ΡΠΈΡ, ΡΡΠΎ ΠΈΠ·ΠΌΠ΅Π½Π΅Π½ΠΈΠ΅ ΠΌΠ°Π³Π½ΠΈΡΠ½ΠΎΠ³ΠΎ ΠΏΠΎΠ»Ρ ΡΠΎΠ·Π΄Π°Π΅Ρ ΡΠ»Π΅ΠΊΡΡΠΈΡΠ΅ΡΠΊΠΎΠ΅ ΠΏΠΎΠ»Π΅: [Π»Π°ΡΠ΅ΠΊΡ]\varepsilon = -\frac{\partial \Phi_\text{B}}{\partial \text{ Ρ}}[/Π»Π°ΡΠ΅ΠΊΡ].
Π¦Π΅Π»ΠΈ ΠΎΠ±ΡΡΠ΅Π½ΠΈΡ
ΠΠΏΠΈΡΠ°ΡΡ Π²Π·Π°ΠΈΠΌΠΎΡΠ²ΡΠ·Ρ ΠΌΠ΅ΠΆΠ΄Ρ ΠΈΠ·ΠΌΠ΅Π½ΡΡΡΠΈΠΌΡΡ ΠΌΠ°Π³Π½ΠΈΡΠ½ΡΠΌ ΠΏΠΎΠ»Π΅ΠΌ ΠΈ ΡΠ»Π΅ΠΊΡΡΠΈΡΠ΅ΡΠΊΠΈΠΌ ΠΏΠΎΠ»Π΅ΠΌ
ΠΠ»ΡΡΠ΅Π²ΡΠ΅ Π²ΡΠ²ΠΎΠ΄Ρ
ΠΠ»ΡΡΠ΅Π²ΡΠ΅ ΠΌΠΎΠΌΠ΅Π½ΡΡ
- ΠΠ°ΠΊΠΎΠ½ ΠΈΠ½Π΄ΡΠΊΡΠΈΠΈ Π€Π°ΡΠ°Π΄Π΅Ρ β ΡΡΠΎ ΠΎΡΠ½ΠΎΠ²Π½ΠΎΠΉ Π·Π°ΠΊΠΎΠ½ ΡΠ»Π΅ΠΊΡΡΠΎΠΌΠ°Π³Π½Π΅ΡΠΈΠ·ΠΌΠ°, ΠΊΠΎΡΠΎΡΡΠΉ ΠΏΡΠ΅Π΄ΡΠΊΠ°Π·ΡΠ²Π°Π΅Ρ, ΠΊΠ°ΠΊ ΠΌΠ°Π³Π½ΠΈΡΠ½ΠΎΠ΅ ΠΏΠΎΠ»Π΅ Π±ΡΠ΄Π΅Ρ Π²Π·Π°ΠΈΠΌΠΎΠ΄Π΅ΠΉΡΡΠ²ΠΎΠ²Π°ΡΡ Ρ ΡΠ»Π΅ΠΊΡΡΠΈΡΠ΅ΡΠΊΠΎΠΉ ΡΠ΅ΠΏΡΡ Π΄Π»Ρ ΡΠΎΠ·Π΄Π°Π½ΠΈΡ ΡΠ»Π΅ΠΊΡΡΠΎΠ΄Π²ΠΈΠΆΡΡΠ΅ΠΉ ΡΠΈΠ»Ρ.
- ΠΠ»ΡΡΠ΅ΡΠ½Π°ΡΠΈΠ²Π½Π°Ρ Π΄ΠΈΡΡΠ΅ΡΠ΅Π½ΡΠΈΠ°Π»ΡΠ½Π°Ρ ΡΠΎΡΠΌΠ° Π·Π°ΠΊΠΎΠ½Π° ΠΈΠ½Π΄ΡΠΊΡΠΈΠΈ Π€Π°ΡΠ°Π΄Π΅Ρ Π²ΡΡΠ°ΠΆΠ°Π΅ΡΡΡ Π² ΡΡΠ°Π²Π½Π΅Π½ΠΈΠΈ [Π»Π°ΡΠ΅ΠΊΡ]\nabla \times \vec {\text{E}} = — \frac{\partial \vec {\text{B}}}{ \partial \text{t}}[/latex].
- ΠΠ°ΠΊΠΎΠ½ ΠΈΠ½Π΄ΡΠΊΡΠΈΠΈ Π€Π°ΡΠ°Π΄Π΅Ρ β ΠΎΠ΄Π½ΠΎ ΠΈΠ· ΡΠ΅ΡΡΡΠ΅Ρ ΡΡΠ°Π²Π½Π΅Π½ΠΈΠΉ ΠΠ°ΠΊΡΠ²Π΅Π»Π»Π°, ΡΠΏΡΠ°Π²Π»ΡΡΡΠΈΡ Π²ΡΠ΅ΠΌΠΈ ΡΠ»Π΅ΠΊΡΡΠΎΠΌΠ°Π³Π½ΠΈΡΠ½ΡΠΌΠΈ ΡΠ²Π»Π΅Π½ΠΈΡΠΌΠΈ.
ΠΠ»ΡΡΠ΅Π²ΡΠ΅ ΡΠ΅ΡΠΌΠΈΠ½Ρ
- ΠΏΠ»ΠΎΡΠ°Π΄Ρ Π²Π΅ΠΊΡΠΎΡΠ° : ΠΠ΅ΠΊΡΠΎΡ, Π²Π΅Π»ΠΈΡΠΈΠ½Π° ΠΊΠΎΡΠΎΡΠΎΠ³ΠΎ ΠΏΡΠ΅Π΄ΡΡΠ°Π²Π»ΡΠ΅Ρ ΡΠΎΠ±ΠΎΠΉ ΡΠ°ΡΡΠΌΠ°ΡΡΠΈΠ²Π°Π΅ΠΌΡΡ ΠΏΠ»ΠΎΡΠ°Π΄Ρ ΠΈ Π½Π°ΠΏΡΠ°Π²Π»Π΅Π½ΠΈΠ΅ ΠΊΠΎΡΠΎΡΠΎΠ³ΠΎ ΠΏΠ΅ΡΠΏΠ΅Π½Π΄ΠΈΠΊΡΠ»ΡΡΠ½ΠΎ ΠΏΠ»ΠΎΡΠΊΠΎΡΡΠΈ.
- Π£ΡΠ°Π²Π½Π΅Π½ΠΈΡ ΠΠ°ΠΊΡΠ²Π΅Π»Π»Π° : ΠΠ°Π±ΠΎΡ ΡΡΠ°Π²Π½Π΅Π½ΠΈΠΉ, ΠΎΠΏΠΈΡΡΠ²Π°ΡΡΠΈΡ , ΠΊΠ°ΠΊ ΡΠ»Π΅ΠΊΡΡΠΈΡΠ΅ΡΠΊΠΈΠ΅ ΠΈ ΠΌΠ°Π³Π½ΠΈΡΠ½ΡΠ΅ ΠΏΠΎΠ»Ρ Π³Π΅Π½Π΅ΡΠΈΡΡΡΡΡΡ ΠΈ ΠΈΠ·ΠΌΠ΅Π½ΡΡΡΡΡ Π΄ΡΡΠ³ Π΄ΡΡΠ³ΠΎΠΌ, Π° ΡΠ°ΠΊΠΆΠ΅ Π·Π°ΡΡΠ΄Π°ΠΌΠΈ ΠΈ ΡΠΎΠΊΠ°ΠΌΠΈ.
- Π’Π΅ΠΎΡΠ΅ΠΌΠ° Π‘ΡΠΎΠΊΡΠ° : ΡΡΠ²Π΅ΡΠΆΠ΄Π΅Π½ΠΈΠ΅ ΠΎΠ± ΠΈΠ½ΡΠ΅Π³ΡΠΈΡΠΎΠ²Π°Π½ΠΈΠΈ Π΄ΠΈΡΡΠ΅ΡΠ΅Π½ΡΠΈΠ°Π»ΡΠ½ΡΡ ΡΠΎΡΠΌ Π½Π° ΠΌΠ½ΠΎΠ³ΠΎΠΎΠ±ΡΠ°Π·ΠΈΡΡ , ΠΊΠΎΡΠΎΡΠΎΠ΅ ΠΎΠ΄Π½ΠΎΠ²ΡΠ΅ΠΌΠ΅Π½Π½ΠΎ ΡΠΏΡΠΎΡΠ°Π΅Ρ ΠΈ ΠΎΠ±ΠΎΠ±ΡΠ°Π΅Ρ Π½Π΅ΡΠΊΠΎΠ»ΡΠΊΠΎ ΡΠ΅ΠΎΡΠ΅ΠΌ Π²Π΅ΠΊΡΠΎΡΠ½ΠΎΠ³ΠΎ ΠΈΡΡΠΈΡΠ»Π΅Π½ΠΈΡ.
ΠΡ ΠΈΠ·ΡΡΠΈΠ»ΠΈ Π·Π°ΠΊΠΎΠ½ ΠΈΠ½Π΄ΡΠΊΡΠΈΠΈ Π€Π°ΡΠ°Π΄Π΅Ρ Π² ΠΏΡΠ΅Π΄ΡΠ΄ΡΡΠΈΡ Π°ΡΠΎΠΌΠ°Ρ . ΠΡ ΡΠ·Π½Π°Π»ΠΈ Π²Π·Π°ΠΈΠΌΠΎΡΠ²ΡΠ·Ρ ΠΌΠ΅ΠΆΠ΄Ρ ΠΈΠ½Π΄ΡΡΠΈΡΠΎΠ²Π°Π½Π½ΠΎΠΉ ΡΠ»Π΅ΠΊΡΡΠΎΠ΄Π²ΠΈΠΆΡΡΠ΅ΠΉ ΡΠΈΠ»ΠΎΠΉ (ΠΠΠ‘) ΠΈ ΠΌΠ°Π³Π½ΠΈΡΠ½ΡΠΌ ΠΏΠΎΡΠΎΠΊΠΎΠΌ. Π Π΄Π²ΡΡ ΡΠ»ΠΎΠ²Π°Ρ , Π·Π°ΠΊΠΎΠ½ Π³Π»Π°ΡΠΈΡ, ΡΡΠΎ ΠΈΠ·ΠΌΠ΅Π½Π΅Π½ΠΈΠ΅ ΠΌΠ°Π³Π½ΠΈΡΠ½ΠΎΠ³ΠΎ ΠΏΠΎΠ»Ρ [Π»Π°ΡΠ΅ΠΊΡ](\frac{\text{d} \Phi_\text{B}}{\text{dt}})[/latex] ΡΠΎΠ·Π΄Π°Π΅Ρ ΡΠ»Π΅ΠΊΡΡΠΈΡΠ΅ΡΠΊΠΎΠ΅ ΠΏΠΎΠ»Π΅ [Π»Π°ΡΠ΅ΠΊΡ] (\varepsilon)[/latex], Π·Π°ΠΊΠΎΠ½ ΠΈΠ½Π΄ΡΠΊΡΠΈΠΈ Π€Π°ΡΠ°Π΄Π΅Ρ Π²ΡΡΠ°ΠΆΠ°Π΅ΡΡΡ ΠΊΠ°ΠΊ [latex]\varepsilon = -\frac{\partial \Phi_\text{B}}{\partial \text{t}}[/latex], Π³Π΄Π΅ [latex]\varepsilon[/latex] β ΠΠΠ‘ ΠΈΠ½Π΄ΡΠΊΡΠΈΠΈ, Π° [latex]\Phi_\text{B}[/latex] β ΠΌΠ°Π³Π½ΠΈΡΠ½ΡΠΉ ΠΏΠΎΡΠΎΠΊ. (Β«NΒ» ΠΎΠΏΡΡΠ΅Π½ΠΎ ΠΈΠ· Π½Π°ΡΠ΅Π³ΠΎ ΠΏΡΠ΅Π΄ΡΠ΄ΡΡΠ΅Π³ΠΎ Π²ΡΡΠ°ΠΆΠ΅Π½ΠΈΡ. ΠΠΎΠ»ΠΈΡΠ΅ΡΡΠ²ΠΎ Π²ΠΊΠ»ΡΡΠ΅Π½Π½ΡΡ Π²ΠΈΡΠΊΠΎΠ² ΠΊΠ°ΡΡΡΠΊΠΈ ΠΌΠΎΠΆΠ΅Ρ Π±ΡΡΡ Π²ΠΊΠ»ΡΡΠ΅Π½ΠΎ Π² ΠΌΠ°Π³Π½ΠΈΡΠ½ΡΠΉ ΠΏΠΎΡΠΎΠΊ, ΠΏΠΎΡΡΠΎΠΌΡ ΡΡΠΎΡ ΠΊΠΎΡΡΡΠΈΡΠΈΠ΅Π½Ρ Π½Π΅ ΡΠ²Π»ΡΠ΅ΡΡΡ ΠΎΠ±ΡΠ·Π°ΡΠ΅Π»ΡΠ½ΡΠΌ.) ΠΠ°ΠΊΠΎΠ½ ΠΈΠ½Π΄ΡΠΊΡΠΈΠΈ Π€Π°ΡΠ°Π΄Π΅Ρ β ΡΡΠΎ ΠΎΡΠ½ΠΎΠ²Π½ΠΎΠΉ Π·Π°ΠΊΠΎΠ½ ΡΠ»Π΅ΠΊΡΡΠΎΠΌΠ°Π³Π½Π΅ΡΠΈΠ·ΠΌΠ°, ΠΊΠΎΡΠΎΡΡΠΉ ΠΏΡΠ΅Π΄ΡΠΊΠ°Π·ΡΠ²Π°Π΅Ρ, ΠΊΠ°ΠΊ ΠΌΠ°Π³Π½ΠΈΡΠ½ΠΎΠ΅ ΠΏΠΎΠ»Π΅ Π±ΡΠ΄Π΅Ρ Π²Π·Π°ΠΈΠΌΠΎΠ΄Π΅ΠΉΡΡΠ²ΠΎΠ²Π°ΡΡ Ρ ΡΠ»Π΅ΠΊΡΡΠΈΡΠ΅ΡΠΊΠΎΠΉ ΡΠ΅ΠΏΡΡ, ΡΠΎΠ·Π΄Π°Π²Π°Ρ ΡΠ»Π΅ΠΊΡΡΠΎΠ΄Π²ΠΈΠΆΡΡΡΡ ΡΠΈΠ»Ρ (ΠΠΠ‘). Π ΡΡΠΎΠΌ ΠΡΠΎΠΌΠ΅ ΠΌΡ ΡΠ·Π½Π°Π΅ΠΌ ΠΎΠ± Π°Π»ΡΡΠ΅ΡΠ½Π°ΡΠΈΠ²Π½ΠΎΠΌ ΠΌΠ°ΡΠ΅ΠΌΠ°ΡΠΈΡΠ΅ΡΠΊΠΎΠΌ Π²ΡΡΠ°ΠΆΠ΅Π½ΠΈΠΈ Π·Π°ΠΊΠΎΠ½Π°.
ΠΠΊΡΠΏΠ΅ΡΠΈΠΌΠ΅Π½Ρ Π€Π°ΡΠ°Π΄Π΅Ρ : ΠΠΊΡΠΏΠ΅ΡΠΈΠΌΠ΅Π½Ρ Π€Π°ΡΠ°Π΄Π΅Ρ, ΠΏΠΎΠΊΠ°Π·ΡΠ²Π°ΡΡΠΈΠΉ ΠΈΠ½Π΄ΡΠΊΡΠΈΡ ΠΌΠ΅ΠΆΠ΄Ρ Π²ΠΈΡΠΊΠ°ΠΌΠΈ ΠΏΡΠΎΠ²ΠΎΠ΄Π°: ΠΠΈΠ΄ΠΊΠΎΡΡΠ½Π°Ρ Π±Π°ΡΠ°ΡΠ΅Ρ (ΡΠΏΡΠ°Π²Π°) ΠΎΠ±Π΅ΡΠΏΠ΅ΡΠΈΠ²Π°Π΅Ρ ΡΠΎΠΊ, ΠΊΠΎΡΠΎΡΡΠΉ ΡΠ΅ΡΠ΅Ρ ΡΠ΅ΡΠ΅Π· ΠΌΠ°Π»Π΅Π½ΡΠΊΡΡ ΠΊΠ°ΡΡΡΠΊΡ (Π), ΡΠΎΠ·Π΄Π°Π²Π°Ρ ΠΌΠ°Π³Π½ΠΈΡΠ½ΠΎΠ΅ ΠΏΠΎΠ»Π΅. ΠΠΎΠ³Π΄Π° ΠΊΠ°ΡΡΡΠΊΠΈ Π½Π΅ΠΏΠΎΠ΄Π²ΠΈΠΆΠ½Ρ, ΡΠΎΠΊ Π½Π΅ ΠΈΠ½Π΄ΡΡΠΈΡΡΠ΅ΡΡΡ. ΠΠΎ ΠΊΠΎΠ³Π΄Π° ΠΌΠ°Π»Π΅Π½ΡΠΊΡΡ ΠΊΠ°ΡΡΡΠΊΡ ΠΏΠ΅ΡΠ΅ΠΌΠ΅ΡΠ°ΡΡ Π²Π½ΡΡΡΡ ΠΈΠ»ΠΈ Π½Π°ΡΡΠΆΡ ΠΈΠ· Π±ΠΎΠ»ΡΡΠΎΠΉ ΠΊΠ°ΡΡΡΠΊΠΈ (Π), ΠΌΠ°Π³Π½ΠΈΡΠ½ΡΠΉ ΠΏΠΎΡΠΎΠΊ ΡΠ΅ΡΠ΅Π· Π±ΠΎΠ»ΡΡΡΡ ΠΊΠ°ΡΡΡΠΊΡ ΠΈΠ·ΠΌΠ΅Π½ΡΠ΅ΡΡΡ, Π²ΡΠ·ΡΠ²Π°Ρ ΡΠΎΠΊ, ΠΊΠΎΡΠΎΡΡΠΉ ΡΠ΅Π³ΠΈΡΡΡΠΈΡΡΠ΅ΡΡΡ Π³Π°Π»ΡΠ²Π°Π½ΠΎΠΌΠ΅ΡΡΠΎΠΌ (G).
ΠΠΈΡΡΠ΅ΡΠ΅Π½ΡΠΈΠ°Π»ΡΠ½Π°Ρ ΡΠΎΡΠΌΠ° Π·Π°ΠΊΠΎΠ½Π° Π€Π°ΡΠ°Π΄Π΅Ρ
ΠΠ°Π³Π½ΠΈΡΠ½ΡΠΉ ΠΏΠΎΡΠΎΠΊ ΡΠ°Π²Π΅Π½ [Π»Π°ΡΠ΅ΠΊΡ]\Phi_\text{B} = \int_\text{S} \vec {\text{B}} \cdot \text{d} \vec {\text{A}}[ /latex], Π³Π΄Π΅ [latex]\vec {\text{A}}[/latex] β ΠΏΠ»ΠΎΡΠ°Π΄Ρ Π²Π΅ΠΊΡΠΎΡΠ° Π½Π°Π΄ Π·Π°ΠΌΠΊΠ½ΡΡΠΎΠΉ ΠΏΠΎΠ²Π΅ΡΡ Π½ΠΎΡΡΡΡ S. Π£ΡΡΡΠΎΠΉΡΡΠ²ΠΎ, ΡΠΏΠΎΡΠΎΠ±Π½ΠΎΠ΅ ΠΏΠΎΠ΄Π΄Π΅ΡΠΆΠΈΠ²Π°ΡΡ ΡΠ°Π·Π½ΠΎΡΡΡ ΠΏΠΎΡΠ΅Π½ΡΠΈΠ°Π»ΠΎΠ², Π½Π΅ΡΠΌΠΎΡΡΡ Π½Π° ΠΏΡΠΎΡΠ΅ΠΊΠ°Π½ΠΈΠ΅ ΡΠΎΠΊΠ°, ΡΠ²Π»ΡΠ΅ΡΡΡ ΠΈΡΡΠΎΡΠ½ΠΈΠΊΠΎΠΌ ΡΠ»Π΅ΠΊΡΡΠΎΠ΄Π²ΠΈΠΆΡΡΠ΅ΠΉ ΡΠΈΠ»Ρ. ΡΠΈΠ»Π°. (ΠΠΠ) ΠΠΏΡΠ΅Π΄Π΅Π»Π΅Π½ΠΈΠ΅ ΠΌΠ°ΡΠ΅ΠΌΠ°ΡΠΈΡΠ΅ΡΠΊΠΈ [Π»Π°ΡΠ΅ΠΊΡ]\varepsilon = \oint_\text{C} \vec {\text{E}} \cdot \text{d}\vec {\text{s}}[/latex], Π³Π΄Π΅ ΠΈΠ½ΡΠ΅Π³ΡΠ°Π» Π²ΡΡΠΈΡΠ»ΡΠ΅ΡΡΡ ΠΏΠΎ Π·Π°ΠΌΠΊΠ½ΡΡΠΎΠΌΡ ΠΊΠΎΠ½ΡΡΡΡ C.
ΠΠ°ΠΊΠΎΠ½ Π€Π°ΡΠ°Π΄Π΅Ρ ΡΠ΅ΠΏΠ΅ΡΡ ΠΌΠΎΠΆΠ½ΠΎ ΠΏΠ΅ΡΠ΅ΠΏΠΈΡΠ°ΡΡ [Π»Π°ΡΠ΅ΠΊΡ]\oint_\text{C} \vec {\text{E}} \cdot \text{d}\vec {\text{s}} = -\frac{\partial }{\partial \text{t}} (\int \vec {\text{B}} \cdot \text{d}\vec {\text{A}})[/latex]. ΠΡΠΏΠΎΠ»ΡΠ·ΡΡ ΡΠ΅ΠΎΡΠ΅ΠΌΡ Π‘ΡΠΎΠΊΡΠ° Π² Π²Π΅ΠΊΡΠΎΡΠ½ΠΎΠΌ ΠΈΡΡΠΈΡΠ»Π΅Π½ΠΈΠΈ, Π»Π΅Π²Π°Ρ ΡΠ°ΡΡΡ ΡΠ°Π²Π½Π° [Π»Π°ΡΠ΅ΠΊΡ]\oint_\text{C} \vec {\text{E}} \cdot \text{d}\vec {\text{s}} = \int_\text{S} (\nabla \times \vec {\text{E}}) \cdot \text{d}\vec {\text{A}}[/latex]. Π’Π°ΠΊΠΆΠ΅ ΠΎΠ±ΡΠ°ΡΠΈΡΠ΅ Π²Π½ΠΈΠΌΠ°Π½ΠΈΠ΅, ΡΡΠΎ Π² ΠΏΡΠ°Π²ΠΎΠΉ ΡΠ°ΡΡΠΈ [Π»Π°ΡΠ΅ΠΊΡ]\frac{\partial}{\partial \text{t}} (\int \vec {\text{B}} \cdot \text{d}\vec {\ ΡΠ΅ΠΊΡΡ {A}}) = \ int \ frac {\ partial \ vec {\ text {B}}} {\ partial \ text {t}} \ cdot \ text {d} \ vec {\ text {A}} [ /Π»Π°ΡΠ΅ΠΊΡ]. Π’Π°ΠΊΠΈΠΌ ΠΎΠ±ΡΠ°Π·ΠΎΠΌ, ΠΌΡ ΠΏΠΎΠ»ΡΡΠ°Π΅ΠΌ Π°Π»ΡΡΠ΅ΡΠ½Π°ΡΠΈΠ²Π½ΡΡ ΡΠΎΡΠΌΡ Π·Π°ΠΊΠΎΠ½Π° ΠΈΠ½Π΄ΡΠΊΡΠΈΠΈ Π€Π°ΡΠ°Π΄Π΅Ρ: \text{t}}[/latex].ΠΡΠΎ ΡΠ°ΠΊΠΆΠ΅ Π½Π°Π·ΡΠ²Π°Π΅ΡΡΡ Π΄ΠΈΡΡΠ΅ΡΠ΅Π½ΡΠΈΠ°Π»ΡΠ½ΠΎΠΉ ΡΠΎΡΠΌΠΎΠΉ Π·Π°ΠΊΠΎΠ½Π° Π€Π°ΡΠ°Π΄Π΅Ρ. ΠΡΠΎ ΠΎΠ΄Π½ΠΎ ΠΈΠ· ΡΠ΅ΡΡΡΠ΅Ρ ΡΡΠ°Π²Π½Π΅Π½ΠΈΠΉ ΠΠ°ΠΊΡΠ²Π΅Π»Π»Π°, ΡΠΏΡΠ°Π²Π»ΡΡΡΠΈΡ Π²ΡΠ΅ΠΌΠΈ ΡΠ»Π΅ΠΊΡΡΠΎΠΌΠ°Π³Π½ΠΈΡΠ½ΡΠΌΠΈ ΡΠ²Π»Π΅Π½ΠΈΡΠΌΠΈ.
ΠΠ»Π΅ΠΊΡΡΠΈΡΠ΅ΡΠΊΠΈΠ΅ Π³Π΅Π½Π΅ΡΠ°ΡΠΎΡΡ
ΠΠ»Π΅ΠΊΡΡΠΈΡΠ΅ΡΠΊΠΈΠ΅ Π³Π΅Π½Π΅ΡΠ°ΡΠΎΡΡ ΠΏΡΠ΅ΠΎΠ±ΡΠ°Π·ΡΡΡ ΠΌΠ΅Ρ Π°Π½ΠΈΡΠ΅ΡΠΊΡΡ ΡΠ½Π΅ΡΠ³ΠΈΡ Π² ΡΠ»Π΅ΠΊΡΡΠΈΡΠ΅ΡΠΊΡΡ; ΠΎΠ½ΠΈ ΠΈΠ½Π΄ΡΡΠΈΡΡΡΡ ΠΠΠ‘, Π²ΡΠ°ΡΠ°Ρ ΠΊΠ°ΡΡΡΠΊΡ Π² ΠΌΠ°Π³Π½ΠΈΡΠ½ΠΎΠΌ ΠΏΠΎΠ»Π΅.
Π¦Π΅Π»ΠΈ ΠΎΠ±ΡΡΠ΅Π½ΠΈΡ
ΠΠ±ΡΡΡΠ½ΠΈΡΡ, ΠΊΠ°ΠΊ Π² ΡΠ»Π΅ΠΊΡΡΠΈΡΠ΅ΡΠΊΠΈΡ Π³Π΅Π½Π΅ΡΠ°ΡΠΎΡΠ°Ρ ΠΈΠ½Π΄ΡΡΠΈΡΡΠ΅ΡΡΡ ΡΠ»Π΅ΠΊΡΡΠΎΠ΄Π²ΠΈΠΆΡΡΠ°Ρ ΡΠΈΠ»Π°
ΠΠ»ΡΡΠ΅Π²ΡΠ΅ Π²ΡΠ²ΠΎΠ΄Ρ
ΠΠ»ΡΡΠ΅Π²ΡΠ΅ ΠΌΠΎΠΌΠ΅Π½ΡΡ
- ΠΠ»Π΅ΠΊΡΡΠΈΡΠ΅ΡΠΊΠΈΠΉ Π³Π΅Π½Π΅ΡΠ°ΡΠΎΡ Π²ΡΠ°ΡΠ°Π΅Ρ ΠΊΠ°ΡΡΡΠΊΡ Π² ΠΌΠ°Π³Π½ΠΈΡΠ½ΠΎΠΌ ΠΏΠΎΠ»Π΅, ΠΈΠ½Π΄ΡΡΠΈΡΡΡ ΠΠΠ‘, Π·Π°Π΄Π°Π½Π½ΡΡ ΠΊΠ°ΠΊ ΡΡΠ½ΠΊΡΠΈΡ Π²ΡΠ΅ΠΌΠ΅Π½ΠΈ Ξ΅= NABw ΡΠΈΠ½ΟΡ. ΠΠ΅Π½Π΅ΡΠ°ΡΠΎΡΡ
- ΠΎΠ±Π΅ΡΠΏΠ΅ΡΠΈΠ²Π°ΡΡ ΠΏΠΎΡΡΠΈ Π²ΡΡ ΠΌΠΎΡΠ½ΠΎΡΡΡ Π΄Π»Ρ ΡΠ»Π΅ΠΊΡΡΠΎΡΠ΅ΡΠ΅ΠΉ, ΠΊΠΎΡΠΎΡΡΠ΅ ΠΎΠ±Π΅ΡΠΏΠ΅ΡΠΈΠ²Π°ΡΡ Π±ΠΎΠ»ΡΡΡΡ ΡΠ°ΡΡΡ ΠΌΠΈΡΠΎΠ²ΠΎΠΉ ΡΠ»Π΅ΠΊΡΡΠΎΡΠ½Π΅ΡΠ³ΠΈΠΈ.
- ΠΠ²ΠΈΠ³Π°ΡΠ΅Π»Ρ ΡΡΠ°Π½ΠΎΠ²ΠΈΡΡΡ Π³Π΅Π½Π΅ΡΠ°ΡΠΎΡΠΎΠΌ, ΠΊΠΎΠ³Π΄Π° Π΅Π³ΠΎ Π²Π°Π» Π²ΡΠ°ΡΠ°Π΅ΡΡΡ.
ΠΡΠ½ΠΎΠ²Π½ΡΠ΅ ΡΠ΅ΡΠΌΠΈΠ½Ρ
- ΡΠ»Π΅ΠΊΡΡΠΎΠ΄Π²ΠΈΠΆΡΡΠ°Ρ ΡΠΈΠ»Π° : (ΠΠΠ‘) β Π½Π°ΠΏΡΡΠΆΠ΅Π½ΠΈΠ΅, ΡΠΎΠ·Π΄Π°Π²Π°Π΅ΠΌΠΎΠ΅ Π±Π°ΡΠ°ΡΠ΅Π΅ΠΉ ΠΈΠ»ΠΈ ΠΌΠ°Π³Π½ΠΈΡΠ½ΠΎΠΉ ΡΠΈΠ»ΠΎΠΉ Π² ΡΠΎΠΎΡΠ²Π΅ΡΡΡΠ²ΠΈΠΈ Ρ Π·Π°ΠΊΠΎΠ½ΠΎΠΌ Π€Π°ΡΠ°Π΄Π΅Ρ. ΠΠ½Π° ΠΈΠ·ΠΌΠ΅ΡΡΠ΅ΡΡΡ Π² Π²ΠΎΠ»ΡΡΠ°Ρ , Π° Π½Π΅ Π² Π½ΡΡΡΠΎΠ½Π°Ρ , ΠΈ, ΡΠ»Π΅Π΄ΠΎΠ²Π°ΡΠ΅Π»ΡΠ½ΠΎ, Π½Π° ΡΠ°ΠΌΠΎΠΌ Π΄Π΅Π»Π΅ Π½Π΅ ΡΠ²Π»ΡΠ΅ΡΡΡ ΡΠΈΠ»ΠΎΠΉ.
- ΡΡΡΠ±ΠΈΠ½Π° : ΠΡΠ±Π°Ρ ΠΈΠ· ΡΠ°Π·Π»ΠΈΡΠ½ΡΡ Π²ΡΠ°ΡΠ°ΡΡΠΈΡ ΡΡ ΠΌΠ°ΡΠΈΠ½, ΠΊΠΎΡΠΎΡΡΠ΅ ΠΈΡΠΏΠΎΠ»ΡΠ·ΡΡΡ ΠΊΠΈΠ½Π΅ΡΠΈΡΠ΅ΡΠΊΡΡ ΡΠ½Π΅ΡΠ³ΠΈΡ Π½Π΅ΠΏΡΠ΅ΡΡΠ²Π½ΠΎΠ³ΠΎ ΠΏΠΎΡΠΎΠΊΠ° ΠΆΠΈΠ΄ΠΊΠΎΡΡΠΈ (ΠΆΠΈΠ΄ΠΊΠΎΡΡΠΈ ΠΈΠ»ΠΈ Π³Π°Π·Π°) Π΄Π»Ρ Π²ΡΠ°ΡΠ΅Π½ΠΈΡ Π²Π°Π»Π°.
ΠΠ»Π΅ΠΊΡΡΠΈΡΠ΅ΡΠΊΠΈΠ΅ Π³Π΅Π½Π΅ΡΠ°ΡΠΎΡΡ β ΡΡΠΎ ΡΡΡΡΠΎΠΉΡΡΠ²Π°, ΠΏΡΠ΅ΠΎΠ±ΡΠ°Π·ΡΡΡΠΈΠ΅ ΠΌΠ΅Ρ Π°Π½ΠΈΡΠ΅ΡΠΊΡΡ ΡΠ½Π΅ΡΠ³ΠΈΡ Π² ΡΠ»Π΅ΠΊΡΡΠΈΡΠ΅ΡΠΊΡΡ. ΠΠ½ΠΈ ΠΈΠ½Π΄ΡΡΠΈΡΡΡΡ ΡΠ»Π΅ΠΊΡΡΠΎΠ΄Π²ΠΈΠΆΡΡΡΡ ΡΠΈΠ»Ρ (ΠΠΠ‘), Π²ΡΠ°ΡΠ°Ρ ΠΊΠ°ΡΡΡΠΊΡ Π² ΠΌΠ°Π³Π½ΠΈΡΠ½ΠΎΠΌ ΠΏΠΎΠ»Π΅. ΠΡΠΎ ΡΡΡΡΠΎΠΉΡΡΠ²ΠΎ, ΠΊΠΎΡΠΎΡΠΎΠ΅ ΠΏΡΠ΅ΠΎΠ±ΡΠ°Π·ΡΠ΅Ρ ΠΌΠ΅Ρ Π°Π½ΠΈΡΠ΅ΡΠΊΡΡ ΡΠ½Π΅ΡΠ³ΠΈΡ Π² ΡΠ»Π΅ΠΊΡΡΠΈΡΠ΅ΡΠΊΡΡ. ΠΠ΅Π½Π΅ΡΠ°ΡΠΎΡ Π·Π°ΡΡΠ°Π²Π»ΡΠ΅Ρ ΡΠ»Π΅ΠΊΡΡΠΈΡΠ΅ΡΠΊΠΈΠΉ Π·Π°ΡΡΠ΄ (ΠΎΠ±ΡΡΠ½ΠΎ ΠΏΠ΅ΡΠ΅Π½ΠΎΡΠΈΠΌΡΠΉ ΡΠ»Π΅ΠΊΡΡΠΎΠ½Π°ΠΌΠΈ) ΡΠ΅ΡΡ ΡΠ΅ΡΠ΅Π· Π²Π½Π΅ΡΠ½ΡΡ ΡΠ»Π΅ΠΊΡΡΠΈΡΠ΅ΡΠΊΡΡ ΡΠ΅ΠΏΡ. Π Π²ΠΎΠ·ΠΌΠΎΠΆΠ½ΡΠΌ ΠΈΡΡΠΎΡΠ½ΠΈΠΊΠ°ΠΌ ΠΌΠ΅Ρ Π°Π½ΠΈΡΠ΅ΡΠΊΠΎΠΉ ΡΠ½Π΅ΡΠ³ΠΈΠΈ ΠΎΡΠ½ΠΎΡΡΡΡΡ: ΠΏΠΎΡΡΠ½Π΅Π²ΠΎΠΉ ΠΈΠ»ΠΈ ΡΡΡΠ±ΠΈΠ½Π½ΡΠΉ ΠΏΠ°ΡΠΎΠ²ΠΎΠΉ Π΄Π²ΠΈΠ³Π°ΡΠ΅Π»Ρ, Π²ΠΎΠ΄Π°, ΠΏΠ°Π΄Π°ΡΡΠ°Ρ ΡΠ΅ΡΠ΅Π· ΡΡΡΠ±ΠΈΠ½Ρ ΠΈΠ»ΠΈ Π²ΠΎΠ΄ΡΠ½ΠΎΠ΅ ΠΊΠΎΠ»Π΅ΡΠΎ, Π΄Π²ΠΈΠ³Π°ΡΠ΅Π»Ρ Π²Π½ΡΡΡΠ΅Π½Π½Π΅Π³ΠΎ ΡΠ³ΠΎΡΠ°Π½ΠΈΡ, Π²Π΅ΡΡΡΠ½Π°Ρ ΡΡΡΠ±ΠΈΠ½Π°, ΡΡΠΊΠΎΡΡΠΊΠ°, ΡΠΆΠ°ΡΡΠΉ Π²ΠΎΠ·Π΄ΡΡ ΠΈΠ»ΠΈ Π»ΡΠ±ΠΎΠΉ Π΄ΡΡΠ³ΠΎΠΉ ΠΈΡΡΠΎΡΠ½ΠΈΠΊ ΠΌΠ΅Ρ Π°Π½ΠΈΡΠ΅ΡΠΊΠΎΠΉ ΡΠ½Π΅ΡΠ³ΠΈΠΈ. ΠΠ΅Π½Π΅ΡΠ°ΡΠΎΡΡ ΠΎΠ±Π΅ΡΠΏΠ΅ΡΠΈΠ²Π°ΡΡ ΠΏΠΎΡΡΠΈ Π²ΡΡ ΠΌΠΎΡΠ½ΠΎΡΡΡ Π΄Π»Ρ ΡΠ»Π΅ΠΊΡΡΠΎΡΠ΅ΡΠ΅ΠΉ, ΠΊΠΎΡΠΎΡΡΠ΅ ΠΎΠ±Π΅ΡΠΏΠ΅ΡΠΈΠ²Π°ΡΡ Π±ΠΎΠ»ΡΡΡΡ ΡΠ°ΡΡΡ ΠΌΠΈΡΠΎΠ²ΠΎΠΉ ΡΠ»Π΅ΠΊΡΡΠΎΡΠ½Π΅ΡΠ³ΠΈΠΈ.
ΠΠ΅Π½Π΅ΡΠ°ΡΠΎΡ ΠΏΠ°ΡΠΎΠ²ΠΎΠΉ ΡΡΡΠ±ΠΈΠ½Ρ : Π‘ΠΎΠ²ΡΠ΅ΠΌΠ΅Π½Π½ΡΠΉ Π³Π΅Π½Π΅ΡΠ°ΡΠΎΡ ΠΏΠ°ΡΠΎΠ²ΠΎΠΉ ΡΡΡΠ±ΠΈΠ½Ρ.
ΠΠ°Π·ΠΎΠ²Π°Ρ ΡΡΡΠ°Π½ΠΎΠ²ΠΊΠ°
Π Π°ΡΡΠΌΠΎΡΡΠΈΠΌ ΡΡΡΠ°Π½ΠΎΠ²ΠΊΡ, ΠΏΠΎΠΊΠ°Π·Π°Π½Π½ΡΡ Π½Π° ΡΠΈΡ. ΠΠ°ΡΡΠ΄Ρ Π² ΠΏΡΠΎΠ²ΠΎΠ΄Π°Ρ ΠΏΠ΅ΡΠ»ΠΈ ΠΈΡΠΏΡΡΡΠ²Π°ΡΡ ΠΌΠ°Π³Π½ΠΈΡΠ½ΡΡ ΡΠΈΠ»Ρ, ΠΏΠΎΡΠΎΠΌΡ ΡΡΠΎ ΠΎΠ½ΠΈ Π΄Π²ΠΈΠΆΡΡΡΡ Π² ΠΌΠ°Π³Π½ΠΈΡΠ½ΠΎΠΌ ΠΏΠΎΠ»Π΅. ΠΠ° Π·Π°ΡΡΠ΄Ρ Π² Π²Π΅ΡΡΠΈΠΊΠ°Π»ΡΠ½ΡΡ ΠΏΡΠΎΠ²ΠΎΠ΄Π°Ρ Π΄Π΅ΠΉΡΡΠ²ΡΡΡ ΡΠΈΠ»Ρ, ΠΏΠ°ΡΠ°Π»Π»Π΅Π»ΡΠ½ΡΠ΅ ΠΏΡΠΎΠ²ΠΎΠ΄Ρ, Π²ΡΠ·ΡΠ²Π°ΡΡΠΈΠ΅ ΡΠΎΠΊΠΈ. ΠΠ΄Π½Π°ΠΊΠΎ ΡΠ΅, ΠΊΡΠΎ Π½Π°Ρ ΠΎΠ΄ΠΈΡΡΡ Π² Π²Π΅ΡΡ Π½Π΅ΠΌ ΠΈ Π½ΠΈΠΆΠ½Π΅ΠΌ ΡΠ΅Π³ΠΌΠ΅Π½ΡΠ°Ρ , ΡΡΠ²ΡΡΠ²ΡΡΡ ΡΠΈΠ»Ρ, ΠΏΠ΅ΡΠΏΠ΅Π½Π΄ΠΈΠΊΡΠ»ΡΡΠ½ΡΡ ΠΏΡΠΎΠ²ΠΎΠ΄Ρ; ΡΡΠ° ΡΠΈΠ»Π° Π½Π΅ Π²ΡΠ·ΡΠ²Π°Π΅Ρ ΡΠΎΠΊΠ°. Π’Π°ΠΊΠΈΠΌ ΠΎΠ±ΡΠ°Π·ΠΎΠΌ, ΠΌΡ ΠΌΠΎΠΆΠ΅ΠΌ Π½Π°ΠΉΡΠΈ ΠΠΠ‘ ΠΈΠ½Π΄ΡΠΊΡΠΈΠΈ, ΡΠ°ΡΡΠΌΠ°ΡΡΠΈΠ²Π°Ρ ΡΠΎΠ»ΡΠΊΠΎ Π±ΠΎΠΊΠΎΠ²ΡΠ΅ ΠΏΡΠΎΠ²ΠΎΠ΄Π°. ΠΠΠ‘ Π΄Π²ΠΈΠΆΠ΅Π½ΠΈΡ ΠΎΠΏΡΠ΅Π΄Π΅Π»ΡΠ΅ΡΡΡ ΠΊΠ°ΠΊ ΠΠΠ‘=Bβv, Π³Π΄Π΅ ΡΠΊΠΎΡΠΎΡΡΡ v ΠΏΠ΅ΡΠΏΠ΅Π½Π΄ΠΈΠΊΡΠ»ΡΡΠ½Π° ΠΌΠ°Π³Π½ΠΈΡΠ½ΠΎΠΌΡ ΠΏΠΎΠ»Ρ B (ΡΠΌ. Π½Π°Ρ Π°ΡΠΎΠΌ Π½Π° Β«ΠΠΠ‘ Π΄Π²ΠΈΠΆΠ΅Π½ΠΈΡΒ»). ΠΠ΄Π΅ΡΡ ΡΠΊΠΎΡΠΎΡΡΡ ΡΠΎΡΡΠ°Π²Π»ΡΠ΅Ρ ΡΠ³ΠΎΠ» ΞΈ Ρ B, ΡΠ°ΠΊ ΡΡΠΎ Π΅Π΅ ΡΠΎΡΡΠ°Π²Π»ΡΡΡΠ°Ρ, ΠΏΠ΅ΡΠΏΠ΅Π½Π΄ΠΈΠΊΡΠ»ΡΡΠ½Π°Ρ B, ΡΠ°Π²Π½Π° vsinΞΈ.
Π‘Ρ Π΅ΠΌΠ° ΡΠ»Π΅ΠΊΡΡΠΈΡΠ΅ΡΠΊΠΎΠ³ΠΎ Π³Π΅Π½Π΅ΡΠ°ΡΠΎΡΠ° : ΠΠ΅Π½Π΅ΡΠ°ΡΠΎΡ Ρ ΠΎΠ΄Π½ΠΎΠΉ ΠΏΡΡΠΌΠΎΡΠ³ΠΎΠ»ΡΠ½ΠΎΠΉ ΠΊΠ°ΡΡΡΠΊΠΎΠΉ, Π²ΡΠ°ΡΠ°ΡΡΠ΅ΠΉΡΡ Ρ ΠΏΠΎΡΡΠΎΡΠ½Π½ΠΎΠΉ ΡΠ³Π»ΠΎΠ²ΠΎΠΉ ΡΠΊΠΎΡΠΎΡΡΡΡ Π² ΠΎΠ΄Π½ΠΎΡΠΎΠ΄Π½ΠΎΠΌ ΠΌΠ°Π³Π½ΠΈΡΠ½ΠΎΠΌ ΠΏΠΎΠ»Π΅, ΠΏΡΠΎΠΈΠ·Π²ΠΎΠ΄ΠΈΡ ΠΠΠ‘, ΠΊΠΎΡΠΎΡΠ°Ρ ΠΈΠ·ΠΌΠ΅Π½ΡΠ΅ΡΡΡ ΡΠΈΠ½ΡΡΠΎΠΈΠ΄Π°Π»ΡΠ½ΠΎ Π²ΠΎ Π²ΡΠ΅ΠΌΠ΅Π½ΠΈ. ΠΠ±ΡΠ°ΡΠΈΡΠ΅ Π²Π½ΠΈΠΌΠ°Π½ΠΈΠ΅, ΡΡΠΎ Π³Π΅Π½Π΅ΡΠ°ΡΠΎΡ ΠΏΠΎΡ ΠΎΠΆ Π½Π° Π΄Π²ΠΈΠ³Π°ΡΠ΅Π»Ρ, Π·Π° ΠΈΡΠΊΠ»ΡΡΠ΅Π½ΠΈΠ΅ΠΌ ΡΠΎΠ³ΠΎ, ΡΡΠΎ Π²Π°Π» Π²ΡΠ°ΡΠ°Π΅ΡΡΡ Π΄Π»Ρ ΡΠΎΠ·Π΄Π°Π½ΠΈΡ ΡΠΎΠΊΠ°, Π° Π½Π΅ Π½Π°ΠΎΠ±ΠΎΡΠΎΡ.
Π’Π°ΠΊΠΈΠΌ ΠΎΠ±ΡΠ°Π·ΠΎΠΌ, Π² ΡΡΠΎΠΌ ΡΠ»ΡΡΠ°Π΅ ΠΠΠ‘, ΠΈΠ½Π΄ΡΡΠΈΡΠΎΠ²Π°Π½Π½Π°Ρ Ρ ΠΊΠ°ΠΆΠ΄ΠΎΠΉ ΡΡΠΎΡΠΎΠ½Ρ, ΡΠ°Π²Π½Π° ΠΠΠ‘=BβvsinΞΈ, ΠΈ ΠΎΠ½ΠΈ ΠΈΠΌΠ΅ΡΡ ΠΎΠ΄ΠΈΠ½Π°ΠΊΠΎΠ²ΠΎΠ΅ Π½Π°ΠΏΡΠ°Π²Π»Π΅Π½ΠΈΠ΅. ΠΠΎΠ»Π½Π°Ρ ΠΠΠ‘ [Π»Π°ΡΠ΅ΠΊΡ]\varepsilon[/Π»Π°ΡΠ΅ΠΊΡ] Π²ΠΎΠΊΡΡΠ³ ΠΊΠΎΠ½ΡΡΡΠ° ΡΠΎΠ³Π΄Π°:
[Π»Π°ΡΠ΅ΠΊΡ]\varepsilon = 2 \text{Blv} \sin{\theta}[/latex].
ΠΡΠΎ Π²ΡΡΠ°ΠΆΠ΅Π½ΠΈΠ΅ Π²Π΅ΡΠ½ΠΎ, Π½ΠΎ ΠΎΠ½ΠΎ Π½Π΅ Π΄Π°Π΅Ρ ΠΠΠ‘ ΠΊΠ°ΠΊ ΡΡΠ½ΠΊΡΠΈΡ Π²ΡΠ΅ΠΌΠ΅Π½ΠΈ. ΠΠ»Ρ Π½Π°Ρ ΠΎΠΆΠ΄Π΅Π½ΠΈΡ Π·Π°Π²ΠΈΡΠΈΠΌΠΎΡΡΠΈ ΠΠΠ‘ ΠΎΡ Π²ΡΠ΅ΠΌΠ΅Π½ΠΈ ΠΏΡΠ΅Π΄ΠΏΠΎΠ»ΠΎΠΆΠΈΠΌ, ΡΡΠΎ ΠΊΠ°ΡΡΡΠΊΠ° Π²ΡΠ°ΡΠ°Π΅ΡΡΡ Ρ ΠΏΠΎΡΡΠΎΡΠ½Π½ΠΎΠΉ ΡΠ³Π»ΠΎΠ²ΠΎΠΉ ΡΠΊΠΎΡΠΎΡΡΡΡ Ο. Π£Π³ΠΎΠ» ΞΈ ΡΠ²ΡΠ·Π°Π½ Ρ ΡΠ³Π»ΠΎΠ²ΠΎΠΉ ΡΠΊΠΎΡΠΎΡΡΡΡ ΡΠΎΠΎΡΠ½ΠΎΡΠ΅Π½ΠΈΠ΅ΠΌ ΞΈ=Οt, ΡΠ°ΠΊ ΡΡΠΎ:
[Π»Π°ΡΠ΅ΠΊΡ]\varepsilon = 2 \text{Blv} \sin{\omega \text{t}}[/latex].
Π’Π΅ΠΏΠ΅ΡΡ Π»ΠΈΠ½Π΅ΠΉΠ½Π°Ρ ΡΠΊΠΎΡΠΎΡΡΡ v ΡΠ²ΡΠ·Π°Π½Π° Ρ ΡΠ³Π»ΠΎΠ²ΠΎΠΉ ΡΠΊΠΎΡΠΎΡΡΡΡ ΡΠΎΠΎΡΠ½ΠΎΡΠ΅Π½ΠΈΠ΅ΠΌ v=rΟ. ΠΠ΄Π΅ΡΡ r=w/2, ΡΠ°ΠΊ ΡΡΠΎ v=(w/2)Ο, ΠΈ:
[Π»Π°ΡΠ΅ΠΊΡ]\varepsilon = 2 \text{Bl} \frac{\text{w}}{2} \omega \sin{\omega \text{t}} = (\text{lw})\text{ B}\omega \sin{\omega \text{t}}[/latex].
ΠΠ°ΠΌΠ΅ΡΠΈΠ², ΡΡΠΎ ΠΏΠ»ΠΎΡΠ°Π΄Ρ ΠΏΠ΅ΡΠ»ΠΈ ΡΠ°Π²Π½Π° A=βw, ΠΈ ΡΡΠΈΡΡΠ²Π°Ρ N ΠΏΠ΅ΡΠ΅Π»Ρ, ΠΌΡ Π½Π°Ρ ΠΎΠ΄ΠΈΠΌ, ΡΡΠΎ:
[Π»Π°ΡΠ΅ΠΊΡ]\varepsilon = \text{NABw}~\sin{\omega \text{t}}[ /Π»Π°ΡΠ΅ΠΊΡ] β ΡΡΠΎ ΠΠΠ‘, ΠΈΠ½Π΄ΡΡΠΈΡΡΠ΅ΠΌΠ°Ρ Π² ΠΊΠ°ΡΡΡΠΊΠ΅ Π³Π΅Π½Π΅ΡΠ°ΡΠΎΡΠ° ΠΈΠ· N Π²ΠΈΡΠΊΠΎΠ² ΠΈ ΠΎΠ±Π»Π°ΡΡΠΈ Π, Π²ΡΠ°ΡΠ°ΡΡΠ΅ΠΉΡΡ Ρ ΠΏΠΎΡΡΠΎΡΠ½Π½ΠΎΠΉ ΡΠ³Π»ΠΎΠ²ΠΎΠΉ ΡΠΊΠΎΡΠΎΡΡΡΡ Π² ΠΎΠ΄Π½ΠΎΡΠΎΠ΄Π½ΠΎΠΌ ΠΌΠ°Π³Π½ΠΈΡΠ½ΠΎΠΌ ΠΏΠΎΠ»Π΅ Π.
ΠΠ΅Π½Π΅ΡΠ°ΡΠΎΡΡ, ΠΏΠΎΠΊΠ°Π·Π°Π½Π½ΡΠ΅ Π² ΡΡΠΎΠΌ Π°ΡΠΎΠΌΠ΅, ΠΎΡΠ΅Π½Ρ ΠΏΠΎΡ ΠΎΠΆΠΈ Π½Π° Π΄Π²ΠΈΠ³Π°ΡΠ΅Π»ΠΈ, ΠΏΠΎΠΊΠ°Π·Π°Π½Π½ΡΠ΅ ΡΠ°Π½Π΅Π΅. ΠΡΠΎ Π½Π΅ ΡΠ»ΡΡΠ°ΠΉΠ½ΠΎ. Π€Π°ΠΊΡΠΈΡΠ΅ΡΠΊΠΈ Π΄Π²ΠΈΠ³Π°ΡΠ΅Π»Ρ ΡΡΠ°Π½ΠΎΠ²ΠΈΡΡΡ Π³Π΅Π½Π΅ΡΠ°ΡΠΎΡΠΎΠΌ, ΠΊΠΎΠ³Π΄Π° Π΅Π³ΠΎ Π²Π°Π» Π²ΡΠ°ΡΠ°Π΅ΡΡΡ.
ΠΠ»Π΅ΠΊΡΡΠΎΠ΄Π²ΠΈΠ³Π°ΡΠ΅Π»ΠΈ
ΠΠ»Π΅ΠΊΡΡΠΎΠ΄Π²ΠΈΠ³Π°ΡΠ΅Π»Ρ β ΡΡΠΎ ΡΡΡΡΠΎΠΉΡΡΠ²ΠΎ, ΠΏΡΠ΅ΠΎΠ±ΡΠ°Π·ΡΡΡΠ΅Π΅ ΡΠ»Π΅ΠΊΡΡΠΈΡΠ΅ΡΠΊΡΡ ΡΠ½Π΅ΡΠ³ΠΈΡ Π² ΠΌΠ΅Ρ Π°Π½ΠΈΡΠ΅ΡΠΊΡΡ.
Π¦Π΅Π»ΠΈ ΠΎΠ±ΡΡΠ΅Π½ΠΈΡ
ΠΠ±ΡΡΡΠ½ΠΈΡΡ, ΠΊΠ°ΠΊ ΡΠΈΠ»Π° Π³Π΅Π½Π΅ΡΠΈΡΡΠ΅ΡΡΡ Π² ΡΠ»Π΅ΠΊΡΡΠΎΠ΄Π²ΠΈΠ³Π°ΡΠ΅Π»ΡΡ
ΠΡΠ½ΠΎΠ²Π½ΡΠ΅ Π²ΡΠ²ΠΎΠ΄Ρ
ΠΠ»ΡΡΠ΅Π²ΡΠ΅ ΠΌΠΎΠΌΠ΅Π½ΡΡ
- Π Π±ΠΎΠ»ΡΡΠΈΠ½ΡΡΠ²Π΅ ΡΠ»Π΅ΠΊΡΡΠΎΠ΄Π²ΠΈΠ³Π°ΡΠ΅Π»Π΅ΠΉ Π΄Π»Ρ ΡΠΎΠ·Π΄Π°Π½ΠΈΡ ΡΠΈΠ»Ρ ΠΈΡΠΏΠΎΠ»ΡΠ·ΡΠ΅ΡΡΡ Π²Π·Π°ΠΈΠΌΠΎΠ΄Π΅ΠΉΡΡΠ²ΠΈΠ΅ ΠΌΠ°Π³Π½ΠΈΡΠ½ΡΡ ΠΏΠΎΠ»Π΅ΠΉ ΠΈ ΠΏΡΠΎΠ²ΠΎΠ΄Π½ΠΈΠΊΠΎΠ² Ρ ΡΠΎΠΊΠΎΠΌ.
- Π’ΠΎΠΊ Π² ΠΏΡΠΎΠ²ΠΎΠ΄Π½ΠΈΠΊΠ΅ ΡΠΎΡΡΠΎΠΈΡ ΠΈΠ· Π΄Π²ΠΈΠΆΡΡΠΈΡ ΡΡ Π·Π°ΡΡΠ΄ΠΎΠ². Π‘Π»Π΅Π΄ΠΎΠ²Π°ΡΠ΅Π»ΡΠ½ΠΎ, ΠΊΠ°ΡΡΡΠΊΠ° Ρ ΡΠΎΠΊΠΎΠΌ Π² ΠΌΠ°Π³Π½ΠΈΡΠ½ΠΎΠΌ ΠΏΠΎΠ»Π΅ ΡΠ°ΠΊΠΆΠ΅ Π±ΡΠ΄Π΅Ρ ΠΎΡΡΡΠ°ΡΡ ΡΠΈΠ»Ρ ΠΠΎΡΠ΅Π½ΡΠ°.
- Π Π΄Π²ΠΈΠ³Π°ΡΠ΅Π»Π΅ ΠΊΠ°ΡΡΡΠΊΠ° Ρ ΡΠΎΠΊΠΎΠΌ Π² ΠΌΠ°Π³Π½ΠΈΡΠ½ΠΎΠΌ ΠΏΠΎΠ»Π΅ ΠΈΡΠΏΡΡΡΠ²Π°Π΅Ρ ΡΠΈΠ»Ρ Ρ ΠΎΠ±Π΅ΠΈΡ ΡΡΠΎΡΠΎΠ½ ΠΊΠ°ΡΡΡΠΊΠΈ, ΠΊΠΎΡΠΎΡΠ°Ρ ΡΠΎΠ·Π΄Π°Π΅Ρ ΠΊΡΡΡΡΡΡΡ ΡΠΈΠ»Ρ (Π½Π°Π·ΡΠ²Π°Π΅ΠΌΡΡ ΠΊΡΡΡΡΡΠΈΠΌ ΠΌΠΎΠΌΠ΅Π½ΡΠΎΠΌ), ΠΊΠΎΡΠΎΡΠ°Ρ Π·Π°ΡΡΠ°Π²Π»ΡΠ΅Ρ Π΅Π΅ Π²ΡΠ°ΡΠ°ΡΡΡΡ.
ΠΠ»ΡΡΠ΅Π²ΡΠ΅ ΡΠ΅ΡΠΌΠΈΠ½Ρ
- Π‘ΠΈΠ»Π° ΠΠΎΡΠ΅Π½ΡΠ° : Π‘ΠΈΠ»Π°, Π΄Π΅ΠΉΡΡΠ²ΡΡΡΠ°Ρ Π½Π° Π·Π°ΡΡΠΆΠ΅Π½Π½ΡΡ ΡΠ°ΡΡΠΈΡΡ Π² ΡΠ»Π΅ΠΊΡΡΠΎΠΌΠ°Π³Π½ΠΈΡΠ½ΠΎΠΌ ΠΏΠΎΠ»Π΅.
- ΠΊΡΡΡΡΡΠΈΠΉ ΠΌΠΎΠΌΠ΅Π½Ρ : Π²ΡΠ°ΡΠ°ΡΠ΅Π»ΡΠ½ΠΎΠ΅ ΠΈΠ»ΠΈ ΡΠΊΡΡΡΠΈΠ²Π°ΡΡΠ΅Π΅ Π΄Π΅ΠΉΡΡΠ²ΠΈΠ΅ ΡΠΈΠ»Ρ; (Π΅Π΄ΠΈΠ½ΠΈΡΠ° Π‘Π Π½ΡΡΡΠΎΠ½-ΠΌΠ΅ΡΡ ΠΈΠ»ΠΈ ΠΠΌ; Π±ΡΠΈΡΠ°Π½ΡΠΊΠ°Ρ Π΅Π΄ΠΈΠ½ΠΈΡΠ° ΡΡΡ-ΡΡΠ½Ρ ΠΈΠ»ΠΈ ΡΡΡ-ΡΡΠ½Ρ)
ΠΡΠ½ΠΎΠ²Π½ΡΠ΅ ΠΏΡΠΈΠ½ΡΠΈΠΏΡ ΡΠ°Π±ΠΎΡΡ Π΄Π²ΠΈΠ³Π°ΡΠ΅Π»Ρ ΡΠ°ΠΊΠΈΠ΅ ΠΆΠ΅, ΠΊΠ°ΠΊ Ρ Π³Π΅Π½Π΅ΡΠ°ΡΠΎΡΠ°, Π·Π° ΠΈΡΠΊΠ»ΡΡΠ΅Π½ΠΈΠ΅ΠΌ ΡΠΎΠ³ΠΎ, ΡΡΠΎ Π΄Π²ΠΈΠ³Π°ΡΠ΅Π»Ρ ΠΏΡΠ΅ΠΎΠ±ΡΠ°Π·ΡΠ΅Ρ ΡΠ»Π΅ΠΊΡΡΠΈΡΠ΅ΡΠΊΡΡ ΡΠ½Π΅ΡΠ³ΠΈΡ Π² ΠΌΠ΅Ρ Π°Π½ΠΈΡΠ΅ΡΠΊΡΡ (Π΄Π²ΠΈΠΆΠ΅Π½ΠΈΠ΅). (Π‘Π½Π°ΡΠ°Π»Π° ΠΏΡΠΎΡΠΈΡΠ°ΠΉΡΠ΅ Π½Π°Ρ Π°ΡΠΎΠΌ ΠΎΠ± ΡΠ»Π΅ΠΊΡΡΠΈΡΠ΅ΡΠΊΠΈΡ Π³Π΅Π½Π΅ΡΠ°ΡΠΎΡΠ°Ρ .) ΠΠΎΠ»ΡΡΠΈΠ½ΡΡΠ²ΠΎ ΡΠ»Π΅ΠΊΡΡΠΎΠ΄Π²ΠΈΠ³Π°ΡΠ΅Π»Π΅ΠΉ ΠΈΡΠΏΠΎΠ»ΡΠ·ΡΡΡ Π²Π·Π°ΠΈΠΌΠΎΠ΄Π΅ΠΉΡΡΠ²ΠΈΠ΅ ΠΌΠ°Π³Π½ΠΈΡΠ½ΡΡ ΠΏΠΎΠ»Π΅ΠΉ ΠΈ ΠΏΡΠΎΠ²ΠΎΠ΄Π½ΠΈΠΊΠΎΠ² Ρ ΡΠΎΠΊΠΎΠΌ Π΄Π»Ρ ΡΠΎΠ·Π΄Π°Π½ΠΈΡ ΡΠΈΠ»Ρ. ΠΠ»Π΅ΠΊΡΡΠΎΠ΄Π²ΠΈΠ³Π°ΡΠ΅Π»ΠΈ ΠΈΡΠΏΠΎΠ»ΡΠ·ΡΡΡΡΡ Π² ΡΠ°ΠΊΠΈΡ ΡΠ°Π·Π½ΠΎΠΎΠ±ΡΠ°Π·Π½ΡΡ ΠΎΠ±Π»Π°ΡΡΡΡ , ΠΊΠ°ΠΊ ΠΏΡΠΎΠΌΡΡΠ»Π΅Π½Π½ΡΠ΅ Π²Π΅Π½ΡΠΈΠ»ΡΡΠΎΡΡ, Π²ΠΎΠ·Π΄ΡΡ ΠΎΠ΄ΡΠ²ΠΊΠΈ ΠΈ Π½Π°ΡΠΎΡΡ, ΡΡΠ°Π½ΠΊΠΈ, Π±ΡΡΠΎΠ²Π°Ρ ΡΠ΅Ρ Π½ΠΈΠΊΠ°, ΡΠ»Π΅ΠΊΡΡΠΎΠΈΠ½ΡΡΡΡΠΌΠ΅Π½ΡΡ ΠΈ Π΄ΠΈΡΠΊΠΎΠ²ΡΠ΅ Π½Π°ΠΊΠΎΠΏΠΈΡΠ΅Π»ΠΈ.
Π‘ΠΈΠ»Π° ΠΠΎΡΠ΅Π½ΡΠ°
ΠΡΠ»ΠΈ Π±Ρ Π²Ρ ΠΏΠΎΠΌΠ΅ΡΡΠΈΠ»ΠΈ Π΄Π²ΠΈΠΆΡΡΡΡΡΡ Π·Π°ΡΡΠΆΠ΅Π½Π½ΡΡ ΡΠ°ΡΡΠΈΡΡ Π² ΠΌΠ°Π³Π½ΠΈΡΠ½ΠΎΠ΅ ΠΏΠΎΠ»Π΅, ΠΎΠ½Π° ΠΈΡΠΏΡΡΠ°Π»Π° Π±Ρ ΡΠΈΠ»Ρ, Π½Π°Π·ΡΠ²Π°Π΅ΠΌΡΡ ΡΠΈΠ»ΠΎΠΉ ΠΠΎΡΠ΅Π½ΡΠ°:
[Π»Π°ΡΠ΅ΠΊΡ]\ΡΠ΅ΠΊΡΡ{F}=\ΡΠ΅ΠΊΡΡ{q}\times \text {v}\times \text{B}[/latex]
ΠΡΠ°Π²ΠΈΠ»ΠΎ ΠΏΡΠ°Π²ΠΎΠΉ ΡΡΠΊΠΈ : ΠΡΠ°Π²ΠΈΠ»ΠΎ ΠΏΡΠ°Π²ΠΎΠΉ ΡΡΠΊΠΈ, ΠΏΠΎΠΊΠ°Π·ΡΠ²Π°ΡΡΠ΅Π΅ Π½Π°ΠΏΡΠ°Π²Π»Π΅Π½ΠΈΠ΅ ΡΠΈΠ»Ρ ΠΠΎΡΠ΅Π½ΡΠ°
, Π³Π΄Π΅ v — ΡΠΊΠΎΡΠΎΡΡΡ Π΄Π²ΠΈΠΆΡΡΠ΅Π³ΠΎΡΡ Π·Π°ΡΡΠ΄Π°, q β Π·Π°ΡΡΠ΄, Π β ΠΌΠ°Π³Π½ΠΈΡΠ½ΠΎΠ΅ ΠΏΠΎΠ»Π΅. Π’ΠΎΠΊ Π² ΠΏΡΠΎΠ²ΠΎΠ΄Π½ΠΈΠΊΠ΅ ΡΠΎΡΡΠΎΠΈΡ ΠΈΠ· Π΄Π²ΠΈΠΆΡΡΠΈΡ ΡΡ Π·Π°ΡΡΠ΄ΠΎΠ². Π‘Π»Π΅Π΄ΠΎΠ²Π°ΡΠ΅Π»ΡΠ½ΠΎ, ΠΊΠ°ΡΡΡΠΊΠ° Ρ ΡΠΎΠΊΠΎΠΌ Π² ΠΌΠ°Π³Π½ΠΈΡΠ½ΠΎΠΌ ΠΏΠΎΠ»Π΅ ΡΠ°ΠΊΠΆΠ΅ Π±ΡΠ΄Π΅Ρ ΠΎΡΡΡΠ°ΡΡ ΡΠΈΠ»Ρ ΠΠΎΡΠ΅Π½ΡΠ°. ΠΠ»Ρ ΠΏΡΡΠΌΠΎΠ»ΠΈΠ½Π΅ΠΉΠ½ΠΎΠ³ΠΎ ΠΏΡΠΎΠ²ΠΎΠ΄Π° Ρ ΡΠΎΠΊΠΎΠΌ, ΠΊΠΎΡΠΎΡΡΠΉ Π½Π΅ Π΄Π²ΠΈΠΆΠ΅ΡΡΡ, ΡΠΈΠ»Π° ΠΠΎΡΠ΅Π½ΡΠ° ΡΠ°Π²Π½Π°:
[Π»Π°ΡΠ΅ΠΊΡ]\ΡΠ΅ΠΊΡΡ{F}=\ΡΠ΅ΠΊΡΡ{I}\times \text{L}\times \text{B}[/latex]
Π³Π΄Π΅ F β ΡΠΈΠ»Π° (Π² Π½ΡΡΡΠΎΠ½Π°Ρ , Π), I β ΡΠΈΠ»Π° ΡΠΎΠΊΠ° Π² ΠΏΡΠΎΠ²ΠΎΠ΄Π΅ (Π² Π°ΠΌΠΏΠ΅ΡΠ°Ρ , Π), L β Π΄Π»ΠΈΠ½Π° ΠΏΡΠΎΠ²ΠΎΠ΄Π°, Π½Π°Ρ ΠΎΠ΄ΡΡΠ΅Π³ΠΎΡΡ Π² ΠΌΠ°Π³Π½ΠΈΡΠ½ΠΎΠΌ ΠΏΠΎΠ»Π΅ (Π² ΠΌ), B β Π½Π°ΠΏΡΡΠΆΠ΅Π½Π½ΠΎΡΡΡ ΠΌΠ°Π³Π½ΠΈΡΠ½ΠΎΠ³ΠΎ ΠΏΠΎΠ»Ρ (Π² ΡΠ΅ΡΠ»Π°Ρ , Π’Π» ). ΠΠ°ΠΏΡΠ°Π²Π»Π΅Π½ΠΈΠ΅ ΡΠΈΠ»Ρ ΠΠΎΡΠ΅Π½ΡΠ° ΠΏΠ΅ΡΠΏΠ΅Π½Π΄ΠΈΠΊΡΠ»ΡΡΠ½ΠΎ ΠΊΠ°ΠΊ Π½Π°ΠΏΡΠ°Π²Π»Π΅Π½ΠΈΡ ΡΠΎΠΊΠ°, ΡΠ°ΠΊ ΠΈ ΠΌΠ°Π³Π½ΠΈΡΠ½ΠΎΠ³ΠΎ ΠΏΠΎΠ»Ρ, ΠΈ Π΅Π³ΠΎ ΠΌΠΎΠΆΠ½ΠΎ Π½Π°ΠΉΡΠΈ Ρ ΠΏΠΎΠΌΠΎΡΡΡ ΠΏΡΠ°Π²ΠΈΠ»Π° ΠΏΡΠ°Π²ΠΎΠΉ ΡΡΠΊΠΈ, ΠΏΠΎΠΊΠ°Π·Π°Π½Π½ΠΎΠ³ΠΎ Π½Π° ΡΠΈΡ. ΠΡΠ°Π²ΠΎΠΉ ΡΡΠΊΠΎΠΉ Π½Π°ΠΏΡΠ°Π²ΡΡΠ΅ Π±ΠΎΠ»ΡΡΠΎΠΉ ΠΏΠ°Π»Π΅Ρ Π² Π½Π°ΠΏΡΠ°Π²Π»Π΅Π½ΠΈΠΈ ΡΠΎΠΊΠ°, ΠΈ ΡΠΊΠ°ΠΆΠΈΡΠ΅ ΠΏΠ°Π»ΡΡΠ΅ΠΌ Π² Π½Π°ΠΏΡΠ°Π²Π»Π΅Π½ΠΈΠΈ ΠΌΠ°Π³Π½ΠΈΡΠ½ΠΎΠ³ΠΎ ΠΏΠΎΠ»Ρ. Π’Π΅ΠΏΠ΅ΡΡ Π²Π°Ρ Π±Π΅Π·ΡΠΌΡΠ½Π½ΡΠΉ ΠΏΠ°Π»Π΅Ρ Π±ΡΠ΄Π΅Ρ ΡΠΊΠ°Π·ΡΠ²Π°ΡΡ Π² Π½Π°ΠΏΡΠ°Π²Π»Π΅Π½ΠΈΠΈ ΡΠΈΠ»Ρ.
ΠΡΡΡΡΡΠΈΠΉ ΠΌΠΎΠΌΠ΅Π½Ρ : Π‘ΠΈΠ»Π° Π½Π° ΠΏΡΠΎΡΠΈΠ²ΠΎΠΏΠΎΠ»ΠΎΠΆΠ½ΡΡ ΡΡΠΎΡΠΎΠ½Π°Ρ ΠΊΠ°ΡΡΡΠΊΠΈ Π±ΡΠ΄Π΅Ρ Π΄Π΅ΠΉΡΡΠ²ΠΎΠ²Π°ΡΡ Π² ΠΏΡΠΎΡΠΈΠ²ΠΎΠΏΠΎΠ»ΠΎΠΆΠ½ΡΡ Π½Π°ΠΏΡΠ°Π²Π»Π΅Π½ΠΈΡΡ , ΠΏΠΎΡΠΎΠΌΡ ΡΡΠΎ Π·Π°ΡΡΠ΄Ρ Π΄Π²ΠΈΠΆΡΡΡΡ Π² ΠΏΡΠΎΡΠΈΠ²ΠΎΠΏΠΎΠ»ΠΎΠΆΠ½ΡΡ Π½Π°ΠΏΡΠ°Π²Π»Π΅Π½ΠΈΡΡ . ΠΡΠΎ ΠΎΠ·Π½Π°ΡΠ°Π΅Ρ, ΡΡΠΎ ΠΊΠ°ΡΡΡΠΊΠ° Π±ΡΠ΄Π΅Ρ Π²ΡΠ°ΡΠ°ΡΡΡΡ.
ΠΠ΅Ρ Π°Π½ΠΈΠΊΠ° Π΄Π²ΠΈΠ³Π°ΡΠ΅Π»Ρ
Π Π΄Π²ΠΈΠ³Π°ΡΠ΅Π»ΠΈ, ΠΈ Π³Π΅Π½Π΅ΡΠ°ΡΠΎΡΡ ΠΌΠΎΠΆΠ½ΠΎ ΠΎΠ±ΡΡΡΠ½ΠΈΡΡ Ρ ΡΠΎΡΠΊΠΈ Π·ΡΠ΅Π½ΠΈΡ ΠΊΠ°ΡΡΡΠΊΠΈ, ΠΊΠΎΡΠΎΡΠ°Ρ Π²ΡΠ°ΡΠ°Π΅ΡΡΡ Π² ΠΌΠ°Π³Π½ΠΈΡΠ½ΠΎΠΌ ΠΏΠΎΠ»Π΅. Π Π³Π΅Π½Π΅ΡΠ°ΡΠΎΡΠ΅ ΠΊΠ°ΡΡΡΠΊΠ° ΠΏΠΎΠ΄ΠΊΠ»ΡΡΠ°Π΅ΡΡΡ ΠΊ Π²Π½Π΅ΡΠ½Π΅ΠΉ ΡΠ΅ΠΏΠΈ, ΠΊΠΎΡΠΎΡΠ°Ρ Π·Π°ΡΠ΅ΠΌ Π²ΠΊΠ»ΡΡΠ°Π΅ΡΡΡ. ΠΡΠΎ ΠΏΡΠΈΠ²ΠΎΠ΄ΠΈΡ ΠΊ ΠΈΠ·ΠΌΠ΅Π½Π΅Π½ΠΈΡ ΠΏΠΎΡΠΎΠΊΠ°, ΠΊΠΎΡΠΎΡΡΠΉ ΠΈΠ½Π΄ΡΡΠΈΡΡΠ΅Ρ ΡΠ»Π΅ΠΊΡΡΠΎΠΌΠ°Π³Π½ΠΈΡΠ½ΠΎΠ΅ ΠΏΠΎΠ»Π΅. Π Π΄Π²ΠΈΠ³Π°ΡΠ΅Π»Π΅ ΠΊΠ°ΡΡΡΠΊΠ° Ρ ΡΠΎΠΊΠΎΠΌ Π² ΠΌΠ°Π³Π½ΠΈΡΠ½ΠΎΠΌ ΠΏΠΎΠ»Π΅ ΠΈΡΠΏΡΡΡΠ²Π°Π΅Ρ ΡΠΈΠ»Ρ Ρ ΠΎΠ±Π΅ΠΈΡ ΡΡΠΎΡΠΎΠ½ ΠΊΠ°ΡΡΡΠΊΠΈ, ΠΊΠΎΡΠΎΡΠ°Ρ ΡΠΎΠ·Π΄Π°Π΅Ρ ΠΊΡΡΡΡΡΡΡ ΡΠΈΠ»Ρ (Π½Π°Π·ΡΠ²Π°Π΅ΠΌΡΡ ΠΊΡΡΡΡΡΠΈΠΌ ΠΌΠΎΠΌΠ΅Π½ΡΠΎΠΌ), ΠΊΠΎΡΠΎΡΠ°Ρ Π·Π°ΡΡΠ°Π²Π»ΡΠ΅Ρ Π΅Π΅ Π²ΡΠ°ΡΠ°ΡΡΡΡ. ΠΡΠ±Π°Ρ ΠΊΠ°ΡΡΡΠΊΠ° Ρ ΡΠΎΠΊΠΎΠΌ ΠΌΠΎΠΆΠ΅Ρ ΡΡΠ²ΡΡΠ²ΠΎΠ²Π°ΡΡ ΡΠΈΠ»Ρ Π² ΠΌΠ°Π³Π½ΠΈΡΠ½ΠΎΠΌ ΠΏΠΎΠ»Π΅. ΠΡΠ° ΡΠΈΠ»Π° ΠΏΡΠ΅Π΄ΡΡΠ°Π²Π»ΡΠ΅Ρ ΡΠΎΠ±ΠΎΠΉ ΡΠΈΠ»Ρ ΠΠΎΡΠ΅Π½ΡΠ°, Π΄Π΅ΠΉΡΡΠ²ΡΡΡΡΡ Π½Π° Π΄Π²ΠΈΠΆΡΡΠΈΠ΅ΡΡ Π·Π°ΡΡΠ΄Ρ Π² ΠΏΡΠΎΠ²ΠΎΠ΄Π½ΠΈΠΊΠ΅. Π‘ΠΈΠ»Π° Π½Π° ΠΏΡΠΎΡΠΈΠ²ΠΎΠΏΠΎΠ»ΠΎΠΆΠ½ΡΡ ΡΡΠΎΡΠΎΠ½Π°Ρ ΠΊΠ°ΡΡΡΠΊΠΈ Π±ΡΠ΄Π΅Ρ ΠΏΡΠΎΡΠΈΠ²ΠΎΠΏΠΎΠ»ΠΎΠΆΠ½ΠΎΠΉ, ΠΏΠΎΡΠΎΠΌΡ ΡΡΠΎ Π·Π°ΡΡΠ΄Ρ Π΄Π²ΠΈΠΆΡΡΡΡ Π² ΠΏΡΠΎΡΠΈΠ²ΠΎΠΏΠΎΠ»ΠΎΠΆΠ½ΡΡ Π½Π°ΠΏΡΠ°Π²Π»Π΅Π½ΠΈΡΡ . ΠΡΠΎ ΠΎΠ·Π½Π°ΡΠ°Π΅Ρ, ΡΡΠΎ ΠΊΠ°ΡΡΡΠΊΠ° Π±ΡΠ΄Π΅Ρ Π²ΡΠ°ΡΠ°ΡΡΡΡ.
ΠΠ½Π΄ΡΠΊΡΠΈΠ²Π½ΠΎΡΡΡ
ΠΠ½Π΄ΡΠΊΡΠΈΠ²Π½ΠΎΡΡΡ β ΡΡΠΎ ΡΠ²ΠΎΠΉΡΡΠ²ΠΎ ΡΡΡΡΠΎΠΉΡΡΠ²Π°, ΠΊΠΎΡΠΎΡΠΎΠ΅ ΠΏΠΎΠΊΠ°Π·ΡΠ²Π°Π΅Ρ, Π½Π°ΡΠΊΠΎΠ»ΡΠΊΠΎ ΡΡΡΠ΅ΠΊΡΠΈΠ²Π½ΠΎ ΠΎΠ½ΠΎ ΠΈΠ½Π΄ΡΡΠΈΡΡΠ΅Ρ ΠΠΠ‘ Π² Π΄ΡΡΠ³ΠΎΠΌ ΡΡΡΡΠΎΠΉΡΡΠ²Π΅ ΠΈΠ»ΠΈ Π½Π° ΡΠ°ΠΌΠΎΠΌ ΡΠ΅Π±Π΅.
Π¦Π΅Π»ΠΈ ΠΎΠ±ΡΡΠ΅Π½ΠΈΡ
ΠΠΏΠΈΡΠ°ΡΡ ΡΠ²ΠΎΠΉΡΡΠ²Π° ΠΊΠ°ΡΡΡΠΊΠΈ ΠΈΠ½Π΄ΡΠΊΡΠΈΠ²Π½ΠΎΡΡΠΈ, ΡΠ°Π·Π»ΠΈΡΠ°Ρ Π²Π·Π°ΠΈΠΌΠ½ΡΡ ΠΈΠ½Π΄ΡΠΊΡΠΈΠ²Π½ΠΎΡΡΡ ΠΈ ΡΠ°ΠΌΠΎΠΈΠ½Π΄ΡΠΊΡΠΈΡ
ΠΠ»ΡΡΠ΅Π²ΡΠ΅ Π²ΡΠ²ΠΎΠ΄Ρ
ΠΠ»ΡΡΠ΅Π²ΡΠ΅ ΠΌΠΎΠΌΠ΅Π½ΡΡ
- ΠΠ·Π°ΠΈΠΌΠ½Π°Ρ ΠΈΠ½Π΄ΡΠΊΡΠΈΠ²Π½ΠΎΡΡΡ β ΡΡΠΎ Π΄Π΅ΠΉΡΡΠ²ΠΈΠ΅ Π΄Π²ΡΡ ΡΡΡΡΠΎΠΉΡΡΠ², ΠΈΠ½Π΄ΡΡΠΈΡΡΡΡΠΈΡ ΠΠΠ‘ Π΄ΡΡΠ³ Π² Π΄ΡΡΠ³Π΅. ΠΠ·ΠΌΠ΅Π½Π΅Π½ΠΈΠ΅ ΡΠΎΠΊΠ° ΞI 1 /Ξt Π² ΠΎΠ΄Π½ΠΎΠΌ ΠΈΠ½Π΄ΡΡΠΈΡΡΠ΅Ρ ΠΠΠ‘ ΡΠ΄Ρ2 Π²ΠΎ Π²ΡΠΎΡΠΎΠΌ: ΠΠΠ‘ 2 = βM ΞI 1 /Ξt, Π³Π΄Π΅ M ΠΎΠΏΡΠ΅Π΄Π΅Π»ΡΠ΅ΡΡΡ ΠΊΠ°ΠΊ Π²Π·Π°ΠΈΠΌΠ½Π°Ρ ΠΈΠ½Π΄ΡΠΊΡΠΈΠ²Π½ΠΎΡΡΡ ΠΌΠ΅ΠΆΠ΄Ρ Π΄Π²ΡΠΌΡ ΡΡΡΡΠΎΠΉΡΡΠ²Π°ΠΌΠΈ.
- Π‘Π°ΠΌΠΎΠΈΠ½Π΄ΡΠΊΡΠΈΡ — ΡΡΠΎ ΡΡΡΠ΅ΠΊΡ ΡΡΡΡΠΎΠΉΡΡΠ²Π°, ΠΈΠ½Π΄ΡΡΠΈΡΡΡΡΠ΅Π³ΠΎ ΠΠΠ‘ ΡΠ°ΠΌΠΎ ΠΏΠΎ ΡΠ΅Π±Π΅.
- Π£ΡΡΡΠΎΠΉΡΡΠ²ΠΎ, ΠΎΠ±Π»Π°Π΄Π°ΡΡΠ΅Π΅ Π·Π½Π°ΡΠΈΡΠ΅Π»ΡΠ½ΠΎΠΉ ΡΠΎΠ±ΡΡΠ²Π΅Π½Π½ΠΎΠΉ ΠΈΠ½Π΄ΡΠΊΡΠΈΠ²Π½ΠΎΡΡΡΡ, Π½Π°Π·ΡΠ²Π°Π΅ΡΡΡ ΠΈΠ½Π΄ΡΠΊΡΠΎΡΠΎΠΌ, Π° ΠΠΠ‘, ΠΈΠ½Π΄ΡΡΠΈΡΡΠ΅ΠΌΠ°Ρ Π² Π½Π΅ΠΌ ΠΈΠ·ΠΌΠ΅Π½Π΅Π½ΠΈΠ΅ΠΌ ΡΠΎΠΊΠ° ΡΠ΅ΡΠ΅Π· Π½Π΅Π³ΠΎ, ΡΠ°Π²Π½Π° ΠΠΠ‘ = βL βI/βt.
ΠΠ»ΡΡΠ΅Π²ΡΠ΅ ΡΠ΅ΡΠΌΠΈΠ½Ρ
- ΠΠ°ΠΊΠΎΠ½ ΠΈΠ½Π΄ΡΠΊΡΠΈΠΈ Π€Π°ΡΠ°Π΄Π΅Ρ : ΠΡΠ½ΠΎΠ²Π½ΠΎΠΉ Π·Π°ΠΊΠΎΠ½ ΡΠ»Π΅ΠΊΡΡΠΎΠΌΠ°Π³Π½Π΅ΡΠΈΠ·ΠΌΠ°, ΠΏΡΠ΅Π΄ΡΠΊΠ°Π·ΡΠ²Π°ΡΡΠΈΠΉ, ΠΊΠ°ΠΊ ΠΌΠ°Π³Π½ΠΈΡΠ½ΠΎΠ΅ ΠΏΠΎΠ»Π΅ Π±ΡΠ΄Π΅Ρ Π²Π·Π°ΠΈΠΌΠΎΠ΄Π΅ΠΉΡΡΠ²ΠΎΠ²Π°ΡΡ Ρ ΡΠ»Π΅ΠΊΡΡΠΈΡΠ΅ΡΠΊΠΎΠΉ ΡΠ΅ΠΏΡΡ, ΡΠΎΠ·Π΄Π°Π²Π°Ρ ΡΠ»Π΅ΠΊΡΡΠΎΠ΄Π²ΠΈΠΆΡΡΡΡ ΡΠΈΠ»Ρ (ΠΠΠ‘).
- ΡΡΠ°Π½ΡΡΠΎΡΠΌΠ°ΡΠΎΡ : Π‘ΡΠ°ΡΠΈΡΠ΅ΡΠΊΠΎΠ΅ ΡΡΡΡΠΎΠΉΡΡΠ²ΠΎ, ΠΊΠΎΡΠΎΡΠΎΠ΅ ΠΏΠ΅ΡΠ΅Π΄Π°Π΅Ρ ΡΠ»Π΅ΠΊΡΡΠΈΡΠ΅ΡΠΊΡΡ ΡΠ½Π΅ΡΠ³ΠΈΡ ΠΎΡ ΠΎΠ΄Π½ΠΎΠΉ ΡΠ΅ΠΏΠΈ ΠΊ Π΄ΡΡΠ³ΠΎΠΉ ΠΏΠΎΡΡΠ΅Π΄ΡΡΠ²ΠΎΠΌ ΠΌΠ°Π³Π½ΠΈΡΠ½ΠΎΠΉ ΡΠ²ΡΠ·ΠΈ. ΠΡ ΠΎΡΠ½ΠΎΠ²Π½ΠΎΠ΅ ΠΏΡΠΈΠΌΠ΅Π½Π΅Π½ΠΈΠ΅ Π·Π°ΠΊΠ»ΡΡΠ°Π΅ΡΡΡ Π² ΠΏΠ΅ΡΠ΅Π΄Π°ΡΠ΅ ΡΠ½Π΅ΡΠ³ΠΈΠΈ ΠΌΠ΅ΠΆΠ΄Ρ ΡΠ°Π·Π»ΠΈΡΠ½ΡΠΌΠΈ ΡΡΠΎΠ²Π½ΡΠΌΠΈ Π½Π°ΠΏΡΡΠΆΠ΅Π½ΠΈΡ, ΡΡΠΎ ΠΏΠΎΠ·Π²ΠΎΠ»ΡΠ΅Ρ Π²ΡΠ±ΡΠ°ΡΡ Π½Π°ΠΈΠ±ΠΎΠ»Π΅Π΅ ΠΏΠΎΠ΄Ρ ΠΎΠ΄ΡΡΠ΅Π΅ Π½Π°ΠΏΡΡΠΆΠ΅Π½ΠΈΠ΅ Π΄Π»Ρ ΠΏΡΠΎΠΈΠ·Π²ΠΎΠ΄ΡΡΠ²Π°, ΠΏΠ΅ΡΠ΅Π΄Π°ΡΠΈ ΠΈ ΡΠ°ΡΠΏΡΠ΅Π΄Π΅Π»Π΅Π½ΠΈΡ ΡΠ»Π΅ΠΊΡΡΠΎΡΠ½Π΅ΡΠ³ΠΈΠΈ ΠΏΠΎ ΠΎΡΠ΄Π΅Π»ΡΠ½ΠΎΡΡΠΈ.
ΠΠ½Π΄ΡΠΊΡΠΈΡ β ΡΡΠΎ ΠΏΡΠΎΡΠ΅ΡΡ, ΠΏΡΠΈ ΠΊΠΎΡΠΎΡΠΎΠΌ ΠΠΠ‘ ΠΈΠ½Π΄ΡΡΠΈΡΡΠ΅ΡΡΡ ΠΈΠ·ΠΌΠ΅Π½Π΅Π½ΠΈΠ΅ΠΌ ΠΌΠ°Π³Π½ΠΈΡΠ½ΠΎΠ³ΠΎ ΠΏΠΎΡΠΎΠΊΠ°. Π’ΡΠ°Π½ΡΡΠΎΡΠΌΠ°ΡΠΎΡΡ, Π½Π°ΠΏΡΠΈΠΌΠ΅Ρ, ΡΠΏΡΠΎΠ΅ΠΊΡΠΈΡΠΎΠ²Π°Π½Ρ ΡΠ°ΠΊ, ΡΡΠΎΠ±Ρ Π±ΡΡΡ ΠΎΡΠΎΠ±Π΅Π½Π½ΠΎ ΡΡΡΠ΅ΠΊΡΠΈΠ²Π½ΡΠΌΠΈ Π΄Π»Ρ Π½Π°Π²Π΅Π΄Π΅Π½ΠΈΡ ΠΆΠ΅Π»Π°Π΅ΠΌΠΎΠ³ΠΎ Π½Π°ΠΏΡΡΠΆΠ΅Π½ΠΈΡ ΠΈ ΡΠΎΠΊΠ° Ρ ΠΎΡΠ΅Π½Ρ Π½Π΅Π±ΠΎΠ»ΡΡΠΎΠΉ ΠΏΠΎΡΠ΅ΡΠ΅ΠΉ ΡΠ½Π΅ΡΠ³ΠΈΠΈ Π² Π΄ΡΡΠ³ΠΈΠ΅ ΡΠΎΡΠΌΡ (ΡΠΌ. Π½Π°Ρ Atom Π² Β«Π’ΡΠ°Π½ΡΡΠΎΡΠΌΠ°ΡΠΎΡΠ°Ρ Β»). Π‘ΡΡΠ΅ΡΡΠ²ΡΠ΅Ρ Π»ΠΈ ΠΏΠΎΠ»Π΅Π·Π½Π°Ρ ΡΠΈΠ·ΠΈΡΠ΅ΡΠΊΠ°Ρ Π²Π΅Π»ΠΈΡΠΈΠ½Π°, ΡΠ²ΡΠ·Π°Π½Π½Π°Ρ Ρ ΡΠ΅ΠΌ, Π½Π°ΡΠΊΠΎΠ»ΡΠΊΠΎ Β«ΡΡΡΠ΅ΠΊΡΠΈΠ²Π΅Π½Β» ΡΡΠ°Π½ΡΡΠΎΡΠΌΠ°ΡΠΎΡ? Π΄Π°Π½Π½ΠΎΠ΅ ΡΡΡΡΠΎΠΉΡΡΠ²ΠΎ Π΅ΡΡΡ? ΠΡΠ²Π΅Ρ ΠΏΠΎΠ»ΠΎΠΆΠΈΡΠ΅Π»ΡΠ½ΡΠΉ, ΠΈ ΡΡΠ° ΡΠΈΠ·ΠΈΡΠ΅ΡΠΊΠ°Ρ Π²Π΅Π»ΠΈΡΠΈΠ½Π° Π½Π°Π·ΡΠ²Π°Π΅ΡΡΡ ΠΈΠ½Π΄ΡΠΊΡΠΈΠ²Π½ΠΎΡΡΡΡ.
ΠΠ·Π°ΠΈΠΌΠ½Π°Ρ ΠΈΠ½Π΄ΡΠΊΡΠΈΠ²Π½ΠΎΡΡΡ
ΠΠ·Π°ΠΈΠΌΠ½Π°Ρ ΠΈΠ½Π΄ΡΠΊΡΠΈΠ²Π½ΠΎΡΡΡ β ΡΡΠΎ Π΄Π΅ΠΉΡΡΠ²ΠΈΠ΅ Π·Π°ΠΊΠΎΠ½Π° ΠΈΠ½Π΄ΡΠΊΡΠΈΠΈ Π€Π°ΡΠ°Π΄Π΅Ρ Π΄Π»Ρ ΠΎΠ΄Π½ΠΎΠ³ΠΎ ΡΡΡΡΠΎΠΉΡΡΠ²Π° Π½Π° Π΄ΡΡΠ³ΠΎΠ΅, Π½Π°ΠΏΡΠΈΠΌΠ΅Ρ ΠΏΠ΅ΡΠ²ΠΈΡΠ½Π°Ρ ΠΊΠ°ΡΡΡΠΊΠ° ΠΏΡΠΈ ΠΏΠ΅ΡΠ΅Π΄Π°ΡΠ΅ ΡΠ½Π΅ΡΠ³ΠΈΠΈ Π²ΡΠΎΡΠΈΡΠ½ΠΎΠΉ Π² ΡΡΠ°Π½ΡΡΠΎΡΠΌΠ°ΡΠΎΡΠ΅. ΠΠΎΡΠΌΠΎΡΡΠΈΡΠ΅, Π³Π΄Π΅ ΠΏΡΠΎΡΡΡΠ΅ ΠΊΠ°ΡΡΡΠΊΠΈ Π½Π°Π²ΠΎΠ΄ΡΡ Π΄ΡΡΠ³ Π² Π΄ΡΡΠ³Π΅ ΠΠΠ‘.
ΠΠ·Π°ΠΈΠΌΠ½Π°Ρ ΠΈΠ½Π΄ΡΠΊΡΠΈΠ²Π½ΠΎΡΡΡ Π² ΠΊΠ°ΡΡΡΠΊΠ°Ρ : ΠΡΠΈ ΠΊΠ°ΡΡΡΠΊΠΈ ΠΌΠΎΠ³ΡΡ ΠΈΠ½Π΄ΡΡΠΈΡΠΎΠ²Π°ΡΡ ΠΠΠ‘ Π΄ΡΡΠ³ Π² Π΄ΡΡΠ³Π΅, ΠΊΠ°ΠΊ Π½Π΅ΡΡΡΠ΅ΠΊΡΠΈΠ²Π½ΡΠΉ ΡΡΠ°Π½ΡΡΠΎΡΠΌΠ°ΡΠΎΡ. ΠΡ Π²Π·Π°ΠΈΠΌΠ½Π°Ρ ΠΈΠ½Π΄ΡΠΊΡΠΈΠ²Π½ΠΎΡΡΡ Π ΡΠΊΠ°Π·ΡΠ²Π°Π΅Ρ Π½Π° ΡΡΡΠ΅ΠΊΡΠΈΠ²Π½ΠΎΡΡΡ ΡΠ²ΡΠ·ΠΈ ΠΌΠ΅ΠΆΠ΄Ρ Π½ΠΈΠΌΠΈ. ΠΠ΄Π΅ΡΡ Π²ΠΈΠ΄Π½ΠΎ, ΡΡΠΎ ΠΈΠ·ΠΌΠ΅Π½Π΅Π½ΠΈΠ΅ ΡΠΎΠΊΠ° Π² ΠΊΠ°ΡΡΡΠΊΠ΅ 1 ΠΈΠ½Π΄ΡΡΠΈΡΡΠ΅Ρ ΠΠΠ‘ Π² ΠΊΠ°ΡΡΡΠΊΠ΅ 2. (ΠΠ±ΡΠ°ΡΠΈΡΠ΅ Π²Π½ΠΈΠΌΠ°Π½ΠΈΠ΅, ΡΡΠΎ Β«E2 ΠΈΠ½Π΄ΡΡΠΈΡΡΠ΅ΡΡΡΒ» ΠΏΡΠ΅Π΄ΡΡΠ°Π²Π»ΡΠ΅Ρ ΠΠΠ‘ ΠΈΠ½Π΄ΡΠΊΡΠΈΠΈ Π² ΠΊΠ°ΡΡΡΠΊΠ΅ 2.)
ΠΠΎ ΠΌΠ½ΠΎΠ³ΠΈΡ ΡΠ»ΡΡΠ°ΡΡ , ΠΊΠΎΠ³Π΄Π° Π³Π΅ΠΎΠΌΠ΅ΡΡΠΈΡ ΡΡΡΡΠΎΠΉΡΡΠ² ΡΠΈΠΊΡΠΈΡΠΎΠ²Π°Π½Π°, ΠΏΠΎΡΠΎΠΊ ΠΈΠ·ΠΌΠ΅Π½ΡΠ΅ΡΡΡ Π·Π° ΡΡΠ΅Ρ ΠΈΠ·ΠΌΠ΅Π½Π΅Π½ΠΈΡ ΡΠΎΠΊΠ°. ΠΠΎΡΡΠΎΠΌΡ ΠΌΡ ΡΠΎΡΡΠ΅Π΄ΠΎΡΠΎΡΠΈΠΌΡΡ Π½Π° ΡΠΊΠΎΡΠΎΡΡΠΈ ΠΈΠ·ΠΌΠ΅Π½Π΅Π½ΠΈΡ ΡΠΎΠΊΠ° ΞI/Ξt ΠΊΠ°ΠΊ Π½Π° ΠΏΡΠΈΡΠΈΠ½Π΅ ΠΈΠ½Π΄ΡΠΊΡΠΈΠΈ. ΠΠ·ΠΌΠ΅Π½Π΅Π½ΠΈΠ΅ ΡΠΎΠΊΠ° I 1 Π² ΠΎΠ΄Π½ΠΎΠΌ ΡΡΡΡΠΎΠΉΡΡΠ²Π΅, ΠΊΠ°ΡΡΡΠΊΠ° 1, ΠΈΠ½Π΄ΡΡΠΈΡΡΠ΅Ρ ΠΠΠ‘ 2 Π² Π΄ΡΡΠ³ΠΎΠΌ. ΠΡ Π²ΡΡΠ°ΠΆΠ°Π΅ΠΌ ΡΡΠΎ Π² Π²ΠΈΠ΄Π΅ ΡΡΠ°Π²Π½Π΅Π½ΠΈΡ ΠΊΠ°ΠΊ
[Π»Π°ΡΠ΅ΠΊΡ]\ΡΠ΅ΠΊΡΡ{ΠΠΠ‘}_2 = -\ΡΠ΅ΠΊΡΡ{Π} \frac{\Delta \text{I}_1}{\Delta \text{t}}[/latex] ,
, Π³Π΄Π΅ M ΠΎΠΏΡΠ΅Π΄Π΅Π»ΡΠ΅ΡΡΡ ΠΊΠ°ΠΊ Π²Π·Π°ΠΈΠΌΠ½Π°Ρ ΠΈΠ½Π΄ΡΠΊΡΠΈΠ²Π½ΠΎΡΡΡ ΠΌΠ΅ΠΆΠ΄Ρ Π΄Π²ΡΠΌΡ ΡΡΡΡΠΎΠΉΡΡΠ²Π°ΠΌΠΈ. ΠΠ½Π°ΠΊ ΠΌΠΈΠ½ΡΡ ΡΠ²Π»ΡΠ΅ΡΡΡ Π²ΡΡΠ°ΠΆΠ΅Π½ΠΈΠ΅ΠΌ Π·Π°ΠΊΠΎΠ½Π° ΠΠ΅Π½ΡΠ°. Π§Π΅ΠΌ Π±ΠΎΠ»ΡΡΠ΅ Π²Π·Π°ΠΈΠΌΠ½Π°Ρ ΠΈΠ½Π΄ΡΠΊΡΠΈΠ²Π½ΠΎΡΡΡ Π, ΡΠ΅ΠΌ ΡΡΡΠ΅ΠΊΡΠΈΠ²Π½Π΅Π΅ ΡΠ²ΡΠ·Ρ.
ΠΡΠΈΡΠΎΠ΄Π° Π·Π΄Π΅ΡΡ ΡΠΈΠΌΠΌΠ΅ΡΡΠΈΡΠ½Π°. ΠΡΠ»ΠΈ ΠΌΡ ΠΈΠ·ΠΌΠ΅Π½ΠΈΠΌ ΡΠΎΠΊ I2 Π² ΠΊΠ°ΡΡΡΠΊΠ΅ 2, ΠΌΡ ΠΈΠ½Π΄ΡΡΠΈΡΡΠ΅ΠΌ ΠΠΠ‘1 Π² ΠΊΠ°ΡΡΡΠΊΠ΅ 1, ΠΊΠΎΡΠΎΡΠ°Ρ ΠΎΠΏΡΠ΅Π΄Π΅Π»ΡΠ΅ΡΡΡ ΠΊΠ°ΠΊ
[Π»Π°ΡΠ΅ΠΊΡ]\text{ΠΠΠ‘}_1 = -\text{M} \frac{\Delta \text{I} _2}{\Delta \text{t}}[/latex],
, Π³Π΄Π΅ M ΡΠ°ΠΊΠΎΠ΅ ΠΆΠ΅, ΠΊΠ°ΠΊ ΠΈ Π΄Π»Ρ ΠΎΠ±ΡΠ°ΡΠ½ΠΎΠ³ΠΎ ΠΏΡΠΎΡΠ΅ΡΡΠ°. Π’ΡΠ°Π½ΡΡΠΎΡΠΌΠ°ΡΠΎΡΡ ΡΠ°Π±ΠΎΡΠ°ΡΡ Π² ΠΎΠ±ΡΠ°ΡΠ½ΠΎΠΌ Π½Π°ΠΏΡΠ°Π²Π»Π΅Π½ΠΈΠΈ Ρ ΡΠΎΠΉ ΠΆΠ΅ ΡΡΡΠ΅ΠΊΡΠΈΠ²Π½ΠΎΡΡΡΡ, ΠΈΠ»ΠΈ Π²Π·Π°ΠΈΠΌΠ½ΠΎΠΉ ΠΈΠ½Π΄ΡΠΊΡΠΈΠ²Π½ΠΎΡΡΡΡ Π.
Π‘Π°ΠΌΠΎΠΈΠ½Π΄ΡΠΊΡΠΈΡ
Π‘Π°ΠΌΠΎΠΈΠ½Π΄ΡΠΊΡΠΈΡ, Π΄Π΅ΠΉΡΡΠ²ΠΈΠ΅ Π·Π°ΠΊΠΎΠ½Π° Π€Π°ΡΠ°Π΄Π΅Ρ ΠΈΠ½Π΄ΡΠΊΡΠΈΠΈ ΡΡΡΡΠΎΠΉΡΡΠ²Π° Π½Π° ΡΠ΅Π±Ρ, ΡΠ°ΠΊΠΆΠ΅ ΡΡΡΠ΅ΡΡΠ²ΡΠ΅Ρ. ΠΠΎΠ³Π΄Π°, Π½Π°ΠΏΡΠΈΠΌΠ΅Ρ, ΡΠΎΠΊ ΡΠ΅ΡΠ΅Π· ΠΊΠ°ΡΡΡΠΊΡ ΡΠ²Π΅Π»ΠΈΡΠΈΠ²Π°Π΅ΡΡΡ, ΠΌΠ°Π³Π½ΠΈΡΠ½ΠΎΠ΅ ΠΏΠΎΠ»Π΅ ΠΈ ΠΏΠΎΡΠΎΠΊ ΡΠ°ΠΊΠΆΠ΅ ΡΠ²Π΅Π»ΠΈΡΠΈΠ²Π°ΡΡΡΡ, ΠΈΠ½Π΄ΡΡΠΈΡΡΡ ΠΏΡΠΎΡΠΈΠ²ΠΎ-ΠΠΠ‘, ΠΊΠ°ΠΊ ΡΠΎΠ³ΠΎ ΡΡΠ΅Π±ΡΠ΅Ρ Π·Π°ΠΊΠΎΠ½ ΠΠ΅Π½ΡΠ°. Π Π½Π°ΠΎΠ±ΠΎΡΠΎΡ, Π΅ΡΠ»ΠΈ ΡΠΎΠΊ ΡΠΌΠ΅Π½ΡΡΠ°Π΅ΡΡΡ, ΠΈΠ½Π΄ΡΡΠΈΡΡΠ΅ΡΡΡ ΠΠΠ‘, ΠΏΡΠ΅ΠΏΡΡΡΡΠ²ΡΡΡΠ°Ρ ΡΠΌΠ΅Π½ΡΡΠ΅Π½ΠΈΡ. ΠΠΎΠ»ΡΡΠΈΠ½ΡΡΠ²ΠΎ ΡΡΡΡΠΎΠΉΡΡΠ² ΠΈΠΌΠ΅ΡΡ ΡΠΈΠΊΡΠΈΡΠΎΠ²Π°Π½Π½ΡΡ Π³Π΅ΠΎΠΌΠ΅ΡΡΠΈΡ, ΠΏΠΎΡΡΠΎΠΌΡ ΠΈΠ·ΠΌΠ΅Π½Π΅Π½ΠΈΠ΅ ΠΏΠΎΡΠΎΠΊΠ° ΠΏΠΎΠ»Π½ΠΎΡΡΡΡ ΡΠ²ΡΠ·Π°Π½ΠΎ Ρ ΠΈΠ·ΠΌΠ΅Π½Π΅Π½ΠΈΠ΅ΠΌ ΡΠΎΠΊΠ° ΞI ΡΠ΅ΡΠ΅Π· ΡΡΡΡΠΎΠΉΡΡΠ²ΠΎ. ΠΠΠ‘ ΠΈΠ½Π΄ΡΠΊΡΠΈΠΈ ΡΠ²ΡΠ·Π°Π½Π° Ρ ΡΠΈΠ·ΠΈΡΠ΅ΡΠΊΠΎΠΉ Π³Π΅ΠΎΠΌΠ΅ΡΡΠΈΠ΅ΠΉ ΡΡΡΡΠΎΠΉΡΡΠ²Π° ΠΈ ΡΠΊΠΎΡΠΎΡΡΡΡ ΠΈΠ·ΠΌΠ΅Π½Π΅Π½ΠΈΡ ΡΠΎΠΊΠ°. ΠΡΠΎ Π΄Π°Π΅ΡΡΡ
[Π»Π°ΡΠ΅ΠΊΡ]\ΡΠ΅ΠΊΡΡ{ΠΠΠ‘} = -\ΡΠ΅ΠΊΡΡ{L} \frac{\Delta \text{I}}{\Delta \text{t}}[/latex],
, Π³Π΄Π΅ L — ΡΠ°ΠΌΠΎ- ΠΈΠ½Π΄ΡΠΊΡΠΈΠ²Π½ΠΎΡΡΡ ΡΡΡΡΠΎΠΉΡΡΠ²Π°. Π£ΡΡΡΠΎΠΉΡΡΠ²ΠΎ, ΠΎΠ±Π»Π°Π΄Π°ΡΡΠ΅Π΅ Π·Π½Π°ΡΠΈΡΠ΅Π»ΡΠ½ΠΎΠΉ ΡΠΎΠ±ΡΡΠ²Π΅Π½Π½ΠΎΠΉ ΠΈΠ½Π΄ΡΠΊΡΠΈΠ²Π½ΠΎΡΡΡΡ, Π½Π°Π·ΡΠ²Π°Π΅ΡΡΡ ΠΈΠ½Π΄ΡΠΊΡΠΎΡΠΎΠΌ. ΠΠΏΡΡΡ ΠΆΠ΅, Π·Π½Π°ΠΊ ΠΌΠΈΠ½ΡΡ ΡΠ²Π»ΡΠ΅ΡΡΡ Π²ΡΡΠ°ΠΆΠ΅Π½ΠΈΠ΅ΠΌ Π·Π°ΠΊΠΎΠ½Π° ΠΠ΅Π½ΡΠ°, ΡΠΊΠ°Π·ΡΠ²Π°ΡΡΠΈΠΌ, ΡΡΠΎ ΠΠΠ‘ ΠΏΡΠΎΡΠΈΠ²ΠΎΠ΄Π΅ΠΉΡΡΠ²ΡΠ΅Ρ ΠΈΠ·ΠΌΠ΅Π½Π΅Π½ΠΈΡ ΡΠΎΠΊΠ°.
A ΠΠΎΠ»ΠΈΡΠ΅ΡΡΠ²Π΅Π½Π½Π°Ρ ΠΈΠ½ΡΠ΅ΡΠΏΡΠ΅ΡΠ°ΡΠΈΡ ΠΠΠ‘ Π΄Π²ΠΈΠΆΠ΅Π½ΠΈΡ
A ΠΠΠ‘ Π΄Π²ΠΈΠΆΠ΅Π½ΠΈΡ β ΡΡΠΎ ΡΠ»Π΅ΠΊΡΡΠΎΠ΄Π²ΠΈΠΆΡΡΠ°Ρ ΡΠΈΠ»Π° (ΠΠΠ‘), ΠΈΠ½Π΄ΡΡΠΈΡΠΎΠ²Π°Π½Π½Π°Ρ Π΄Π²ΠΈΠΆΠ΅Π½ΠΈΠ΅ΠΌ ΠΎΡΠ½ΠΎΡΠΈΡΠ΅Π»ΡΠ½ΠΎ ΠΌΠ°Π³Π½ΠΈΡΠ½ΠΎΠ³ΠΎ ΠΏΠΎΠ»Ρ B.
Π¦Π΅Π»ΠΈ ΠΎΠ±ΡΡΠ΅Π½ΠΈΡ
Π‘ΡΠΎΡΠΌΡΠ»ΠΈΡΠΎΠ²Π°ΡΡ Π΄Π²Π΅ ΡΠΎΡΠΊΠΈ Π·ΡΠ΅Π½ΠΈΡ, ΠΊΠΎΡΠΎΡΡΠ΅ ΠΏΡΠΈΠΌΠ΅Π½ΡΡΡΡΡ Π΄Π»Ρ ΡΠ°ΡΡΠ΅ΡΠ° ΡΠ»Π΅ΠΊΡΡΠΎΠ΄Π²ΠΈΠΆΡΡΠ΅ΠΉ ΡΠΈΠ»Ρ
ΠΠ»ΡΡΠ΅Π²ΡΠ΅ Π²ΡΠ²ΠΎΠ΄Ρ
ΠΠ»ΡΡΠ΅Π²ΡΠ΅ ΠΌΠΎΠΌΠ΅Π½ΡΡ
- ΠΠΠ‘ Π΄Π²ΠΈΠΆΠ΅Π½ΠΈΡ ΠΈ ΠΈΠ½Π΄ΡΠΊΡΠΈΠΈ β ΡΡΠΎ ΠΎΠ΄Π½ΠΎ ΠΈ ΡΠΎ ΠΆΠ΅ ΡΠ²Π»Π΅Π½ΠΈΠ΅, ΠΏΡΠΎΡΡΠΎ Π½Π°Π±Π»ΡΠ΄Π°Π΅ΠΌΠΎΠ΅ Π² ΡΠ°Π·Π½ΡΡ ΡΠΈΡΡΠ΅ΠΌΠ°Ρ ΠΎΡΡΡΠ΅ΡΠ°. ΠΠΊΠ²ΠΈΠ²Π°Π»Π΅Π½ΡΠ½ΠΎΡΡΡ ΡΡΠΈΡ Π΄Π²ΡΡ ΡΠ²Π»Π΅Π½ΠΈΠΉ ΠΏΠΎΠ±ΡΠ΄ΠΈΠ»Π° ΠΠΉΠ½ΡΡΠ΅ΠΉΠ½Π° Π·Π°Π½ΡΡΡΡΡ ΡΠΏΠ΅ΡΠΈΠ°Π»ΡΠ½ΠΎΠΉ ΡΠ΅ΠΎΡΠΈΠ΅ΠΉ ΠΎΡΠ½ΠΎΡΠΈΡΠ΅Π»ΡΠ½ΠΎΡΡΠΈ.
- ΠΠΠ‘, Π²ΠΎΠ·Π½ΠΈΠΊΠ°ΡΡΠ°Ρ ΠΈΠ·-Π·Π° ΠΎΡΠ½ΠΎΡΠΈΡΠ΅Π»ΡΠ½ΠΎΠ³ΠΎ Π΄Π²ΠΈΠΆΠ΅Π½ΠΈΡ ΠΏΠ΅ΡΠ»ΠΈ ΠΈ ΠΌΠ°Π³Π½ΠΈΡΠ°, ΠΎΠΏΡΠ΅Π΄Π΅Π»ΡΠ΅ΡΡΡ ΠΊΠ°ΠΊ [Π»Π°ΡΠ΅ΠΊΡ]\varepsilon_{\text{Π΄Π²ΠΈΠΆΠ΅Π½ΠΈΠ΅}} = \text{vB} \times \text{L}[/latex] (ΡΡΠ°Π²Π½Π΅Π½ΠΈΠ΅ 1 ), Π³Π΄Π΅ L β Π΄Π»ΠΈΠ½Π° ΠΎΠ±ΡΠ΅ΠΊΡΠ°, Π΄Π²ΠΈΠΆΡΡΠ΅Π³ΠΎΡΡ ΡΠΎ ΡΠΊΠΎΡΠΎΡΡΡΡ v ΠΎΡΠ½ΠΎΡΠΈΡΠ΅Π»ΡΠ½ΠΎ ΠΌΠ°Π³Π½ΠΈΡΠ°.
- ΠΠΠ‘ ΠΌΠΎΠΆΠ½ΠΎ ΡΠ°ΡΡΡΠΈΡΠ°ΡΡ Ρ Π΄Π²ΡΡ ΡΠ°Π·Π½ΡΡ ΡΠΎΡΠ΅ΠΊ Π·ΡΠ΅Π½ΠΈΡ: 1) ΡΠ΅ΡΠ΅Π· ΠΌΠ°Π³Π½ΠΈΡΠ½ΡΡ ΡΠΈΠ»Ρ, Π΄Π΅ΠΉΡΡΠ²ΡΡΡΡΡ Π½Π° Π΄Π²ΠΈΠΆΡΡΠΈΠ΅ΡΡ ΡΠ»Π΅ΠΊΡΡΠΎΠ½Ρ Π² ΠΌΠ°Π³Π½ΠΈΡΠ½ΠΎΠΌ ΠΏΠΎΠ»Π΅, ΠΈ 2) ΡΠ΅ΡΠ΅Π· ΡΠΊΠΎΡΠΎΡΡΡ ΠΈΠ·ΠΌΠ΅Π½Π΅Π½ΠΈΡ ΠΌΠ°Π³Π½ΠΈΡΠ½ΠΎΠ³ΠΎ ΠΏΠΎΡΠΎΠΊΠ°. ΠΠ±Π° Π΄Π°ΡΡ ΠΎΠ΄ΠΈΠ½Π°ΠΊΠΎΠ²ΡΠΉ ΡΠ΅Π·ΡΠ»ΡΡΠ°Ρ.
ΠΠ»ΡΡΠ΅Π²ΡΠ΅ ΡΠ΅ΡΠΌΠΈΠ½Ρ
- ΡΠΏΠ΅ΡΠΈΠ°Π»ΡΠ½Π°Ρ ΡΠ΅ΠΎΡΠΈΡ ΠΎΡΠ½ΠΎΡΠΈΡΠ΅Π»ΡΠ½ΠΎΡΡΠΈ : Π’Π΅ΠΎΡΠΈΡ, ΠΊΠΎΡΠΎΡΠ°Ρ (ΠΏΡΠ΅Π½Π΅Π±ΡΠ΅Π³Π°Ρ Π²Π»ΠΈΡΠ½ΠΈΠ΅ΠΌ Π³ΡΠ°Π²ΠΈΡΠ°ΡΠΈΠΈ) ΡΠΎΠ³Π»Π°ΡΠΎΠ²ΡΠ²Π°Π΅Ρ ΠΏΡΠΈΠ½ΡΠΈΠΏ ΠΎΡΠ½ΠΎΡΠΈΡΠ΅Π»ΡΠ½ΠΎΡΡΠΈ Ρ Π½Π°Π±Π»ΡΠ΄Π΅Π½ΠΈΠ΅ΠΌ, ΡΡΠΎ ΡΠΊΠΎΡΠΎΡΡΡ ΡΠ²Π΅ΡΠ° ΠΏΠΎΡΡΠΎΡΠ½Π½Π° Π²ΠΎ Π²ΡΠ΅Ρ ΡΠΈΡΡΠ΅ΠΌΠ°Ρ ΠΎΡΡΡΠ΅ΡΠ°.
- ΠΌΠ°Π³Π½ΠΈΡΠ½ΠΎΠ΅ ΠΏΠΎΠ»Π΅ : Π‘ΠΎΡΡΠΎΡΠ½ΠΈΠ΅ Π² ΠΏΡΠΎΡΡΡΠ°Π½ΡΡΠ²Π΅ Π²ΠΎΠΊΡΡΠ³ ΠΌΠ°Π³Π½ΠΈΡΠ° ΠΈΠ»ΠΈ ΡΠ»Π΅ΠΊΡΡΠΈΡΠ΅ΡΠΊΠΎΠ³ΠΎ ΡΠΎΠΊΠ°, Π² ΠΊΠΎΡΠΎΡΠΎΠΌ ΠΏΡΠΈΡΡΡΡΡΠ²ΡΠ΅Ρ ΠΎΠΏΡΠ΅Π΄Π΅Π»ΡΠ΅ΠΌΠ°Ρ ΠΌΠ°Π³Π½ΠΈΡΠ½Π°Ρ ΡΠΈΠ»Π° ΠΈ Π³Π΄Π΅ ΠΏΡΠΈΡΡΡΡΡΠ²ΡΡΡ Π΄Π²Π° ΠΌΠ°Π³Π½ΠΈΡΠ½ΡΡ ΠΏΠΎΠ»ΡΡΠ°.
- ΡΠΈΡΡΠ΅ΠΌΠ° ΠΎΡΡΡΠ΅ΡΠ° : Π‘ΠΈΡΡΠ΅ΠΌΠ° ΠΊΠΎΠΎΡΠ΄ΠΈΠ½Π°Ρ ΠΈΠ»ΠΈ Π½Π°Π±ΠΎΡ ΠΎΡΠ΅ΠΉ, Π² ΠΊΠΎΡΠΎΡΡΡ ΠΈΠ·ΠΌΠ΅ΡΡΠ΅ΡΡΡ ΠΏΠΎΠ»ΠΎΠΆΠ΅Π½ΠΈΠ΅, ΠΎΡΠΈΠ΅Π½ΡΠ°ΡΠΈΡ ΠΈ Π΄ΡΡΠ³ΠΈΠ΅ ΡΠ²ΠΎΠΉΡΡΠ²Π° ΠΎΠ±ΡΠ΅ΠΊΡΠΎΠ² Π² Π½Π΅ΠΉ.
ΠΠ»Π΅ΠΊΡΡΠΎΠ΄Π²ΠΈΠΆΡΡΠ°Ρ ΡΠΈΠ»Π° (ΠΠΠ‘), Π²ΡΠ·Π²Π°Π½Π½Π°Ρ Π΄Π²ΠΈΠΆΠ΅Π½ΠΈΠ΅ΠΌ ΠΎΡΠ½ΠΎΡΠΈΡΠ΅Π»ΡΠ½ΠΎ ΠΌΠ°Π³Π½ΠΈΡΠ½ΠΎΠ³ΠΎ ΠΏΠΎΠ»Ρ B, Π½Π°Π·ΡΠ²Π°Π΅ΡΡΡ ΠΠΠ‘ Π΄Π²ΠΈΠΆΠ΅Π½ΠΈΡ. ΠΡ ΠΌΠΎΠ³Π»ΠΈ Π·Π°ΠΌΠ΅ΡΠΈΡΡ, ΡΡΠΎ ΠΠΠ‘ Π΄Π²ΠΈΠΆΠ΅Π½ΠΈΡ ΠΎΡΠ΅Π½Ρ ΠΏΠΎΡ ΠΎΠΆΠ° Π½Π° ΠΈΠ½Π΄ΡΡΠΈΡΠΎΠ²Π°Π½Π½ΡΡ ΠΠΠ‘, Π²ΡΠ·Π²Π°Π½Π½ΡΡ ΠΈΠ·ΠΌΠ΅Π½ΡΡΡΠΈΠΌΡΡ ΠΌΠ°Π³Π½ΠΈΡΠ½ΡΠΌ ΠΏΠΎΠ»Π΅ΠΌ. Π ΡΡΠΎΠΌ ΠΡΠΎΠΌΠ΅ ΠΌΡ Π²ΠΈΠ΄ΠΈΠΌ, ΡΡΠΎ ΡΡΠΎ Π΄Π΅ΠΉΡΡΠ²ΠΈΡΠ΅Π»ΡΠ½ΠΎ ΠΎΠ΄Π½ΠΎ ΠΈ ΡΠΎ ΠΆΠ΅ ΡΠ²Π»Π΅Π½ΠΈΠ΅, ΠΏΠΎΠΊΠ°Π·Π°Π½Π½ΠΎΠ΅ Π² ΡΠ°Π·Π½ΡΡ ΡΠΈΡΡΠ΅ΠΌΠ°Ρ ΠΎΡΡΡΠ΅ΡΠ°.
ΠΠΠ‘ Π΄Π²ΠΈΠΆΠ΅Π½ΠΈΡ
Π ΡΠ»ΡΡΠ°Π΅, ΠΊΠΎΠ³Π΄Π° ΠΏΠ΅ΡΠ»Ρ ΠΏΡΠΎΠ²ΠΎΠ΄Π½ΠΈΠΊΠ° Π΄Π²ΠΈΠΆΠ΅ΡΡΡ Π² ΠΌΠ°Π³Π½ΠΈΡ, ΠΏΠΎΠΊΠ°Π·Π°Π½Π½ΡΠΉ Π½Π° (Π°), ΠΌΠ°Π³Π½ΠΈΡΠ½Π°Ρ ΡΠΈΠ»Π°, Π΄Π΅ΠΉΡΡΠ²ΡΡΡΠ°Ρ Π½Π° Π΄Π²ΠΈΠΆΡΡΠΈΠΉΡΡ Π·Π°ΡΡΠ΄ Π² ΠΏΠ΅ΡΠ»Π΅, ΠΎΠΏΡΠ΅Π΄Π΅Π»ΡΠ΅ΡΡΡ Π²ΡΡΠ°ΠΆΠ΅Π½ΠΈΠ΅ΠΌ [Π»Π°ΡΠ΅ΠΊΡ]evB[/Π»Π°ΡΠ΅ΠΊΡ] (ΡΠΈΠ»Π° ΠΠΎΡΠ΅Π½ΡΠ°, Π΅: ΡΠ»Π΅ΠΊΡΡΠΎΠ½ ΠΎΠ±Π²ΠΈΠ½Π΅Π½ΠΈΠ΅).
ΠΠ΅ΡΠ»Ρ ΠΏΡΠΎΠ²ΠΎΠ΄Π½ΠΈΠΊΠ°, Π΄Π²ΠΈΠΆΡΡΠ°ΡΡΡ Π² ΠΌΠ°Π³Π½ΠΈΡΠ΅ : (a) ΠΠΠ‘ Π΄Π²ΠΈΠΆΠ΅Π½ΠΈΡ. ΠΠ΅ΡΠ»Ρ Ρ ΡΠΎΠΊΠΎΠΌ Π΄Π²ΠΈΠΆΠ΅ΡΡΡ Π² Π½Π΅ΠΏΠΎΠ΄Π²ΠΈΠΆΠ½ΡΠΉ ΠΌΠ°Π³Π½ΠΈΡ. ΠΠ°ΠΏΡΠ°Π²Π»Π΅Π½ΠΈΠ΅ ΠΌΠ°Π³Π½ΠΈΡΠ½ΠΎΠ³ΠΎ ΠΏΠΎΠ»Ρ Π½Π° ΡΠΊΡΠ°Π½. (Π±) ΠΠ°Π²Π΅Π΄Π΅Π½Π½Π°Ρ ΠΠΠ‘. Π’ΠΎΠΊΠΎΠ²Π°Ρ ΠΏΠ΅ΡΠ»Ρ Π½Π΅ΠΏΠΎΠ΄Π²ΠΈΠΆΠ½Π°, Π° ΠΌΠ°Π³Π½ΠΈΡ Π΄Π²ΠΈΠΆΠ΅ΡΡΡ.
ΠΠΎΠ΄ Π΄Π΅ΠΉΡΡΠ²ΠΈΠ΅ΠΌ ΡΠΈΠ»Ρ ΡΠ»Π΅ΠΊΡΡΠΎΠ½Ρ Π±ΡΠ΄ΡΡ Π½Π°ΠΊΠ°ΠΏΠ»ΠΈΠ²Π°ΡΡΡΡ Π½Π° ΠΎΠ΄Π½ΠΎΠΉ ΡΡΠΎΡΠΎΠ½Π΅ (Π½ΠΈΠΆΠ½ΠΈΠΉ ΠΊΠΎΠ½Π΅Ρ ΡΠΈΡΡΠ½ΠΊΠ°) Π΄ΠΎ ΡΠ΅Ρ ΠΏΠΎΡ, ΠΏΠΎΠΊΠ° Π½Π° ΡΡΠ΅ΡΠΆΠ½Π΅ Π½Π΅ ΡΡΡΠ°Π½ΠΎΠ²ΠΈΡΡΡ Π΄ΠΎΡΡΠ°ΡΠΎΡΠ½ΠΎΠ΅ ΡΠ»Π΅ΠΊΡΡΠΈΡΠ΅ΡΠΊΠΎΠ΅ ΠΏΠΎΠ»Π΅, ΠΏΡΠΎΡΠΈΠ²ΠΎΠ΄Π΅ΠΉΡΡΠ²ΡΡΡΠ΅Π΅ Π΄Π²ΠΈΠΆΠ΅Π½ΠΈΡ ΡΠ»Π΅ΠΊΡΡΠΎΠ½ΠΎΠ², ΡΠΎ Π΅ΡΡΡ [Π»Π°ΡΠ΅ΠΊΡ]\ΡΠ΅ΠΊΡΡ{eE}[ /Π»Π°ΡΠ΅ΠΊΡ]. ΠΡΠΈΡΠ°Π²Π½ΠΈΠ²Π°Ρ Π΄Π²Π΅ ΡΠΈΠ»Ρ, ΠΏΠΎΠ»ΡΡΠ°Π΅ΠΌ [Π»Π°ΡΠ΅ΠΊΡ]\ΡΠ΅ΠΊΡΡ{Π} = \ΡΠ΅ΠΊΡΡ{vB}[/Π»Π°ΡΠ΅ΠΊΡ].
Π‘Π»Π΅Π΄ΠΎΠ²Π°ΡΠ΅Π»ΡΠ½ΠΎ, ΠΠΠ‘ Π΄Π²ΠΈΠΆΠ΅Π½ΠΈΡ ΠΏΠΎ Π΄Π»ΠΈΠ½Π΅ L ΡΡΠΎΡΠΎΠ½Ρ ΠΏΠ΅ΡΠ»ΠΈ ΠΎΠΏΡΠ΅Π΄Π΅Π»ΡΠ΅ΡΡΡ ΠΊΠ°ΠΊ [Π»Π°ΡΠ΅ΠΊΡ]\varepsilon_{\text{motion}} = \text{vB} \times \text{L}[/latex] ( Π£ΡΠ°Π²Π½Π΅Π½ΠΈΠ΅ 1), Π³Π΄Π΅ L β Π΄Π»ΠΈΠ½Π° ΠΎΠ±ΡΠ΅ΠΊΡΠ°, Π΄Π²ΠΈΠΆΡΡΠ΅Π³ΠΎΡΡ ΡΠΎ ΡΠΊΠΎΡΠΎΡΡΡΡ v ΠΎΡΠ½ΠΎΡΠΈΡΠ΅Π»ΡΠ½ΠΎ ΠΌΠ°Π³Π½ΠΈΡΠ°.
ΠΠΠ‘ ΠΈΠ½Π΄ΡΠΊΡΠΈΠΈ
ΠΠΎΡΠΊΠΎΠ»ΡΠΊΡ ΡΠΊΠΎΡΠΎΡΡΡ ΠΈΠ·ΠΌΠ΅Π½Π΅Π½ΠΈΡ ΠΌΠ°Π³Π½ΠΈΡΠ½ΠΎΠ³ΠΎ ΠΏΠΎΡΠΎΠΊΠ°, ΠΏΡΠΎΡ ΠΎΠ΄ΡΡΠ΅Π³ΠΎ ΡΠ΅ΡΠ΅Π· ΠΏΠ΅ΡΠ»Ρ, ΡΠ°Π²Π½Π° [Π»Π°ΡΠ΅ΠΊΡ]\ΡΠ΅ΠΊΡΡ{Π}\frac{\text{dA}}{\text{dt}}[/Π»Π°ΡΠ΅ΠΊΡ](Π : ΠΏΠ»ΠΎΡΠ°Π΄Ρ ΠΏΠ΅ΡΠ»ΠΈ, ΡΠ΅ΡΠ΅Π· ΠΊΠΎΡΠΎΡΡΡ ΠΏΡΠΎΡ ΠΎΠ΄ΠΈΡ ΠΌΠ°Π³Π½ΠΈΡΠ½ΠΎΠ΅ ΠΏΠΎΠ»Π΅), ΠΠΠ‘ ΠΈΠ½Π΄ΡΠΊΡΠΈΠΈ [Π»Π°ΡΠ΅ΠΊΡ]\varepsilon_{\text{ΠΈΠ½Π΄ΡΡΠΈΡΡΠ΅ΠΌΠ°Ρ}} = \text{BLv}[/Π»Π°ΡΠ΅ΠΊΡ] (ΡΡΠ°Π²Π½Π΅Π½ΠΈΠ΅ 2).
ΠΠΊΠ²ΠΈΠ²Π°Π»Π΅Π½ΡΠ½ΠΎΡΡΡ ΠΠΠ‘ Π΄Π²ΠΈΠΆΠ΅Π½ΠΈΡ ΠΈ ΠΈΠ½Π΄ΡΠΊΡΠΈΠΈ
ΠΠ· ΡΡΠ°Π²Π½Π΅Π½ΠΈΡ. 1 ΠΈ ΡΡΠ°Π²Π½Π΅Π½ΠΈΠ΅ 2 ΠΌΡ ΠΌΠΎΠΆΠ΅ΠΌ ΠΏΠΎΠ΄ΡΠ²Π΅ΡΠ΄ΠΈΡΡ, ΡΡΠΎ ΠΠΠ‘ Π΄Π²ΠΈΠΆΠ΅Π½ΠΈΡ ΠΈ Π½Π°Π²Π΅Π΄Π΅Π½ΠΈΡ Π΄Π°ΡΡ ΠΎΠ΄ΠΈΠ½Π°ΠΊΠΎΠ²ΡΠΉ ΡΠ΅Π·ΡΠ»ΡΡΠ°Ρ. ΠΠ° ΡΠ°ΠΌΠΎΠΌ Π΄Π΅Π»Π΅ ΡΠΊΠ²ΠΈΠ²Π°Π»Π΅Π½ΡΠ½ΠΎΡΡΡ ΡΡΠΈΡ Π΄Π²ΡΡ ΡΠ²Π»Π΅Π½ΠΈΠΉ ΠΈ ΠΏΠΎΠ±ΡΠ΄ΠΈΠ»Π° ΠΠ»ΡΠ±Π΅ΡΡΠ° ΠΠΉΠ½ΡΡΠ΅ΠΉΠ½Π° Π·Π°Π½ΡΡΡΡΡ ΡΠΏΠ΅ΡΠΈΠ°Π»ΡΠ½ΠΎΠΉ ΡΠ΅ΠΎΡΠΈΠ΅ΠΉ ΠΎΡΠ½ΠΎΡΠΈΡΠ΅Π»ΡΠ½ΠΎΡΡΠΈ. Π ΡΠ²ΠΎΠ΅ΠΉ ΠΎΡΠ½ΠΎΠ²ΠΎΠΏΠΎΠ»Π°Π³Π°ΡΡΠ΅ΠΉ ΡΡΠ°ΡΡΠ΅ ΠΏΠΎ ΡΠΏΠ΅ΡΠΈΠ°Π»ΡΠ½ΠΎΠΉ ΡΠ΅ΠΎΡΠΈΠΈ ΠΎΡΠ½ΠΎΡΠΈΡΠ΅Π»ΡΠ½ΠΎΡΡΠΈ, ΠΎΠΏΡΠ±Π»ΠΈΠΊΠΎΠ²Π°Π½Π½ΠΎΠΉ Π² 1905 Π³ΠΎΠ΄Ρ, ΠΠΉΠ½ΡΡΠ΅ΠΉΠ½ Π½Π°ΡΠΈΠ½Π°Π΅Ρ Ρ ΡΠΏΠΎΠΌΠΈΠ½Π°Π½ΠΈΡ ΠΎΠ± ΡΠΊΠ²ΠΈΠ²Π°Π»Π΅Π½ΡΠ½ΠΎΡΡΠΈ Π΄Π²ΡΡ ΡΠ²Π»Π΅Π½ΠΈΠΉ:
Β«β¦β¦ Π½Π°ΠΏΡΠΈΠΌΠ΅Ρ, Π²Π·Π°ΠΈΠΌΠ½ΠΎΠ΅ ΡΠ»Π΅ΠΊΡΡΠΎΠ΄ΠΈΠ½Π°ΠΌΠΈΡΠ΅ΡΠΊΠΎΠ΅ Π΄Π΅ΠΉΡΡΠ²ΠΈΠ΅ ΠΌΠ°Π³Π½ΠΈΡΠ° ΠΈ ΠΏΡΠΎΠ²ΠΎΠ΄Π½ΠΈΠΊΠ°. ΠΠ°Π±Π»ΡΠ΄Π°Π΅ΠΌΠΎΠ΅ Π·Π΄Π΅ΡΡ ΡΠ²Π»Π΅Π½ΠΈΠ΅ Π·Π°Π²ΠΈΡΠΈΡ ΡΠΎΠ»ΡΠΊΠΎ ΠΎΡ ΠΎΡΠ½ΠΎΡΠΈΡΠ΅Π»ΡΠ½ΠΎΠ³ΠΎ Π΄Π²ΠΈΠΆΠ΅Π½ΠΈΡ ΠΏΡΠΎΠ²ΠΎΠ΄Π½ΠΈΠΊΠ° ΠΈ ΠΌΠ°Π³Π½ΠΈΡΠ°, ΡΠΎΠ³Π΄Π° ΠΊΠ°ΠΊ ΠΎΠ±ΡΡΠ½Π°Ρ ΡΠΎΡΠΊΠ° Π·ΡΠ΅Π½ΠΈΡ ΠΏΡΠΎΠ²ΠΎΠ΄ΠΈΡ ΡΠ΅Π·ΠΊΠΎΠ΅ ΡΠ°Π·Π»ΠΈΡΠΈΠ΅ ΠΌΠ΅ΠΆΠ΄Ρ Π΄Π²ΡΠΌΡ ΡΠ»ΡΡΠ°ΡΠΌΠΈ, Π² ΠΊΠΎΡΠΎΡΡΡ Π΄Π²ΠΈΠΆΠ΅ΡΡΡ ΡΠΎ ΠΎΠ΄Π½ΠΎ, ΡΠΎ Π΄ΡΡΠ³ΠΎΠ΅ ΠΈΠ· ΡΡΠΈΡ ΡΠ΅Π». Π ΡΠ°ΠΌΠΎΠΌ Π΄Π΅Π»Π΅, Π΅ΡΠ»ΠΈ ΠΌΠ°Π³Π½ΠΈΡ Π½Π°Ρ ΠΎΠ΄ΠΈΡΡΡ Π² Π΄Π²ΠΈΠΆΠ΅Π½ΠΈΠΈ, Π° ΠΏΡΠΎΠ²ΠΎΠ΄Π½ΠΈΠΊ ΠΏΠΎΠΊΠΎΠΈΡΡΡ, ΡΠΎ Π²Π±Π»ΠΈΠ·ΠΈ ΠΌΠ°Π³Π½ΠΈΡΠ° Π²ΠΎΠ·Π½ΠΈΠΊΠ°Π΅Ρ ΡΠ»Π΅ΠΊΡΡΠΈΡΠ΅ΡΠΊΠΎΠ΅ ΠΏΠΎΠ»Π΅ Ρ Π½Π΅ΠΊΠΎΡΠΎΡΡΠΌ ΠΎΠΏΡΠ΅Π΄Π΅Π»Π΅Π½Π½ΡΠΌ ΡΠ½Π΅ΡΠ³ΠΈΡ , ΠΏΡΠΎΠΈΠ·Π²ΠΎΠ΄ΡΡΠ°Ρ ΡΠΎΠΊ Π² ΠΌΠ΅ΡΡΠ°Ρ ΡΠ°ΡΠΏΠΎΠ»ΠΎΠΆΠ΅Π½ΠΈΡ ΡΠ°ΡΡΠ΅ΠΉ ΠΏΡΠΎΠ²ΠΎΠ΄Π½ΠΈΠΊΠ°. ΠΠΎ Π΅ΡΠ»ΠΈ ΠΌΠ°Π³Π½ΠΈΡ Π½Π΅ΠΏΠΎΠ΄Π²ΠΈΠΆΠ΅Π½, Π° ΠΏΡΠΎΠ²ΠΎΠ΄Π½ΠΈΠΊ Π΄Π²ΠΈΠΆΠ΅ΡΡΡ, ΡΠΎ Π²Π±Π»ΠΈΠ·ΠΈ ΠΌΠ°Π³Π½ΠΈΡΠ° Π½Π΅ Π²ΠΎΠ·Π½ΠΈΠΊΠ°Π΅Ρ Π½ΠΈΠΊΠ°ΠΊΠΎΠ³ΠΎ ΡΠ»Π΅ΠΊΡΡΠΈΡΠ΅ΡΠΊΠΎΠ³ΠΎ ΠΏΠΎΠ»Ρ. Π ΠΏΡΠΎΠ²ΠΎΠ΄Π½ΠΈΠΊΠ΅, ΠΎΠ΄Π½Π°ΠΊΠΎ, ΠΌΡ Π½Π°Ρ ΠΎΠ΄ΠΈΠΌ ΡΠ»Π΅ΠΊΡΡΠΎΠ΄Π²ΠΈΠΆΡΡΡΡ ΡΠΈΠ»Ρ, ΠΊΠΎΡΠΎΡΠΎΠΉ ΡΠ°ΠΌΠΎΠΉ ΠΏΠΎ ΡΠ΅Π±Π΅ Π½Π΅Ρ ΡΠΎΠΎΡΠ²Π΅ΡΡΡΠ²ΡΡΡΠ΅ΠΉ ΡΠ½Π΅ΡΠ³ΠΈΠΈ, Π½ΠΎ ΠΊΠΎΡΠΎΡΠ°Ρ ΠΏΠΎΡΠΎΠΆΠ΄Π°Π΅Ρ β ΠΏΡΠΈ ΡΡΠ»ΠΎΠ²ΠΈΠΈ ΡΠ°Π²Π΅Π½ΡΡΠ²Π° ΠΎΡΠ½ΠΎΡΠΈΡΠ΅Π»ΡΠ½ΠΎΠ³ΠΎ Π΄Π²ΠΈΠΆΠ΅Π½ΠΈΡ Π² Π΄Π²ΡΡ ΡΠ°ΡΡΠΌΠΎΡΡΠ΅Π½Π½ΡΡ ΡΠ»ΡΡΠ°ΡΡ β ΡΠ»Π΅ΠΊΡΡΠΈΡΠ΅ΡΠΊΠΈΠ΅ ΡΠΎΠΊΠΈ ΡΠΎΠ³ΠΎ ΠΆΠ΅ ΠΏΡΡΠΈ ΠΈ ΡΠΈΠ»Ρ, ΡΡΠΎ ΠΈ ΠΏΡΠΎΠΈΠ·Π²ΠΎΠ΄ΠΈΠΌΡΠ΅ ΡΠ»Π΅ΠΊΡΡΠΈΡΠ΅ΡΠΊΠΈΠΌΠΈ ΡΠΈΠ»Π°ΠΌΠΈ Π² ΠΏΠ΅ΡΠ²ΠΎΠΌ ΡΠ»ΡΡΠ°Π΅.
ΠΠ΅Ρ Π°Π½ΠΈΡΠ΅ΡΠΊΠ°Ρ ΡΠ°Π±ΠΎΡΠ° ΠΈ ΡΠ»Π΅ΠΊΡΡΠΈΡΠ΅ΡΠΊΠ°Ρ ΡΠ½Π΅ΡΠ³ΠΈΡ
ΠΠ΅Ρ Π°Π½ΠΈΡΠ΅ΡΠΊΠ°Ρ ΡΠ°Π±ΠΎΡΠ°, ΡΠΎΠ²Π΅ΡΡΠ°Π΅ΠΌΠ°Ρ Π²Π½Π΅ΡΠ½Π΅ΠΉ ΡΠΈΠ»ΠΎΠΉ Π΄Π»Ρ ΡΠΎΠ·Π΄Π°Π½ΠΈΡ ΠΠΠ‘ Π΄Π²ΠΈΠΆΠ΅Π½ΠΈΡ, ΠΏΡΠ΅ΠΎΠ±ΡΠ°Π·ΡΠ΅ΡΡΡ Π² ΡΠ΅ΠΏΠ»ΠΎΠ²ΡΡ ΡΠ½Π΅ΡΠ³ΠΈΡ; Π² ΠΏΡΠΎΡΠ΅ΡΡΠ΅ ΡΠΎΡ ΡΠ°Π½ΡΠ΅ΡΡΡ ΡΠ½Π΅ΡΠ³ΠΈΡ.
Π¦Π΅Π»ΠΈ ΠΎΠ±ΡΡΠ΅Π½ΠΈΡ
ΠΡΠΈΠΌΠ΅Π½ΠΈΡΡ Π·Π°ΠΊΠΎΠ½ ΡΠΎΡ ΡΠ°Π½Π΅Π½ΠΈΡ ΡΠ½Π΅ΡΠ³ΠΈΠΈ Π΄Π»Ρ ΠΎΠΏΠΈΡΠ°Π½ΠΈΡ ΠΏΡΠΎΠΈΠ·Π²ΠΎΠ΄ΡΡΠ²Π° Π΄Π²ΠΈΠΆΡΡΠ΅ΠΉ ΡΠ»Π΅ΠΊΡΡΠΎΠ΄Π²ΠΈΠΆΡΡΠ΅ΠΉ ΡΠΈΠ»Ρ Ρ ΠΌΠ΅Ρ Π°Π½ΠΈΡΠ΅ΡΠΊΠΎΠΉ ΡΠ°Π±ΠΎΡΠΎΠΉ Π»Π°ΡΠ΅ΠΊΡ]\varepsilon = \text{Blv}[/Π»Π°ΡΠ΅ΠΊΡ].
ΠΠ»ΡΡΠ΅Π²ΡΠ΅ ΡΠ΅ΡΠΌΠΈΠ½Ρ
- ΠΠΠ‘ Π΄Π²ΠΈΠΆΠ΅Π½ΠΈΡ : ΠΠΠ‘ (ΡΠ»Π΅ΠΊΡΡΠΎΠ΄Π²ΠΈΠΆΡΡΠ°Ρ ΡΠΈΠ»Π°), ΠΈΠ½Π΄ΡΡΠΈΡΠΎΠ²Π°Π½Π½Π°Ρ Π΄Π²ΠΈΠΆΠ΅Π½ΠΈΠ΅ΠΌ ΠΎΡΠ½ΠΎΡΠΈΡΠ΅Π»ΡΠ½ΠΎ ΠΌΠ°Π³Π½ΠΈΡΠ½ΠΎΠ³ΠΎ ΠΏΠΎΠ»Ρ.
- ΠΠ°ΠΊΠΎΠ½ ΠΈΠ½Π΄ΡΠΊΡΠΈΠΈ Π€Π°ΡΠ°Π΄Π΅Ρ : ΠΡΠ½ΠΎΠ²Π½ΠΎΠΉ Π·Π°ΠΊΠΎΠ½ ΡΠ»Π΅ΠΊΡΡΠΎΠΌΠ°Π³Π½Π΅ΡΠΈΠ·ΠΌΠ°, ΠΊΠΎΡΠΎΡΡΠΉ ΠΏΡΠ΅Π΄ΡΠΊΠ°Π·ΡΠ²Π°Π΅Ρ, ΠΊΠ°ΠΊ ΠΌΠ°Π³Π½ΠΈΡΠ½ΠΎΠ΅ ΠΏΠΎΠ»Π΅ Π±ΡΠ΄Π΅Ρ Π²Π·Π°ΠΈΠΌΠΎΠ΄Π΅ΠΉΡΡΠ²ΠΎΠ²Π°ΡΡ Ρ ΡΠ»Π΅ΠΊΡΡΠΈΡΠ΅ΡΠΊΠΎΠΉ ΡΠ΅ΠΏΡΡ, ΡΠΎΠ·Π΄Π°Π²Π°Ρ ΡΠ»Π΅ΠΊΡΡΠΎΠ΄Π²ΠΈΠΆΡΡΡΡ ΡΠΈΠ»Ρ (ΠΠΠ‘).
Π Π°Π½Π΅Π΅ ΠΌΡ ΡΠ·Π½Π°Π»ΠΈ ΠΎΠ± ΠΠΠ‘ Π΄Π²ΠΈΠΆΠ΅Π½ΠΈΡ (ΡΠΌ. Π½Π°Ρ ΠΡΠΎΠΌ Π² ΡΠ°Π·Π΄Π΅Π»Π΅ Β«ΠΠΠ‘ Π΄Π²ΠΈΠΆΠ΅Π½ΠΈΡΒ»). ΠΠ»Ρ ΠΏΡΠΎΡΡΠΎΠΉ ΡΡΡΠ°Π½ΠΎΠ²ΠΊΠΈ, ΠΏΠΎΠΊΠ°Π·Π°Π½Π½ΠΎΠΉ Π½ΠΈΠΆΠ΅, ΠΠΠ‘ Π΄Π²ΠΈΠΆΠ΅Π½ΠΈΡ [Π»Π°ΡΠ΅ΠΊΡ](\varepsilon)[/latex], ΡΠΎΠ·Π΄Π°Π²Π°Π΅ΠΌΠ°Ρ Π΄Π²ΠΈΠΆΡΡΠΈΠΌΡΡ ΠΏΡΠΎΠ²ΠΎΠ΄Π½ΠΈΠΊΠΎΠΌ (Π² ΠΎΠ΄Π½ΠΎΡΠΎΠ΄Π½ΠΎΠΌ ΠΏΠΎΠ»Π΅), ΠΎΠΏΡΠ΅Π΄Π΅Π»ΡΠ΅ΡΡΡ ΡΠ»Π΅Π΄ΡΡΡΠΈΠΌ ΠΎΠ±ΡΠ°Π·ΠΎΠΌ:
[Π»Π°ΡΠ΅ΠΊΡ]\varepsilon = \text{Blv}[ /Π»Π°ΡΠ΅ΠΊΡ]
, Π³Π΄Π΅ B β ΠΌΠ°Π³Π½ΠΈΡΠ½ΠΎΠ΅ ΠΏΠΎΠ»Π΅, l β Π΄Π»ΠΈΠ½Π° ΠΏΡΠΎΠ²ΠΎΠ΄ΡΡΠ΅Π³ΠΎ ΡΡΠ΅ΡΠΆΠ½Ρ, Π° v β (ΠΏΠΎΡΡΠΎΡΠ½Π½Π°Ρ) ΡΠΊΠΎΡΠΎΡΡΡ Π΅Π³ΠΎ Π΄Π²ΠΈΠΆΠ΅Π½ΠΈΡ. ( Π , Π» ΠΈ v Π²ΡΠ΅ ΠΏΠ΅ΡΠΏΠ΅Π½Π΄ΠΈΠΊΡΠ»ΡΡΠ½Ρ Π΄ΡΡΠ³ Π΄ΡΡΠ³Ρ, ΠΊΠ°ΠΊ ΠΏΠΎΠΊΠ°Π·Π°Π½ΠΎ Π½Π° ΠΈΠ·ΠΎΠ±ΡΠ°ΠΆΠ΅Π½ΠΈΠΈ Π½ΠΈΠΆΠ΅.)
ΠΠΠ‘ Π΄Π²ΠΈΠΆΠ΅Π½ΠΈΡ : (a) ΠΠΠ‘ Π΄Π²ΠΈΠΆΠ΅Π½ΠΈΡ = Bβv ΠΈΠ½Π΄ΡΡΠΈΡΡΠ΅ΡΡΡ ΠΌΠ΅ΠΆΠ΄Ρ ΡΠ΅Π»ΡΡΠ°ΠΌΠΈ, ΠΊΠΎΠ³Π΄Π° ΡΡΠΎΡ ΡΡΠ΅ΡΠΆΠ΅Π½Ρ Π΄Π²ΠΈΠΆΠ΅ΡΡΡ Π²ΠΏΡΠ°Π²ΠΎ Π² ΠΎΠ΄Π½ΠΎΡΠΎΠ΄Π½ΠΎΠΌ ΠΌΠ°Π³Π½ΠΈΡΠ½ΠΎΠΌ ΠΏΠΎΠ»Π΅. ΠΠ°Π³Π½ΠΈΡΠ½ΠΎΠ΅ ΠΏΠΎΠ»Π΅ B Π½Π°ΠΏΡΠ°Π²Π»Π΅Π½ΠΎ Π²Π½ΡΡΡΡ ΡΡΡΠ°Π½ΠΈΡΡ, ΠΏΠ΅ΡΠΏΠ΅Π½Π΄ΠΈΠΊΡΠ»ΡΡΠ½ΠΎ Π΄Π²ΠΈΠΆΡΡΠΈΠΌΡΡ ΡΡΠ΅ΡΠΆΠ½Ρ ΠΈ ΡΠ΅Π»ΡΡΠ°ΠΌ ΠΈ, ΡΠ»Π΅Π΄ΠΎΠ²Π°ΡΠ΅Π»ΡΠ½ΠΎ, ΠΎΠ³ΡΠ°Π½ΠΈΡΠ΅Π½Π½ΠΎΠΉ ΠΈΠΌΠΈ ΠΎΠ±Π»Π°ΡΡΠΈ. (b) ΠΠ°ΠΊΠΎΠ½ ΠΠ΅Π½ΡΠ° Π΄Π°Π΅Ρ Π½Π°ΠΏΡΠ°Π²Π»Π΅Π½ΠΈΡ ΠΈΠ½Π΄ΡΡΠΈΡΠΎΠ²Π°Π½Π½ΠΎΠ³ΠΎ ΠΏΠΎΠ»Ρ ΠΈ ΡΠΎΠΊΠ°, Π° ΡΠ°ΠΊΠΆΠ΅ ΠΏΠΎΠ»ΡΡΠ½ΠΎΡΡΡ ΠΈΠ½Π΄ΡΡΠΈΡΠΎΠ²Π°Π½Π½ΠΎΠΉ ΠΠΠ‘. ΠΠΎΡΠΊΠΎΠ»ΡΠΊΡ ΠΏΠΎΡΠΎΠΊ ΡΠ²Π΅Π»ΠΈΡΠΈΠ²Π°Π΅ΡΡΡ, ΠΈΠ½Π΄ΡΡΠΈΡΠΎΠ²Π°Π½Π½ΠΎΠ΅ ΠΏΠΎΠ»Π΅ Π½Π°ΠΏΡΠ°Π²Π»Π΅Π½ΠΎ Π² ΠΏΡΠΎΡΠΈΠ²ΠΎΠΏΠΎΠ»ΠΎΠΆΠ½ΠΎΠΌ Π½Π°ΠΏΡΠ°Π²Π»Π΅Π½ΠΈΠΈ ΠΈΠ»ΠΈ Π²ΡΡ ΠΎΠ΄ΠΈΡ Π·Π° ΠΏΡΠ΅Π΄Π΅Π»Ρ ΡΡΡΠ°Π½ΠΈΡΡ. ΠΡΠ°Π²ΠΈΠ»ΠΎ ΠΏΡΠ°Π²ΠΎΠΉ ΡΡΠΊΠΈ Π΄Π°Π΅Ρ ΠΏΠΎΠΊΠ°Π·Π°Π½Π½ΠΎΠ΅ Π½Π°ΠΏΡΠ°Π²Π»Π΅Π½ΠΈΠ΅ ΡΠΎΠΊΠ°, ΠΈ ΠΏΠΎΠ»ΡΡΠ½ΠΎΡΡΡ ΡΡΠ΅ΡΠΆΠ½Ρ Π±ΡΠ΄Π΅Ρ ΡΠΏΡΠ°Π²Π»ΡΡΡ ΡΠ°ΠΊΠΈΠΌ ΡΠΎΠΊΠΎΠΌ.
Π‘ΠΎΡ ΡΠ°Π½Π΅Π½ΠΈΠ΅ ΡΠ½Π΅ΡΠ³ΠΈΠΈ
Π ΡΡΠΎΠΌ Π°ΡΠΎΠΌΠ΅ ΠΌΡ ΡΠ°ΡΡΠΌΠΎΡΡΠΈΠΌ ΡΠΈΡΡΠ΅ΠΌΡ Ρ ΡΠΎΡΠΊΠΈ Π·ΡΠ΅Π½ΠΈΡ ΡΠ½Π΅ΡΠ³ΠΈΠΈ . ΠΡΠΈ Π΄Π²ΠΈΠΆΠ΅Π½ΠΈΠΈ ΡΡΠ΅ΡΠΆΠ½Ρ Ρ ΡΠΎΠΊΠΎΠΌ i Π½Π° Π½Π΅Π³ΠΎ Π΄Π΅ΠΉΡΡΠ²ΡΠ΅Ρ ΡΠΈΠ»Π° ΠΠΎΡΠ΅Π½ΡΠ°
[Π»Π°ΡΠ΅ΠΊΡ]\ΡΠ΅ΠΊΡΡ{F}_\ΡΠ΅ΠΊΡΡ{L} = \text{iBL}[/Π»Π°ΡΠ΅ΠΊΡ].
Π§ΡΠΎΠ±Ρ ΡΡΠ΅ΡΠΆΠ΅Π½Ρ Π΄Π²ΠΈΠ³Π°Π»ΡΡ Ρ ΠΏΠΎΡΡΠΎΡΠ½Π½ΠΎΠΉ ΡΠΊΠΎΡΠΎΡΡΡΡ v , ΠΊ ΡΡΠ΅ΡΠΆΠ½Ρ Π²Π΄ΠΎΠ»Ρ Π΅Π³ΠΎ Π΄Π²ΠΈΠΆΠ΅Π½ΠΈΠ΅. Π’Π°ΠΊ ΠΊΠ°ΠΊ ΡΡΠ΅ΡΠΆΠ΅Π½Ρ Π΄Π²ΠΈΠΆΠ΅ΡΡΡ Π² v , ΠΌΠΎΡΠ½ΠΎΡΡΡ P , ΠΏΠ΅ΡΠ΅Π΄Π°Π½Π½Π°Ρ Π²Π½Π΅ΡΠ½Π΅ΠΉ ΡΠΈΠ»ΠΎΠΉ, Π±ΡΠ΄Π΅Ρ:
[Π»Π°ΡΠ΅ΠΊΡ]\text{P} = \text{F}_{\text{ext}} \text{v} = (\text {iBL})\times \text{v} = \text{i} \varepsilon[/latex].
ΠΠ° ΠΏΠΎΡΠ»Π΅Π΄Π½Π΅ΠΌ ΡΠ°Π³Π΅ ΠΌΡ ΠΈΡΠΏΠΎΠ»ΡΠ·ΠΎΠ²Π°Π»ΠΈ ΠΏΠ΅ΡΠ²ΠΎΠ΅ ΡΡΠ°Π²Π½Π΅Π½ΠΈΠ΅, ΠΎ ΠΊΠΎΡΠΎΡΠΎΠΌ Π³ΠΎΠ²ΠΎΡΠΈΠ»ΠΈ. ΠΠ±ΡΠ°ΡΠΈΡΠ΅ Π²Π½ΠΈΠΌΠ°Π½ΠΈΠ΅, ΡΡΠΎ ΡΡΠΎ ΠΈΠΌΠ΅Π½Π½ΠΎ ΠΌΠΎΡΠ½ΠΎΡΡΡ, ΡΠ°ΡΡΠ΅ΠΈΠ²Π°Π΅ΠΌΠ°Ρ Π² ΠΏΠ΅ΡΠ»Π΅ (= ΡΠΎΠΊ [Π»Π°ΡΠ΅ΠΊΡ]\ΡΠΌΠ½ΠΎΠΆΠΈΡΡ [/Π»Π°ΡΠ΅ΠΊΡ] Π½Π°ΠΏΡΡΠΆΠ΅Π½ΠΈΠ΅). Π’Π°ΠΊΠΈΠΌ ΠΎΠ±ΡΠ°Π·ΠΎΠΌ, ΠΌΡ Π·Π°ΠΊΠ»ΡΡΠ°Π΅ΠΌ, ΡΡΠΎ ΠΌΠ΅Ρ Π°Π½ΠΈΡΠ΅ΡΠΊΠ°Ρ ΡΠ°Π±ΠΎΡΠ°, ΡΠΎΠ²Π΅ΡΡΠ°Π΅ΠΌΠ°Ρ Π²Π½Π΅ΡΠ½Π΅ΠΉ ΡΠΈΠ»ΠΎΠΉ Π΄Π»Ρ ΠΏΠΎΠ΄Π΄Π΅ΡΠΆΠ°Π½ΠΈΡ Π΄Π²ΠΈΠΆΠ΅Π½ΠΈΡ ΡΡΠ΅ΡΠΆΠ½Ρ Ρ ΠΏΠΎΡΡΠΎΡΠ½Π½ΠΎΠΉ ΡΠΊΠΎΡΠΎΡΡΡΡ, ΠΏΡΠ΅ΠΎΠ±ΡΠ°Π·ΡΠ΅ΡΡΡ Π² ΡΠ΅ΠΏΠ»ΠΎΠ²ΡΡ ΡΠ½Π΅ΡΠ³ΠΈΡ Π² ΠΏΠ΅ΡΠ»Π΅. Π Π±ΠΎΠ»Π΅Π΅ ΠΎΠ±ΡΠ΅ΠΌ ΡΠΌΡΡΠ»Π΅ ΠΌΠ΅Ρ Π°Π½ΠΈΡΠ΅ΡΠΊΠ°Ρ ΡΠ°Π±ΠΎΡΠ°, ΡΠΎΠ²Π΅ΡΡΠ°Π΅ΠΌΠ°Ρ Π²Π½Π΅ΡΠ½Π΅ΠΉ ΡΠΈΠ»ΠΎΠΉ Π΄Π»Ρ ΡΠΎΠ·Π΄Π°Π½ΠΈΡ ΠΠΠ‘ Π΄Π²ΠΈΠΆΠ΅Π½ΠΈΡ, ΠΏΡΠ΅ΠΎΠ±ΡΠ°Π·ΡΠ΅ΡΡΡ Π² ΡΠ΅ΠΏΠ»ΠΎΠ²ΡΡ ΡΠ½Π΅ΡΠ³ΠΈΡ. ΠΠ½Π΅ΡΠ³ΠΈΡ ΡΠΎΡ ΡΠ°Π½ΡΠ΅ΡΡΡ Π² ΠΏΡΠΎΡΠ΅ΡΡΠ΅.
ΠΠ°ΠΊΠΎΠ½ ΠΠ΅Π½ΡΠ°
Π Β«ΠΡΠΎΠΌΠ΅Β» Β«ΠΠ°ΠΊΠΎΠ½ ΠΈΠ½Π΄ΡΠΊΡΠΈΠΈ Π€Π°ΡΠ°Π΄Π΅Ρ ΠΈ Π·Π°ΠΊΠΎΠ½ ΠΠ΅Π½ΡΠ°Β» ΠΌΡ ΡΠ·Π½Π°Π»ΠΈ, ΡΡΠΎ Π·Π°ΠΊΠΎΠ½ ΠΠ΅Π½ΡΠ° ΡΠ²Π»ΡΠ΅ΡΡΡ ΠΏΡΠΎΡΠ²Π»Π΅Π½ΠΈΠ΅ΠΌ Π·Π°ΠΊΠΎΠ½Π° ΡΠΎΡ ΡΠ°Π½Π΅Π½ΠΈΡ ΡΠ½Π΅ΡΠ³ΠΈΠΈ. ΠΠ°ΠΊ ΠΌΡ Π²ΠΈΠ΄ΠΈΠΌ Π½Π° ΠΏΡΠΈΠΌΠ΅ΡΠ΅ Ρ ΡΡΠΈΠΌ Π°ΡΠΎΠΌΠΎΠΌ, Π·Π°ΠΊΠΎΠ½ ΠΠ΅Π½ΡΠ° Π³Π°ΡΠ°Π½ΡΠΈΡΡΠ΅Ρ, ΡΡΠΎ Π΄Π²ΠΈΠΆΠ΅Π½ΠΈΠ΅ ΡΡΠ΅ΡΠΆΠ½Ρ ΠΏΡΠΎΡΠΈΠ²ΠΎΠΏΠΎΠ»ΠΎΠΆΠ½ΠΎ ΠΈΠ·-Π·Π° ΡΠ΅Π½Π΄Π΅Π½ΡΠΈΠΈ ΠΏΡΠΈΡΠΎΠ΄Ρ ΡΠΎΠΏΡΠΎΡΠΈΠ²Π»ΡΡΡΡΡ ΠΈΠ·ΠΌΠ΅Π½Π΅Π½ΠΈΡ ΠΌΠ°Π³Π½ΠΈΡΠ½ΠΎΠ³ΠΎ ΠΏΠΎΠ»Ρ. ΠΡΠ»ΠΈ Π±Ρ ΠΈΠ½Π΄ΡΡΠΈΡΠΎΠ²Π°Π½Π½Π°Ρ ΠΠΠ‘ Π±ΡΠ»Π° Π½Π°ΠΏΡΠ°Π²Π»Π΅Π½Π° ββΠ² ΡΡ ΠΆΠ΅ ΡΡΠΎΡΠΎΠ½Ρ, ΡΡΠΎ ΠΈ ΠΈΠ·ΠΌΠ΅Π½Π΅Π½ΠΈΠ΅ ΠΏΠΎΡΠΎΠΊΠ°, ΡΠΎ ΡΡΡΠ΅ΡΡΠ²ΠΎΠ²Π°Π»Π° Π±Ρ ΠΏΠΎΠ»ΠΎΠΆΠΈΡΠ΅Π»ΡΠ½Π°Ρ ΠΎΠ±ΡΠ°ΡΠ½Π°Ρ ΡΠ²ΡΠ·Ρ, Π·Π°ΡΡΠ°Π²Π»ΡΡΡΠ°Ρ ΡΡΠ΅ΡΠΆΠ΅Π½Ρ ΡΠ»Π΅ΡΠ°ΡΡ ΠΎΡ ΠΌΠ°Π»Π΅ΠΉΡΠ΅Π³ΠΎ Π²ΠΎΠ·ΠΌΡΡΠ΅Π½ΠΈΡ.
ΠΠ½Π΅ΡΠ³ΠΈΡ Π² ΠΌΠ°Π³Π½ΠΈΡΠ½ΠΎΠΌ ΠΏΠΎΠ»Π΅
ΠΠ°Π³Π½ΠΈΡΠ½ΠΎΠ΅ ΠΏΠΎΠ»Π΅ Π½Π°ΠΊΠ°ΠΏΠ»ΠΈΠ²Π°Π΅Ρ ΡΠ½Π΅ΡΠ³ΠΈΡ. ΠΠ»ΠΎΡΠ½ΠΎΡΡΡ ΡΠ½Π΅ΡΠ³ΠΈΠΈ ΠΎΠΏΡΠ΅Π΄Π΅Π»ΡΠ΅ΡΡΡ ΠΊΠ°ΠΊ [Π»Π°ΡΠ΅ΠΊΡ]\ΡΠ΅ΠΊΡΡ{u} = \frac{\mathbf{\text{B}}\cdot\mathbf{\text{B}}}{2\mu}[/latex].
Π¦Π΅Π»ΠΈ ΠΎΠ±ΡΡΠ΅Π½ΠΈΡ
ΠΡΡΠ°Π·ΠΈΡΠ΅ ΠΏΠ»ΠΎΡΠ½ΠΎΡΡΡ ΡΠ½Π΅ΡΠ³ΠΈΠΈ ΠΌΠ°Π³Π½ΠΈΡΠ½ΠΎΠ³ΠΎ ΠΏΠΎΠ»Ρ Π² ΡΠΎΡΠΌΠ΅ ΡΡΠ°Π²Π½Π΅Π½ΠΈΡ
ΠΡΠ½ΠΎΠ²Π½ΡΠ΅ Π²ΡΠ²ΠΎΠ΄Ρ
ΠΠ»ΡΡΠ΅Π²ΡΠ΅ ΠΌΠΎΠΌΠ΅Π½ΡΡ
- ΠΠ½Π΅ΡΠ³ΠΈΡ Π½Π΅ΠΎΠ±Ρ ΠΎΠ΄ΠΈΠΌΠ° Π΄Π»Ρ ΡΠΎΠ·Π΄Π°Π½ΠΈΡ ΠΌΠ°Π³Π½ΠΈΡΠ½ΠΎΠ³ΠΎ ΠΏΠΎΠ»Ρ, ΠΊΠ°ΠΊ Π΄Π»Ρ ΡΠ°Π±ΠΎΡΡ ΠΏΡΠΎΡΠΈΠ² ΡΠ»Π΅ΠΊΡΡΠΈΡΠ΅ΡΠΊΠΎΠ³ΠΎ ΠΏΠΎΠ»Ρ, ΡΠ°ΠΊ ΠΈ Π΄Π»Ρ ΡΠ°Π±ΠΎΡΡ Ρ ΠΈΠ·ΠΌΠ΅Π½ΡΡΡΠΈΠΌΡΡ ΠΌΠ°Π³Π½ΠΈΡΠ½ΡΠΌ ΠΏΠΎΠ»Π΅ΠΌ. ΡΠΎΠ·Π΄Π°Π΅Ρ ΠΈ ΠΈΠ·ΠΌΠ΅Π½ΡΠ΅Ρ Π½Π°ΠΌΠ°Π³Π½ΠΈΡΠ΅Π½Π½ΠΎΡΡΡ Π»ΡΠ±ΠΎΠ³ΠΎ ΠΌΠ°ΡΠ΅ΡΠΈΠ°Π»Π° Π² ΠΏΡΠ΅Π΄Π΅Π»Π°Ρ ΠΌΠ°Π³Π½ΠΈΡΠ½ΠΎΠ³ΠΎ ΠΏΠΎΠ»Ρ. 92[/Π»Π°ΡΠ΅ΠΊΡ].
ΠΠ»ΡΡΠ΅Π²ΡΠ΅ ΡΠ΅ΡΠΌΠΈΠ½Ρ
- ΠΏΡΠΎΠ½ΠΈΡΠ°Π΅ΠΌΠΎΡΡΡ : ΠΠΎΠ»ΠΈΡΠ΅ΡΡΠ²Π΅Π½Π½Π°Ρ ΠΌΠ΅ΡΠ° ΡΡΠ΅ΠΏΠ΅Π½ΠΈ Π½Π°ΠΌΠ°Π³Π½ΠΈΡΠ΅Π½Π½ΠΎΡΡΠΈ ΠΌΠ°ΡΠ΅ΡΠΈΠ°Π»Π° Π² ΠΏΡΠΈΡΡΡΡΡΠ²ΠΈΠΈ ΠΏΡΠΈΠ»ΠΎΠΆΠ΅Π½Π½ΠΎΠ³ΠΎ ΠΌΠ°Π³Π½ΠΈΡΠ½ΠΎΠ³ΠΎ ΠΏΠΎΠ»Ρ (ΠΈΠ·ΠΌΠ΅ΡΡΠ΅ΡΡΡ Π² Π½ΡΡΡΠΎΠ½Π°Ρ Π½Π° Π°ΠΌΠΏΠ΅Ρ Π² ΠΊΠ²Π°Π΄ΡΠ°ΡΠ΅ Π² Π΅Π΄ΠΈΠ½ΠΈΡΠ°Ρ Π‘Π).
- ΠΈΠ½Π΄ΡΠΊΡΠΎΡ : ΠΏΠ°ΡΡΠΈΠ²Π½ΠΎΠ΅ ΡΡΡΡΠΎΠΉΡΡΠ²ΠΎ, Π²Π²ΠΎΠ΄ΡΡΠ΅Π΅ ΠΈΠ½Π΄ΡΠΊΡΠΈΠ²Π½ΠΎΡΡΡ Π² ΡΠ»Π΅ΠΊΡΡΠΈΡΠ΅ΡΠΊΡΡ ΡΠ΅ΠΏΡ.
- ΡΠ΅ΡΡΠΎΠΌΠ°Π³Π½Π΅ΡΠΈΠΊ : ΠΠ°ΡΠ΅ΡΠΈΠ°Π»Ρ, ΠΎΠ±Π»Π°Π΄Π°ΡΡΠΈΠ΅ ΠΏΠΎΡΡΠΎΡΠ½Π½ΡΠΌ ΠΌΠ°Π³Π½ΠΈΡΠ½ΡΠΌ ΡΠ²ΠΎΠΉΡΡΠ²ΠΎΠΌ.
ΠΠ½Π΅ΡΠ³ΠΈΡ Π½Π΅ΠΎΠ±Ρ ΠΎΠ΄ΠΈΠΌΠ° Π΄Π»Ρ ΡΠΎΠ·Π΄Π°Π½ΠΈΡ ΠΌΠ°Π³Π½ΠΈΡΠ½ΠΎΠ³ΠΎ ΠΏΠΎΠ»Ρ ΠΊΠ°ΠΊ Π΄Π»Ρ ΡΠ°Π±ΠΎΡΡ ΠΏΡΠΎΡΠΈΠ² ΡΠ»Π΅ΠΊΡΡΠΈΡΠ΅ΡΠΊΠΎΠ³ΠΎ ΠΏΠΎΠ»Ρ, ΡΠΎΠ·Π΄Π°Π²Π°Π΅ΠΌΠΎΠ³ΠΎ ΠΈΠ·ΠΌΠ΅Π½ΡΡΡΠΈΠΌΡΡ ΠΌΠ°Π³Π½ΠΈΡΠ½ΡΠΌ ΠΏΠΎΠ»Π΅ΠΌ, ΡΠ°ΠΊ ΠΈ Π΄Π»Ρ ΠΈΠ·ΠΌΠ΅Π½Π΅Π½ΠΈΡ Π½Π°ΠΌΠ°Π³Π½ΠΈΡΠ΅Π½Π½ΠΎΡΡΠΈ Π»ΡΠ±ΠΎΠ³ΠΎ ΠΌΠ°ΡΠ΅ΡΠΈΠ°Π»Π° Π² ΠΏΡΠ΅Π΄Π΅Π»Π°Ρ ΠΌΠ°Π³Π½ΠΈΡΠ½ΠΎΠ³ΠΎ ΠΏΠΎΠ»Ρ. ΠΠ»Ρ Π½Π΅Π΄ΠΈΡΠΏΠ΅ΡΡΠΈΠΎΠ½Π½ΡΡ ΠΌΠ°ΡΠ΅ΡΠΈΠ°Π»ΠΎΠ² ΡΡΠ° ΠΆΠ΅ ΡΠ½Π΅ΡΠ³ΠΈΡ Π²ΡΡΠ²ΠΎΠ±ΠΎΠΆΠ΄Π°Π΅ΡΡΡ ΠΏΡΠΈ ΡΠ°Π·ΡΡΡΠ΅Π½ΠΈΠΈ ΠΌΠ°Π³Π½ΠΈΡΠ½ΠΎΠ³ΠΎ ΠΏΠΎΠ»Ρ. Π‘Π»Π΅Π΄ΠΎΠ²Π°ΡΠ΅Π»ΡΠ½ΠΎ, ΡΡΡ ΡΠ½Π΅ΡΠ³ΠΈΡ ΠΌΠΎΠΆΠ½ΠΎ ΡΠΌΠΎΠ΄Π΅Π»ΠΈΡΠΎΠ²Π°ΡΡ ΠΊΠ°ΠΊ Β«Π·Π°ΠΏΠ°ΡΠ΅Π½Π½ΡΡΒ» Π² ΠΌΠ°Π³Π½ΠΈΡΠ½ΠΎΠΌ ΠΏΠΎΠ»Π΅.
ΠΠ°Π³Π½ΠΈΡΠ½ΠΎΠ΅ ΠΏΠΎΠ»Π΅, ΡΠΎΠ·Π΄Π°Π²Π°Π΅ΠΌΠΎΠ΅ ΡΠΎΠ»Π΅Π½ΠΎΠΈΠ΄ΠΎΠΌ : ΠΠ°Π³Π½ΠΈΡΠ½ΠΎΠ΅ ΠΏΠΎΠ»Π΅, ΡΠΎΠ·Π΄Π°Π²Π°Π΅ΠΌΠΎΠ΅ ΡΠΎΠ»Π΅Π½ΠΎΠΈΠ΄ΠΎΠΌ (Π²ΠΈΠ΄ Π² ΠΏΠΎΠΏΠ΅ΡΠ΅ΡΠ½ΠΎΠΌ ΡΠ΅ΡΠ΅Π½ΠΈΠΈ), ΠΎΠΏΠΈΡΡΠ²Π°Π΅ΠΌΠΎΠ΅ Π»ΠΈΠ½ΠΈΡΠΌΠΈ ΠΏΠΎΠ»Ρ. ΠΠ½Π΅ΡΠ³ΠΈΡ Β«Π·Π°ΠΏΠ°ΡΠ°Π΅ΡΡΡΒ» Π² ΠΌΠ°Π³Π½ΠΈΡΠ½ΠΎΠΌ ΠΏΠΎΠ»Π΅.
ΠΠ½Π΅ΡΠ³ΠΈΡ, Π½Π°ΠΊΠΎΠΏΠ»Π΅Π½Π½Π°Ρ Π² ΠΌΠ°Π³Π½ΠΈΡΠ½ΠΎΠΌ ΠΏΠΎΠ»Π΅
ΠΠ»Ρ Π»ΠΈΠ½Π΅ΠΉΠ½ΡΡ Π½Π΅Π΄ΠΈΡΠΏΠ΅ΡΡΠΈΠΎΠ½Π½ΡΡ ΠΌΠ°ΡΠ΅ΡΠΈΠ°Π»ΠΎΠ² (ΡΠ°ΠΊΠΈΡ , ΡΡΠΎ B = Β΅ H, Π³Π΄Π΅ Β΅ Π½Π°Π·ΡΠ²Π°Π΅ΡΡΡ ΠΌΠ°Π³Π½ΠΈΡΠ½ΠΎΠΉ ΠΏΡΠΎΠ½ΠΈΡΠ°Π΅ΠΌΠΎΡΡΡΡ, Π½Π΅ Π·Π°Π²ΠΈΡΠΈΡ ΠΎΡ ΡΠ°ΡΡΠΎΡΡ), ΠΏΠ»ΠΎΡΠ½ΠΎΡΡΡ ΡΠ½Π΅ΡΠ³ΠΈΠΈ ΡΠΎΡΡΠ°Π²Π»ΡΠ΅Ρ:
[ Π»Π°ΡΠ΅ΠΊΡ] \ ΡΠ΅ΠΊΡΡ {u} = \ frac {\ mathbf {\ text {B}} \ cdot \ mathbf {\ text {B}}} {2 \ mu} = \ frac {\ mu \ mathbf {\ text {H }}\cdot\mathbf{\text{H}}}{2}[/Π»Π°ΡΠ΅ΠΊΡ].
ΠΠ»ΠΎΡΠ½ΠΎΡΡΡ ΡΠ½Π΅ΡΠ³ΠΈΠΈ β ΡΡΠΎ ΠΊΠΎΠ»ΠΈΡΠ΅ΡΡΠ²ΠΎ ΡΠ½Π΅ΡΠ³ΠΈΠΈ, Ρ ΡΠ°Π½ΡΡΠ΅ΠΉΡΡ Π² Π΄Π°Π½Π½ΠΎΠΉ ΡΠΈΡΡΠ΅ΠΌΠ΅ ΠΈΠ»ΠΈ ΠΎΠ±Π»Π°ΡΡΠΈ ΠΏΡΠΎΡΡΡΠ°Π½ΡΡΠ²Π° Π½Π° Π΅Π΄ΠΈΠ½ΠΈΡΡ ΠΎΠ±ΡΠ΅ΠΌΠ°. ΠΡΠ»ΠΈ ΠΏΠΎΠ±Π»ΠΈΠ·ΠΎΡΡΠΈ Π½Π΅Ρ ΠΌΠ°Π³Π½ΠΈΡΠ½ΡΡ ΠΌΠ°ΡΠ΅ΡΠΈΠ°Π»ΠΎΠ², ΞΌ ΠΌΠΎΠΆΠ½ΠΎ Π·Π°ΠΌΠ΅Π½ΠΈΡΡ Π½Π° ΞΌ 0 . ΠΠ΄Π½Π°ΠΊΠΎ ΠΏΡΠΈΠ²Π΅Π΄Π΅Π½Π½ΠΎΠ΅ Π²ΡΡΠ΅ ΡΡΠ°Π²Π½Π΅Π½ΠΈΠ΅ Π½Π΅Π»ΡΠ·Ρ ΠΈΡΠΏΠΎΠ»ΡΠ·ΠΎΠ²Π°ΡΡ Π΄Π»Ρ Π½Π΅Π»ΠΈΠ½Π΅ΠΉΠ½ΡΡ ΠΌΠ°ΡΠ΅ΡΠΈΠ°Π»ΠΎΠ²; Π½Π΅ΠΎΠ±Ρ ΠΎΠ΄ΠΈΠΌΠΎ ΠΈΡΠΏΠΎΠ»ΡΠ·ΠΎΠ²Π°ΡΡ Π±ΠΎΠ»Π΅Π΅ ΠΎΠ±ΡΠ΅Π΅ Π²ΡΡΠ°ΠΆΠ΅Π½ΠΈΠ΅ (ΠΏΡΠΈΠ²Π΅Π΄Π΅Π½Π½ΠΎΠ΅ Π½ΠΈΠΆΠ΅).
Π ΠΎΠ±ΡΠ΅ΠΌ, Π΄ΠΎΠΏΠΎΠ»Π½ΠΈΡΠ΅Π»ΡΠ½Π°Ρ ΡΠ°Π±ΠΎΡΠ° Π½Π° Π΅Π΄ΠΈΠ½ΠΈΡΡ ΠΎΠ±ΡΠ΅ΠΌΠ° Ξ΄W , Π½Π΅ΠΎΠ±Ρ ΠΎΠ΄ΠΈΠΌΠ°Ρ Π΄Π»Ρ ΡΠΎΠ³ΠΎ, ΡΡΠΎΠ±Ρ Π²ΡΠ·Π²Π°ΡΡ Π½Π΅Π±ΠΎΠ»ΡΡΠΎΠ΅ ΠΈΠ·ΠΌΠ΅Π½Π΅Π½ΠΈΠ΅ ΠΌΠ°Π³Π½ΠΈΡΠ½ΠΎΠ³ΠΎ ΠΏΠΎΠ»Ρ Ξ΄ B, ΡΠΎΡΡΠ°Π²Π»ΡΠ΅Ρ:
[Π»Π°ΡΠ΅ΠΊΡ]\Π΄Π΅Π»ΡΡΠ°\ΡΠ΅ΠΊΡΡ{W} = \mathbf{\ΡΠ΅ΠΊΡΡ{Π}}\cdot\Π΄Π΅Π»ΡΡΠ°\mathbf{\ΡΠ΅ΠΊΡΡ{Π}}[/Π»Π°ΡΠ΅ΠΊΡ].
ΠΠΎΠ³Π΄Π° ΠΎΡΠ½ΠΎΡΠ΅Π½ΠΈΠ΅ ΠΌΠ΅ΠΆΠ΄Ρ H ΠΈ B ΠΈΠ·Π²Π΅ΡΡΠ½ΠΎ, ΡΡΠΎ ΡΡΠ°Π²Π½Π΅Π½ΠΈΠ΅ ΠΈΡΠΏΠΎΠ»ΡΠ·ΡΠ΅ΡΡΡ Π΄Π»Ρ ΠΎΠΏΡΠ΅Π΄Π΅Π»Π΅Π½ΠΈΡ ΡΠ°Π±ΠΎΡΡ, Π½Π΅ΠΎΠ±Ρ ΠΎΠ΄ΠΈΠΌΠΎΠΉ Π΄Π»Ρ Π΄ΠΎΡΡΠΈΠΆΠ΅Π½ΠΈΡ Π΄Π°Π½Π½ΠΎΠ³ΠΎ ΠΌΠ°Π³Π½ΠΈΡΠ½ΠΎΠ³ΠΎ ΡΠΎΡΡΠΎΡΠ½ΠΈΡ. ΠΠ»Ρ Π³ΠΈΡΡΠ΅ΡΠ΅Π·ΠΈΡΠ½ΡΡ ΠΌΠ°ΡΠ΅ΡΠΈΠ°Π»ΠΎΠ², ΡΠ°ΠΊΠΈΡ ΠΊΠ°ΠΊ ΡΠ΅ΡΡΠΎΠΌΠ°Π³Π½Π΅ΡΠΈΠΊΠΈ ΠΈ ΡΠ²Π΅ΡΡ ΠΏΡΠΎΠ²ΠΎΠ΄Π½ΠΈΠΊΠΈ, Π½Π΅ΠΎΠ±Ρ ΠΎΠ΄ΠΈΠΌΠ°Ρ ΡΠ°Π±ΠΎΡΠ° ΡΠ°ΠΊΠΆΠ΅ Π·Π°Π²ΠΈΡΠΈΡ ΠΎΡ ΡΠΎΠ³ΠΎ, ΠΊΠ°ΠΊ ΡΠΎΠ·Π΄Π°Π΅ΡΡΡ ΠΌΠ°Π³Π½ΠΈΡΠ½ΠΎΠ΅ ΠΏΠΎΠ»Π΅. ΠΠ΄Π½Π°ΠΊΠΎ Π΄Π»Ρ Π»ΠΈΠ½Π΅ΠΉΠ½ΡΡ Π½Π΅Π΄ΠΈΡΠΏΠ΅ΡΡΠΈΠΎΠ½Π½ΡΡ ΠΌΠ°ΡΠ΅ΡΠΈΠ°Π»ΠΎΠ² ΠΎΠ±ΡΠ΅Π΅ ΡΡΠ°Π²Π½Π΅Π½ΠΈΠ΅ ΠΏΡΠΈΠ²ΠΎΠ΄ΠΈΡ Π½Π΅ΠΏΠΎΡΡΠ΅Π΄ΡΡΠ²Π΅Π½Π½ΠΎ ΠΊ Π±ΠΎΠ»Π΅Π΅ ΠΏΡΠΎΡΡΠΎΠΌΡ ΡΡΠ°Π²Π½Π΅Π½ΠΈΡ ΠΏΠ»ΠΎΡΠ½ΠΎΡΡΠΈ ΡΠ½Π΅ΡΠ³ΠΈΠΈ, ΠΏΡΠΈΠ²Π΅Π΄Π΅Π½Π½ΠΎΠΌΡ Π²ΡΡΠ΅. 92[/Π»Π°ΡΠ΅ΠΊΡ].
Π’ΡΠ°Π½ΡΡΠΎΡΠΌΠ°ΡΠΎΡΡ
Π’ΡΠ°Π½ΡΡΠΎΡΠΌΠ°ΡΠΎΡΡ ΠΏΡΠ΅ΠΎΠ±ΡΠ°Π·ΡΡΡ Π½Π°ΠΏΡΡΠΆΠ΅Π½ΠΈΠ΅ ΠΈΠ· ΠΎΠ΄Π½ΠΎΠ³ΠΎ Π·Π½Π°ΡΠ΅Π½ΠΈΡ Π² Π΄ΡΡΠ³ΠΎΠ΅; Π΅Π³ΠΎ ΡΡΠ½ΠΊΡΠΈΡ ΠΎΠΏΡΠ΅Π΄Π΅Π»ΡΠ΅ΡΡΡ ΡΡΠ°Π²Π½Π΅Π½ΠΈΠ΅ΠΌ ΡΡΠ°Π½ΡΡΠΎΡΠΌΠ°ΡΠΎΡΠ°.
Π¦Π΅Π»ΠΈ ΠΎΠ±ΡΡΠ΅Π½ΠΈΡ
ΠΡΠΈΠΌΠ΅Π½Π΅Π½ΠΈΠ΅ ΡΡΠ°Π²Π½Π΅Π½ΠΈΡ ΡΡΠ°Π½ΡΡΠΎΡΠΌΠ°ΡΠΎΡΠ° Π΄Π»Ρ ΡΡΠ°Π²Π½Π΅Π½ΠΈΡ Π²ΡΠΎΡΠΈΡΠ½ΠΎΠ³ΠΎ ΠΈ ΠΏΠ΅ΡΠ²ΠΈΡΠ½ΠΎΠ³ΠΎ Π½Π°ΠΏΡΡΠΆΠ΅Π½ΠΈΡ
ΠΠ»ΡΡΠ΅Π²ΡΠ΅ Π²ΡΠ²ΠΎΠ΄Ρ
ΠΠ»ΡΡΠ΅Π²ΡΠ΅ ΠΌΠΎΠΌΠ΅Π½ΡΡ
- Π’ΡΠ°Π½ΡΡΠΎΡΠΌΠ°ΡΠΎΡΡ ΡΠ°ΡΡΠΎ ΠΈΡΠΏΠΎΠ»ΡΠ·ΡΡΡΡΡ Π² Π½Π΅ΡΠΊΠΎΠ»ΡΠΊΠΈΡ ΡΠΎΡΠΊΠ°Ρ Π² ΡΠΈΡΡΠ΅ΠΌΠ°Ρ ΡΠ°ΡΠΏΡΠ΅Π΄Π΅Π»Π΅Π½ΠΈΡ ΡΠ»Π΅ΠΊΡΡΠΎΡΠ½Π΅ΡΠ³ΠΈΠΈ, Π° ΡΠ°ΠΊΠΆΠ΅ Π²ΠΎ ΠΌΠ½ΠΎΠ³ΠΈΡ Π±ΡΡΠΎΠ²ΡΡ Π°Π΄Π°ΠΏΡΠ΅ΡΠ°Ρ ΠΏΠΈΡΠ°Π½ΠΈΡ.
- Π£ΡΠ°Π²Π½Π΅Π½ΠΈΠ΅ ΡΡΠ°Π½ΡΡΠΎΡΠΌΠ°ΡΠΎΡΠ° Π³Π»Π°ΡΠΈΡ, ΡΡΠΎ ΠΎΡΠ½ΠΎΡΠ΅Π½ΠΈΠ΅ Π²ΡΠΎΡΠΈΡΠ½ΡΡ ΠΈ ΠΏΠ΅ΡΠ²ΠΈΡΠ½ΡΡ Π½Π°ΠΏΡΡΠΆΠ΅Π½ΠΈΠΉ Π² ΡΡΠ°Π½ΡΡΠΎΡΠΌΠ°ΡΠΎΡΠ΅ ΡΠ°Π²Π½ΠΎ ΠΎΡΠ½ΠΎΡΠ΅Π½ΠΈΡ ΠΊΠΎΠ»ΠΈΡΠ΅ΡΡΠ²Π° Π²ΠΈΡΠΊΠΎΠ² Π² ΠΈΡ ΠΊΠ°ΡΡΡΠΊΠ°Ρ : [Π»Π°ΡΠ΅ΠΊΡ]\frac{\text{V}_\text{s}}{\text{ V}_\text{p}} = \frac{\text{N}_\text{s}}{\text{N}_\text{p}}[/latex].
- ΠΡΠ΅Π΄ΠΏΠΎΠ»Π°Π³Π°Ρ, ΠΊΠ°ΠΊ ΠΈ ΠΌΡ, ΡΡΠΎ ΡΠΎΠΏΡΠΎΡΠΈΠ²Π»Π΅Π½ΠΈΠ΅ ΠΏΡΠ΅Π½Π΅Π±ΡΠ΅ΠΆΠΈΠΌΠΎ ΠΌΠ°Π»ΠΎ, Π²ΡΡ ΠΎΠ΄Π½Π°Ρ ΡΠ»Π΅ΠΊΡΡΠΈΡΠ΅ΡΠΊΠ°Ρ ΠΌΠΎΡΠ½ΠΎΡΡΡ ΡΡΠ°Π½ΡΡΠΎΡΠΌΠ°ΡΠΎΡΠ° ΡΠ°Π²Π½Π° Π΅Π³ΠΎ Π²Ρ ΠΎΠ΄Π½ΠΎΠΉ ΠΌΠΎΡΠ½ΠΎΡΡΠΈ. ΠΡΠΎ ΠΏΡΠΈΠ²ΠΎΠ΄ΠΈΡ Π½Π°Ρ ΠΊ Π΄ΡΡΠ³ΠΎΠΌΡ ΠΏΠΎΠ»Π΅Π·Π½ΠΎΠΌΡ ΡΡΠ°Π²Π½Π΅Π½ΠΈΡ: [Π»Π°ΡΠ΅ΠΊΡ]\frac{\text{I}_\text{s}}{\text{I}_\text{p}} = \frac{\text{N}_\ text{p}}{\text{N}_\text{s}}[/latex]. ΠΡΠ»ΠΈ Π½Π°ΠΏΡΡΠΆΠ΅Π½ΠΈΠ΅ ΡΠ²Π΅Π»ΠΈΡΠΈΠ²Π°Π΅ΡΡΡ, ΡΠΎΠΊ ΡΠΌΠ΅Π½ΡΡΠ°Π΅ΡΡΡ. Π Π½Π°ΠΎΠ±ΠΎΡΠΎΡ, Π΅ΡΠ»ΠΈ Π½Π°ΠΏΡΡΠΆΠ΅Π½ΠΈΠ΅ ΡΠΌΠ΅Π½ΡΡΠ°Π΅ΡΡΡ, ΡΠΎΠΊ ΡΠ²Π΅Π»ΠΈΡΠΈΠ²Π°Π΅ΡΡΡ.
ΠΠ»ΡΡΠ΅Π²ΡΠ΅ ΡΠ΅ΡΠΌΠΈΠ½Ρ
- ΠΌΠ°Π³Π½ΠΈΡΠ½ΡΠΉ ΠΏΠΎΡΠΎΠΊ : ΠΠ΅ΡΠ° ΡΠΈΠ»Ρ ΠΌΠ°Π³Π½ΠΈΡΠ½ΠΎΠ³ΠΎ ΠΏΠΎΠ»Ρ Π² Π΄Π°Π½Π½ΠΎΠΉ ΠΎΠ±Π»Π°ΡΡΠΈ.
- ΠΠ°ΠΊΠΎΠ½ ΠΈΠ½Π΄ΡΠΊΡΠΈΠΈ Π€Π°ΡΠ°Π΄Π΅Ρ : ΠΡΠ½ΠΎΠ²Π½ΠΎΠΉ Π·Π°ΠΊΠΎΠ½ ΡΠ»Π΅ΠΊΡΡΠΎΠΌΠ°Π³Π½Π΅ΡΠΈΠ·ΠΌΠ°, ΠΊΠΎΡΠΎΡΡΠΉ ΠΏΡΠ΅Π΄ΡΠΊΠ°Π·ΡΠ²Π°Π΅Ρ, ΠΊΠ°ΠΊ ΠΌΠ°Π³Π½ΠΈΡΠ½ΠΎΠ΅ ΠΏΠΎΠ»Π΅ Π±ΡΠ΄Π΅Ρ Π²Π·Π°ΠΈΠΌΠΎΠ΄Π΅ΠΉΡΡΠ²ΠΎΠ²Π°ΡΡ Ρ ΡΠ»Π΅ΠΊΡΡΠΈΡΠ΅ΡΠΊΠΎΠΉ ΡΠ΅ΠΏΡΡ, ΡΠΎΠ·Π΄Π°Π²Π°Ρ ΡΠ»Π΅ΠΊΡΡΠΎΠ΄Π²ΠΈΠΆΡΡΡΡ ΡΠΈΠ»Ρ (ΠΠΠ‘).
Π’ΡΠ°Π½ΡΡΠΎΡΠΌΠ°ΡΠΎΡΡ ΠΈΠ·ΠΌΠ΅Π½ΡΡΡ Π½Π°ΠΏΡΡΠΆΠ΅Π½ΠΈΠ΅ Ρ ΠΎΠ΄Π½ΠΎΠ³ΠΎ Π·Π½Π°ΡΠ΅Π½ΠΈΡ Π½Π° Π΄ΡΡΠ³ΠΎΠ΅. ΠΠ°ΠΏΡΠΈΠΌΠ΅Ρ, ΡΠ°ΠΊΠΈΠ΅ ΡΡΡΡΠΎΠΉΡΡΠ²Π°, ΠΊΠ°ΠΊ ΡΠΎΡΠΎΠ²ΡΠ΅ ΡΠ΅Π»Π΅ΡΠΎΠ½Ρ, Π½ΠΎΡΡΠ±ΡΠΊΠΈ, Π²ΠΈΠ΄Π΅ΠΎΠΈΠ³ΡΡ, ΡΠ»Π΅ΠΊΡΡΠΎΠΈΠ½ΡΡΡΡΠΌΠ΅Π½ΡΡ ΠΈ Π½Π΅Π±ΠΎΠ»ΡΡΠΈΠ΅ Π±ΡΡΠΎΠ²ΡΠ΅ ΠΏΡΠΈΠ±ΠΎΡΡ, ΠΈΠΌΠ΅ΡΡ ΡΡΠ°Π½ΡΡΠΎΡΠΌΠ°ΡΠΎΡ (Π²ΡΡΡΠΎΠ΅Π½Π½ΡΠΉ Π² ΠΈΡ ΡΡΠ΅ΠΌΠ½ΡΠΉ Π±Π»ΠΎΠΊ), ΠΊΠΎΡΠΎΡΡΠΉ ΠΏΡΠ΅ΠΎΠ±ΡΠ°Π·ΡΠ΅Ρ 120 Π Π² Π½Π°ΠΏΡΡΠΆΠ΅Π½ΠΈΠ΅, ΠΏΠΎΠ΄Ρ ΠΎΠ΄ΡΡΠ΅Π΅ Π΄Π»Ρ ΡΡΡΡΠΎΠΉΡΡΠ²Π°. Π’ΡΠ°Π½ΡΡΠΎΡΠΌΠ°ΡΠΎΡΡ ΡΠ°ΠΊΠΆΠ΅ ΠΈΡΠΏΠΎΠ»ΡΠ·ΡΡΡΡΡ Π² Π½Π΅ΡΠΊΠΎΠ»ΡΠΊΠΈΡ ΡΠΎΡΠΊΠ°Ρ Π² ΡΠΈΡΡΠ΅ΠΌΠ°Ρ ΡΠ°ΡΠΏΡΠ΅Π΄Π΅Π»Π΅Π½ΠΈΡ ΡΠ»Π΅ΠΊΡΡΠΎΡΠ½Π΅ΡΠ³ΠΈΠΈ, ΠΊΠ°ΠΊ ΠΏΠΎΠΊΠ°Π·Π°Π½ΠΎ Π½Π° ΡΠΈΡ. . ΠΠ½Π΅ΡΠ³ΠΈΡ ΠΏΠ΅ΡΠ΅Π΄Π°Π΅ΡΡΡ Π½Π° Π±ΠΎΠ»ΡΡΠΈΠ΅ ΡΠ°ΡΡΡΠΎΡΠ½ΠΈΡ ΠΏΡΠΈ Π²ΡΡΠΎΠΊΠΎΠΌ Π½Π°ΠΏΡΡΠΆΠ΅Π½ΠΈΠΈ, ΠΏΠΎΡΠΊΠΎΠ»ΡΠΊΡ Π΄Π»Ρ Π·Π°Π΄Π°Π½Π½ΠΎΠΉ ΠΌΠΎΡΠ½ΠΎΡΡΠΈ ΡΡΠ΅Π±ΡΠ΅ΡΡΡ ΠΌΠ΅Π½ΡΡΠΈΠΉ ΡΠΎΠΊ (ΡΡΠΎ ΠΎΠ·Π½Π°ΡΠ°Π΅Ρ ΠΌΠ΅Π½ΡΡΠΈΠ΅ ΠΏΠΎΡΠ΅ΡΠΈ Π² Π»ΠΈΠ½ΠΈΠΈ). ΠΠΎΡΠΊΠΎΠ»ΡΠΊΡ Π²ΡΡΠΎΠΊΠΎΠ΅ Π½Π°ΠΏΡΡΠΆΠ΅Π½ΠΈΠ΅ ΠΏΡΠ΅Π΄ΡΡΠ°Π²Π»ΡΠ΅Ρ Π±ΠΎΠ»ΡΡΡΡ ΠΎΠΏΠ°ΡΠ½ΠΎΡΡΡ, Π΄Π»Ρ ΠΏΠΎΠ»ΡΡΠ΅Π½ΠΈΡ Π±ΠΎΠ»Π΅Π΅ Π½ΠΈΠ·ΠΊΠΎΠ³ΠΎ Π½Π°ΠΏΡΡΠΆΠ΅Π½ΠΈΡ Π² ΠΌΠ΅ΡΡΠ΅ ΡΠ°ΡΠΏΠΎΠ»ΠΎΠΆΠ΅Π½ΠΈΡ ΠΏΠΎΠ»ΡΠ·ΠΎΠ²Π°ΡΠ΅Π»Ρ ΠΈΡΠΏΠΎΠ»ΡΠ·ΡΡΡΡΡ ΡΡΠ°Π½ΡΡΠΎΡΠΌΠ°ΡΠΎΡΡ.
ΠΠ°ΡΡΡΠΎΠΉΠΊΠ° ΡΡΠ°Π½ΡΡΠΎΡΠΌΠ°ΡΠΎΡΠ° : Π’ΡΠ°Π½ΡΡΠΎΡΠΌΠ°ΡΠΎΡΡ ΠΈΠ·ΠΌΠ΅Π½ΡΡΡ Π½Π°ΠΏΡΡΠΆΠ΅Π½ΠΈΠ΅ Π² Π½Π΅ΡΠΊΠΎΠ»ΡΠΊΠΈΡ ΡΠΎΡΠΊΠ°Ρ ΡΠΈΡΡΠ΅ΠΌΡ ΡΠ°ΡΠΏΡΠ΅Π΄Π΅Π»Π΅Π½ΠΈΡ ΡΠ»Π΅ΠΊΡΡΠΎΡΠ½Π΅ΡΠ³ΠΈΠΈ. ΠΠ»Π΅ΠΊΡΡΠΎΡΠ½Π΅ΡΠ³ΠΈΡ ΠΎΠ±ΡΡΠ½ΠΎ Π²ΡΡΠ°Π±Π°ΡΡΠ²Π°Π΅ΡΡΡ ΠΏΡΠΈ Π½Π°ΠΏΡΡΠΆΠ΅Π½ΠΈΠΈ Π±ΠΎΠ»Π΅Π΅ 10 ΠΊΠ ΠΈ ΠΏΠ΅ΡΠ΅Π΄Π°Π΅ΡΡΡ Π½Π° Π±ΠΎΠ»ΡΡΠΈΠ΅ ΡΠ°ΡΡΡΠΎΡΠ½ΠΈΡ ΠΏΡΠΈ Π½Π°ΠΏΡΡΠΆΠ΅Π½ΠΈΠΈ Π±ΠΎΠ»Π΅Π΅ 200 ΠΊΠ, ΠΈΠ½ΠΎΠ³Π΄Π° Π΄ΠΎ 700 ΠΊΠ, Π΄Π»Ρ ΠΎΠ³ΡΠ°Π½ΠΈΡΠ΅Π½ΠΈΡ ΠΏΠΎΡΠ΅ΡΡ ΡΠ½Π΅ΡΠ³ΠΈΠΈ. ΠΠ΅ΡΡΠ½ΠΎΠ΅ ΡΠ°ΡΠΏΡΠ΅Π΄Π΅Π»Π΅Π½ΠΈΠ΅ ΡΠ»Π΅ΠΊΡΡΠΎΡΠ½Π΅ΡΠ³ΠΈΠΈ Π² ΡΠ°ΠΉΠΎΠ½Ρ ΠΈΠ»ΠΈ ΠΏΡΠ΅Π΄ΠΏΡΠΈΡΡΠΈΡ ΠΏΡΠΎΡ ΠΎΠ΄ΠΈΡ ΡΠ΅ΡΠ΅Π· ΠΏΠΎΠ΄ΡΡΠ°Π½ΡΠΈΡ ββΠΈ ΠΏΠ΅ΡΠ΅Π΄Π°Π΅ΡΡΡ Π½Π° ΠΊΠΎΡΠΎΡΠΊΠΈΠ΅ ΡΠ°ΡΡΡΠΎΡΠ½ΠΈΡ ΠΏΡΠΈ Π½Π°ΠΏΡΡΠΆΠ΅Π½ΠΈΠΈ ΠΎΡ 5 Π΄ΠΎ 13 ΠΊΠ. ΠΠ½ΠΎ ΡΠ½ΠΈΠΆΠ΅Π½ΠΎ Π΄ΠΎ 120, 240 ΠΈΠ»ΠΈ 480 Π Π΄Π»Ρ ΠΎΠ±Π΅ΡΠΏΠ΅ΡΠ΅Π½ΠΈΡ Π±Π΅Π·ΠΎΠΏΠ°ΡΠ½ΠΎΡΡΠΈ Π½Π° ΠΎΠ±ΡΠ΅ΠΊΡΠ΅ ΠΎΡΠ΄Π΅Π»ΡΠ½ΠΎΠ³ΠΎ ΠΏΠΎΠ»ΡΠ·ΠΎΠ²Π°ΡΠ΅Π»Ρ.
Π’ΠΈΠΏ ΡΡΠ°Π½ΡΡΠΎΡΠΌΠ°ΡΠΎΡΠ°, ΡΠ°ΡΡΠΌΠ°ΡΡΠΈΠ²Π°Π΅ΠΌΠΎΠ³ΠΎ Π·Π΄Π΅ΡΡ, ΠΎΡΠ½ΠΎΠ²Π°Π½ Π½Π° Π·Π°ΠΊΠΎΠ½Π΅ ΠΈΠ½Π΄ΡΠΊΡΠΈΠΈ Π€Π°ΡΠ°Π΄Π΅Ρ ΠΈ ΠΎΡΠ΅Π½Ρ ΠΏΠΎΡ ΠΎΠΆ ΠΏΠΎ ΠΊΠΎΠ½ΡΡΡΡΠΊΡΠΈΠΈ Π½Π° Π°ΠΏΠΏΠ°ΡΠ°Ρ Π€Π°ΡΠ°Π΄Π΅Ρ, ΠΊΠΎΡΠΎΡΡΠΉ ΠΈΡΠΏΠΎΠ»ΡΠ·ΠΎΠ²Π°Π»ΡΡ Π΄Π»Ρ Π΄Π΅ΠΌΠΎΠ½ΡΡΡΠ°ΡΠΈΠΈ ΡΠΎΠ³ΠΎ, ΡΡΠΎ ΠΌΠ°Π³Π½ΠΈΡΠ½ΡΠ΅ ΠΏΠΎΠ»Ρ ΠΌΠΎΠ³ΡΡ ΡΠΎΠ·Π΄Π°Π²Π°ΡΡ ΡΠΎΠΊΠΈ (ΠΏΠΎΠΊΠ°Π·Π°Π½ΠΎ Π½Π° ). ΠΠ²Π΅ ΠΊΠ°ΡΡΡΠΊΠΈ Π½Π°Π·ΡΠ²Π°ΡΡΡΡ ΠΏΠ΅ΡΠ²ΠΈΡΠ½ΠΎΠΉ ΠΈ Π²ΡΠΎΡΠΈΡΠ½ΠΎΠΉ ΠΊΠ°ΡΡΡΠΊΠ°ΠΌΠΈ. ΠΡΠΈ Π½ΠΎΡΠΌΠ°Π»ΡΠ½ΠΎΠΌ ΠΈΡΠΏΠΎΠ»ΡΠ·ΠΎΠ²Π°Π½ΠΈΠΈ Π²Ρ ΠΎΠ΄Π½ΠΎΠ΅ Π½Π°ΠΏΡΡΠΆΠ΅Π½ΠΈΠ΅ ΠΏΠΎΠ΄Π°Π΅ΡΡΡ Π½Π° ΠΏΠ΅ΡΠ²ΠΈΡΠ½ΡΡ ΠΎΠ±ΠΌΠΎΡΠΊΡ, Π° Π²ΡΠΎΡΠΈΡΠ½Π°Ρ ΡΠΎΠ·Π΄Π°Π΅Ρ ΠΏΡΠ΅ΠΎΠ±ΡΠ°Π·ΠΎΠ²Π°Π½Π½ΠΎΠ΅ Π²ΡΡ ΠΎΠ΄Π½ΠΎΠ΅ Π½Π°ΠΏΡΡΠΆΠ΅Π½ΠΈΠ΅. ΠΠ΅Π»Π΅Π·Π½ΡΠΉ ΡΠ΅ΡΠ΄Π΅ΡΠ½ΠΈΠΊ Π½Π΅ ΡΠΎΠ»ΡΠΊΠΎ ΡΠ»Π°Π²Π»ΠΈΠ²Π°Π΅Ρ ΠΌΠ°Π³Π½ΠΈΡΠ½ΠΎΠ΅ ΠΏΠΎΠ»Π΅, ΡΠΎΠ·Π΄Π°Π²Π°Π΅ΠΌΠΎΠ΅ ΠΏΠ΅ΡΠ²ΠΈΡΠ½ΠΎΠΉ ΠΊΠ°ΡΡΡΠΊΠΎΠΉ, Π½ΠΎ ΠΈ ΡΠ²Π΅Π»ΠΈΡΠΈΠ²Π°Π΅Ρ Π΅Π³ΠΎ Π½Π°ΠΌΠ°Π³Π½ΠΈΡΠ΅Π½Π½ΠΎΡΡΡ. ΠΠΎΡΠΊΠΎΠ»ΡΠΊΡ Π²Ρ ΠΎΠ΄Π½ΠΎΠ΅ Π½Π°ΠΏΡΡΠΆΠ΅Π½ΠΈΠ΅ ΠΏΠ΅ΡΠ΅ΠΌΠ΅Π½Π½ΠΎΠ³ΠΎ ΡΠΎΠΊΠ°, ΠΈΠ·ΠΌΠ΅Π½ΡΡΡΠΈΠΉΡΡ Π²ΠΎ Π²ΡΠ΅ΠΌΠ΅Π½ΠΈ ΠΌΠ°Π³Π½ΠΈΡΠ½ΡΠΉ ΠΏΠΎΡΠΎΠΊ Π½Π°ΠΏΡΠ°Π²Π»ΡΠ΅ΡΡΡ Π½Π° Π²ΡΠΎΡΠΈΡΠ½ΡΡ ΠΎΠ±ΠΌΠΎΡΠΊΡ, ΠΈΠ½Π΄ΡΡΠΈΡΡΡ Π΅Π΅ Π²ΡΡ ΠΎΠ΄Π½ΠΎΠ΅ Π½Π°ΠΏΡΡΠΆΠ΅Π½ΠΈΠ΅ ΠΏΠ΅ΡΠ΅ΠΌΠ΅Π½Π½ΠΎΠ³ΠΎ ΡΠΎΠΊΠ°.
ΠΡΠΎΡΡΠΎΠΉ ΡΡΠ°Π½ΡΡΠΎΡΠΌΠ°ΡΠΎΡ : Π’ΠΈΠΏΠΈΡΠ½Π°Ρ ΠΊΠΎΠ½ΡΡΡΡΠΊΡΠΈΡ ΠΏΡΠΎΡΡΠΎΠ³ΠΎ ΡΡΠ°Π½ΡΡΠΎΡΠΌΠ°ΡΠΎΡΠ° ΡΠΎΡΡΠΎΠΈΡ ΠΈΠ· Π΄Π²ΡΡ ΠΊΠ°ΡΡΡΠ΅ΠΊ, Π½Π°ΠΌΠΎΡΠ°Π½Π½ΡΡ Π½Π° ΡΠ΅ΡΡΠΎΠΌΠ°Π³Π½ΠΈΡΠ½ΡΠΉ ΡΠ΅ΡΠ΄Π΅ΡΠ½ΠΈΠΊ, Π»Π°ΠΌΠΈΠ½ΠΈΡΠΎΠ²Π°Π½Π½ΡΠΉ Π΄Π»Ρ ΠΌΠΈΠ½ΠΈΠΌΠΈΠ·Π°ΡΠΈΠΈ Π²ΠΈΡ ΡΠ΅Π²ΡΡ ΡΠΎΠΊΠΎΠ². ΠΠ°Π³Π½ΠΈΡΠ½ΠΎΠ΅ ΠΏΠΎΠ»Π΅, ΡΠΎΠ·Π΄Π°Π²Π°Π΅ΠΌΠΎΠ΅ ΠΏΠ΅ΡΠ²ΠΈΡΠ½ΠΎΠΉ ΠΎΠ±ΠΌΠΎΡΠΊΠΎΠΉ, Π² ΠΎΡΠ½ΠΎΠ²Π½ΠΎΠΌ ΠΎΠ³ΡΠ°Π½ΠΈΡΠΈΠ²Π°Π΅ΡΡΡ ΠΈ ΡΡΠΈΠ»ΠΈΠ²Π°Π΅ΡΡΡ ΡΠ΅ΡΠ΄Π΅ΡΠ½ΠΈΠΊΠΎΠΌ, ΠΊΠΎΡΠΎΡΡΠΉ ΠΏΠ΅ΡΠ΅Π΄Π°Π΅Ρ Π΅Π³ΠΎ Π²ΡΠΎΡΠΈΡΠ½ΠΎΠΉ ΠΎΠ±ΠΌΠΎΡΠΊΠ΅. ΠΡΠ±ΠΎΠ΅ ΠΈΠ·ΠΌΠ΅Π½Π΅Π½ΠΈΠ΅ ΡΠΎΠΊΠ° Π² ΠΏΠ΅ΡΠ²ΠΈΡΠ½ΠΎΠΉ ΠΎΠ±ΠΌΠΎΡΠΊΠ΅ ΠΈΠ½Π΄ΡΡΠΈΡΡΠ΅Ρ ΡΠΎΠΊ Π²ΠΎ Π²ΡΠΎΡΠΈΡΠ½ΠΎΠΉ ΠΎΠ±ΠΌΠΎΡΠΊΠ΅. ΠΠ° ΡΠΈΡΡΠ½ΠΊΠ΅ ΠΏΠΎΠΊΠ°Π·Π°Π½ ΠΏΡΠΎΡΡΠΎΠΉ ΡΡΠ°Π½ΡΡΠΎΡΠΌΠ°ΡΠΎΡ Ρ Π΄Π²ΡΠΌΡ ΠΊΠ°ΡΡΡΠΊΠ°ΠΌΠΈ, Π½Π°ΠΌΠΎΡΠ°Π½Π½ΡΠΌΠΈ ΠΏΠΎ ΠΎΠ±Π΅ΠΈΠΌ ΡΡΠΎΡΠΎΠ½Π°ΠΌ ΠΌΠ½ΠΎΠ³ΠΎΡΠ»ΠΎΠΉΠ½ΠΎΠ³ΠΎ ΡΠ΅ΡΡΠΎΠΌΠ°Π³Π½ΠΈΡΠ½ΠΎΠ³ΠΎ ΡΠ΅ΡΠ΄Π΅ΡΠ½ΠΈΠΊΠ°. ΠΠ°Π±ΠΎΡ ΠΊΠ°ΡΡΡΠ΅ΠΊ Π½Π° Π»Π΅Π²ΠΎΠΉ ΡΡΠΎΡΠΎΠ½Π΅ ΡΠ΅ΡΠ΄Π΅ΡΠ½ΠΈΠΊΠ° ΠΏΠΎΠΌΠ΅ΡΠ΅Π½ ΠΊΠ°ΠΊ ΠΏΠ΅ΡΠ²ΠΈΡΠ½ΡΠΉ, Π° Π΅Π³ΠΎ Π½ΠΎΠΌΠ΅Ρ ΡΠΊΠ°Π·Π°Π½ ΠΊΠ°ΠΊ N p. ΠΠ°ΠΏΡΡΠΆΠ΅Π½ΠΈΠ΅ Π½Π° ΠΏΠ΅ΡΠ²ΠΈΡΠ½ΠΎΠΉ ΠΎΠ±ΠΌΠΎΡΠΊΠ΅ ΠΎΠΏΡΠ΅Π΄Π΅Π»ΡΠ΅ΡΡΡ ΠΊΠ°ΠΊ V p. ΠΠ°Π±ΠΎΡ ΠΊΠ°ΡΡΡΠ΅ΠΊ Π½Π° ΠΏΡΠ°Π²ΠΎΠΉ ΡΡΠΎΡΠΎΠ½Π΅ ΡΠ΅ΡΠ΄Π΅ΡΠ½ΠΈΠΊΠ° ΠΏΠΎΠΌΠ΅ΡΠ΅Π½ ΠΊΠ°ΠΊ Π²ΡΠΎΡΠΈΡΠ½ΡΠΉ, Π° ΠΈΡ ΠΊΠΎΠ»ΠΈΡΠ΅ΡΡΠ²ΠΎ ΠΏΡΠ΅Π΄ΡΡΠ°Π²Π»Π΅Π½ΠΎ ΠΊΠ°ΠΊ Ns. ΠΠ°ΠΏΡΡΠΆΠ΅Π½ΠΈΠ΅ Π½Π° Π²ΡΠΎΡΠΈΡΠ½ΠΎΠΉ ΠΎΠ±ΠΌΠΎΡΠΊΠ΅ ΠΎΠΏΡΠ΅Π΄Π΅Π»ΡΠ΅ΡΡΡ ΠΊΠ°ΠΊ V s. Π‘ΠΈΠΌΠ²ΠΎΠ» ΡΡΠ°Π½ΡΡΠΎΡΠΌΠ°ΡΠΎΡΠ° ΡΠ°ΠΊΠΆΠ΅ ΠΏΠΎΠΊΠ°Π·Π°Π½ ΠΏΠΎΠ΄ Π΄ΠΈΠ°Π³ΡΠ°ΠΌΠΌΠΎΠΉ. ΠΠ½ ΡΠΎΡΡΠΎΠΈΡ ΠΈΠ· Π΄Π²ΡΡ ΠΊΠ°ΡΡΡΠ΅ΠΊ ΠΈΠ½Π΄ΡΠΊΡΠΈΠ²Π½ΠΎΡΡΠΈ, ΡΠ°Π·Π΄Π΅Π»Π΅Π½Π½ΡΡ Π΄Π²ΡΠΌΡ ΡΠ°Π²Π½ΡΠΌΠΈ ΠΏΠ°ΡΠ°Π»Π»Π΅Π»ΡΠ½ΡΠΌΠΈ Π»ΠΈΠ½ΠΈΡΠΌΠΈ, ΠΏΡΠ΅Π΄ΡΡΠ°Π²Π»ΡΡΡΠΈΠΌΠΈ ΡΠ΅ΡΠ΄Π΅ΡΠ½ΠΈΠΊ.
Π£ΡΠ°Π²Π½Π΅Π½ΠΈΠ΅ ΡΡΠ°Π½ΡΡΠΎΡΠΌΠ°ΡΠΎΡΠ°
ΠΠ»Ρ ΠΏΡΠΎΡΡΠΎΠ³ΠΎ ΡΡΠ°Π½ΡΡΠΎΡΠΌΠ°ΡΠΎΡΠ°, ΠΏΠΎΠΊΠ°Π·Π°Π½Π½ΠΎΠ³ΠΎ Π½Π° ΡΠΈΡ., Π²ΡΡ ΠΎΠ΄Π½ΠΎΠ΅ Π½Π°ΠΏΡΡΠΆΠ΅Π½ΠΈΠ΅ V s ΠΏΠΎΡΡΠΈ ΠΏΠΎΠ»Π½ΠΎΡΡΡΡ Π·Π°Π²ΠΈΡΠΈΡ ΠΎΡ Π²Ρ ΠΎΠ΄Π½ΠΎΠ³ΠΎ Π½Π°ΠΏΡΡΠΆΠ΅Π½ΠΈΡ V p ΠΈ ΡΠΎΠΎΡΠ½ΠΎΡΠ΅Π½ΠΈΡ ΡΠΈΡΠ»Π° Π²ΠΈΡΠΊΠΎΠ² Π² ΠΏΠ΅ΡΠ²ΠΈΡΠ½ΠΎΠΉ ΠΈ Π²ΡΠΎΡΠΈΡΠ½ΠΎΠΉ ΠΎΠ±ΠΌΠΎΡΠΊΠ°Ρ . ΠΠ°ΠΊΠΎΠ½ ΠΈΠ½Π΄ΡΠΊΡΠΈΠΈ Π€Π°ΡΠ°Π΄Π΅Ρ Π΄Π»Ρ Π²ΡΠΎΡΠΈΡΠ½ΠΎΠΉ ΠΊΠ°ΡΡΡΠΊΠΈ Π΄Π°Π΅Ρ ΠΈΠ½Π΄ΡΡΠΈΡΠΎΠ²Π°Π½Π½ΠΎΠ΅ Π²ΡΡ ΠΎΠ΄Π½ΠΎΠ΅ Π½Π°ΠΏΡΡΠΆΠ΅Π½ΠΈΠ΅ V Ρ ΠΊΠ°ΠΊ:
[Π»Π°ΡΠ΅ΠΊΡ]\ΡΠ΅ΠΊΡΡ{V}_\ΡΠ΅ΠΊΡΡ{s} = -\text{N}_\text{s} \frac {\Delta \Phi}{\Delta \text{t}}[/latex],
, Π³Π΄Π΅ N s β ΠΊΠΎΠ»ΠΈΡΠ΅ΡΡΠ²ΠΎ Π²ΠΈΡΠΊΠΎΠ² Π²ΡΠΎΡΠΈΡΠ½ΠΎΠΉ ΠΎΠ±ΠΌΠΎΡΠΊΠΈ, Π° Ξ/Ξt β ΡΠΊΠΎΡΠΎΡΡΡ ΠΈΠ·ΠΌΠ΅Π½Π΅Π½ΠΈΡ ΠΌΠ°Π³Π½ΠΈΡΠ½ΠΎΠ³ΠΎ ΠΏΠΎΡΠΎΠΊΠ°. ΠΠ±ΡΠ°ΡΠΈΡΠ΅ Π²Π½ΠΈΠΌΠ°Π½ΠΈΠ΅, ΡΡΠΎ Π²ΡΡ ΠΎΠ΄Π½ΠΎΠ΅ Π½Π°ΠΏΡΡΠΆΠ΅Π½ΠΈΠ΅ ΡΠ°Π²Π½ΠΎ ΠΠΠ‘ ΠΈΠ½Π΄ΡΠΊΡΠΈΠΈ (V Ρ = ΠΠΠ‘ Ρ ), Π΅ΡΠ»ΠΈ ΡΠΎΠΏΡΠΎΡΠΈΠ²Π»Π΅Π½ΠΈΠ΅ ΠΊΠ°ΡΡΡΠΊΠΈ ΠΌΠ°Π»ΠΎ. ΠΠ»ΠΎΡΠ°Π΄Ρ ΠΏΠΎΠΏΠ΅ΡΠ΅ΡΠ½ΠΎΠ³ΠΎ ΡΠ΅ΡΠ΅Π½ΠΈΡ ΠΊΠ°ΡΡΡΠ΅ΠΊ ΠΎΠ΄ΠΈΠ½Π°ΠΊΠΎΠ²Π° Ρ ΠΎΠ±Π΅ΠΈΡ ΡΡΠΎΡΠΎΠ½, ΠΊΠ°ΠΊ ΠΈ Π½Π°ΠΏΡΡΠΆΠ΅Π½Π½ΠΎΡΡΡ ΠΌΠ°Π³Π½ΠΈΡΠ½ΠΎΠ³ΠΎ ΠΏΠΎΠ»Ρ, ΠΏΠΎΡΡΠΎΠΌΡ /Ξt ΠΎΠ΄ΠΈΠ½Π°ΠΊΠΎΠ²ΠΎ Ρ ΠΎΠ±Π΅ΠΈΡ ΡΡΠΎΡΠΎΠ½. ΠΡ ΠΎΠ΄Π½ΠΎΠ΅ ΠΏΠ΅ΡΠ²ΠΈΡΠ½ΠΎΠ΅ Π½Π°ΠΏΡΡΠΆΠ΅Π½ΠΈΠ΅ V p ΡΠ°ΠΊΠΆΠ΅ ΡΠ²ΡΠ·Π°Π½ΠΎ Ρ ΠΈΠ·ΠΌΠ΅Π½Π΅Π½ΠΈΠ΅ΠΌ ΠΏΠΎΡΠΎΠΊΠ°:
[Π»Π°ΡΠ΅ΠΊΡ]\text{V}_\text{p} = -\text{N}_\text{p} \frac{\Delta \Phi}{\Delta \text{t}}[/latex].
Π‘ΠΎΠΎΡΠ½ΠΎΡΠ΅Π½ΠΈΠ΅ ΡΡΠΈΡ Π΄Π²ΡΡ ΠΏΠΎΡΠ»Π΅Π΄Π½ΠΈΡ ΡΡΠ°Π²Π½Π΅Π½ΠΈΠΉ Π΄Π°Π΅Ρ ΠΏΠΎΠ»Π΅Π·Π½ΠΎΠ΅ ΡΠΎΠΎΡΠ½ΠΎΡΠ΅Π½ΠΈΠ΅:
[Π»Π°ΡΠ΅ΠΊΡ]\frac{\text{V}_\text{s}}{\text{V}_\text{p}} = \ frac{\text{N}_\text{s}}{\text{N}_\text{p}}[/latex].
ΠΡΠΎ ΠΈΠ·Π²Π΅ΡΡΠ½ΠΎ ΠΊΠ°ΠΊ ΡΡΠ°Π²Π½Π΅Π½ΠΈΠ΅ ΡΡΠ°Π½ΡΡΠΎΡΠΌΠ°ΡΠΎΡΠ° , ΠΊΠΎΡΠΎΡΠΎΠ΅ ΠΏΡΠΎΡΡΠΎ ΡΡΠ²Π΅ΡΠΆΠ΄Π°Π΅Ρ, ΡΡΠΎ ΠΎΡΠ½ΠΎΡΠ΅Π½ΠΈΠ΅ Π²ΡΠΎΡΠΈΡΠ½ΠΎΠ³ΠΎ Π½Π°ΠΏΡΡΠΆΠ΅Π½ΠΈΡ ΠΊ ΠΏΠ΅ΡΠ²ΠΈΡΠ½ΠΎΠΌΡ Π² ΡΡΠ°Π½ΡΡΠΎΡΠΌΠ°ΡΠΎΡΠ΅ ΡΠ°Π²Π½ΠΎ ΠΎΡΠ½ΠΎΡΠ΅Π½ΠΈΡ ΠΊΠΎΠ»ΠΈΡΠ΅ΡΡΠ²Π° Π²ΠΈΡΠΊΠΎΠ² Π² Π΅Π³ΠΎ ΠΊΠ°ΡΡΡΠΊΠ°Ρ . ΠΡΡ ΠΎΠ΄Π½ΠΎΠ΅ Π½Π°ΠΏΡΡΠΆΠ΅Π½ΠΈΠ΅ ΡΡΠ°Π½ΡΡΠΎΡΠΌΠ°ΡΠΎΡΠ° ΠΌΠΎΠΆΠ΅Ρ Π±ΡΡΡ ΠΌΠ΅Π½ΡΡΠ΅, Π±ΠΎΠ»ΡΡΠ΅ ΠΈΠ»ΠΈ ΡΠ°Π²Π½ΠΎ Π²Ρ ΠΎΠ΄Π½ΠΎΠΌΡ Π½Π°ΠΏΡΡΠΆΠ΅Π½ΠΈΡ, Π² Π·Π°Π²ΠΈΡΠΈΠΌΠΎΡΡΠΈ ΠΎΡ ΡΠΎΠΎΡΠ½ΠΎΡΠ΅Π½ΠΈΡ ΡΠΈΡΠ»Π° Π²ΠΈΡΠΊΠΎΠ² Π² ΠΈΡ ΠΊΠ°ΡΡΡΠΊΠ°Ρ . ΠΠ΅ΠΊΠΎΡΠΎΡΡΠ΅ ΡΡΠ°Π½ΡΡΠΎΡΠΌΠ°ΡΠΎΡΡ Π΄Π°ΠΆΠ΅ ΠΎΠ±Π΅ΡΠΏΠ΅ΡΠΈΠ²Π°ΡΡ ΠΏΠ΅ΡΠ΅ΠΌΠ΅Π½Π½ΡΡ ΠΌΠΎΡΠ½ΠΎΡΡΡ, ΠΏΠΎΠ·Π²ΠΎΠ»ΡΡ Π²ΡΠΏΠΎΠ»Π½ΡΡΡ ΠΏΠΎΠ΄ΠΊΠ»ΡΡΠ΅Π½ΠΈΠ΅ Π² ΡΠ°Π·Π½ΡΡ ΡΠΎΡΠΊΠ°Ρ Π²ΡΠΎΡΠΈΡΠ½ΠΎΠΉ ΠΎΠ±ΠΌΠΎΡΠΊΠΈ. ΠΠΎΠ²ΡΡΠ°ΡΡΠΈΠΉ ΡΡΠ°Π½ΡΡΠΎΡΠΌΠ°ΡΠΎΡ β ΡΡΠΎ ΡΠΎΡ, ΠΊΠΎΡΠΎΡΡΠΉ ΡΠ²Π΅Π»ΠΈΡΠΈΠ²Π°Π΅Ρ Π½Π°ΠΏΡΡΠΆΠ΅Π½ΠΈΠ΅, ΡΠΎΠ³Π΄Π° ΠΊΠ°ΠΊ ΠΏΠΎΠ½ΠΈΠΆΠ°ΡΡΠΈΠΉ ΡΡΠ°Π½ΡΡΠΎΡΠΌΠ°ΡΠΎΡ ΡΠΌΠ΅Π½ΡΡΠ°Π΅Ρ Π½Π°ΠΏΡΡΠΆΠ΅Π½ΠΈΠ΅.
ΠΡΠ΅Π΄ΠΏΠΎΠ»Π°Π³Π°Ρ, ΠΊΠ°ΠΊ ΠΈ ΠΌΡ, ΡΡΠΎ ΡΠΎΠΏΡΠΎΡΠΈΠ²Π»Π΅Π½ΠΈΠ΅ ΠΏΡΠ΅Π½Π΅Π±ΡΠ΅ΠΆΠΈΠΌΠΎ ΠΌΠ°Π»ΠΎ, Π²ΡΡ ΠΎΠ΄Π½Π°Ρ ΡΠ»Π΅ΠΊΡΡΠΈΡΠ΅ΡΠΊΠ°Ρ ΠΌΠΎΡΠ½ΠΎΡΡΡ ΡΡΠ°Π½ΡΡΠΎΡΠΌΠ°ΡΠΎΡΠ° ΡΠ°Π²Π½Π° Π΅Π³ΠΎ Π²Ρ ΠΎΠ΄Π½ΠΎΠΉ ΠΌΠΎΡΠ½ΠΎΡΡΠΈ. ΠΡΠΈΡΠ°Π²Π½ΠΈΠ²Π°Ρ Π²Ρ ΠΎΠ΄Π½ΡΡ ΠΈ Π²ΡΡ ΠΎΠ΄Π½ΡΡ ΠΌΠΎΡΠ½ΠΎΡΡΡ,
[Π»Π°ΡΠ΅ΠΊΡ]\ΡΠ΅ΠΊΡΡ{P}_\text{p} = \text{I}_\text{p} \text{V}_\text{p} = \text{ I}_\text{s} \text{V}_\text{s} = \text{P}_\text{s}[/latex].
ΠΠΎΠΌΠ±ΠΈΠ½ΠΈΡΡΡ ΡΡΠΎΡ ΡΠ΅Π·ΡΠ»ΡΡΠ°Ρ Ρ ΡΡΠ°Π²Π½Π΅Π½ΠΈΠ΅ΠΌ ΡΡΠ°Π½ΡΡΠΎΡΠΌΠ°ΡΠΎΡΠ°, ΠΌΡ Π½Π°Ρ ΠΎΠ΄ΠΈΠΌ:
[Π»Π°ΡΠ΅ΠΊΡ]\frac{\text{I}_\text{s}}{\text{I}_\text{p}} = \frac{ \text{N}_\text{p}}{\text{N}_\text{s}}[/latex].
Π’Π°ΠΊΠΈΠΌ ΠΎΠ±ΡΠ°Π·ΠΎΠΌ, Π΅ΡΠ»ΠΈ Π½Π°ΠΏΡΡΠΆΠ΅Π½ΠΈΠ΅ ΡΠ²Π΅Π»ΠΈΡΠΈΠ²Π°Π΅ΡΡΡ, ΡΠΎΠΊ ΡΠΌΠ΅Π½ΡΡΠ°Π΅ΡΡΡ. Π Π½Π°ΠΎΠ±ΠΎΡΠΎΡ, Π΅ΡΠ»ΠΈ Π½Π°ΠΏΡΡΠΆΠ΅Π½ΠΈΠ΅ ΡΠΌΠ΅Π½ΡΡΠ°Π΅ΡΡΡ, ΡΠΎΠΊ ΡΠ²Π΅Π»ΠΈΡΠΈΠ²Π°Π΅ΡΡΡ.
ΠΠΏΡΠ΅Π΄Π΅Π»Π΅Π½ΠΈΠ΅ ΠΠΠ‘ Π΄Π²ΠΈΠΆΠ΅Π½ΠΈΡ Π² ΡΠΈΠ·ΠΈΠΊΠ΅.
(ΡΡΡΠ΅ΡΡΠ²ΠΈΡΠ΅Π»ΡΠ½ΠΎΠ΅)
ΠΠΠ‘ (ΡΠ»Π΅ΠΊΡΡΠΎΠ΄Π²ΠΈΠΆΡΡΠ°Ρ ΡΠΈΠ»Π°), Π²ΡΠ·Π²Π°Π½Π½Π°Ρ Π΄Π²ΠΈΠΆΠ΅Π½ΠΈΠ΅ΠΌ ΠΎΡΠ½ΠΎΡΠΈΡΠ΅Π»ΡΠ½ΠΎ ΠΌΠ°Π³Π½ΠΈΡΠ½ΠΎΠ³ΠΎ ΠΏΠΎΠ»Ρ.
ΠΠΎΠ»ΠΈΡΠ΅ΡΡΠ²Π΅Π½Π½Π°Ρ ΠΈΠ½ΡΠ΅ΡΠΏΡΠ΅ΡΠ°ΡΠΈΡ ΠΠΠ‘ Π΄Π²ΠΈΠΆΠ΅Π½ΠΈΡ
- A a Π΄Π²ΠΈΠΆΡΡΠ°Ρ ΠΠΠ‘ — ΡΠ»Π΅ΠΊΡΡΠΎΠ΄Π²ΠΈΠΆΡΡΠ°Ρ ΡΠΈΠ»Π° ( ΠΠΠ‘ ), ΠΈΠ½Π΄ΡΡΠΈΡΠΎΠ²Π°Π½Π½Π°Ρ Π΄Π²ΠΈΠΆΠ΅Π½ΠΈΠ΅ΠΌ ΠΎΡΠ½ΠΎΡΠΈΡΠ΅Π»ΡΠ½ΠΎ ΠΌΠ°Π³Π½ΠΈΡΠ½ΠΎΠ³ΠΎ ΠΏΠΎΠ»Ρ B.
- ΠΠ»Π΅ΠΊΡΡΠΎΠ΄Π²ΠΈΠΆΡΡΠ°Ρ ΡΠΈΠ»Π° ( ΠΠΠ‘ ) , Π²ΡΠ·Π²Π°Π½Π½Π°Ρ Π΄Π²ΠΈΠΆΠ΅Π½ΠΈΠ΅ΠΌ ΠΎΡΠ½ΠΎΡΠΈΡΠ΅Π»ΡΠ½ΠΎ ΠΌΠ°Π³Π½ΠΈΡΠ½ΠΎΠ³ΠΎ ΠΏΠΎΠ»Ρ B , Π½Π°Π·ΡΠ²Π°Π΅ΡΡΡ Π΄Π²ΠΈΠΆΡΡΠ΅ΠΉ ΠΠΠ‘ .
- ΠΡ ΠΌΠΎΠ³Π»ΠΈ Π·Π°ΠΌΠ΅ΡΠΈΡΡ, ΡΡΠΎ Π΄Π²ΠΈΠΆΡΡΠ°ΡΡΡ ΠΠΠ‘ ΠΎΡΠ΅Π½Ρ ΠΏΠΎΡ ΠΎΠΆΠ° Π½Π° ΠΈΠ½Π΄ΡΡΠΈΡΠΎΠ²Π°Π½Π½ΡΡ ΠΠΠ‘ , Π²ΡΠ·Π²Π°Π½Π½ΡΡ ΠΈΠ·ΠΌΠ΅Π½ΡΡΡΠΈΠΌΡΡ ΠΌΠ°Π³Π½ΠΈΡΠ½ΡΠΌ ΠΏΠΎΠ»Π΅ΠΌ.
- ΠΠ· ΡΡΠ°Π²Π½Π΅Π½ΠΈΡ. 1 ΠΈ ΡΡΠ°Π²Π½Π΅Π½ΠΈΠ΅ 2 ΠΌΡ ΠΌΠΎΠΆΠ΅ΠΌ ΠΏΠΎΠ΄ΡΠ²Π΅ΡΠ΄ΠΈΡΡ, ΡΡΠΎ Π΄Π²ΠΈΠΆΡΡΠΈΡ ΡΡ ΠΈ ΠΈΠ½Π΄ΡΡΠΈΡΠΎΠ²Π°Π½Π½ΡΡ ΠΠΠ‘ Π΄Π°ΡΡ ΠΎΠ΄ΠΈΠ½Π°ΠΊΠΎΠ²ΡΠΉ ΡΠ΅Π·ΡΠ»ΡΡΠ°Ρ.
- (Π°) ΠΠ²ΠΈΠΆΠ΅Π½ΠΈΠ΅ ΠΠΠ‘ .
ΠΠΠ‘ Π΄Π²ΠΈΠΆΠ΅Π½ΠΈΡ
- ΠΠ²ΠΈΠΆΠ΅Π½ΠΈΠ΅ Π² ΡΡΠ°ΡΠΈΠΎΠ½Π°ΡΠ½ΠΎΠΌ ΠΎΡΠ½ΠΎΡΠΈΡΠ΅Π»ΡΠ½ΠΎ ΠΠ΅ΠΌΠ»ΠΈ ΠΌΠ°Π³Π½ΠΈΡΠ½ΠΎΠΌ ΠΏΠΎΠ»Π΅ ΠΈΠ½Π΄ΡΡΠΈΡΡΠ΅Ρ Π΄Π²ΠΈΠΆΡΡΡΡ ΠΠΠ‘ (ΡΠ»Π΅ΠΊΡΡΠΎΠ΄Π²ΠΈΠΆΡΡΠ°Ρ ΡΠΈΠ»Π°).
- ΠΠ²ΠΈΠΆΠ΅Π½ΠΈΠ΅ ΡΠ²Π»ΡΠ΅ΡΡΡ ΠΎΠ΄Π½ΠΎΠΉ ΠΈΠ· ΠΎΡΠ½ΠΎΠ²Π½ΡΡ ΠΏΡΠΈΡΠΈΠ½ ΠΈΠ½Π΄ΡΠΊΡΠΈΠΈ.
- ΠΠ°ΠΏΡΠΈΠΌΠ΅Ρ, ΠΌΠ°Π³Π½ΠΈΡ, ΠΏΠ΅ΡΠ΅ΠΌΠ΅ΡΠ°Π΅ΠΌΡΠΉ ΠΏΠΎ Π½Π°ΠΏΡΠ°Π²Π»Π΅Π½ΠΈΡ ΠΊ ΠΊΠ°ΡΡΡΠΊΠ΅, ΠΈΠ½Π΄ΡΡΠΈΡΡΠ΅Ρ ΠΠΠ‘ , Π° ΠΊΠ°ΡΡΡΠΊΠ°, ΠΏΠ΅ΡΠ΅ΠΌΠ΅ΡΠ°Π΅ΠΌΠ°Ρ ΠΏΠΎ Π½Π°ΠΏΡΠ°Π²Π»Π΅Π½ΠΈΡ ΠΊ ΠΌΠ°Π³Π½ΠΈΡΡ, ΡΠΎΠ·Π΄Π°Π΅Ρ Π°Π½Π°Π»ΠΎΠ³ΠΈΡΠ½ΡΡ ΠΠΠ‘ .
- Π ΡΡΠΎΠΌ Π°ΡΠΎΠΌΠ΅ ΠΌΡ ΠΊΠΎΠ½ΡΠ΅Π½ΡΡΠΈΡΡΠ΅ΠΌΡΡ Π½Π° Π΄Π²ΠΈΠΆΠ΅Π½ΠΈΠ΅ Π² ΠΌΠ°Π³Π½ΠΈΡΠ½ΠΎΠΌ ΠΏΠΎΠ»Π΅, ΡΡΠ°ΡΠΈΠΎΠ½Π°ΡΠ½ΠΎΠΌ ΠΎΡΠ½ΠΎΡΠΈΡΠ΅Π»ΡΠ½ΠΎ ΠΠ΅ΠΌΠ»ΠΈ, ΠΏΡΠΎΠΈΠ·Π²ΠΎΠ΄Ρ ΡΠΎ, ΡΡΠΎ ΡΡΠ»ΠΎΠ²Π½ΠΎ Π½Π°Π·ΡΠ²Π°ΡΡ Π΄Π²ΠΈΠΆΡΡΠΈΠΌΡΡ ΠΠΠ‘ .
- (a) Π΄Π²ΠΈΠΆΡΡΠ°ΡΡΡ ΠΠΠ‘ =Bβv ΠΈΠ½Π΄ΡΡΠΈΡΡΠ΅ΡΡΡ ΠΌΠ΅ΠΆΠ΄Ρ ΡΠ΅Π»ΡΡΠ°ΠΌΠΈ, ΠΊΠΎΠ³Π΄Π° ΡΡΠΎΡ ΡΡΠ΅ΡΠΆΠ΅Π½Ρ Π΄Π²ΠΈΠΆΠ΅ΡΡΡ Π²ΠΏΡΠ°Π²ΠΎ Π² ΠΎΠ΄Π½ΠΎΡΠΎΠ΄Π½ΠΎΠΌ ΠΌΠ°Π³Π½ΠΈΡΠ½ΠΎΠΌ ΠΏΠΎΠ»Π΅.
ΠΠ΅Ρ Π°Π½ΠΈΡΠ΅ΡΠΊΠ°Ρ ΡΠ°Π±ΠΎΡΠ° ΠΈ ΡΠ»Π΅ΠΊΡΡΠΈΡΠ΅ΡΠΊΠ°Ρ ΡΠ½Π΅ΡΠ³ΠΈΡ
- ΠΠ΅Ρ Π°Π½ΠΈΡΠ΅ΡΠΊΠ°Ρ ΡΠ°Π±ΠΎΡΠ°, ΡΠΎΠ²Π΅ΡΡΠ°Π΅ΠΌΠ°Ρ Π²Π½Π΅ΡΠ½Π΅ΠΉ ΡΠΈΠ»ΠΎΠΉ Π΄Π»Ρ ΡΠΎΠ·Π΄Π°Π½ΠΈΡ Π΄Π²ΠΈΠΆΠ΅Π½ΠΈΡ ΠΠΠ‘ ΠΏΡΠ΅ΠΎΠ±ΡΠ°Π·ΡΠ΅ΡΡΡ Π² ΡΠ΅ΠΏΠ»ΠΎΠ²ΡΡ ΡΠ½Π΅ΡΠ³ΠΈΡ; Π² ΠΏΡΠΎΡΠ΅ΡΡΠ΅ ΡΠΎΡ ΡΠ°Π½ΡΠ΅ΡΡΡ ΡΠ½Π΅ΡΠ³ΠΈΡ.
- Π Motional EMF ΠΌΡ ΡΠ·Π½Π°Π»ΠΈ ΡΠ°Π½Π΅Π΅ (ΡΠΌ. Π½Π°Ρ ΠΡΠΎΠΌ Π½Π° « Motional EMF «).
- ΠΠ»Ρ ΠΏΡΠΎΡΡΠΎΠΉ ΡΡΡΠ°Π½ΠΎΠ²ΠΊΠΈ, ΠΏΠΎΠΊΠ°Π·Π°Π½Π½ΠΎΠΉ Π½ΠΈΠΆΠ΅, Π΄Π²ΠΈΠΆΡΡΠ°ΡΡΡ ΠΠΠ‘ $(\varepsilon)$, ΡΠΎΠ·Π΄Π°Π²Π°Π΅ΠΌΠ°Ρ Π΄Π²ΠΈΠΆΡΡΠΈΠΌΡΡ ΠΏΡΠΎΠ²ΠΎΠ΄Π½ΠΈΠΊΠΎΠΌ (Π² ΠΎΠ΄Π½ΠΎΡΠΎΠ΄Π½ΠΎΠΌ ΠΏΠΎΠ»Π΅), ΠΎΠΏΡΠ΅Π΄Π΅Π»ΡΠ΅ΡΡΡ ΡΠ»Π΅Π΄ΡΡΡΠΈΠΌ ΠΎΠ±ΡΠ°Π·ΠΎΠΌ:
- Π Π±ΠΎΠ»Π΅Π΅ ΠΎΠ±ΡΠ΅ΠΌ ΡΠΌΡΡΠ»Π΅ ΠΌΠ΅Ρ Π°Π½ΠΈΡΠ΅ΡΠΊΠ°Ρ ΡΠ°Π±ΠΎΡΠ°, ΡΠΎΠ²Π΅ΡΡΠ°Π΅ΠΌΠ°Ρ Π²Π½Π΅ΡΠ½Π΅ΠΉ ΡΠΈΠ»ΠΎΠΉ Π΄Π»Ρ ΠΏΡΠΎΠΈΠ·Π²ΠΎΠ΄ΡΡΠ²Π° Π΄Π²ΠΈΠΆΠ΅Π½ΠΈΡ ΠΠΠ‘ , ΠΏΡΠ΅ΠΎΠ±ΡΠ°Π·ΡΠ΅ΡΡΡ Π² ΡΠ΅ΠΏΠ»ΠΎΠ²ΡΡ ΡΠ½Π΅ΡΠ³ΠΈΡ.
- (Π°) A Motional ΠΠΠ‘ =Bβv ΠΈΠ½Π΄ΡΡΠΈΡΡΠ΅ΡΡΡ ΠΌΠ΅ΠΆΠ΄Ρ ΡΠ΅Π»ΡΡΠ°ΠΌΠΈ, ΠΊΠΎΠ³Π΄Π° ΡΡΠΎΡ ΡΡΠ΅ΡΠΆΠ΅Π½Ρ Π΄Π²ΠΈΠΆΠ΅ΡΡΡ Π²ΠΏΡΠ°Π²ΠΎ Π² ΠΎΠ΄Π½ΠΎΡΠΎΠ΄Π½ΠΎΠΌ ΠΌΠ°Π³Π½ΠΈΡΠ½ΠΎΠΌ ΠΏΠΎΠ»Π΅.
ΠΠ»Π΅ΠΊΡΡΠΎΠ³Π΅Π½Π΅ΡΠ°ΡΠΎΡΡ
- ΠΠ½ΠΈ ΠΈΠ½Π΄ΡΡΠΈΡΡΡΡ ΡΠ»Π΅ΠΊΡΡΠΎΠ΄Π²ΠΈΠΆΡΡΡΡ ΡΠΈΠ»Ρ ( ΠΠΠ‘ ), Π²ΡΠ°ΡΠ°Ρ ΠΊΠ°ΡΡΡΠΊΡ Π² ΠΌΠ°Π³Π½ΠΈΡΠ½ΠΎΠΌ ΠΏΠΎΠ»Π΅.
- Π’Π°ΠΊΠΈΠΌ ΠΎΠ±ΡΠ°Π·ΠΎΠΌ, ΠΌΡ ΠΌΠΎΠΆΠ΅ΠΌ Π½Π°ΠΉΡΠΈ ΠΈΠ½Π΄ΡΡΠΈΡΠΎΠ²Π°Π½Π½ΡΡ ΠΠΠ‘ , ΡΠ°ΡΡΠΌΠ°ΡΡΠΈΠ²Π°Ρ ΡΠΎΠ»ΡΠΊΠΎ Π±ΠΎΠΊΠΎΠ²ΡΠ΅ ΠΏΡΠΎΠ²ΠΎΠ΄Π°.
- ΠΠ²ΠΈΠΆΡΡΠ°Ρ ΠΠΠ‘ ΠΏΡΠΈΠ½ΠΈΠΌΠ°Π΅ΡΡΡ ΡΠ°Π²Π½ΠΎΠΉ ΠΠΠ‘ =Bβv, Π³Π΄Π΅ ΡΠΊΠΎΡΠΎΡΡΡ v ΠΏΠ΅ΡΠΏΠ΅Π½Π΄ΠΈΠΊΡΠ»ΡΡΠ½Π° ΠΌΠ°Π³Π½ΠΈΡΠ½ΠΎΠΌΡ ΠΏΠΎΠ»Ρ B (ΡΠΌ. Π½Π°Ρ ΠΡΠΎΠΌ Π½Π° « Motional EMF «).
- Π’Π°ΠΊΠΈΠΌ ΠΎΠ±ΡΠ°Π·ΠΎΠΌ, Π² ΡΡΠΎΠΌ ΡΠ»ΡΡΠ°Π΅ ΠΠΠ‘ , ΠΈΠ½Π΄ΡΡΠΈΡΠΎΠ²Π°Π½Π½Π°Ρ Ρ ΠΊΠ°ΠΆΠ΄ΠΎΠΉ ΡΡΠΎΡΠΎΠ½Ρ, ΡΠ°Π²Π½Π° ΠΠΠ‘ =BβvsinΞΈ, ΠΈ ΠΎΠ½ΠΈ ΠΈΠΌΠ΅ΡΡ ΠΎΠ΄ΠΈΠ½Π°ΠΊΠΎΠ²ΠΎΠ΅ Π½Π°ΠΏΡΠ°Π²Π»Π΅Π½ΠΈΠ΅.
- Π‘ΡΠΌΠΌΠ°ΡΠ½ΠΎΠ΅ ΠΠΠ‘ $\varepsilon$ Π²ΠΎΠΊΡΡΠ³ ΠΊΠΎΠ½ΡΡΡΠ° ΡΠΎΠ³Π΄Π°:
ΠΠ±ΡΠ°ΡΠ½Π°Ρ ΠΠΠ‘, Π²ΠΈΡ ΡΠ΅Π²ΡΠ΅ ΡΠΎΠΊΠΈ ΠΈ ΠΌΠ°Π³Π½ΠΈΡΠ½ΠΎΠ΅ Π΄Π΅ΠΌΠΏΡΠΈΡΠΎΠ²Π°Π½ΠΈΠ΅
- ΠΠ°Π·Π°Π΄ ΠΠΠ‘ , Π²ΠΈΡ ΡΠ΅Π²ΡΠ΅ ΡΠΎΠΊΠΈ ΠΈ ΠΌΠ°Π³Π½ΠΈΡΠ½ΠΎΠ΅ Π΄Π΅ΠΌΠΏΡΠΈΡΠΎΠ²Π°Π½ΠΈΠ΅ ΡΠ²Π»ΡΡΡΡΡ ΡΠ»Π΅Π΄ΡΡΠ²ΠΈΠ΅ΠΌ Π½Π°Π²Π΅Π΄Π΅Π½Π½ΠΎΠΉ ΠΠΠ‘ ΠΈ ΠΌΠΎΠ³ΡΡ Π±ΡΡΡ ΠΎΠ±ΡΡΡΠ½Π΅Π½Ρ Π·Π°ΠΊΠΎΠ½ΠΎΠΌ ΠΈΠ½Π΄ΡΠΊΡΠΈΠΈ Π€Π°ΡΠ°Π΄Π΅Ρ.
- ΠΠ°ΠΊΠΎΠ½ ΠΠ΅Π½ΡΠ° Π³ΠΎΠ²ΠΎΡΠΈΡ Π½Π°ΠΌ, ΡΡΠΎ ΠΈΠ½Π΄ΡΡΠΈΡΠΎΠ²Π°Π½Π½Π°Ρ ΠΠΠ‘ ΠΏΡΠΎΡΠΈΠ²ΠΎΠ΄Π΅ΠΉΡΡΠ²ΡΠ΅Ρ Π»ΡΠ±ΠΎΠΌΡ ΠΈΠ·ΠΌΠ΅Π½Π΅Π½ΠΈΡ, ΡΠ°ΠΊ ΡΡΠΎ Π²Ρ ΠΎΠ΄Π½ΠΎΠΉ ΠΠΠ‘ , ΠΏΠΈΡΠ°ΡΡΠ΅ΠΉ Π΄Π²ΠΈΠ³Π°ΡΠ΅Π»Ρ, Π±ΡΠ΄Π΅Ρ ΠΏΡΠΎΡΠΈΠ²ΠΎΠ΄Π΅ΠΉΡΡΠ²ΠΎΠ²Π°ΡΡ ΡΠ°ΠΌΠΎΠ³Π΅Π½Π΅ΡΠΈΡΡΠ΅ΠΌΠ°Ρ ΠΠΠ‘ Π΄Π²ΠΈΠ³Π°ΡΠ΅Π»Ρ, Π½Π°Π·ΡΠ²Π°Π΅ΠΌΠ°Ρ ΠΎΠ±ΡΠ°ΡΠ½ΠΎΠΉ ΠΠΠ‘ Π΄Π²ΠΈΠ³Π°ΡΠ΅Π»Ρ.
- ΠΠ°ΠΊ ΠΎΠ±ΡΡΠΆΠ΄Π°Π»ΠΎΡΡ Π² Β« Motional EMF Β», Motional EMF ΠΈΠ½Π΄ΡΡΠΈΡΡΠ΅ΡΡΡ, ΠΊΠΎΠ³Π΄Π° ΠΏΡΠΎΠ²ΠΎΠ΄Π½ΠΈΠΊ Π΄Π²ΠΈΠΆΠ΅ΡΡΡ Π² ΠΌΠ°Π³Π½ΠΈΡΠ½ΠΎΠΌ ΠΏΠΎΠ»Π΅ ΠΈΠ»ΠΈ ΠΊΠΎΠ³Π΄Π° ΠΌΠ°Π³Π½ΠΈΡΠ½ΠΎΠ΅ ΠΏΠΎΠ»Π΅ Π΄Π²ΠΈΠΆΠ΅ΡΡΡ ΠΎΡΠ½ΠΎΡΠΈΡΠ΅Π»ΡΠ½ΠΎ ΠΏΡΠΎΠ²ΠΎΠ΄Π½ΠΈΠΊΠ°.
- ΠΡΠ»ΠΈ ΠΏΠΎΠ΄Π²ΠΈΠΆΠ½ΡΠΉ ΠΠΠ‘ ΠΌΠΎΠΆΠ΅Ρ Π²ΡΠ·Π²Π°ΡΡ ΠΏΠ΅ΡΠ»Ρ ΡΠΎΠΊΠ° Π² ΠΏΡΠΎΠ²ΠΎΠ΄Π½ΠΈΠΊΠ΅, ΠΌΡ Π½Π°Π·ΡΠ²Π°Π΅ΠΌ ΡΡΠΎΡ ΡΠΎΠΊ Π²ΠΈΡ ΡΠ΅Π²ΡΠΌ ΡΠΎΠΊΠΎΠΌ.
- ΠΠ°Π³Π½ΠΈΡΠ½Π°Ρ ΡΠΈΠ»Π° Π½Π° ΡΠΎΠΊΠΎΠ²ΠΎΠΉ ΠΏΠ΅ΡΠ»Π΅ ΠΏΡΠΎΡΠΈΠ²ΠΎΠ΄Π΅ΠΉΡΡΠ²ΡΠ΅Ρ Π΄Π²ΠΈΠΆΠ΅Π½ΠΈΡ .
ΠΠ°ΡΡΠ΄ΠΊΠ° Π°ΠΊΠΊΡΠΌΡΠ»ΡΡΠΎΡΠ°: ΠΏΠΎΡΠ»Π΅Π΄ΠΎΠ²Π°ΡΠ΅Π»ΡΠ½ΠΎΠ΅ ΠΈ ΠΏΠ°ΡΠ°Π»Π»Π΅Π»ΡΠ½ΠΎΠ΅ ΠΠΠ‘
- ΠΠ±ΡΡΠ½ΠΎ ΡΠ»Π΅ΠΌΠ΅Π½ΡΡ ΡΠΎΠ΅Π΄ΠΈΠ½Π΅Π½Ρ ΠΏΠΎΡΠ»Π΅Π΄ΠΎΠ²Π°ΡΠ΅Π»ΡΠ½ΠΎ Π΄Π»Ρ ΠΏΠΎΠ»ΡΡΠ΅Π½ΠΈΡ Π±ΠΎΠ»ΡΡΠ΅ΠΉ ΠΎΠ±ΡΠ΅ΠΉ Ρ.Π΄.Ρ. .
- ΠΠΎ, Π΅ΡΠ»ΠΈ ΠΊΠ»Π΅ΡΠΊΠΈ ΠΏΡΠΎΡΠΈΠ²ΠΎΡΡΠΎΡΡ Π΄ΡΡΠ³ Π΄ΡΡΠ³Ρ, Π½Π°ΠΏΡΠΈΠΌΠ΅Ρ, Π΅ΡΠ»ΠΈ ΠΎΠ΄Π½Ρ ΠΈΠ· Π½ΠΈΡ ΠΏΠΎΠΌΠ΅ΡΡΠΈΡΡ Π² ΠΏΡΠΈΠ±ΠΎΡ Π·Π°Π΄ΠΎΠΌ Π½Π°ΠΏΠ΅ΡΡΠ΄, Π²ΡΠ΅Π³ΠΎ ΠΠΠ‘ ΠΌΠ΅Π½ΡΡΠ΅, ΡΠ°ΠΊ ΠΊΠ°ΠΊ ΡΡΠΎ Π°Π»Π³Π΅Π±ΡΠ°ΠΈΡΠ΅ΡΠΊΠ°Ρ ΡΡΠΌΠΌΠ° ΠΎΡΠ΄Π΅Π»ΡΠ½ΡΡ ΠΠΠ‘ .
- ΠΠΎΠ³Π΄Π° Π΄Π²Π° ΠΈΡΡΠΎΡΠ½ΠΈΠΊΠ° Π½Π°ΠΏΡΡΠΆΠ΅Π½ΠΈΡ Ρ ΠΎΠ΄ΠΈΠ½Π°ΠΊΠΎΠ²ΡΠΌΠΈ ΠΠΠ‘ ΡΠΎΠ΅Π΄ΠΈΠ½Π΅Π½Ρ ΠΏΠ°ΡΠ°Π»Π»Π΅Π»ΡΠ½ΠΎ, Π° ΡΠ°ΠΊΠΆΠ΅ ΠΏΠΎΠ΄ΠΊΠ»ΡΡΠ΅Π½Ρ ΠΊ ΡΠΎΠΏΡΠΎΡΠΈΠ²Π»Π΅Π½ΠΈΡ Π½Π°Π³ΡΡΠ·ΠΊΠΈ, ΠΎΠ±ΡΠ°Ρ ΠΠΠ‘ ΡΠ°ΠΊΠ°Ρ ΠΆΠ΅, ΠΊΠ°ΠΊ ΠΎΡΠ΄Π΅Π»ΡΠ½ΡΠ΅ ΠΠΠ‘ .
- ΠΠ°ΡΡΠ΄Π½ΠΎΠ΅ ΡΡΡΡΠΎΠΉΡΡΠ²ΠΎ Π΄ΠΎΠ»ΠΆΠ½ΠΎ ΠΈΠΌΠ΅ΡΡ Π±ΠΎΠ»ΡΡΡΡ Ρ.Π΄.Ρ. , ΡΠ΅ΠΌ Π°ΠΊΠΊΡΠΌΡΠ»ΡΡΠΎΡ, ΡΡΠΎΠ±Ρ ΠΏΡΠΎΡΠ΅ΠΊΠ°ΡΡΠΈΠΉ ΡΠ΅ΡΠ΅Π· Π½Π΅Π³ΠΎ ΠΎΠ±ΡΠ°ΡΠ½ΡΠΉ ΡΠΎΠΊ.
- ΠΠ²Π° ΠΈΡΡΠΎΡΠ½ΠΈΠΊΠ° Π½Π°ΠΏΡΡΠΆΠ΅Π½ΠΈΡ Ρ ΠΎΠ΄ΠΈΠ½Π°ΠΊΠΎΠ²ΡΠΌΠΈ ΠΠΠ‘ (ΠΎΠ±ΠΎΠ·Π½Π°ΡΠ΅Π½Ρ Π±ΡΠΊΠ²ΠΎΠΉ E), ΡΠΎΠ΅Π΄ΠΈΠ½Π΅Π½Π½ΡΠ΅ ΠΏΠ°ΡΠ°Π»Π»Π΅Π»ΡΠ½ΠΎ, Π΄Π°ΡΡ ΠΎΠ΄ΠΈΠ½Π°ΠΊΠΎΠ²ΡΠ΅ ΠΠΠ‘ , Π½ΠΎ ΠΈΠΌΠ΅ΡΡ ΠΌΠ΅Π½ΡΡΠ΅Π΅ ΠΎΠ±ΡΠ΅Π΅ Π²Π½ΡΡΡΠ΅Π½Π½Π΅Π΅ ΡΠΎΠΏΡΠΎΡΠΈΠ²Π»Π΅Π½ΠΈΠ΅, ΡΠ΅ΠΌ ΠΎΡΠ΄Π΅Π»ΡΠ½ΡΠ΅ ΠΈΡΡΠΎΡΠ½ΠΈΠΊΠΈ.
ΠΠ²ΡΠΊΠΎΠ²ΡΠ΅ ΡΠΈΡΡΠ΅ΠΌΡ, ΠΊΠΎΠΌΠΏΡΡΡΠ΅ΡΠ½Π°Ρ ΠΏΠ°ΠΌΡΡΡ, ΡΠ΅ΠΉΡΠΌΠΎΠ³ΡΠ°Ρ, GFCI
- ΠΠΈΠΊΡΠΎΡΠΎΠ½ ΡΠ°Π±ΠΎΡΠ°Π΅Ρ ΠΏΠΎ ΠΈΠ½Π΄ΡΠΊΡΠΈΠΈ, ΡΠ°ΠΊ ΠΊΠ°ΠΊ Π²ΠΈΠ±ΡΠΈΡΡΡΡΠ°Ρ ΠΌΠ΅ΠΌΠ±ΡΠ°Π½Π° Π½Π°Π²ΠΎΠ΄ΠΈΡ Π² ΠΊΠ°ΡΡΡΠΊΠ΅ ΠΠΠ‘ .
- Π’Π°ΠΊΠΈΠΌ ΠΎΠ±ΡΠ°Π·ΠΎΠΌ, ΠΊΠΎΠ»Π΅Π±Π°Π½ΠΈΡ ΡΠ»Π΅ΠΊΡΡΠΈΡΠ΅ΡΠΊΠΎΠ³ΠΎ ΡΠΎΠΊΠ°, ΠΏΡΠΎΡ ΠΎΠ΄ΡΡΠ΅Π³ΠΎ ΡΠ΅ΡΠ΅Π· Π΄ΠΈΠ½Π°ΠΌΠΈΠΊ, ΠΏΡΠ΅ΠΎΠ±ΡΠ°Π·ΡΡΡΡΡ Π² ΠΏΠ΅ΡΠ΅ΠΌΠ΅Π½Π½ΡΠ΅ ΠΌΠ°Π³Π½ΠΈΡΠ½ΡΠ΅ ΡΠΈΠ»Ρ, ΠΊΠΎΡΠΎΡΡΠ΅ ΠΏΠ΅ΡΠ΅ΠΌΠ΅ΡΠ°ΡΡ Π΄ΠΈΠ°ΡΡΠ°Π³ΠΌΡ Π΄ΠΈΠ½Π°ΠΌΠΈΠΊΠ°, Π·Π°ΡΡΠ°Π²Π»ΡΡ Π΄ΡΠ°ΠΉΠ²Π΅Ρ ΠΏΡΠΎΠΈΠ·Π²ΠΎΠ΄ΠΈΡΡ Π΄Π²ΠΈΠΆΠ΅Π½ΠΈΠ΅ Π²ΠΎΠ·Π΄ΡΡ Π° , ΠΊΠΎΡΠΎΡΡΠΉ Π°Π½Π°Π»ΠΎΠ³ΠΈΡΠ΅Π½ ΠΈΡΡ ΠΎΠ΄Π½ΠΎΠΌΡ ΡΠΈΠ³Π½Π°Π»Ρ ΡΡΠΈΠ»ΠΈΡΠ΅Π»Ρ.
- ΠΡΠ»ΠΈ GFCI ΠΎΠ±Π½Π°ΡΡΠΆΠΈΠ²Π°Π΅Ρ ΡΡΠ΅ΡΠΊΡ ΡΠΎΠΊΠ°, ΠΎΠ½ ΡΠΎΠ·Π΄Π°Π΅Ρ ΠΠΠ‘ ΠΈ ΡΠΎΠΊ Π² Π½Π°ΠΏΡΠ°Π²Π»Π΅Π½ΠΈΠΈ, ΠΏΡΠΎΡΠΈΠ²ΠΎΠΏΠΎΠ»ΠΎΠΆΠ½ΠΎΠΌ ΠΈΡΡ ΠΎΠ΄Π½ΠΎΠΌΡ ΡΠΎΠΊΡ.
ΠΠ°ΠΊΠΎΠ½ ΠΈΠ½Π΄ΡΠΊΡΠΈΠΈ Π€Π°ΡΠ°Π΄Π΅Ρ ΠΈ Π·Π°ΠΊΠΎΠ½ ΠΠ΅Π½ΡΠ°
- ΠΠ°ΠΊΠΎΠ½ ΠΈΠ½Π΄ΡΠΊΡΠΈΠΈ Π€Π°ΡΠ°Π΄Π΅Ρ Π³Π»Π°ΡΠΈΡ, ΡΡΠΎ ΠΠΠ‘ , ΠΈΠ½Π΄ΡΡΠΈΡΠΎΠ²Π°Π½Π½Π°Ρ ΠΈΠ·ΠΌΠ΅Π½Π΅Π½ΠΈΠ΅ΠΌ ΠΌΠ°Π³Π½ΠΈΡΠ½ΠΎΠ³ΠΎ ΠΏΠΎΡΠΎΠΊΠ°, ΡΠ°Π²Π½Π° $ ΠΠΠ‘ = -N\frac{\Delta \Phi}{\Delta t}$, ΠΊΠΎΠ³Π΄Π° ΠΏΠΎΡΠΎΠΊ ΠΈΠ·ΠΌΠ΅Π½ΡΠ΅ΡΡΡ Π½Π° Ξ Π·Π° Π²ΡΠ΅ΠΌΡ Ξt.
- ΠΠΎ-ΠΏΠ΅ΡΠ²ΡΡ , ΠΠΠ‘ ΠΏΡΡΠΌΠΎ ΠΏΡΠΎΠΏΠΎΡΡΠΈΠΎΠ½Π°Π»ΡΠ½Π° ΠΈΠ·ΠΌΠ΅Π½Π΅Π½ΠΈΡ ΠΏΠΎΡΠΎΠΊΠ° Ξ.
- Π‘Π΅ΠΊΡΠ½Π΄Π°, ΠΠΠ‘ ΠΌΠ°ΠΊΡΠΈΠΌΠ°Π»ΡΠ½Π°, ΠΊΠΎΠ³Π΄Π° ΠΈΠ·ΠΌΠ΅Π½Π΅Π½ΠΈΠ΅ Π²ΠΎ Π²ΡΠ΅ΠΌΠ΅Π½ΠΈ Ξt Π½Π°ΠΈΠΌΠ΅Π½ΡΡΠ΅Π΅, ΡΠΎ Π΅ΡΡΡ ΠΠΠ‘ ΠΎΠ±ΡΠ°ΡΠ½ΠΎ ΠΏΡΠΎΠΏΠΎΡΡΠΈΠΎΠ½Π°Π»ΡΠ½Π° Ξt.
- ΠΠ°ΠΊΠΎΠ½Π΅Ρ, Π΅ΡΠ»ΠΈ ΠΊΠ°ΡΡΡΠΊΠ° ΠΈΠΌΠ΅Π΅Ρ N Π²ΠΈΡΠΊΠΎΠ², Π±ΡΠ΄Π΅Ρ ΠΏΡΠΎΠΈΠ·Π²Π΅Π΄Π΅Π½Π° ΠΠΠ‘ , ΠΊΠΎΡΠΎΡΠ°Ρ Π² N ΡΠ°Π· Π±ΠΎΠ»ΡΡΠ΅, ΡΠ΅ΠΌ Π΄Π»Ρ ΠΎΠ΄ΠΈΠ½ΠΎΡΠ½ΠΎΠΉ ΠΊΠ°ΡΡΡΠΊΠΈ, ΡΠ°ΠΊ ΡΡΠΎ ΠΠΠ‘ ΠΏΡΡΠΌΠΎ ΠΏΡΠΎΠΏΠΎΡΡΠΈΠΎΠ½Π°Π»ΡΠ½Π° N.
- ΠΠ΄ΠΈΠ½ΠΈΡΠ°ΠΌΠΈ Π΄Π»Ρ EMF , ΠΊΠ°ΠΊ ΠΎΠ±ΡΡΠ½ΠΎ, ΡΠ²Π»ΡΡΡΡΡ Π²ΠΎΠ»ΡΡΡ.
ΠΠ½Π΄ΡΠΊΡΠΈΠ²Π½ΠΎΡΡΡ
- ΠΠ½Π΄ΡΠΊΡΠΈΡ β ΡΡΠΎ ΠΏΡΠΎΡΠ΅ΡΡ, ΠΏΡΠΈ ΠΊΠΎΡΠΎΡΠΎΠΌ ΠΠΠ‘ ΠΈΠ½Π΄ΡΡΠΈΡΡΠ΅ΡΡΡ ΠΈΠ·ΠΌΠ΅Π½Π΅Π½ΠΈΠ΅ΠΌ ΠΌΠ°Π³Π½ΠΈΡΠ½ΠΎΠ³ΠΎ ΠΏΠΎΡΠΎΠΊΠ°.
- Π‘ΠΌ. , Π³Π΄Π΅ ΠΏΡΠΎΡΡΡΠ΅ ΠΊΠ°ΡΡΡΠΊΠΈ ΠΈΠ½Π΄ΡΡΠΈΡΡΡΡ Π΄ΡΡΠ³ Π² Π΄ΡΡΠ³Π΅ ΠΠΠ‘ .
- ΠΡΠΈ ΠΊΠ°ΡΡΡΠΊΠΈ ΠΌΠΎΠ³ΡΡ ΠΈΠ½Π΄ΡΡΠΈΡΠΎΠ²Π°ΡΡ ΠΠΠ‘ Π΄ΡΡΠ³ Π² Π΄ΡΡΠ³Π΅, ΠΊΠ°ΠΊ Π½Π΅ΡΡΡΠ΅ΠΊΡΠΈΠ²Π½ΡΠΉ ΡΡΠ°Π½ΡΡΠΎΡΠΌΠ°ΡΠΎΡ.
- ΠΠ΄Π΅ΡΡ Π²ΠΈΠ΄Π½ΠΎ, ΡΡΠΎ ΠΈΠ·ΠΌΠ΅Π½Π΅Π½ΠΈΠ΅ ΡΠΎΠΊΠ° Π² ΠΊΠ°ΡΡΡΠΊΠ΅ 1 ΠΈΠ½Π΄ΡΡΠΈΡΡΠ΅Ρ ΠΠΠ‘ Π² ΠΊΠ°ΡΡΡΠΊΠ΅ 2.
- (ΠΠ±ΡΠ°ΡΠΈΡΠ΅ Π²Π½ΠΈΠΌΠ°Π½ΠΈΠ΅, ΡΡΠΎ Β«ΠΈΠ½Π΄ΡΡΠΈΡΡΠ΅ΠΌΠ°Ρ E2Β» ΠΏΡΠ΅Π΄ΡΡΠ°Π²Π»ΡΠ΅Ρ ΡΠΎΠ±ΠΎΠΉ ΠΈΠ½Π΄ΡΡΠΈΡΠΎΠ²Π°Π½Π½ΡΡ ΠΠΠ‘ Π² ΠΊΠ°ΡΡΡΠΊΠ΅ 2. )
ΠΡΡΠΎΡΠ½ΠΈΠΊΠΈ ΠΠΠ
- Π‘ΠΎΠ·Π΄Π°Π²Π°Π΅ΠΌΠ°Ρ ΡΠ°Π·Π½ΠΎΡΡΡ ΡΠ»Π΅ΠΊΡΡΠΈΡΠ΅ΡΠΊΠΈΡ ΠΏΠΎΡΠ΅Π½ΡΠΈΠ°Π»ΠΎΠ² ΡΠΏΡΠ°Π²Π»ΡΠ΅Ρ ΠΏΡΠΎΡΠ΅ΠΊΠ°Π½ΠΈΠ΅ΠΌ ΡΠΎΠΊΠ°, Π΅ΡΠ»ΠΈ ΡΠ΅ΠΏΡ ΠΏΠΎΠ΄ΠΊΠ»ΡΡΠ΅Π½Π° ΠΊ ΠΈΡΡΠΎΡΠ½ΠΈΠΊΡ ΠΠΠ‘ .
- Π‘Π½ΠΎΠ²Π° ΠΠΠ‘ ΠΏΡΠΎΡΠΈΠ²ΠΎΡΡΠΎΠΈΡ ΡΠ»Π΅ΠΊΡΡΠΈΡΠ΅ΡΠΊΠΎΠΌΡ Π½Π°ΠΏΡΡΠΆΠ΅Π½ΠΈΡ ΠΈΠ·-Π·Π° ΡΠ°Π·Π΄Π΅Π»Π΅Π½ΠΈΡ Π·Π°ΡΡΠ΄Π°.
- ΠΠ±ΡΠΈΠΉ ΠΏΡΠΈΠ½ΡΠΈΠΏ, ΡΠ΅Π³ΡΠ»ΠΈΡΡΡΡΠΈΠΉ ΠΠΠ‘ Π² ΡΠ°ΠΊΠΈΡ ΡΠ»Π΅ΠΊΡΡΠΈΡΠ΅ΡΠΊΠΈΡ ΠΌΠ°ΡΠΈΠ½Π°Ρ , β ΡΡΠΎ Π·Π°ΠΊΠΎΠ½ ΠΈΠ½Π΄ΡΠΊΡΠΈΠΈ Π€Π°ΡΠ°Π΄Π΅Ρ.
- Π ΠΏΡΠΈΡΠΎΠ΄Π΅ ΠΠΠ‘ Π³Π΅Π½Π΅ΡΠΈΡΡΠ΅ΡΡΡ Π²ΡΡΠΊΠΈΠΉ ΡΠ°Π·, ΠΊΠΎΠ³Π΄Π° ΡΠ»ΡΠΊΡΡΠ°ΡΠΈΠΈ ΠΌΠ°Π³Π½ΠΈΡΠ½ΠΎΠ³ΠΎ ΠΏΠΎΠ»Ρ ΠΏΡΠΎΡ ΠΎΠ΄ΡΡ ΡΠ΅ΡΠ΅Π· ΠΏΠΎΠ²Π΅ΡΡ Π½ΠΎΡΡΡ.
- ΠΡΡ ΠΎΠ΄Π½ΠΎΠ΅ Π½Π°ΠΏΡΡΠΆΠ΅Π½ΠΈΠ΅ ΠΊΠ°ΠΆΠ΄ΠΎΠ³ΠΎ Π·Π°Π²ΠΈΡΠΈΡ ΠΎΡ Π΅Π³ΠΎ ΠΊΠΎΠ½ΡΡΡΡΠΊΡΠΈΠΈ ΠΈ Π½Π°Π³ΡΡΠ·ΠΊΠΈ ΠΈ ΡΠ°Π²Π½ΡΠ΅ΡΡΡ Ρ.Π΄.Ρ. ΡΠΎΠ»ΡΠΊΠΎ ΠΏΡΠΈ ΠΎΡΡΡΡΡΡΠ²ΠΈΠΈ Π½Π°Π³ΡΡΠ·ΠΊΠΈ.
ΠΠΠ‘ ΠΈΠ½Π΄ΡΠΊΡΠΈΠΈ ΠΈ ΠΌΠ°Π³Π½ΠΈΡΠ½ΡΠΉ ΠΏΠΎΡΠΎΠΊ
Π¦Π΅Π»ΠΈ ΠΎΠ±ΡΡΠ΅Π½ΠΈΡ
Π ΠΊΠΎΠ½ΡΡ ΡΡΠΎΠ³ΠΎ ΡΠ°Π·Π΄Π΅Π»Π° Π²Ρ ΡΠΌΠΎΠΆΠ΅ΡΠ΅:
- Π Π°ΡΡΡΠΈΡΠ°ΡΡ ΠΏΠΎΡΠΎΠΊ ΠΎΠ΄Π½ΠΎΡΠΎΠ΄Π½ΠΎΠ³ΠΎ ΠΌΠ°Π³Π½ΠΈΡΠ½ΠΎΠ³ΠΎ ΠΏΠΎΠ»Ρ ΡΠ΅ΡΠ΅Π· ΠΏΠ΅ΡΠ»Ρ ΠΏΡΠΎΠΈΠ·Π²ΠΎΠ»ΡΠ½ΠΎΠΉ ΠΎΡΠΈΠ΅Π½ΡΠ°ΡΠΈΠΈ.
- ΠΠΏΠΈΡΠ°ΡΡ ΠΌΠ΅ΡΠΎΠ΄Ρ ΡΠΎΠ·Π΄Π°Π½ΠΈΡ ΡΠ»Π΅ΠΊΡΡΠΎΠ΄Π²ΠΈΠΆΡΡΠ΅ΠΉ ΡΠΈΠ»Ρ (ΠΠΠ‘) Ρ ΠΏΠΎΠΌΠΎΡΡΡ ΠΌΠ°Π³Π½ΠΈΡΠ½ΠΎΠ³ΠΎ ΠΏΠΎΠ»Ρ ΠΈΠ»ΠΈ ΠΌΠ°Π³Π½ΠΈΡΠ° ΠΈ ΠΏΡΠΎΠ²ΠΎΠ»ΠΎΡΠ½ΠΎΠΉ ΠΏΠ΅ΡΠ»ΠΈ.
Π£ΡΡΡΠΎΠΉΡΡΠ²ΠΎ, ΠΈΡΠΏΠΎΠ»ΡΠ·ΠΎΠ²Π°Π½Π½ΠΎΠ΅ Π€Π°ΡΠ°Π΄Π΅Π΅ΠΌ Π΄Π»Ρ Π΄Π΅ΠΌΠΎΠ½ΡΡΡΠ°ΡΠΈΠΈ ΡΠΎΠ³ΠΎ, ΡΡΠΎ ΠΌΠ°Π³Π½ΠΈΡΠ½ΡΠ΅ ΠΏΠΎΠ»Ρ ΠΌΠΎΠ³ΡΡ ΡΠΎΠ·Π΄Π°Π²Π°ΡΡ ΡΠΎΠΊΠΈ, ΠΏΠΎΠΊΠ°Π·Π°Π½ΠΎ Π½Π° ΡΠΈΡ. 1. ΠΠΎΠ³Π΄Π° ΠΏΠ΅ΡΠ΅ΠΊΠ»ΡΡΠ°ΡΠ΅Π»Ρ Π·Π°ΠΌΠΊΠ½ΡΡ, Π² ΠΊΠ°ΡΡΡΠΊΠ΅ Π² Π²Π΅ΡΡ Π½Π΅ΠΉ ΡΠ°ΡΡΠΈ ΠΆΠ΅Π»Π΅Π·Π½ΠΎΠ³ΠΎ ΠΊΠΎΠ»ΡΡΠ° ΡΠΎΠ·Π΄Π°Π΅ΡΡΡ ΠΌΠ°Π³Π½ΠΈΡΠ½ΠΎΠ΅ ΠΏΠΎΠ»Π΅, ΠΊΠΎΡΠΎΡΠΎΠ΅ ΠΏΠ΅ΡΠ΅Π΄Π°Π΅ΡΡΡ Π½Π° ΠΊΠ°ΡΡΡΠΊΡ Π² Π²Π΅ΡΡ Π½Π΅ΠΉ ΡΠ°ΡΡΠΈ ΠΆΠ΅Π»Π΅Π·Π½ΠΎΠ³ΠΎ ΠΊΠΎΠ»ΡΡΠ°. Π½ΠΈΠΆΠ½ΡΡ ΡΠ°ΡΡΡ ΠΊΠΎΠ»ΡΡΠ°. ΠΠ°Π»ΡΠ²Π°Π½ΠΎΠΌΠ΅ΡΡ ΠΈΡΠΏΠΎΠ»ΡΠ·ΡΠ΅ΡΡΡ Π΄Π»Ρ ΠΎΠ±Π½Π°ΡΡΠΆΠ΅Π½ΠΈΡ Π»ΡΠ±ΠΎΠ³ΠΎ ΡΠΎΠΊΠ°, Π½Π°Π²Π΅Π΄Π΅Π½Π½ΠΎΠ³ΠΎ Π² ΠΊΠ°ΡΡΡΠΊΠ΅ Π½Π° Π΄Π½Π΅. ΠΡΠ»ΠΎ ΠΎΠ±Π½Π°ΡΡΠΆΠ΅Π½ΠΎ, ΡΡΠΎ ΠΊΠ°ΠΆΠ΄ΡΠΉ ΡΠ°Π·, ΠΊΠΎΠ³Π΄Π° ΠΏΠ΅ΡΠ΅ΠΊΠ»ΡΡΠ°ΡΠ΅Π»Ρ Π·Π°ΠΌΡΠΊΠ°Π΅ΡΡΡ, Π³Π°Π»ΡΠ²Π°Π½ΠΎΠΌΠ΅ΡΡ ΡΠ΅Π³ΠΈΡΡΡΠΈΡΡΠ΅Ρ ΡΠΎΠΊ Π² ΠΎΠ΄Π½ΠΎΠΌ Π½Π°ΠΏΡΠ°Π²Π»Π΅Π½ΠΈΠΈ Π² ΠΊΠ°ΡΡΡΠΊΠ΅ Π½Π° Π΄Π½Π΅. (ΠΡ ΡΠ°ΠΊΠΆΠ΅ ΠΌΠΎΠΆΠ΅ΡΠ΅ Π½Π°Π±Π»ΡΠ΄Π°ΡΡ ΡΡΠΎ Π² ΡΠΈΠ·ΠΈΡΠ΅ΡΠΊΠΎΠΉ Π»Π°Π±ΠΎΡΠ°ΡΠΎΡΠΈΠΈ.) ΠΠ°ΠΆΠ΄ΡΠΉ ΡΠ°Π·, ΠΊΠΎΠ³Π΄Π° ΠΏΠ΅ΡΠ΅ΠΊΠ»ΡΡΠ°ΡΠ΅Π»Ρ ΡΠ°Π·ΠΌΡΠΊΠ°Π΅ΡΡΡ, Π³Π°Π»ΡΠ²Π°Π½ΠΎΠΌΠ΅ΡΡ ΠΎΠ±Π½Π°ΡΡΠΆΠΈΠ²Π°Π΅Ρ ΡΠΎΠΊ Π² ΠΏΡΠΎΡΠΈΠ²ΠΎΠΏΠΎΠ»ΠΎΠΆΠ½ΠΎΠΌ Π½Π°ΠΏΡΠ°Π²Π»Π΅Π½ΠΈΠΈ. ΠΠ½ΡΠ΅ΡΠ΅ΡΠ½ΠΎ, ΡΡΠΎ Π΅ΡΠ»ΠΈ ΠΏΠ΅ΡΠ΅ΠΊΠ»ΡΡΠ°ΡΠ΅Π»Ρ ΠΎΡΡΠ°Π΅ΡΡΡ Π·Π°ΠΌΠΊΠ½ΡΡΡΠΌ ΠΈΠ»ΠΈ ΡΠ°Π·ΠΎΠΌΠΊΠ½ΡΡΡΠΌ ΠΊΠ°ΠΊΠΎΠ΅-ΡΠΎ Π²ΡΠ΅ΠΌΡ, ΡΠΎΠΊ ΡΠ΅ΡΠ΅Π· Π³Π°Π»ΡΠ²Π°Π½ΠΎΠΌΠ΅ΡΡ ΠΎΡΡΡΡΡΡΠ²ΡΠ΅Ρ. ΠΠ°ΠΌΡΠΊΠ°Π½ΠΈΠ΅ ΠΈ ΡΠ°Π·ΠΌΡΠΊΠ°Π½ΠΈΠ΅ ΠΏΠ΅ΡΠ΅ΠΊΠ»ΡΡΠ°ΡΠ΅Π»Ρ ΠΈΠ½Π΄ΡΡΠΈΡΡΠ΅Ρ ΡΠΎΠΊ. ΠΠΌΠ΅Π½Π½ΠΎ ΠΈΠ·ΠΌΠ΅Π½Π΅Π½ΠΈΠ΅ Π² ΠΌΠ°Π³Π½ΠΈΡΠ½ΠΎΠΌ ΠΏΠΎΠ»Π΅ ΡΠΎΠ·Π΄Π°Π΅Ρ ΡΠΎΠΊ. ΠΠΎΠ»Π΅Π΅ Π²Π°ΠΆΠ½ΡΠΌ, ΡΠ΅ΠΌ ΡΠ΅ΠΊΡΡΠΈΠΉ ΡΠΎΠΊ, ΡΠ²Π»ΡΠ΅ΡΡΡ ΠΠΠ‘, ΠΊΠΎΡΠΎΡΠ°Ρ Π΅Π³ΠΎ Π²ΡΠ·ΡΠ²Π°Π΅Ρ. Π’ΠΎΠΊ ΡΠ²Π»ΡΠ΅ΡΡΡ ΡΠ΅Π·ΡΠ»ΡΡΠ°ΡΠΎΠΌ ΠΠΠ‘ , ΠΈΠ½Π΄ΡΡΠΈΡΠΎΠ²Π°Π½Π½ΠΎΠΉ ΠΈΠ·ΠΌΠ΅Π½ΡΡΡΠΈΠΌΡΡ ΠΌΠ°Π³Π½ΠΈΡΠ½ΡΠΌ ΠΏΠΎΠ»Π΅ΠΌ , Π½Π΅Π·Π°Π²ΠΈΡΠΈΠΌΠΎ ΠΎΡ ΡΠΎΠ³ΠΎ, Π΅ΡΡΡ Π»ΠΈ ΠΏΡΡΡ Π΄Π»Ρ ΠΏΡΠΎΡΠ΅ΠΊΠ°Π½ΠΈΡ ΡΠΎΠΊΠ°.
Π ΠΈΡ. 1. ΠΠΏΠΏΠ°ΡΠ°Ρ Π€Π°ΡΠ°Π΄Π΅Ρ Π΄Π»Ρ Π΄Π΅ΠΌΠΎΠ½ΡΡΡΠ°ΡΠΈΠΈ ΡΠΎΠ³ΠΎ, ΡΡΠΎ ΠΌΠ°Π³Π½ΠΈΡΠ½ΠΎΠ΅ ΠΏΠΎΠ»Π΅ ΠΌΠΎΠΆΠ΅Ρ ΠΏΡΠΎΠΈΠ·Π²ΠΎΠ΄ΠΈΡΡ ΡΠΎΠΊ. ΠΠ·ΠΌΠ΅Π½Π΅Π½ΠΈΠ΅ ΠΏΠΎΠ»Ρ, ΡΠΎΠ·Π΄Π°Π²Π°Π΅ΠΌΠΎΠ³ΠΎ Π²Π΅ΡΡ Π½Π΅ΠΉ ΠΊΠ°ΡΡΡΠΊΠΎΠΉ, ΠΈΠ½Π΄ΡΡΠΈΡΡΠ΅Ρ ΠΠΠ‘ ΠΈ, ΡΠ»Π΅Π΄ΠΎΠ²Π°ΡΠ΅Π»ΡΠ½ΠΎ, ΡΠΎΠΊ Π² Π½ΠΈΠΆΠ½Π΅ΠΉ ΠΊΠ°ΡΡΡΠΊΠ΅. ΠΠΎΠ³Π΄Π° ΠΏΠ΅ΡΠ΅ΠΊΠ»ΡΡΠ°ΡΠ΅Π»Ρ ΡΠ°Π·ΠΌΡΠΊΠ°Π΅ΡΡΡ ΠΈ Π·Π°ΠΌΡΠΊΠ°Π΅ΡΡΡ, Π³Π°Π»ΡΠ²Π°Π½ΠΎΠΌΠ΅ΡΡ ΡΠ΅Π³ΠΈΡΡΡΠΈΡΡΠ΅Ρ ΡΠΎΠΊΠΈ Π² ΠΏΡΠΎΡΠΈΠ²ΠΎΠΏΠΎΠ»ΠΎΠΆΠ½ΡΡ Π½Π°ΠΏΡΠ°Π²Π»Π΅Π½ΠΈΡΡ . Π§Π΅ΡΠ΅Π· Π³Π°Π»ΡΠ²Π°Π½ΠΎΠΌΠ΅ΡΡ Π½Π΅ ΠΏΡΠΎΡΠ΅ΠΊΠ°Π΅Ρ ΡΠΎΠΊ, ΠΊΠΎΠ³Π΄Π° ΠΏΠ΅ΡΠ΅ΠΊΠ»ΡΡΠ°ΡΠ΅Π»Ρ ΠΎΡΡΠ°Π΅ΡΡΡ Π·Π°ΠΌΠΊΠ½ΡΡΡΠΌ ΠΈΠ»ΠΈ ΡΠ°Π·ΠΎΠΌΠΊΠ½ΡΡΡΠΌ.
ΠΠΊΡΠΏΠ΅ΡΠΈΠΌΠ΅Π½Ρ, ΠΊΠΎΡΠΎΡΡΠΉ Π»Π΅Π³ΠΊΠΎ ΠΏΡΠΎΠ²Π΅ΡΡΠΈ ΠΈ ΠΊΠΎΡΠΎΡΡΠΉ ΡΠ°ΡΡΠΎ ΠΏΡΠΎΠ²ΠΎΠ΄ΡΡ Π² ΡΠΈΠ·ΠΈΡΠ΅ΡΠΊΠΈΡ Π»Π°Π±ΠΎΡΠ°ΡΠΎΡΠΈΡΡ , ΠΏΠΎΠΊΠ°Π·Π°Π½ Π½Π° ΡΠΈΡ. 2. ΠΠΠ‘ ΠΈΠ½Π΄ΡΡΠΈΡΡΠ΅ΡΡΡ Π² ΠΊΠ°ΡΡΡΠΊΠ΅, ΠΊΠΎΠ³Π΄Π° ΡΡΠ΅ΡΠΆΠ½Π΅Π²ΠΎΠΉ ΠΌΠ°Π³Π½ΠΈΡ Π²ΡΠ°Π»ΠΊΠΈΠ²Π°Π΅ΡΡΡ Π² Π½Π΅Π΅ ΠΈ Π²ΡΠ΄Π²ΠΈΠ³Π°Π΅ΡΡΡ ΠΈΠ· Π½Π΅Π΅. ΠΠΠ‘ ΡΠ°Π·Π½ΡΡ Π·Π½Π°ΠΊΠΎΠ² ΡΠΎΠ·Π΄Π°ΡΡΡΡ Π΄Π²ΠΈΠΆΠ΅Π½ΠΈΠ΅ΠΌ Π² ΠΏΡΠΎΡΠΈΠ²ΠΎΠΏΠΎΠ»ΠΎΠΆΠ½ΡΡ Π½Π°ΠΏΡΠ°Π²Π»Π΅Π½ΠΈΡΡ , Π° ΡΠ°ΠΊΠΆΠ΅ ΠΈΠ·ΠΌΠ΅Π½Π΅Π½ΠΈΠ΅ΠΌ ΠΏΠΎΠ»ΡΡΠ½ΠΎΡΡΠΈ ΠΠΠ‘ Π½Π° ΠΏΡΠΎΡΠΈΠ²ΠΎΠΏΠΎΠ»ΠΎΠΆΠ½ΠΎΠ΅. Π’Π΅ ΠΆΠ΅ ΡΠ΅Π·ΡΠ»ΡΡΠ°ΡΡ ΠΏΠΎΠ»ΡΡΠ°ΡΡΡΡ, Π΅ΡΠ»ΠΈ ΠΏΠ΅ΡΠ΅ΠΌΠ΅ΡΠ°ΡΡ ΠΊΠ°ΡΡΡΠΊΡ, Π° Π½Π΅ ΠΌΠ°Π³Π½ΠΈΡ β Π²Π°ΠΆΠ½ΠΎ ΠΎΡΠ½ΠΎΡΠΈΡΠ΅Π»ΡΠ½ΠΎΠ΅ Π΄Π²ΠΈΠΆΠ΅Π½ΠΈΠ΅. Π§Π΅ΠΌ Π±ΡΡΡΡΠ΅Π΅ Π΄Π²ΠΈΠΆΠ΅Π½ΠΈΠ΅, ΡΠ΅ΠΌ Π±ΠΎΠ»ΡΡΠ΅ ΠΠΠ‘, Π° ΠΊΠΎΠ³Π΄Π° ΠΌΠ°Π³Π½ΠΈΡ Π½Π΅ΠΏΠΎΠ΄Π²ΠΈΠΆΠ΅Π½ ΠΎΡΠ½ΠΎΡΠΈΡΠ΅Π»ΡΠ½ΠΎ ΠΊΠ°ΡΡΡΠΊΠΈ, ΠΠΠ‘ ΠΎΡΡΡΡΡΡΠ²ΡΠ΅Ρ.
Π ΠΈΡ. 2. ΠΠ²ΠΈΠΆΠ΅Π½ΠΈΠ΅ ΠΌΠ°Π³Π½ΠΈΡΠ° ΠΎΡΠ½ΠΎΡΠΈΡΠ΅Π»ΡΠ½ΠΎ ΠΊΠ°ΡΡΡΠΊΠΈ ΡΠΎΠ·Π΄Π°Π΅Ρ ΠΠΠ‘, ΠΊΠ°ΠΊ ΠΏΠΎΠΊΠ°Π·Π°Π½ΠΎ Π½Π° ΡΠΈΡΡΠ½ΠΊΠ΅. Π’Π°ΠΊΠΈΠ΅ ΠΆΠ΅ ΠΠΠ‘ Π²ΠΎΠ·Π½ΠΈΠΊΠ°ΡΡ, Π΅ΡΠ»ΠΈ ΠΊΠ°ΡΡΡΠΊΡ ΠΏΠ΅ΡΠ΅ΠΌΠ΅ΡΠ°ΡΡ ΠΎΡΠ½ΠΎΡΠΈΡΠ΅Π»ΡΠ½ΠΎ ΠΌΠ°Π³Π½ΠΈΡΠ°. Π§Π΅ΠΌ Π±ΠΎΠ»ΡΡΠ΅ ΡΠΊΠΎΡΠΎΡΡΡ, ΡΠ΅ΠΌ Π±ΠΎΠ»ΡΡΠ΅ Π²Π΅Π»ΠΈΡΠΈΠ½Π° ΠΠΠ‘, Π° ΠΠΠ‘ ΡΠ°Π²Π½Π° Π½ΡΠ»Ρ, ΠΊΠΎΠ³Π΄Π° Π½Π΅Ρ Π΄Π²ΠΈΠΆΠ΅Π½ΠΈΡ.
ΠΠ΅ΡΠΎΠ΄ ΠΈΠ½Π΄ΡΠΊΡΠΈΠΈ ΠΠΠ‘, ΠΈΡΠΏΠΎΠ»ΡΠ·ΡΠ΅ΠΌΡΠΉ Π² Π±ΠΎΠ»ΡΡΠΈΠ½ΡΡΠ²Π΅ ΡΠ»Π΅ΠΊΡΡΠΈΡΠ΅ΡΠΊΠΈΡ Π³Π΅Π½Π΅ΡΠ°ΡΠΎΡΠΎΠ², ΠΏΠΎΠΊΠ°Π·Π°Π½ Π½Π° ΡΠΈΡΡΠ½ΠΊΠ΅ 3. ΠΠ°ΡΡΡΠΊΠ° Π²ΡΠ°ΡΠ°Π΅ΡΡΡ Π² ΠΌΠ°Π³Π½ΠΈΡΠ½ΠΎΠΌ ΠΏΠΎΠ»Π΅, ΡΠΎΠ·Π΄Π°Π²Π°Ρ ΠΠΠ‘ ΠΏΠ΅ΡΠ΅ΠΌΠ΅Π½Π½ΠΎΠ³ΠΎ ΡΠΎΠΊΠ°, ΠΊΠΎΡΠΎΡΠ°Ρ Π·Π°Π²ΠΈΡΠΈΡ ΠΎΡ ΡΠΊΠΎΡΠΎΡΡΠΈ Π²ΡΠ°ΡΠ΅Π½ΠΈΡ ΠΈ Π΄ΡΡΠ³ΠΈΡ ΡΠ°ΠΊΡΠΎΡΠΎΠ², ΠΊΠΎΡΠΎΡΡΠ΅ Π±ΡΠ΄ΡΡ ΡΠ°ΡΡΠΌΠΎΡΡΠ΅Π½Ρ Π² ΠΏΠΎΡΠ»Π΅Π΄ΡΡΡΠΈΡ ΡΠ°Π·Π΄Π΅Π»Π°Ρ . ΠΠ±ΡΠ°ΡΠΈΡΠ΅ Π²Π½ΠΈΠΌΠ°Π½ΠΈΠ΅, ΡΡΠΎ Π³Π΅Π½Π΅ΡΠ°ΡΠΎΡ ΠΎΡΠ΅Π½Ρ ΠΏΠΎΡ ΠΎΠΆ ΠΏΠΎ ΠΊΠΎΠ½ΡΡΡΡΠΊΡΠΈΠΈ Π½Π° Π΄Π²ΠΈΠ³Π°ΡΠ΅Π»Ρ (Π΅ΡΠ΅ ΠΎΠ΄Π½Π° ΡΠΈΠΌΠΌΠ΅ΡΡΠΈΡ).
Π ΠΈΡ. 3. ΠΡΠ°ΡΠ΅Π½ΠΈΠ΅ ΠΊΠ°ΡΡΡΠΊΠΈ Π² ΠΌΠ°Π³Π½ΠΈΡΠ½ΠΎΠΌ ΠΏΠΎΠ»Π΅ ΡΠΎΠ·Π΄Π°Π΅Ρ ΠΠΠ‘. ΠΡΠΎ ΠΎΡΠ½ΠΎΠ²Π½Π°Ρ ΠΊΠΎΠ½ΡΡΡΡΠΊΡΠΈΡ Π³Π΅Π½Π΅ΡΠ°ΡΠΎΡΠ°, Π² ΠΊΠΎΡΠΎΡΠΎΠΌ ΡΠ°Π±ΠΎΡΠ° ΠΏΠΎ Π²ΡΠ°ΡΠ΅Π½ΠΈΡ ΠΊΠ°ΡΡΡΠΊΠΈ ΠΏΡΠ΅ΠΎΠ±ΡΠ°Π·ΡΠ΅ΡΡΡ Π² ΡΠ»Π΅ΠΊΡΡΠΈΡΠ΅ΡΠΊΡΡ ΡΠ½Π΅ΡΠ³ΠΈΡ. ΠΠ±ΡΠ°ΡΠΈΡΠ΅ Π²Π½ΠΈΠΌΠ°Π½ΠΈΠ΅, Π³Π΅Π½Π΅ΡΠ°ΡΠΎΡ ΠΎΡΠ΅Π½Ρ ΠΏΠΎΡ ΠΎΠΆ ΠΏΠΎ ΠΊΠΎΠ½ΡΡΡΡΠΊΡΠΈΠΈ Π½Π° Π΄Π²ΠΈΠ³Π°ΡΠ΅Π»Ρ.
ΠΡΠ°ΠΊ, ΠΌΡ Π²ΠΈΠ΄ΠΈΠΌ, ΡΡΠΎ ΠΈΠ·ΠΌΠ΅Π½Π΅Π½ΠΈΠ΅ Π²Π΅Π»ΠΈΡΠΈΠ½Ρ ΠΈΠ»ΠΈ Π½Π°ΠΏΡΠ°Π²Π»Π΅Π½ΠΈΡ ΠΌΠ°Π³Π½ΠΈΡΠ½ΠΎΠ³ΠΎ ΠΏΠΎΠ»Ρ ΡΠΎΠ·Π΄Π°Π΅Ρ ΠΠΠ‘. ΠΠΊΡΠΏΠ΅ΡΠΈΠΌΠ΅Π½ΡΡ ΠΏΠΎΠΊΠ°Π·Π°Π»ΠΈ, ΡΡΠΎ ΡΡΡΠ΅ΡΡΠ²ΡΠ΅Ρ ΡΠ΅ΡΠ°ΡΡΠ°Ρ Π²Π΅Π»ΠΈΡΠΈΠ½Π°, Π½Π°Π·ΡΠ²Π°Π΅ΠΌΠ°Ρ ΠΌΠ°Π³Π½ΠΈΡΠ½ΡΠΌ ΠΏΠΎΡΠΎΠΊΠΎΠΌ , Ξ¦ , ΠΎΠΏΡΠ΅Π΄Π΅Π»ΡΠ΅ΠΌΠ°Ρ Π²ΡΡΠ°ΠΆΠ΅Π½ΠΈΠ΅ΠΌ
Ξ¦ Β =Β BA Β cosΒ ΞΈ ,
Π³Π΄Π΅ B β Π½Π°ΠΏΡΡΠΆΠ΅Π½Π½ΠΎΡΡΡ ΠΌΠ°Π³Π½ΠΈΡΠ½ΠΎΠ³ΠΎ ΠΏΠΎΠ»Ρ Π½Π° ΠΏΠ»ΠΎΡΠ°Π΄ΠΈ A ΠΏΠΎΠ΄ Π»ΡΠ±ΡΠΌ ΡΠ³Π»ΠΎΠΌ 9. ΠΈΠ·ΠΌΠ΅Π½Π΅Π½ΠΈΠ΅ ΠΌΠ°Π³Π½ΠΈΡΠ½ΠΎΠ³ΠΎ ΠΏΠΎΡΠΎΠΊΠ° Ξ¦ ΠΈΠ½Π΄ΡΡΠΈΡΡΠ΅Ρ ΠΠΠ‘. ΠΡΠΎΡ ΠΏΡΠΎΡΠ΅ΡΡ ΠΎΠΏΡΠ΅Π΄Π΅Π»ΡΠ΅ΡΡΡ ΠΊΠ°ΠΊ ΡΠ»Π΅ΠΊΡΡΠΎΠΌΠ°Π³Π½ΠΈΡΠ½Π°Ρ ΠΈΠ½Π΄ΡΠΊΡΠΈΡ . ΠΠ΄ΠΈΠ½ΠΈΡΠ°ΠΌΠΈ ΠΌΠ°Π³Π½ΠΈΡΠ½ΠΎΠ³ΠΎ ΠΏΠΎΡΠΎΠΊΠ° Ξ¦ ΡΠ²Π»ΡΡΡΡΡ Π’ β ΠΌ 2 . ΠΠ°ΠΊ Π²ΠΈΠ΄Π½ΠΎ Π½Π° ΡΠΈΡ. 4, B cos ΞΈ = B β₯ , ΠΊΠΎΡΠΎΡΠ°Ρ ΡΠ²Π»ΡΠ΅ΡΡΡ ΠΊΠΎΠΌΠΏΠΎΠ½Π΅Π½ΡΠΎΠΉ B , ΠΏΠ΅ΡΠΏΠ΅Π½Π΄ΠΈΠΊΡΠ»ΡΡΠ½ΠΎΠΉ ΠΏΠ»ΠΎΡΠ°Π΄ΠΈ A . Π’Π°ΠΊΠΈΠΌ ΠΎΠ±ΡΠ°Π·ΠΎΠΌ, ΠΌΠ°Π³Π½ΠΈΡΠ½ΡΠΉ ΠΏΠΎΡΠΎΠΊ ΡΠ°Π²Π΅Π½ Ξ¦ = B β₯ A , ΠΏΡΠΎΠΈΠ·Π²Π΅Π΄Π΅Π½ΠΈΠ΅ ΠΏΠ»ΠΎΡΠ°Π΄ΠΈ ΠΈ ΡΠΎΡΡΠ°Π²Π»ΡΡΡΠ΅ΠΉ ΠΌΠ°Π³Π½ΠΈΡΠ½ΠΎΠ³ΠΎ ΠΏΠΎΠ»Ρ, ΠΏΠ΅ΡΠΏΠ΅Π½Π΄ΠΈΠΊΡΠ»ΡΡΠ½ΠΎΠΉ Π΅ΠΉ.
Π ΠΈΡ. 4. ΠΠ°Π³Π½ΠΈΡΠ½ΡΠΉ ΠΏΠΎΡΠΎΠΊ Ξ¦ ΡΠ²ΡΠ·Π°Π½ Ρ ΠΌΠ°Π³Π½ΠΈΡΠ½ΡΠΌ ΠΏΠΎΠ»Π΅ΠΌ ΠΈ ΠΏΠ»ΠΎΡΠ°Π΄ΡΡ, Π½Π°Π΄ ΠΊΠΎΡΠΎΡΠΎΠΉ ΠΎΠ½ΠΎ ΡΡΡΠ΅ΡΡΠ²ΡΠ΅Ρ. ΠΠΎΡΠΎΠΊ Ξ¦ = BA cos ΞΈ ΠΎΡΠ½ΠΎΡΠΈΡΡΡ ΠΊ ΠΈΠ½Π΄ΡΠΊΡΠΈΠΈ; Π»ΡΠ±ΠΎΠ΅ ΠΈΠ·ΠΌΠ΅Π½Π΅Π½ΠΈΠ΅ Ξ¦ ΠΈΠ½Π΄ΡΡΠΈΡΡΠ΅Ρ ΠΠΠ‘.
ΠΡΡ ΠΈΠ½Π΄ΡΠΊΡΠΈΡ, Π²ΠΊΠ»ΡΡΠ°Ρ ΠΏΡΠΈΠ²Π΅Π΄Π΅Π½Π½ΡΠ΅ Π²ΡΡΠ΅ ΠΏΡΠΈΠΌΠ΅ΡΡ, Π²ΠΎΠ·Π½ΠΈΠΊΠ°Π΅Ρ ΠΈΠ·-Π·Π° Π½Π΅ΠΊΠΎΡΠΎΡΠΎΠ³ΠΎ ΠΈΠ·ΠΌΠ΅Π½Π΅Π½ΠΈΡ ΠΌΠ°Π³Π½ΠΈΡΠ½ΠΎΠ³ΠΎ ΠΏΠΎΡΠΎΠΊΠ° Ξ¦ . ΠΠ°ΠΏΡΠΈΠΌΠ΅Ρ, Π€Π°ΡΠ°Π΄Π΅ΠΉ ΠΌΠ΅Π½ΡΠ» B ΠΈ, ΡΠ»Π΅Π΄ΠΎΠ²Π°ΡΠ΅Π»ΡΠ½ΠΎ, Ξ¦ ΠΏΡΠΈ ΡΠ°Π·ΠΌΡΠΊΠ°Π½ΠΈΠΈ ΠΈ Π·Π°ΠΌΡΠΊΠ°Π½ΠΈΠΈ ΠΏΠ΅ΡΠ΅ΠΊΠ»ΡΡΠ°ΡΠ΅Π»Ρ Π² ΡΠ²ΠΎΠ΅ΠΌ Π°ΠΏΠΏΠ°ΡΠ°ΡΠ΅ (ΠΏΠΎΠΊΠ°Π·Π°Π½Π½ΠΎΠΌ Π½Π° ΡΠΈΡ. 1). ΠΡΠΎ ΡΠ°ΠΊΠΆΠ΅ Π²Π΅ΡΠ½ΠΎ Π΄Π»Ρ ΡΡΠ΅ΡΠΆΠ½Π΅Π²ΠΎΠ³ΠΎ ΠΌΠ°Π³Π½ΠΈΡΠ° ΠΈ ΠΊΠ°ΡΡΡΠΊΠΈ, ΠΏΠΎΠΊΠ°Π·Π°Π½Π½ΡΡ Π½Π° ΡΠΈΡΡΠ½ΠΊΠ΅ 2. ΠΡΠΈ Π²ΡΠ°ΡΠ΅Π½ΠΈΠΈ ΠΊΠ°ΡΡΡΠΊΠΈ Π³Π΅Π½Π΅ΡΠ°ΡΠΎΡΠ° ΡΠ³ΠΎΠ» ΞΈ ΠΈ, ΡΠ»Π΅Π΄ΠΎΠ²Π°ΡΠ΅Π»ΡΠ½ΠΎ, Ξ¦ ΠΈΠ·ΠΌΠ΅Π½ΡΠ΅ΡΡΡ. Π’ΠΎ, Π½Π°ΡΠΊΠΎΠ»ΡΠΊΠΎ Π²Π΅Π»ΠΈΠΊΠ° ΠΠΠ‘ ΠΈ ΠΊΠ°ΠΊΠΎΠ΅ Π½Π°ΠΏΡΠ°Π²Π»Π΅Π½ΠΈΠ΅ ΠΎΠ½Π° ΠΏΡΠΈΠ½ΠΈΠΌΠ°Π΅Ρ, Π·Π°Π²ΠΈΡΠΈΡ ΠΎΡ ΠΈΠ·ΠΌΠ΅Π½Π΅Π½ΠΈΡ Ξ¦ ΠΈ ΠΎΡ ΡΠΎΠ³ΠΎ, Π½Π°ΡΠΊΠΎΠ»ΡΠΊΠΎ Π±ΡΡΡΡΠΎ ΡΡΠΎ ΠΈΠ·ΠΌΠ΅Π½Π΅Π½ΠΈΠ΅ ΠΏΡΠΎΠΈΡΡ ΠΎΠ΄ΠΈΡ, ΠΊΠ°ΠΊ Π±ΡΠ΄Π΅Ρ ΡΠ°ΡΡΠΌΠΎΡΡΠ΅Π½ΠΎ Π² ΡΠ»Π΅Π΄ΡΡΡΠ΅ΠΌ ΡΠ°Π·Π΄Π΅Π»Π΅.
Π Π΅Π·ΡΠΌΠ΅ ΡΠ°Π·Π΄Π΅Π»Π°
- Π Π΅ΡΠ°ΡΡΠ΅ΠΉ Π²Π΅Π»ΠΈΡΠΈΠ½ΠΎΠΉ ΠΈΠ½Π΄ΡΠΊΡΠΈΠΈ ΡΠ²Π»ΡΠ΅ΡΡΡ ΠΌΠ°Π³Π½ΠΈΡΠ½ΡΠΉ ΠΏΠΎΡΠΎΠΊ Ξ¦ , ΠΎΠΏΡΠ΅Π΄Π΅Π»ΡΠ΅ΠΌΡΠΉ ΠΊΠ°ΠΊ Ξ¦ = BA cos ΞΈ , Π³Π΄Π΅ B β Π½Π°ΠΏΡΡΠΆΠ΅Π½Π½ΠΎΡΡΡ ΠΌΠ°Π³Π½ΠΈΡΠ½ΠΎΠ³ΠΎ ΠΏΠΎΠ»Ρ Π½Π° ΠΏΠ»ΠΎΡΠ°Π΄ΠΈ A ΠΏΠΎΠ΄ ΡΠ³Π»ΠΎΠΌ ΠΏΠ΅ΡΠΏΠ΅Π½Π΄ΠΈΠΊΡΠ»ΡΡ ΠΊ ΠΏΠ»ΠΎΡΠ°Π΄ΠΈ.
- ΠΠ΄ΠΈΠ½ΠΈΡΠ°ΠΌΠΈ ΠΌΠ°Π³Π½ΠΈΡΠ½ΠΎΠ³ΠΎ ΠΏΠΎΡΠΎΠΊΠ° Ξ¦ ΡΠ²Π»ΡΡΡΡΡ Π’Π»β ΠΌ 2 .
- ΠΡΠ±ΠΎΠ΅ ΠΈΠ·ΠΌΠ΅Π½Π΅Π½ΠΈΠ΅ ΠΌΠ°Π³Π½ΠΈΡΠ½ΠΎΠ³ΠΎ ΠΏΠΎΡΠΎΠΊΠ° Ξ¦ ΠΈΠ½Π΄ΡΡΠΈΡΡΠ΅Ρ ΠΠΠ‘ β ΠΏΡΠΎΡΠ΅ΡΡ ΠΎΠΏΡΠ΅Π΄Π΅Π»ΡΠ΅ΡΡΡ ΠΊΠ°ΠΊ ΡΠ»Π΅ΠΊΡΡΠΎΠΌΠ°Π³Π½ΠΈΡΠ½Π°Ρ ΠΈΠ½Π΄ΡΠΊΡΠΈΡ.
ΠΠΎΠ½ΡΠ΅ΠΏΡΡΠ°Π»ΡΠ½ΡΠ΅ Π²ΠΎΠΏΡΠΎΡΡ
1. ΠΠ°ΠΊ ΠΌΠ½ΠΎΠ³ΠΎΠΊΠΎΠ½ΡΡΡΠ½ΡΠ΅ ΠΊΠ°ΡΡΡΠΊΠΈ ΠΈ ΠΆΠ΅Π»Π΅Π·Π½ΠΎΠ΅ ΠΊΠΎΠ»ΡΡΠΎ Π² Π²Π΅ΡΡΠΈΠΈ Π°ΠΏΠΏΠ°ΡΠ°ΡΠ° Π€Π°ΡΠ°Π΄Π΅Ρ, ΠΏΠΎΠΊΠ°Π·Π°Π½Π½ΠΎΠΉ Π½Π° ΡΠΈΡΡΠ½ΠΊΠ΅ 1, ΡΠ»ΡΡΡΠ°ΡΡ Π½Π°Π±Π»ΡΠ΄Π΅Π½ΠΈΠ΅ ΠΠΠ‘ ΠΈΠ½Π΄ΡΠΊΡΠΈΠΈ?
2. ΠΠΎΠ³Π΄Π° ΠΌΠ°Π³Π½ΠΈΡ Π²ΡΡΠ³ΠΈΠ²Π°Π΅ΡΡΡ Π² ΠΊΠ°ΡΡΡΠΊΡ, ΠΊΠ°ΠΊ ΠΏΠΎΠΊΠ°Π·Π°Π½ΠΎ Π½Π° ΡΠΈΡ. 2(Π°), ΠΊΠ°ΠΊ Π½Π°ΠΏΡΠ°Π²Π»Π΅Π½Π° ββΡΠΈΠ»Π°, Π΄Π΅ΠΉΡΡΠ²ΡΡΡΠ°Ρ Π½Π° ΠΌΠ°Π³Π½ΠΈΡ ΡΠΎ ΡΡΠΎΡΠΎΠ½Ρ ΠΊΠ°ΡΡΡΠΊΠΈ? ΠΠ°ΡΠΈΡΡΠΉΡΠ΅ Π΄ΠΈΠ°Π³ΡΠ°ΠΌΠΌΡ, ΠΏΠΎΠΊΠ°Π·ΡΠ²Π°ΡΡΡΡ Π½Π°ΠΏΡΠ°Π²Π»Π΅Π½ΠΈΠ΅ ΡΠΎΠΊΠ°, ΠΈΠ½Π΄ΡΡΠΈΡΡΠ΅ΠΌΠΎΠ³ΠΎ Π² ΠΊΠ°ΡΡΡΠΊΠ΅, ΠΈ ΡΠΎΠ·Π΄Π°Π²Π°Π΅ΠΌΠΎΠ΅ ΠΈΠΌ ΠΌΠ°Π³Π½ΠΈΡΠ½ΠΎΠ΅ ΠΏΠΎΠ»Π΅, ΡΡΠΎΠ±Ρ ΠΎΠ±ΠΎΡΠ½ΠΎΠ²Π°ΡΡ ΡΠ²ΠΎΠΉ ΠΎΡΠ²Π΅Ρ. ΠΠ°ΠΊ Π²Π΅Π»ΠΈΡΠΈΠ½Π° ΡΠΈΠ»Ρ Π·Π°Π²ΠΈΡΠΈΡ ΠΎΡ ΡΠΎΠΏΡΠΎΡΠΈΠ²Π»Π΅Π½ΠΈΡ Π³Π°Π»ΡΠ²Π°Π½ΠΎΠΌΠ΅ΡΡΠ°?
3. ΠΠ±ΡΡΡΠ½ΠΈΡΠ΅, ΠΊΠ°ΠΊ ΠΌΠ°Π³Π½ΠΈΡΠ½ΡΠΉ ΠΏΠΎΡΠΎΠΊ ΠΌΠΎΠΆΠ΅Ρ Π±ΡΡΡ ΡΠ°Π²Π΅Π½ Π½ΡΠ»Ρ, Π΅ΡΠ»ΠΈ ΠΌΠ°Π³Π½ΠΈΡΠ½ΠΎΠ΅ ΠΏΠΎΠ»Π΅ Π½Π΅ ΡΠ°Π²Π½ΠΎ Π½ΡΠ»Ρ.
4. ΠΠ°Π²ΠΎΠ΄ΠΈΡΡΡ Π»ΠΈ ΠΠΠ‘ Π² ΠΊΠ°ΡΡΡΠΊΠ΅ Π½Π° ΡΠΈΡΡΠ½ΠΊΠ΅ 5, ΠΊΠΎΠ³Π΄Π° ΠΎΠ½Π° ΡΠ°ΡΡΡΠ³ΠΈΠ²Π°Π΅ΡΡΡ? ΠΡΠ»ΠΈ Π΄Π°, ΡΠΊΠ°ΠΆΠΈΡΠ΅ ΠΏΠΎΡΠ΅ΠΌΡ ΠΈ ΡΠΊΠ°ΠΆΠΈΡΠ΅ Π½Π°ΠΏΡΠ°Π²Π»Π΅Π½ΠΈΠ΅ ΠΈΠ½Π΄ΡΠΊΡΠΈΠΎΠ½Π½ΠΎΠ³ΠΎ ΡΠΎΠΊΠ°.
Π ΠΈΡ. 5. ΠΡΡΠ³Π»Π°Ρ ΠΊΠ°ΡΡΡΠΊΠ° ΠΏΡΠΎΠ²ΠΎΠ»ΠΎΠΊΠΈ Π½Π°ΡΡΠ½ΡΡΠ° Π² ΠΌΠ°Π³Π½ΠΈΡΠ½ΠΎΠΌ ΠΏΠΎΠ»Π΅.
ΠΠ°Π΄Π°ΡΠΈ ΠΈ ΡΠΏΡΠ°ΠΆΠ½Π΅Π½ΠΈΡ
1. ΠΠ°ΠΊΠΎΠ²Π° Π²Π΅Π»ΠΈΡΠΈΠ½Π° ΠΌΠ°Π³Π½ΠΈΡΠ½ΠΎΠ³ΠΎ ΠΏΠΎΡΠΎΠΊΠ° Π½Π° ΠΊΠ°ΡΡΡΠΊΠ΅ 2 Π½Π° ΡΠΈΡΡΠ½ΠΊΠ΅ 6Β ΠΈΠ·-Π·Π° ΠΊΠ°ΡΡΡΠΊΠΈ 1?
Π ΠΈΡ. 6. (Π°) ΠΠ»ΠΎΡΠΊΠΎΡΡΠΈ Π΄Π²ΡΡ ΠΊΠ°ΡΡΡΠ΅ΠΊ ΠΏΠ΅ΡΠΏΠ΅Π½Π΄ΠΈΠΊΡΠ»ΡΡΠ½Ρ. Π±) ΠΏΡΠΎΠ²ΠΎΠ΄ ΠΏΠ΅ΡΠΏΠ΅Π½Π΄ΠΈΠΊΡΠ»ΡΡΠ΅Π½ ΠΏΠ»ΠΎΡΠΊΠΎΡΡΠΈ ΠΊΠ°ΡΡΡΠΊΠΈ.
2. ΠΠ°ΠΊΠΎΠ²Π° Π²Π΅Π»ΠΈΡΠΈΠ½Π° ΠΌΠ°Π³Π½ΠΈΡΠ½ΠΎΠ³ΠΎ ΠΏΠΎΡΠΎΠΊΠ° ΡΠ΅ΡΠ΅Π· ΠΊΠ°ΡΡΡΠΊΡ Π½Π° ΡΠΈΡ. 6(b) ΠΈΠ·-Π·Π° ΠΏΡΠΎΠ²ΠΎΠ΄Π°?
ΠΠ»ΠΎΡΡΠ°ΡΠΈΠΉ
- ΠΌΠ°Π³Π½ΠΈΡΠ½ΡΠΉ ΠΏΠΎΡΠΎΠΊ:
- Π²Π΅Π»ΠΈΡΠΈΠ½Π° ΠΌΠ°Π³Π½ΠΈΡΠ½ΠΎΠ³ΠΎ ΠΏΠΎΠ»Ρ, ΠΏΡΠΎΡ ΠΎΠ΄ΡΡΠ΅Π³ΠΎ ΡΠ΅ΡΠ΅Π· ΠΎΠΏΡΠ΅Π΄Π΅Π»Π΅Π½Π½ΡΡ ΠΏΠ»ΠΎΡΠ°Π΄Ρ, ΡΠ°ΡΡΡΠΈΡΠ°Π½Π½Π°Ρ ΠΏΠΎ ΡΠΎΡΠΌΡΠ»Π΅ Ξ¦ = B A cos ΞΈ , Π³Π΄Π΅ B β Π½Π°ΠΏΡΡΠΆΠ΅Π½Π½ΠΎΡΡΡ ΠΌΠ°Π³Π½ΠΈΡΠ½ΠΎΠ³ΠΎ ΠΏΠΎΠ»Ρ Π½Π° ΠΏΠ»ΠΎΡΠ°Π΄ΠΈ A ΠΏΠΎΠ΄ ΡΠ³Π»ΠΎΠΌ ΞΈ ΠΊ ΠΏΠ΅ΡΠΏΠ΅Π½Π΄ΠΈΠΊΡΠ»ΡΡΡ ΡΠ°ΠΉΠΎΠ½
- ΡΠ»Π΅ΠΊΡΡΠΎΠΌΠ°Π³Π½ΠΈΡΠ½Π°Ρ ΠΈΠ½Π΄ΡΠΊΡΠΈΡ:
- ΠΏΡΠΎΡΠ΅ΡΡ Π½Π°Π²Π΅Π΄Π΅Π½ΠΈΡ ΠΠΠ‘ (Π½Π°ΠΏΡΡΠΆΠ΅Π½ΠΈΡ) ΠΏΡΠΈ ΠΈΠ·ΠΌΠ΅Π½Π΅Π½ΠΈΠΈ ΠΌΠ°Π³Π½ΠΈΡΠ½ΠΎΠ³ΠΎ ΠΏΠΎΡΠΎΠΊΠ°
Π£ΠΏΡΠ°ΠΆΠ½Π΅Π½ΠΈΡ
1. ΠΠΎΠ»Ρ
Β
ΠΠ»Π΅ΠΊΡΡΠΎΠΌΠ°Π³Π½ΠΈΡΠ½Π°Ρ ΠΈΠ½Π΄ΡΠΊΡΠΈΡ: ΠΎΠΏΡΠ΅Π΄Π΅Π»Π΅Π½ΠΈΠ΅, ΠΏΡΠΈΠΌΠ΅ΡΡ ΠΈ ΠΏΡΠΈΠΌΠ΅Π½Π΅Π½ΠΈΠ΅
Π§ΡΠΎ ΡΠ°ΠΊΠΎΠ΅ ΡΠ»Π΅ΠΊΡΡΠΎΠΌΠ°Π³Π½ΠΈΡΠ½Π°Ρ ΠΈΠ½Π΄ΡΠΊΡΠΈΡ
ΠΌΠ°Π³Π½ΠΈΡΠ½ΡΠΉ ΠΏΠΎΡΠΎΠΊ, ΠΏΡΠΎΡ ΠΎΠ΄ΡΡΠΈΠΉ ΡΠ΅ΡΠ΅Π· Π½Π΅Π³ΠΎ, ΠΈΠ·ΠΌΠ΅Π½ΡΠ΅ΡΡΡ ΡΠΎ Π²ΡΠ΅ΠΌΠ΅Π½Π΅ΠΌ. ΠΠΎΡΠΊΠΎΠ»ΡΠΊΡ ΠΏΠ»ΠΎΡΠ°Π΄Ρ ΠΏΠΎΠΏΠ΅ΡΠ΅ΡΠ½ΠΎΠ³ΠΎ ΡΠ΅ΡΠ΅Π½ΠΈΡ ΠΏΠ΅ΡΠ»ΠΈ ΡΠΈΠΊΡΠΈΡΠΎΠ²Π°Π½Π°, ΠΈΠ·ΠΌΠ΅Π½Π΅Π½ΠΈΠ΅ ΠΏΠΎΡΠΎΠΊΠ° Π²ΡΠ·Π²Π°Π½ΠΎ ΠΈΠ·ΠΌΠ΅Π½Π΅Π½ΠΈΠ΅ΠΌ ΠΌΠ°Π³Π½ΠΈΡΠ½ΠΎΠ³ΠΎ ΠΏΠΎΠ»Ρ. ΠΡΠΎ ΠΈΠ·ΠΌΠ΅Π½ΡΡΡΠ΅Π΅ΡΡ ΠΌΠ°Π³Π½ΠΈΡΠ½ΠΎΠ΅ ΠΏΠΎΠ»Π΅ ΠΏΡΠΈΠΊΠ»Π°Π΄ΡΠ²Π°Π΅Ρ ΡΠΈΠ»Ρ ΠΊ ΡΠ²ΠΎΠ±ΠΎΠ΄Π½ΡΠΌ ΡΠ»Π΅ΠΊΡΡΠΎΠ½Π°ΠΌ Π² ΠΏΡΠΎΠ²ΠΎΠ΄Π½ΠΈΠΊΠ΅. Π ΡΠ΅Π·ΡΠ»ΡΡΠ°ΡΠ΅ ΡΡΠΈ ΡΠ»Π΅ΠΊΡΡΠΎΠ½Ρ Π΄Π²ΠΈΠΆΡΡΡΡ ΠΈ ΡΠΎΠ·Π΄Π°ΡΡ ΡΠΎΠΊ Π²Π½ΡΡΡΠΈ ΠΏΡΠΎΠ²ΠΎΠ΄Π½ΠΈΠΊΠ°. ΠΡΠΎΡ ΡΠΎΠΊ ΠΈΠ·Π²Π΅ΡΡΠ΅Π½ ΠΊΠ°ΠΊ ΠΈΠ½Π΄ΡΡΠΈΡΠΎΠ²Π°Π½Π½ΡΠΉ ΡΠΎΠΊ. Π Π°Π·Π½ΠΎΡΡΡ ΠΏΠΎΡΠ΅Π½ΡΠΈΠ°Π»ΠΎΠ² ΠΌΠ΅ΠΆΠ΄Ρ Π»ΡΠ±ΡΠΌΠΈ Π΄Π²ΡΠΌΡ ΡΠΎΡΠΊΠ°ΠΌΠΈ Π² ΠΏΡΠΎΠ²ΠΎΠ΄Π½ΠΈΠΊΠ΅ ΠΈΠ·Π²Π΅ΡΡΠ½Π° ΠΊΠ°ΠΊ ΠΈΠ½Π΄ΡΡΠΈΡΠΎΠ²Π°Π½Π½Π°Ρ ΠΠΠ‘ (ΡΠ»Π΅ΠΊΡΡΠΎΠ΄Π²ΠΈΠΆΡΡΠ°Ρ ΡΠΈΠ»Π°). ΠΡΠΎΡΠ΅ΡΡ Π³Π΅Π½Π΅ΡΠ°ΡΠΈΠΈ ΡΠ»Π΅ΠΊΡΡΠΈΡΠ΅ΡΠΊΠΎΠ³ΠΎ ΡΠΎΠΊΠ° ΠΈΠ·ΠΌΠ΅Π½ΡΡΡΠΈΠΌΡΡ ΠΌΠ°Π³Π½ΠΈΡΠ½ΡΠΌ ΠΏΠΎΠ»Π΅ΠΌ Π½Π°Π·ΡΠ²Π°Π΅ΡΡΡ ΡΠ»Π΅ΠΊΡΡΠΎΠΌΠ°Π³Π½ΠΈΡΠ½ΠΎΠΉ ΠΈΠ½Π΄ΡΠΊΡΠΈΠ΅ΠΉ.
ΠΠ°Π³Π½ΠΈΡΠ½ΠΎΠ΅ ΠΏΠΎΠ»Π΅ ΠΈΡΡ ΠΎΠ΄ΠΈΡ ΠΎΡ ΠΏΠΎΡΡΠΎΡΠ½Π½ΠΎΠ³ΠΎ ΠΌΠ°Π³Π½ΠΈΡΠ°, ΡΠ°ΠΊΠΎΠ³ΠΎ ΠΊΠ°ΠΊ ΡΡΠ΅ΡΠΆΠ½Π΅Π²ΠΎΠΉ ΠΌΠ°Π³Π½ΠΈΡ. Π―Π²Π»Π΅Π½ΠΈΠ΅ Π½Π°Π·ΡΠ²Π°Π΅ΡΡΡ ΠΈΠ½Π΄ΡΠΊΡΠΈΠ΅ΠΉ, ΠΏΠΎΡΠΎΠΌΡ ΡΡΠΎ ΠΌΠ΅ΠΆΠ΄Ρ ΠΏΡΠΎΠ²ΠΎΠ΄Π½ΠΈΠΊΠΎΠΌ ΠΈ ΠΌΠ°Π³Π½ΠΈΡΠΎΠΌ Π½Π΅Ρ ΡΠΈΠ·ΠΈΡΠ΅ΡΠΊΠΎΠ³ΠΎ ΠΊΠΎΠ½ΡΠ°ΠΊΡΠ°. ΠΠ°Π³Π½ΠΈΡΠ½ΡΠ΅ ΡΠΈΠ»ΠΎΠ²ΡΠ΅ Π»ΠΈΠ½ΠΈΠΈ ΠΏΡΠΎΡ ΠΎΠ΄ΡΡ ΡΠ΅ΡΠ΅Π· Π²ΠΎΠ·Π΄ΡΡ ΠΈΠ»ΠΈ ΡΡΠ΅Π΄Ρ, Π΅ΡΠ»ΠΈ ΠΊΠ°ΡΡΡΠΊΠ° Π½Π°ΠΌΠΎΡΠ°Π½Π° Π½Π° ΠΌΠ΅ΡΠ°Π»Π»ΠΈΡΠ΅ΡΠΊΠΈΠΉ ΡΠ΅ΡΠ΄Π΅ΡΠ½ΠΈΠΊ. Π£ΡΠ»ΠΎΠ²ΠΈΠ΅ ΡΠ»Π΅ΠΊΡΡΠΎΠΌΠ°Π³Π½ΠΈΡΠ½ΠΎΠΉ ΠΈΠ½Π΄ΡΠΊΡΠΈΠΈ ΡΠΎΡΡΠΎΠΈΡ Π² ΡΠΎΠΌ, ΡΡΠΎ ΠΌΠ΅ΠΆΠ΄Ρ ΠΏΡΠΎΠ²ΠΎΠ΄Π½ΠΈΠΊΠΎΠΌ ΠΈ ΠΌΠ°Π³Π½ΠΈΡΠΎΠΌ Π΄ΠΎΠ»ΠΆΠ½ΠΎ Π±ΡΡΡ ΠΎΡΠ½ΠΎΡΠΈΡΠ΅Π»ΡΠ½ΠΎΠ΅ Π΄Π²ΠΈΠΆΠ΅Π½ΠΈΠ΅.
ΠΠ»Π΅ΠΊΡΡΠΎΠΌΠ°Π³Π½ΠΈΡΠ½Π°Ρ ΠΈΠ½Π΄ΡΠΊΡΠΈΡΠΡΠΎ ΠΎΡΠΊΡΡΠ» ΡΠ»Π΅ΠΊΡΡΠΎΠΌΠ°Π³Π½ΠΈΡΠ½ΡΡ ΠΈΠ½Π΄ΡΠΊΡΠΈΡ
ΠΠ½Π³Π»ΠΈΠΉΡΠΊΠΈΠΉ ΡΠΈΠ·ΠΈΠΊ ΠΠ°ΠΉΠΊΠ» Π€Π°ΡΠ°Π΄Π΅ΠΉ ΠΎΡΠΊΡΡΠ» ΡΠ»Π΅ΠΊΡΡΠΎΠΌΠ°Π³Π½ΠΈΡΠ½ΡΡ ΠΈΠ½Π΄ΡΠΊΡΠΈΡ Π² 1831 Π³ΠΎΠ΄Ρ. Π¨ΠΎΡΠ»Π°Π½Π΄ΡΠΊΠΈΠΉ ΡΠΈΠ·ΠΈΠΊ ΠΈ ΠΌΠ°ΡΠ΅ΠΌΠ°ΡΠΈΠΊ ΠΠΆΠ΅ΠΉΠΌΡ ΠΠ»Π΅ΡΠΊ ΠΠ°ΠΊΡΠ²Π΅Π»Π» ΡΠ°Π·ΡΠ°Π±ΠΎΡΠ°Π» ΠΌΠ°ΡΠ΅ΠΌΠ°ΡΠΈΡΠ΅ΡΠΊΠΈΠ΅ ΡΡΠ°Π²Π½Π΅Π½ΠΈΡ ΠΈ ΠΎΠΏΡΠ±Π»ΠΈΠΊΠΎΠ²Π°Π» ΠΈΡ Π² 1865 Π³ΠΎΠ΄Ρ. ΠΡΠΈ ΡΡΠ°Π²Π½Π΅Π½ΠΈΡ ΡΠ΅Π³ΠΎΠ΄Π½Ρ ΠΈΠ·Π²Π΅ΡΡΠ½Ρ ΠΊΠ°ΠΊ ΡΡΠ°Π²Π½Π΅Π½ΠΈΡ ΠΠ°ΠΊΡΠ²Π΅Π»Π»Π°.
ΠΠ°ΠΊ ΡΠ°Π±ΠΎΡΠ°Π΅Ρ ΡΠ»Π΅ΠΊΡΡΠΎΠΌΠ°Π³Π½ΠΈΡΠ½Π°Ρ ΠΈΠ½Π΄ΡΠΊΡΠΈΡ
ΠΠ»Π΅ΠΊΡΡΠΎΠΌΠ°Π³Π½ΠΈΡΠ½ΡΡ ΠΈΠ½Π΄ΡΠΊΡΠΈΡ Π»ΡΡΡΠ΅ Π²ΡΠ΅Π³ΠΎ ΠΎΠ±ΡΡΡΠ½ΠΈΡΡ, ΠΊΠΎΠ³Π΄Π° ΠΏΡΠΎΠ²ΠΎΠ΄Π½ΠΈΠΊ, Π½Π°ΠΌΠΎΡΠ°Π½Π½ΡΠΉ Π² ΠΊΠ°ΡΡΡΠΊΡ, ΠΏΠΎΠΌΠ΅ΡΠ°Π΅ΡΡΡ ΡΡΠ΄ΠΎΠΌ Ρ Π΄Π²ΠΈΠΆΡΡΠΈΠΌΡΡ ΡΡΠ΅ΡΠΆΠ½Π΅Π²ΡΠΌ ΠΌΠ°Π³Π½ΠΈΡΠΎΠΌ, ΠΈΠΌΠ΅ΡΡΠΈΠΌ ΡΠ΅Π²Π΅ΡΠ½ΡΠΉ ΠΈ ΡΠΆΠ½ΡΠΉ ΠΏΠΎΠ»ΡΡΠ°. ΠΠ°Π³Π½ΠΈΡΠ½ΠΎΠ΅ ΠΏΠΎΠ»Π΅ Π² ΡΡΠ΅ΡΠΆΠ½Π΅Π²ΠΎΠΌ ΠΌΠ°Π³Π½ΠΈΡΠ΅ ΠΏΡΠ΅Π΄ΡΡΠ°Π²Π»Π΅Π½ΠΎ ΡΠΈΠ»ΠΎΠ²ΡΠΌΠΈ Π»ΠΈΠ½ΠΈΡΠΌΠΈ, ΠΊΠΎΡΠΎΡΡΠ΅ Π²ΡΡ ΠΎΠ΄ΡΡ ΠΈΠ· ΡΠ΅Π²Π΅ΡΠ½ΠΎΠ³ΠΎ ΠΏΠΎΠ»ΡΡΠ° ΠΈ Π·Π°ΠΊΠ°Π½ΡΠΈΠ²Π°ΡΡΡΡ Π² ΡΠΆΠ½ΠΎΠΌ ΠΏΠΎΠ»ΡΡΠ΅. ΠΠΎΠ»ΠΈΡΠ΅ΡΡΠ²ΠΎ Π»ΠΈΠ½ΠΈΠΉ, ΠΏΡΠΎΡ ΠΎΠ΄ΡΡΠΈΡ ΡΠ΅ΡΠ΅Π· Π΄Π°Π½Π½ΡΡ ΠΎΠ±Π»Π°ΡΡΡ Π² ΠΏΡΠΎΡΡΡΠ°Π½ΡΡΠ²Π΅, ΡΠ²Π»ΡΠ΅ΡΡΡ ΠΌΠ°Π³Π½ΠΈΡΠ½ΡΠΌ ΠΏΠΎΡΠΎΠΊΠΎΠΌ.
Π£ΡΠ°Π²Π½Π΅Π½ΠΈΠ΅ ΠΌΠ°Π³Π½ΠΈΡΠ½ΠΎΠ³ΠΎ ΠΏΠΎΡΠΎΠΊΠ°
ΠΡΠ΅Π΄ΠΏΠΎΠ»ΠΎΠΆΠΈΠΌ, ΡΡΠΎ ΡΡΠ΅ΡΠΆΠ½Π΅Π²ΠΎΠΉ ΠΌΠ°Π³Π½ΠΈΡ ΠΏΡΠ΅Π΄ΡΡΠ°Π²Π»ΡΠ΅Ρ ΡΠΎΠ±ΠΎΠΉ ΠΏΠΎΠ»Π΅ Ρ Π½Π°ΠΏΡΡΠΆΠ΅Π½Π½ΠΎΡΡΡΡ B. Π Π°ΡΡΠΌΠΎΡΡΠΈΠΌ Π΄Π²ΡΠΌΠ΅ΡΠ½ΡΡ ΠΎΠ±Π»Π°ΡΡΡ A Π² ΠΏΡΠΎΡΡΡΠ°Π½ΡΡΠ²Π΅, ΡΠΈΠ»ΠΎΠ²ΡΠ΅ Π»ΠΈΠ½ΠΈΠΈ ΠΊΠΎΡΠΎΡΠΎΠΉ ΠΏΡΠΎΡ ΠΎΠ΄ΡΡ ΡΠ΅ΡΠ΅Π· ΡΡΡ ΠΎΠ±Π»Π°ΡΡΡ ΠΈ ΡΠΎΡΡΠ°Π²Π»ΡΡΡ ΡΠ³ΠΎΠ» ΞΈ Ρ Π²Π΅ΠΊΡΠΎΡΠΎΠΌ ΠΏΠ»ΠΎΡΠ°Π΄ΠΈ. Π’ΠΎΠ³Π΄Π° ΠΏΠ»ΠΎΡΠ°Π΄Ρ, ΡΠΏΡΠΎΠ΅ΡΠΈΡΠΎΠ²Π°Π½Π½Π°Ρ Π½Π° ΠΏΠ»ΠΎΡΠΊΠΎΡΡΡ, ΠΏΠ΅ΡΠΏΠ΅Π½Π΄ΠΈΠΊΡΠ»ΡΡΠ½ΡΡ ΠΏΠΎΠ»Ρ, ΡΠ°Π²Π½Π° A cos ΞΈ. ΠΠ°Π³Π½ΠΈΡΠ½ΡΠΉ ΠΏΠΎΡΠΎΠΊ Ο ΠΎΠΏΡΠ΅Π΄Π΅Π»ΡΠ΅ΡΡΡ ΠΊΠ°ΠΊ ΠΏΡΠΎΠΈΠ·Π²Π΅Π΄Π΅Π½ΠΈΠ΅ ΠΏΠΎΠ»Ρ Π½Π° ΠΏΠ»ΠΎΡΠ°Π΄Ρ ΠΏΡΠΎΠ΅ΠΊΡΠΈΠΈ.
Ο = BA cos ΞΈ
ΠΠΎΠ³Π΄Π° ΡΠ΅Π²Π΅ΡΠ½ΡΠΉ ΠΏΠΎΠ»ΡΡ ΠΏΡΠΈΠ±Π»ΠΈΠΆΠ°Π΅ΡΡΡ ΠΊ ΠΊΠ°ΡΡΡΠΊΠ΅, ΠΏΠΎΡΠΎΠΊ ΡΠ²Π΅Π»ΠΈΡΠΈΠ²Π°Π΅ΡΡΡ ΠΈ Π±ΡΠ΄Π΅Ρ ΠΏΡΠΎΠ΄ΠΎΠ»ΠΆΠ°ΡΡ ΡΠ²Π΅Π»ΠΈΡΠΈΠ²Π°ΡΡΡΡ, ΠΏΠΎΠΊΠ° ΠΌΠ°Π³Π½ΠΈΡ Π½Π°Ρ ΠΎΠ΄ΠΈΡΡΡ Π² Π΄Π²ΠΈΠΆΠ΅Π½ΠΈΠΈ. ΠΠ·ΠΌΠ΅Π½Π΅Π½ΠΈΠ΅ ΠΌΠ°Π³Π½ΠΈΡΠ½ΠΎΠ³ΠΎ ΠΏΠΎΡΠΎΠΊΠ° Π²ΠΎ Π²ΡΠ΅ΠΌΠ΅Π½ΠΈ ΠΏΡΠΈΠ²ΠΎΠ΄ΠΈΡ ΠΊ ΠΈΠ½Π΄ΡΡΠΈΡΠΎΠ²Π°Π½Π½ΠΎΠΌΡ ΡΠΎΠΊΡ. ΠΡΠ»ΠΈ Π°ΠΌΠΏΠ΅ΡΠΌΠ΅ΡΡ ΠΏΠΎΠ΄ΠΊΠ»ΡΡΠΈΡΡ ΠΊ Π΄Π²ΡΠΌ ΠΊΠΎΠ½ΡΠ°ΠΌ ΠΊΠ°ΡΡΡΠΊΠΈ, ΠΎΠ½ Π·Π°ΡΠ΅Π³ΠΈΡΡΡΠΈΡΡΠ΅Ρ ΡΠΎΠΊ. Π’ΠΎΠΊ Π±ΡΠ΄Π΅Ρ ΠΏΡΠΎΠ΄ΠΎΠ»ΠΆΠ°ΡΡ ΡΠ΅ΡΡ, ΠΏΠΎΠΊΠ° ΠΌΠ°Π³Π½ΠΈΡ Π΄Π²ΠΈΠΆΠ΅ΡΡΡ. ΠΡΠ»ΠΈ ΠΌΠ°Π³Π½ΠΈΡ ΠΎΡΠ²Π΅ΡΡΠΈ ΠΎΡ ΠΊΠ°ΡΡΡΠΊΠΈ, ΠΈΠ½Π΄ΡΡΠΈΡΠΎΠ²Π°Π½Π½ΡΠΉ ΡΠΎΠΊ ΠΈΠ·ΠΌΠ΅Π½ΠΈΡ Π½Π°ΠΏΡΠ°Π²Π»Π΅Π½ΠΈΠ΅.
ΠΠ°ΠΊΠΎΠ½Ρ ΡΠ»Π΅ΠΊΡΡΠΎΠΌΠ°Π³Π½ΠΈΡΠ½ΠΎΠΉ ΠΈΠ½Π΄ΡΠΊΡΠΈΠΈ
1. ΠΠ°ΠΊΠΎΠ½ Π€Π°ΡΠ°Π΄Π΅Ρ
ΠΠΎΡΠΊΠΎΠ»ΡΠΊΡ Π² ΡΠ΅ΠΏΠΈ ΠΈΠ½Π΄ΡΡΠΈΡΡΠ΅ΡΡΡ ΡΠΎΠΊ, ΡΠ°ΠΊΠΆΠ΅ Π²ΠΎΠ·Π½ΠΈΠΊΠ°Π΅Ρ ΠΈΠ½Π΄ΡΡΠΈΡΠΎΠ²Π°Π½Π½ΠΎΠ΅ Π½Π°ΠΏΡΡΠΆΠ΅Π½ΠΈΠ΅ ΠΈΠ»ΠΈ ΠΠΠ‘ (ΡΠ»Π΅ΠΊΡΡΠΎΠ΄Π²ΠΈΠΆΡΡΠ°Ρ ΡΠΈΠ»Π°) ΠΈΠ·-Π·Π° ΡΠΎΠΏΡΠΎΡΠΈΠ²Π»Π΅Π½ΠΈΡ ΠΏΡΠΎΠ²ΠΎΠ΄Π°. Π‘ΠΎΠ³Π»Π°ΡΠ½ΠΎ Π·Π°ΠΊΠΎΠ½Ρ Π€Π°ΡΠ°Π΄Π΅Ρ, Π²Π΅Π»ΠΈΡΠΈΠ½Π° ΡΡΠΎΠΉ ΠΠΠ‘ ΠΏΡΠΎΠΏΠΎΡΡΠΈΠΎΠ½Π°Π»ΡΠ½Π° ΡΠΊΠΎΡΠΎΡΡΠΈ ΠΈΠ·ΠΌΠ΅Π½Π΅Π½ΠΈΡ ΠΏΠΎΡΠΎΠΊΠ°. ΠΡΠΎΡ Π·Π°ΠΊΠΎΠ½ ΡΡΡΠ°Π½Π°Π²Π»ΠΈΠ²Π°Π΅Ρ ΠΊΠΎΠ»ΠΈΡΠ΅ΡΡΠ²Π΅Π½Π½ΡΡ ΡΠ²ΡΠ·Ρ ΠΌΠ΅ΠΆΠ΄Ρ ΠΈΠ·ΠΌΠ΅Π½ΡΡΡΠΈΠΌΡΡ ΠΌΠ°Π³Π½ΠΈΡΠ½ΡΠΌ ΠΏΠΎΠ»Π΅ΠΌ ΠΈ ΠΠΠ‘ ΠΈΠ½Π΄ΡΠΊΡΠΈΠΈ Π² ΡΠ»Π΅ΠΊΡΡΠΎΠΌΠ°Π³Π½ΠΈΡΠ½ΠΎΠΉ ΡΠ΅ΠΏΠΈ.
Π£ΡΠ°Π²Π½Π΅Π½ΠΈΠ΅ Π·Π°ΠΊΠΎΠ½Π° Π€Π°ΡΠ°Π΄Π΅Ρ
ΠΡΠ΅Π΄ΠΏΠΎΠ»ΠΎΠΆΠΈΠΌ, ΠΊΠ°ΡΡΡΠΊΠ° ΠΈΠΌΠ΅Π΅Ρ N Π²ΠΈΡΠΊΠΎΠ². ΠΠ½ ΠΏΠΎΠΌΠ΅ΡΠ΅Π½ Π² ΠΌΠ°Π³Π½ΠΈΡΠ½ΠΎΠ΅ ΠΏΠΎΠ»Π΅ ΡΠ°ΠΊΠΈΠΌ ΠΎΠ±ΡΠ°Π·ΠΎΠΌ, ΡΡΠΎ dΟ/dt ΠΏΡΠ΅Π΄ΡΡΠ°Π²Π»ΡΠ΅Ρ ΡΠΊΠΎΡΠΎΡΡΡ ΠΏΠΎΡΠΎΠΊΠ°. Π€ΠΎΡΠΌΡΠ»Ρ Π΄Π»Ρ ΠΠΠ‘ ΠΈΠ½Π΄ΡΠΊΡΠΈΠΈ Ξ΅ ΠΈΠΌΠ΅ΡΡ Π²ΠΈΠ΄,
Ξ΅ = β N dΟ/dt (Π½Π΅ΠΏΡΠ΅ΡΡΠ²Π½Π°Ρ)
Ξ΅ = β N ΞΟ/Ξt (Π΄ΠΈΡΠΊΡΠ΅ΡΠ½Π°Ρ)
Π‘Π ΠΠ΄ΠΈΠ½ΠΈΡΠ° ΠΠΠ‘ : ΠΠΎΠ»ΡΡΡ
, ΡΡΠΎ Π΄ΠΈΠΊΡΡΠ΅ΡΡΡ Π·Π°ΠΊΠΎΠ½ΠΎΠΌ ΠΠ΅Π½ΡΠ°.
Π€Π°ΠΊΡΠΎΡΡ, Π²Π»ΠΈΡΡΡΠΈΠ΅ Π½Π° ΡΠ»Π΅ΠΊΡΡΠΎΠΌΠ°Π³Π½ΠΈΡΠ½ΡΡ ΠΈΠ½Π΄ΡΠΊΡΠΈΡ
1. ΠΠΎΠ»ΠΈΡΠ΅ΡΡΠ²ΠΎ Π²ΠΈΡΠΊΠΎΠ² : ΠΠ°Π²Π΅Π΄Π΅Π½Π½ΠΎΠ΅ Π½Π°ΠΏΡΡΠΆΠ΅Π½ΠΈΠ΅ ΠΏΡΡΠΌΠΎ ΠΏΡΠΎΠΏΠΎΡΡΠΈΠΎΠ½Π°Π»ΡΠ½ΠΎ ΠΊΠΎΠ»ΠΈΡΠ΅ΡΡΠ²Ρ Π²ΠΈΡΠΊΠΎΠ² ΠΏΡΠΎΠ²ΠΎΠ΄Π°. Π‘Π»Π΅Π΄ΠΎΠ²Π°ΡΠ΅Π»ΡΠ½ΠΎ, ΡΠ΅ΠΌ Π±ΠΎΠ»ΡΡΠ΅ ΡΠΈΡΠ»ΠΎ Π²ΠΈΡΠΊΠΎΠ², ΡΠ΅ΠΌ Π²ΡΡΠ΅ ΡΠΎΠ·Π΄Π°Π²Π°Π΅ΠΌΠΎΠ΅ Π½Π°ΠΏΡΡΠΆΠ΅Π½ΠΈΠ΅.
2. ΠΠ·ΠΌΠ΅Π½Π΅Π½ΠΈΠ΅ ΠΌΠ°Π³Π½ΠΈΡΠ½ΠΎΠ³ΠΎ ΠΏΠΎΠ»Ρ : ΠΠ°Π²Π΅Π΄Π΅Π½Π½ΠΎΠ΅ Π½Π°ΠΏΡΡΠΆΠ΅Π½ΠΈΠ΅ ΠΏΡΡΠΌΠΎ ΠΏΡΠΎΠΏΠΎΡΡΠΈΠΎΠ½Π°Π»ΡΠ½ΠΎ ΡΠΊΠΎΡΠΎΡΡΠΈ ΠΏΠΎΡΠΎΠΊΠ°. ΠΠ·ΠΌΠ΅Π½Π΅Π½ΠΈΠ΅ ΠΌΠ°Π³Π½ΠΈΡΠ½ΠΎΠ³ΠΎ ΠΏΠΎΡΠΎΠΊΠ° Π²Π»ΠΈΡΠ΅Ρ Π½Π° ΠΈΠ½Π΄ΡΡΠΈΡΠΎΠ²Π°Π½Π½ΠΎΠ΅ Π½Π°ΠΏΡΡΠΆΠ΅Π½ΠΈΠ΅. ΠΡΠΎ ΠΈΠ·ΠΌΠ΅Π½Π΅Π½ΠΈΠ΅ ΠΌΠΎΠΆΠ΅Ρ Π±ΡΡΡ Π²ΡΠΏΠΎΠ»Π½Π΅Π½ΠΎ ΠΏΡΡΠ΅ΠΌ ΠΏΠ΅ΡΠ΅ΠΌΠ΅ΡΠ΅Π½ΠΈΡ ΠΌΠ°Π³Π½ΠΈΡΠ° Π±Π»ΠΈΠΆΠ΅ ΠΊ ΠΊΠ°ΡΡΡΠΊΠ΅ ΠΈΠ»ΠΈ ΠΎΡ Π½Π΅Π΅ ΠΈ Π½Π°ΠΎΠ±ΠΎΡΠΎΡ.
2. ΠΠ°ΠΊΠΎΠ½ ΠΠ΅Π½ΡΠ°
ΠΠ°ΠΊΠΎΠ½ Π€Π°ΡΠ°Π΄Π΅Ρ Π½ΠΈΡΠ΅Π³ΠΎ Π½Π΅ Π³ΠΎΠ²ΠΎΡΠΈΡ ΠΎ ΡΠΎΡ ΡΠ°Π½Π΅Π½ΠΈΠΈ ΡΠ½Π΅ΡΠ³ΠΈΠΈ. ΠΠ°ΠΊΠΎΠ½ ΠΠ΅Π½ΡΠ° ΠΌΠΎΠΆΠ΅Ρ ΠΎΠ±ΡΡΡΠ½ΠΈΡΡ ΠΎΡΡΠΈΡΠ°ΡΠ΅Π»ΡΠ½ΡΠΉ Π·Π½Π°ΠΊ Π² ΡΡΠ°Π²Π½Π΅Π½ΠΈΠΈ Π·Π°ΠΊΠΎΠ½Π° Π€Π°ΡΠ°Π΄Π΅Ρ. ΠΠ°ΠΊΠΎΠ½ ΠΠ΅Π½ΡΠ° Π³Π»Π°ΡΠΈΡ: Β«ΠΠΎΠ»ΡΡΠ½ΠΎΡΡΡ ΠΠΠ‘ ΠΈΠ½Π΄ΡΠΊΡΠΈΠΈ ΡΠ°ΠΊΠΎΠ²Π°, ΡΡΠΎ ΠΎΠ½Π° ΠΏΡΠΎΡΠΈΠ²ΠΎΠ΄Π΅ΠΉΡΡΠ²ΡΠ΅Ρ ΠΈΠ·ΠΌΠ΅Π½Π΅Π½ΠΈΡ ΠΌΠ°Π³Π½ΠΈΡΠ½ΠΎΠ³ΠΎ ΠΏΠΎΡΠΎΠΊΠ°, Π²ΡΠ·Π²Π°Π²ΡΠ΅ΠΌΡ Π΅Π΅Β». ΠΡΡΠ³ΠΈΠΌΠΈ ΡΠ»ΠΎΠ²Π°ΠΌΠΈ, ΠΈΠ½Π΄ΡΡΠΈΡΠΎΠ²Π°Π½Π½ΡΠΉ ΡΠΎΠΊ Π²ΡΠ΅Π³Π΄Π° Π±ΡΠ΄Π΅Ρ ΠΏΡΠΎΡΠΈΠ²ΠΎΠ΄Π΅ΠΉΡΡΠ²ΠΎΠ²Π°ΡΡ Π΄Π²ΠΈΠΆΠ΅Π½ΠΈΡ, Π²ΡΠ·Π²Π°Π²ΡΠ΅ΠΌΡ Π΅Π³ΠΎ Π² ΠΏΠ΅ΡΠ²ΡΡ ΠΎΡΠ΅ΡΠ΅Π΄Ρ. Π‘Π»Π΅Π΄ΠΎΠ²Π°ΡΠ΅Π»ΡΠ½ΠΎ, Π·Π°ΠΊΠΎΠ½ ΠΠ΅Π½ΡΠ° ΠΌΠΎΠΆΠ΅Ρ ΠΎΠΏΡΠ΅Π΄Π΅Π»ΠΈΡΡ Π½Π°ΠΏΡΠ°Π²Π»Π΅Π½ΠΈΠ΅ ΠΈΠ½Π΄ΡΠΊΡΠΈΠΎΠ½Π½ΠΎΠ³ΠΎ ΡΠΎΠΊΠ°.
ΠΠ°ΠΊΠΎΠ½ ΠΠ΅Π½ΡΠ° Π½Π°Π·Π²Π°Π½ Π² ΡΠ΅ΡΡΡ Π½Π΅ΠΌΠ΅ΡΠΊΠΎΠ³ΠΎ ΡΠΈΠ·ΠΈΠΊΠ° ΠΠ΅Π½ΡΠΈΡ Π° Π€ΡΠΈΠ΄ΡΠΈΡ Π° ΠΠ΅Π½ΡΠ° ΠΏΠΎΡΠ»Π΅ ΡΠΎΠ³ΠΎ, ΠΊΠ°ΠΊ ΠΎΠ½ Π²ΡΠ²Π΅Π» Π΅Π³ΠΎ Π² 1834 Π³ΠΎΠ΄Ρ.
- ΠΠ»Π΅ΠΊΡΡΠΈΡΠ΅ΡΠΊΠΈΠΉ Π³Π΅Π½Π΅ΡΠ°ΡΠΎΡ : ΠΡΠ΅ΠΎΠ±ΡΠ°Π·ΡΠ΅Ρ ΠΊΠΈΠ½Π΅ΡΠΈΡΠ΅ΡΠΊΡΡ ΡΠ½Π΅ΡΠ³ΠΈΡ Π² ΡΠ»Π΅ΠΊΡΡΠΈΡΠ΅ΡΠΊΡΡ ΠΈ ΠΏΡΠΎΠΈΠ·Π²ΠΎΠ΄ΠΈΡ ΡΠ»Π΅ΠΊΡΡΠΎΡΠ½Π΅ΡΠ³ΠΈΡ Π½Π° ΡΠ»Π΅ΠΊΡΡΠΎΡΡΠ°Π½ΡΠΈΡΡ . ΠΠ°ΡΠ΅ΠΌ ΡΠ»Π΅ΠΊΡΡΠΈΡΠ΅ΡΡΠ²ΠΎ ΠΏΠΎΠ΄Π°Π΅ΡΡΡ Π² Π΄ΠΎΠΌΠ°.
- ΠΠ»Π΅ΠΊΡΡΠΈΡΠ΅ΡΠΊΠΈΠΉ ΡΡΠ°Π½ΡΡΠΎΡΠΌΠ°ΡΠΎΡ : ΠΠ½ ΠΈΠ·ΠΌΠ΅Π½ΡΠ΅Ρ Π½Π°ΠΏΡΡΠΆΠ΅Π½ΠΈΠ΅ ΠΈ Π±ΡΠ²Π°Π΅Ρ Π΄Π²ΡΡ ΡΠΈΠΏΠΎΠ² β ΠΏΠΎΠ²ΡΡΠ°ΡΡΠΈΠΉ ΠΈ ΠΏΠΎΠ½ΠΈΠΆΠ°ΡΡΠΈΠΉ. ΠΠΎΠ²ΡΡΠ°ΡΡΠΈΠΉ ΡΡΠ°Π½ΡΡΠΎΡΠΌΠ°ΡΠΎΡ ΡΠ²Π΅Π»ΠΈΡΠΈΠ²Π°Π΅Ρ Π½Π°ΠΏΡΡΠΆΠ΅Π½ΠΈΠ΅, Π° ΠΏΠΎΠ½ΠΈΠΆΠ°ΡΡΠΈΠΉ ΠΏΠΎΠ½ΠΈΠΆΠ°Π΅Ρ Π½Π°ΠΏΡΡΠΆΠ΅Π½ΠΈΠ΅.
- ΠΠΈΠ½Π°ΠΌΠΎ : ΠΡΠΎ Π½Π΅Π±ΠΎΠ»ΡΡΠΎΠΉ Π³Π΅Π½Π΅ΡΠ°ΡΠΎΡ, ΡΠΎΡΡΠΎΡΡΠΈΠΉ ΠΈΠ· ΠΏΠΎΡΡΠΎΡΠ½Π½ΠΎΠ³ΠΎ ΠΌΠ°Π³Π½ΠΈΡΠ°, ΠΊΠΎΡΠΎΡΡΠΉ Π²ΡΠ°ΡΠ°Π΅ΡΡΡ Π²Π½ΡΡΡΠΈ Π½Π΅ΠΏΠΎΠ΄Π²ΠΈΠΆΠ½ΠΎΠΉ ΠΊΠ°ΡΡΡΠΊΠΈ Π·Π° ΡΡΠ΅Ρ Π΄Π²ΠΈΠΆΠ΅Π½ΠΈΡ Π²Π΅Π»ΠΎΡΠΈΠΏΠ΅Π΄Π½ΠΎΠ³ΠΎ ΠΊΠΎΠ»Π΅ΡΠ°.
- ΠΠ½Π΄ΡΠΊΡΠΈΠ²Π½ΠΎΡΡΡ : ΠΡΠΎ ΠΏΡΠΎΡΡΠ°Ρ ΠΊΠ°ΡΡΡΠΊΠ° Ρ ΡΠ΅ΡΡΠΎΠΌΠ°Π³Π½ΠΈΡΠ½ΡΠΌ ΡΠ΅ΡΠ΄Π΅ΡΠ½ΠΈΠΊΠΎΠΌ, ΠΎΠ±ΡΡΠ½ΠΎ ΡΠ²Π»ΡΡΡΠ°ΡΡΡ ΡΠ°ΡΡΡΡ ΡΠ»Π΅ΠΊΡΡΠΈΡΠ΅ΡΠΊΠΎΠΉ ΡΠ΅ΠΏΠΈ. ΠΠ½ Π½Π°ΠΊΠ°ΠΏΠ»ΠΈΠ²Π°Π΅Ρ ΡΠ½Π΅ΡΠ³ΠΈΡ Π² ΠΌΠ°Π³Π½ΠΈΡΠ½ΠΎΠΌ ΠΏΠΎΠ»Π΅ ΠΊΠ°ΡΡΡΠΊΠΈ, ΠΊΠΎΠ³Π΄Π° ΡΠ΅ΡΠ΅Π· Π½Π΅Π΅ ΠΏΡΠΎΡ ΠΎΠ΄ΠΈΡ ΡΠΎΠΊ.
ΠΡΠ΅ΠΈΠΌΡΡΠ΅ΡΡΠ²Π° ΡΠ»Π΅ΠΊΡΡΠΎΠΌΠ°Π³Π½ΠΈΡΠ½ΠΎΠΉ ΠΈΠ½Π΄ΡΠΊΡΠΈΠΈ
ΠΠΎΡ Π½Π΅ΠΊΠΎΡΠΎΡΡΠ΅ ΠΏΡΠ΅ΠΈΠΌΡΡΠ΅ΡΡΠ²Π° ΡΠ»Π΅ΠΊΡΡΠΎΠΌΠ°Π³Π½Π΅ΡΠΈΠ·ΠΌΠ°.
- ΠΠ΅ΡΠ΅Π²ΡΠΉ, ΠΏΡΠΎΡΡΠΎΠΉ, Π½Π°Π΄Π΅ΠΆΠ½ΡΠΉ, ΡΡΡΠ΅ΠΊΡΠΈΠ²Π½ΡΠΉ ΠΈ ΡΠΏΡΠ°Π²Π»ΡΠ΅ΠΌΡΠΉ
- ΠΠ΅Ρ Π°Π½ΠΈΡΠ΅ΡΠΊΠ°Ρ ΡΠ½Π΅ΡΠ³ΠΈΡ ΠΌΠΎΠΆΠ΅Ρ Π±ΡΡΡ ΠΏΡΠ΅ΠΎΠ±ΡΠ°Π·ΠΎΠ²Π°Π½Π° Π½Π΅ΠΏΠΎΡΡΠ΅Π΄ΡΡΠ²Π΅Π½Π½ΠΎ Π² ΡΠ»Π΅ΠΊΡΡΠΈΡΠ΅ΡΠΊΡΡ
- ΠΡΠΎΠΈΠ·Π²ΠΎΠ΄ΠΈΡ ΡΠ»Π΅ΠΊΡΡΠΈΡΠ΅ΡΠΊΠΈΠΉ ΡΠΎΠΊ ΠΈ ΠΏΠΈΡΠ°Π΅Ρ Π΄ΠΎΠΌΠ°
ΠΡΠΈΠΌΠ΅ΡΡ ΠΏΡΠΎΠ±Π»Π΅ΠΌ ΠΈ ΡΠ΅ΡΠ΅Π½ΠΈΠΉ
P.1. ΠΠ°ΠΌΠΊΠ½ΡΡΠ°Ρ ΠΊΠ°ΡΡΡΠΊΠ° ΠΈΠ· 50 Π²ΠΈΡΠΊΠΎΠ² ΠΏΠ»ΠΎΡΠ°Π΄ΡΡ 150 ΡΠΌ 2 Π²ΡΠ°ΡΠ°Π΅ΡΡΡ Π² ΠΌΠ°Π³Π½ΠΈΡΠ½ΠΎΠΌ ΠΏΠΎΠ»Π΅ Ρ ΠΈΠ½Π΄ΡΠΊΡΠΈΠ΅ΠΉ 2 ΠΠ±Β·ΠΌ -2 . ΠΠ½ ΠΏΠΎΠ²ΠΎΡΠ°ΡΠΈΠ²Π°Π΅ΡΡΡ ΠΈΠ· ΠΏΠΎΠ»ΠΎΠΆΠ΅Π½ΠΈΡ, ΠΊΠΎΠ³Π΄Π° Π΅Π³ΠΎ ΠΏΠ»ΠΎΡΠΊΠΎΡΡΡ ΡΠΎΡΡΠ°Π²Π»ΡΠ΅Ρ ΡΠ³ΠΎΠ» 45Β° Ρ ΠΏΠΎΠ»Π΅ΠΌ, Π² ΠΏΠΎΠ»ΠΎΠΆΠ΅Π½ΠΈΠ΅, ΠΏΠ΅ΡΠΏΠ΅Π½Π΄ΠΈΠΊΡΠ»ΡΡΠ½ΠΎΠ΅ ΠΏΠΎΠ»Ρ, Π·Π° Π²ΡΠ΅ΠΌΡ 0,3 Ρ. ΠΠ°ΠΉΡΠΈ Π²Π΅Π»ΠΈΡΠΈΠ½Ρ ΠΠΠ‘, Π½Π°Π²Π΅Π΄Π΅Π½Π½ΠΎΠΉ Π² ΠΊΠ°ΡΡΡΠΊΠ΅ Π·Π° ΡΡΠ΅Ρ Π΅Π΅ Π²ΡΠ°ΡΠ΅Π½ΠΈΡ.
Π‘ΠΎΠ»Π½. , Π΄Π°Π½ΠΎ,
n = 50
B = 2 WB M -2
A = 150 ΡΠΌ 2 = 150 x 10 -4 M 2 = 0,015 M 2 9005
ΞΈ = = 0,015 M 2
ΞΈ = = 0,015 M 2
ΞΈ = = 0,015 M 2 9000
ΞΈ = = 0,015 M 2 9000
ΞΈ = = 0,015 ΠΌ 2
ΞΈ = 0,015 ΠΌ 2 ΞΈ = 0,015 Π 2 .